Вы находитесь на странице: 1из 296

Thinking Your Way to Freedom

Thinking Your Way to Freedom


A Guide to Owning Your Own Practical Reasoning

SUSAN T. GARDNER
Illustrations by Dirk van Stralen

Temple University Press

Philadelphia

Temple University Press 1601 North Broad Street Philadelphia PA 19122 www.temple.edu/tempress Copyright 2009 by Temple University All rights reserved Published 2009 Printed in the United States of America Comics Susan T. Gardner. Used by permission. a The paper used in this publication meets the requirements of the American National Standard for Information SciencesPermanence of Paper for Printed Library Materials, ANSI Z39.481984 Library of Congress Cataloging-in-Publication Data Gardner, Susan T., 1945 Thinking your way to freedom : a guide to owning your own practical reasoning / Susan T. Gardner ; Dirk van Stralen, illustrator. p. cm. Includes bibliographical references and index. ISBN 978-1-59213-867-8 (pbk. : alk. paper) 1. Reasoning. 2. Practical reason. 3. Critical thinking. 4. LibertyPhilosophy. 5. Autonomy (Philosophy). I. van Stralen, Dirk, 1966 ill. II. Title. BC177.G37 2009 160dc22 2008023988 2 4 6 8 9 7 5 3 1

contents
List of Comics ix Acknowledgments xi Introduction 1
Pre-TeSTS 3 Pre-test 1: What Kind of Thinker Are You? 4 Pre-test 2: Demonstrating the Need for Logic 5

PART I section 1

TheORY

The Possibility of Freedom 9


THe GOAL, SUreLY, MUST Be FreeDOM 10 HOW IS FreeDOM POSSIBLe? 10 Negative Freedom, or Self-Direction 11 Positive Freedom, or Autonomy 11 The Dynamics of Value 13 Getting Control of Your Own Values 18 FreeDOM THrOUGH THe IMPArTIAL exAMInATIOn OF VALUeS 18

section 2

Impartial Thinking 23
BIAS neUTrALIZATIOn IS An InTer- (nOT InTrA-) SUBJeCTIVe PrOCeSS 25 JUDGInG QUALITY: eSTIMATInG TrUTH THrOUGH FALSIFICATIOn 28 Establishing Truth through a Falsification Process Is Not Possible, but Estimating truth Is 29 Truth Seeking in Ethics 31 Practical Reasoning Is Inevitably a Two-Step Falsification Process 31 Talking to the Relativist 34 THe MeSSAGe 36 POSTSCrIPT: FreeDOM neeDS DeTerMInISM 40 reVIeW QUeSTIOnS 41 AnSWerS TO reVIeW QUeSTIOnS 42

vi

COnTenTs

PART II section 1

PRACTICe

Learning the Intricacies of Practical Reasoning 49


1. KnOWInG WHAT TO LOOK FOr: reASOnS VerSUS eVIDenCe 52 2. PUSHInG TOWArD PreCISIOn 58 3. TAKInG A LOOK AT YOUr OWn VALUeS (AnD COMInG UP WITH GOOD THeSIS STATeMenTS) 62 4. COMMOn InFOrMAL FAULTS Or FALLACIeS 76 1. Begging the Question 76 2. Ad Hominem/Ad Feminam Attack 80 3. Appeal to Authority 83 4. Strawperson 86 5. Slippery Slope 87 6. False Dilemma 88 7. Distinction without a Difference 90 8. Post Hoc Fallacy 91 9. Analogies: Good and Faulty 93 10. Fallacy of the Golden Mean 98 5. WHAT KInD OF ArGUMenT IS IT? 102 6. SeeInG THe WHOLe ArGUMenT: A VALID DeDUCTIVe ArGUMenT IS A neCeSSArY COnDITIOn FOr eVALUATInG reASOnS (SOUnDneSS) 105 7. eVALUATInG reASOnS Or SOUnDneSS (LOCAL SUFFICIenCY) 112 8. eVALUATInG THe LOCAL SUFFICIenCY OF YOUr OWn AnD YOUr OPPOSITIOnS POSITIOnS 127 9. eVALUATInG THe GLOBAL SUFFICIenCY OF YOUr OWn POSITIOn 131 10. AVOIDInG rOTTWeILer FLIPS: GeTTInG YOUr COUnTerexAMPLeS STrAIGHT 136 11. Are YOU MAKInG A CLAIM ABOUT A SUFFICIenT Or A neCeSSArY COnDITIOn? 142 12. BACK TO SeeInG THe WHOLe ArGUMenT: FInDInG THe HIDDen PreMISe In FOrCeD-CHOICe SITUATIOnS 156 13. reSPOnDInG TO InCOrreCT COUnTerexAMPLeS 159 14. DeDUCInG FrOM COnDITIOnAL Or ALL CLAIMS: VALID AnD InVALID MOVeS 163 15. OVerVIeW 180 POST-TeSTS 184 Post-test 1: What Kind of Thinker Are You? 185 Post-test 2: Logic 186 A PerSOnAL GOOD-BYe 188

COnTenTs

vii

section 2

Thinking and Writing Your Way to Truth 189


InTerACTIVe LeArnInG In YOUr IMAGInATIOn 190 WHAT A GOOD (IMPArTIAL) ArGUMenT LOOKS LIKe 190 DeTAILeD AnALYSIS OF THe FIVe eSSenTIAL ArGUMenT COnSTITUenTS 191 1. A Clear Thesis Statement in Support of One Side of a Highly Contentious Issue 191 2. A Convincing Support for the Thesis Statement 193 3. Articulation of a Strong Opposition 196 4. A Convincing Response to the Opposition 198 5. A Convincing Resolution or Conclusion to the Posed Problem SUMMArY FOr eVALUATInG THe FIVe eSSenTIAL ArGUMenT COnSTITUenTS 204 InTerACTIVe reASOnInG 206

201

appendix I

Answers to exercises 207

appendix II

Analyzing Arguments 233

appendix III

examples of Good Arguments 255

appendix IV appendix V

What Good and Poor Thinkers Look Like 265

Answers to Pre-tests and Post-tests 267


Pre-TeST 1: WHAT KInD OF THInKer Are YOU? SCOrInG POST-TeST 2: LOGIC AnSWerS 268 267

notes 269 Glossary 273 Index 277

list of comics
Introduction
1. Squandering Self-Consciousness 1

PART I section 1
2. 3. 4. 5. 6. 7. 8. 9.

TheORY

The Possibility of Freedom


How Is Freedom Possible? 9 Self-Control Is Not Autonomy (I) 12 Self-Control Is Not Autonomy (II) 13 Learning through Linguistically Enhanced Imagination Is the Human Trump Card 14 Language Tightens the Strings of Social Control 16 Materialism Threatens Freedom 17 Freedom Requires Courage 19 Self-Consciousness Is a Double-Edged Sword 21

section 2

Impartial Thinking
10. Social Determinism Can Be Invisible 23 11. To Be a Chooser Requires Genuine Dialogue with Those Who Think Differently 26 12. You Have to Engage Others in Order to Be Free of Them 13. Truth through Falsification 30 14. Seeking the Impossible Quest 32 15. Truth Seeking Is Always a Two-Step Process 33 16. Not Everyones Opinions Are Equally Good 34 17. What Is Objective Is Faulty Reasoning, Not Truth 35 18. Truth Seeking, Not Truth, Shall Set You Free 37 19. The Stop and Go Signs toward Autonomy 38

27

PART II section 1
20. 21. 22. 23. 24. 25. 26. 27.

PRACTICe

Learning the Intricacies of Practical Reasoning


Seeking a Good You Rather than Good Times 50 Doing Nothing Is Not an Option: It Is Doing Something 51 Wisdom versus Intelligence 53 Freedom Is about the Future 55 Imprecision Is Inefficient in Action Guiding 58 Without Precision, Value Judgments Cannot Act as Magnets 61 If You Do Not Care, You Are Not in Charge 63 The Mind Is Opened by What Is Interesting; What Is Interesting Is Contentious 64

ix

LIsT OF COmICs

28. 29. 30. 31. 32. 33. 34. 35. 36. 37. 38. 39. 40. 41. 42. 43. 44. 45. 46. 47. 48. 49. 50. 51. 52. 53. 54. 55. 56. 57. 58. 59. 60.

Labeling Relevant Behavior Can Be Action Guiding 67 Your Present Values Are Your Window into Your Future 68 Are Your Emotions Those That You Choose? 69 Values Become Yours Only When They Inform Your Actions 71 You May Be Blind to the Person You Are Becoming 73 A Reason That Is Not a Reason: Begging the Question (I) 77 A Reason That Is Not a Reason: Begging the Question (II) 79 Attacking the Arguer Rather than the Argument Misses the Point 81 An Appeal to Tradition Can Sometimes Be a Reason 85 Reasons Are Strong Only by Comparison: Bowling for Truth a Strawpin Argument 87 Reasons Can Sometimes Connect A to Z: Slippery Slope Arguments 89 Wisdom Varies with the Options Considered: False Dilemmas 90 Own Up to an Accurate Description of Your Attitude: Distinction without a Difference 92 Bad Reasoning Can Misplace Values (and Associated Responses) 94 Analogizing Your Way toward Truth 96 Truth Is a Function of Reason and/or Evidence, Not Mathematics: Fallacy of the Golden Mean 98 Reasoning Changes Behavior by Moving Value 106 Breaking the Major Premise 110 The Wonders of Fault Finding 114 I Am Talking about Rapists, Not Enron Executives 116 I Am Talking about Homosexuals, Not Glasses 118 What Would Count as Falsifying Your Belief? 120 The Positivity of Negativity 123 Me Only because of We 124 Getting to Global Sufficiency by Testing Local Sufficiency through Counterexamples 128 All Rottweilers Are Dogsi.e., Being a Rottweiler Is a Sufficient Condition for Being a Dog 146 A Rottweiler Flip: Mistaking a Necessary for a Sufficient Condition 147 A Yorkie Flip 159 Bad Deductive Reasoning Can Lead to Bad Decisions (I) 166 Bad Deductive Reasoning Can Lead to Bad Decisions (II) 167 Bad Deductive Reasoning Can Lead to Bad Decisions (III) 170 Bad Deductive Reasoning Can Lead to Bad Decisions (IV) 172 Bad Deductive Reasoning Can Misplace Values (and Associated Responses) 174

section 2
61. 62. 63. 64. 65. 66.

Thinking and Writing Your Way to Truth


It Is More Important to Be Right than to Appear Right 189 Begging the Question: Boring Hypocritical Repetition 195 Strawperson Argument: Bow-wow Chow 197 Language Games 200 Writing Can Be a Form of Interactive Listeningor Not 202 When Self-Consciousness Takes Up the Challenge, It Is a Wonder to Behold 205

acknowledgments
This text belongs to many extraordinary students who, with laughter and tears, courageously agonized their way through a journey toward their own freedom that was set out for them in many preliminary drafts. It was a privilege and an honor to learn with them. I am also grateful to my professorial, administrative, and staff colleagues at Capilano Universitya gem of an educational institution in North Vancouver, Canada, where excellence in teaching is par for the course and where every effort is made to make all things possible (in this case, my paid educational leave to attend to the finetuning and publishing of this text). In particular, I would like to thank Sally Spires and Barb Smith, who patiently and enthusiastically oversaw various drafts and who sent them every which where as necessary, and Carol Hamshaw, for helping me work with a graphics computer program that was unfamiliar to me. And with respect to the graphics, I owe a great deal to Dirk van Stralen, who labored so patiently to capture the essence of my two pups, Diva and Bennie. I would also like to thank Mick Gusinde-Duffy, acquisitions editor at Temple University Press, who recognized the value and the risk inherent in this new method of critical thinking and who went beyond the call of duty to shepherd this work through to publication. My gratitude also goes to Joan Polsky Vidal, production editor at P. M. Gordon Associates, who worked tirelessly in editing multiple drafts of this text. I am also grateful to Katie Himsworth, who assisted in proofreading the companion Instructors Manual, and Clive Himsworth, who assisted in proofreading the final version of the text. I continue to be indebted to Alan Montefiore, my tutor and friend during my years at Oxford University, who was responsible not only for initiating my lifelong love affair with Immanuel Kant but also for encouraging me to play with obtuse ideas. Such Kantian play is evident in the pages that follow. Ultimately, however, virtually all of what I have come to understand about thinking ones way to freedom is grounded in the interactive journey I have undertaken with my daughter, Chelsea, since the day she was born. Now twenty-five years old and a doctor of veterinary medicine, she continues to be my confidante and my inspiration. I dedicate this book to her.

xi

Thinking Your Way to Freedom

introduction
SQUANDERING SELF-CONSCIOUSNESS
At its best, schooling can be about how to make a life, which is quite different from how to make a living.
NEIL POSTMAN1

introduction

How is it possible that even though what we do has lifelong implications for ourselves as individuals, for our loved ones, and for the communities of which we are members, so little of our formal education focuses on analyzing, let alone upgrading, our practical reasoningthe reasoning that leads to action? Why is this the case? The answer may lie in the fact that most of us assume that we humans are free in the sense of being self-legislating because of our power of thinking per se. This belief is false. Although the emergence of symbolic language has moved humans out of the frying pan of strict conditioning, it has flipped us into the fire of psychological and sociological determinism. With the emergence of linguistic deductive reasoning, outside sources gained the ability to implant values directly into the heads of others. And the fact that rebellion often results when outside sources conflict is no more indicative of freedom than a dog suddenly refusing to jump through a hoop because it had been reinforced by another trainer not to leave the ground. However, we need not be blindly subjected to one anothers invisible linguistic conditioning. Self-legislation is possible, but it requires that we take control of our own practical reasoning. This is not to say that one must simply learn how to supply oneself with reasons. If autonomy is the goal, then there is a certain kind of reasoning that must be mastered: the kind that neutralizes outside influence or bias. After all, to be autonomous means that one can make decisions by oneself, that ones decisions are not determined by outside pressures. Reasoning that is not determined by outside pressure is impartial reasoning. Part I of this book focuses on a theoretical analysis of practical reasoning: how freedom is possible through impartial reasoning (Section 1) and, in general, what impartial reasoning looks like (Section 2). Since one must be convinced that one is not already there in order to be seduced into taking the journey, Part I is the critical motivator for this entire endeavor, whichif autonomy is the goalmust be woven into every judgment that one makes hereafter. In order for impartial practical reasoning to actually fuel autonomy, it must be logically adequate. Logically adequate impartial reasoning requires that one assemble as many potential candidates for truth as possible, test the adequacy of the reasoning behind each option (referred to as establishing local sufficiency), and then determine the least-weak candidate (referred to as establishing global sufficiency). Section 1 of Part II focuses on local sufficiency, that is, estimating the adequacy of individual arguments. Section 2 focuses on global sufficiency, that is, estimating the adequacy of individual arguments relative to those of other contenders. Since there is a dynamic relationship between Sections 1 and 2, students will be instructed to move back and forth between them. However, since Section 2 provides a template for writing any argumentative essay, it has been written as a self-sufficient unit (with only passing reference to Section 1) so that it can serve as a quick reference point for future thinking and writing. What follows, then, is no mere academic exercise. Since you are a self-conscious language user, you have the capacity to create yourself. However, if you do not actively take control of that process, you will remain a slave to the physical, psychological, social, and political forces to which you are constantly subjected. If you do not actively take control of that process, you will continue to be whatever others want you to be. You can instead become your own person, the master of your own fate. The choice is yours.

introduction

Pre-tests
Before you begin the journey laid out for you here, it is recommended that you take the two following pre-tests: What kind of thinker are you? and Demonstrating the need for logic. The first pre-test measures what kind of thinker you are. You will be asked to complete the same test at the end of the course, so that you will have an empirical measure of the degree to which the way you think has changed after taking the journey set out for you here. Thus, although the marking scheme is included in Appendix V, you should not mark this pre-test until you have completed the post-test at the end of the course. The second pre-test focuses on logic. You will be asked to complete a similar post-test at the end of the course. (The answers to both tests are also included in Appendix V.) You should mark the logic pre-test immediately. If your score is low, you should be both concerned and surprised, as this test measures merely how well you understand the meaning of simple sentences. You should also be particularly keen on tackling the logic portion of what is to follow, as it is logic that will help to transform your present messy understanding.

Pre-test 1: What Kind of Thinker Are You?


The following questionnaire was designed to help you understand yourself a little better and find out to what extent you might benefit from exposure to philosophy. Please answer the questions as honestly as you can. This questionnaire is for your benefit only. You will use it later in the course. For items 1 to 13, tick the column with the word that most appropriately fits in the round brackets provided. For items 14 to 17, tick the appropriate answer.
Often 1. I()askalotofquestions. 2. hendiscussingissues,I()getfrustratedwhenpeopledisagreewithme. W 3. tis()awasteoftimetotalktootherswhothinkcompletelydifferentlyfromthe I wayIdo. 4. ()questionothersindetailinordertoclarifywhattheyhavejustsaid. I 5. naheatedexchange,whenothersaretalking,I()usethatopportunitytoconsider I howIcanrestatemyownpositionmorepersuasively. 6. ()findithumiliatingtobackdown. I 7. am()reluctanttochangemypositiononcertainissues,becausemypositionon I thoseissuesispartofwhoIam. 8. ()expressconfidenceinapositionevenif,atthetime,Iamunabletobackitup I convincingly. 9. have()saidthatsinceIdonothaveadequatedataorinformation,Iamnotina I positiontogiveanopiniononanissue. 10. I()findcomplexityannoying. 11. Iam()reluctanttodisagreewithpeoplewhoareimportanttome. 12. ()seriouslyquestionwhatagoodlifeisandwhatkindofpersonIwanttobecome. I 13. I()spendtimejustthinking Yes 14. Ihaveapreciseideaofwhatagoodthinkerlookslike. 15. candefinegoodthinking. I 16. hensomeoneelseusespoorthinking,Iunderstandandcanexplainwhythe W thinkingisfaulty. 17. Havingalotofmoneyisnecessaryformywell-being. Dr. S. T. Gardner, the Vancouver Institute of Philosophy for Children, 1998. Sort of No Sometimes Rarely

Pre-test 2: Demonstrating the Need for Logic


Please circle the answer that you think is most appropriate, and then check the answers for both the pre- and post-test. A comparison of the pre- and post-test scores should give you some insight into the degree to which this journey has helped you acquire a more precise understanding of what you and others mean when you speak to one another. 1. Suppose a doctor says to you, Your grandmother will live only if she has this operation. If this is true, it follows that a. If your grandmother has this operation but she does not live, you will know that the doctor was not telling the truth. b. If the doctor is telling the truth, you should feel relieved because now you know that if your grandmother has this operation, she will live. c. You should make arrangements for your grandmother to have this operation as quickly as possible because you know if she does not have this operation, she will die. 2. Suppose a doctor says to you, You will live if you have this operation. If this is true, it follows that a. You should make arrangements as quickly as possible to have this operation because you know if you do not have this operation, you will die. b. You should feel relieved because now you know that if you have this operation, you will live. c. Both a and b. 3. If it is true that only brilliant people get straight As, it follows that a. Since Jerry did not get straight As, he is not brilliant. b. All brilliant people get straight As. c. Sally must be brilliant, since she got straight As. 4. If it were true that people cannot become neurotic unless they are brought up in an unhappy home, it follows that a. All neurotics come from unhappy homes. b. Your cousin Jane is bound to become neurotic, since she is being brought up in a very unhappy home. c. If you are not neurotic, your home life could not have been that unhappy. 5. If it is true that only the good die young, it follows that a. Maria is a saint. I do not think she is long for this world. b. You do not have to worry about your cousin dying in that car crash. He has been nothing but trouble from an early age. c. Socrates could not have been as good as he has been made out to be. He lived to a ripe old age. 6. If it is true that without patience, one will inevitably fail, it follows that a. Since Jane failed at the task, she had no patience. b. Since John has infinite patience, he is sure to be a success. c. Neither a nor b. 7. If it is true that you will never be popular if you tell people what is really on your mind, then it follows that a. If you want to be popular, you should practice being deceitful, as being deceitful is necessary in order to be popular. b. Since John is a master at deceit, he is bound to be popular. c. Both a and b. 8. If it is true that loving people are honest people, it follows that a. Jane should not marry John. Since everyone knows that John is unbelievably dishonest, we can predict he will never really love her. b. Jane should marry Harry. Since everyone knows that Harry is as honest as the day is long, he is bound to be a loving mate. c. Neither a nor b.

9. If it is true that the person who can smile when things go wrong has found someone to blame it on, it follows that a. Since Harry can never find anyone to blame, you can be sure he will not be smiling when things go wrong. b. Joan is always smiling. That just goes to show that she must have found others to blame when things go wrong. c. Both a and b. 10. There is a saying that dull women have immaculate homes. If this is true, it follows that a. If a woman has a clean home, that means she must be dull. b. If a woman has a dirty home, that means that she is not dull. c. If a woman is not dull, that means you can be sure that she has a dirty home. 11. Let us suppose that someone tells you that s/he has a magic ring and that if you put it on and it turns green, that means that you are in love. On the assumption that the ring is truly magical, it follows that a. Since the ring does not turn green, that means you are not in love even if you think you are. b. Since you are not in love and you know it, it will not turn green. c. Since you are deeply in love and you know it, you can be sure that it will turn green. 12. The school principal said, This locker must belong to a boy, because it is a mess. The school principal must be assuming that a. All boys are messy. b. Some boys are messy. c. Only boys are messy. 13. Jane said, Everyone who takes calculus is a nerd. John said, That is not true. If John is right, then it must follow that a. People who take calculus are not nerds. b. There are lots of nerds who do not take calculus. c. At least one person who is taking calculus is not a nerd. 14. Studies have proven that if the water is treated with sufficient levels of chlorine, it is safe to drink. Since the water in our town has been found unsafe to drink, it follows that a. The water was treated with sufficient levels of chlorine. b. The water was not treated with sufficient levels of chlorine. c. We cannot tell whether or not the water was treated with sufficient levels of chlorine. 15. You heard someone say at a party, All men are selfish. If you wanted to demonstrate that that statement was false, the best way to do so would be to a. Show that there are many women who are selfish. b. Show that there is at least one man who is not selfish. c. Show that men in general are not selfish.

Part I
theory
section 1 section 2
the Possibility of Freedom
through

Impartial thinking

section 1
the Possibility of Freedom
HOW IS FREEDOM POSSIBLE?

10

Part I theory

The Goal, Surely, Must Be Freedom


If educators followed all reasonable advice about what they should teach students, they could fill their students time with more educational hours and days than they do now, and they would still have suggestions left over. So when the question arises about what students should learn, it is important that it be couched in terms of necessary conditions. We need to ask, What is it that is essential that students learn? What is it that, if it were left out, educators could be charged with being irredeemably irresponsible?2 Obviously, the knowledge and fundamental skills that are necessary for making a living are of paramount importance. But what about making a life? Is there something special, or essential, about being human such that, if it were not actuated, a person would fundamentally fall short of what it is to be human? The characteristic that can be described as essentially human is the capacity for linguistically enhanced self-consciousness.3 It is because humans are robustly self-conscious in this sense that they can imagine themselves in various possible future scenarios, that they can evaluate which future they would prefer, that they can estimate the degree to which they can contribute to the possibility of bringing that future into being, and that they can decide whether the cost is worth it. By contrast, merely conscious (i.e., non-selfconscious) entities (such as the local cow) are, by definition, not conscious of themselves i.e., they cannot imagine themselves in varying alternative situations. Merely conscious entities, again by definition, are clearly conscious of their environment (i.e., they respond to stimuli); they are simply not conscious of themselves in it. The behavioral and ethical consequence that accompanies this evolutionary leap in cognitive functioning is that self-conscious language users gain the capacity to direct their own actionsi.e., they can become masters of their own fate in a way and to a degree that non-self-conscious entities cannot. We already implicitly recognize the behavioral consequence of the development of self-consciousness by the fact that we hold self-conscious entitiesin contrast to merely conscious entitiesresponsible for their actions. Holding an entity responsible for its actions makes sense only if we believe that that entity is free to do other than it doesthat it has the capacity to choose. However, although we implicitly recognize the freedom of self-conscious entities, the fact that we remain ignorant of the precise dynamics that underpin freedom ultimately robs us of the possibility of maximizing its potential. The answer to the question initially posed about whether there is something essential that a self-conscious entity should learn must surely be this: Since it is the possibility of self-legislation that is unique and defining of self-conscious agents, whatever else educators should teach students, surely it is essential that they teach them how to harness that capacity and make it their own. Educators should teach students how to maximize their capacity for autonomy. We need to teach freedom. Aside from maximizing the potential for unique existence, there is perhaps a more concrete reason why we should teach freedom. Young humans are forever admonished to think. Colleges and universities encourage students to take critical thinking courses. Businesses list the capacity for critical and creative thinking at the top of important skills. Yet the default assumption of all of the natural and social sciences is that human beings are determined. If human beings are determined, however, then it is not clear how thinking can do any real work. Thus, showing how freedom is possible not only opens the way for maximizing that potential; it also constitutes a necessary precondition for demonstrating how and why critical thinking can enhance an individuals long-term well-being. Thinking and freedom are flip sides of the same human coin.

Self-conscious language users are unique in their capacity for self-direction.

Autonomy requires that self-conscious entities learn how to harness their capacity for self-direction. Thinking and freedom are flip sides of the same human coin.

Every one of us has to make himself into a free man. Freedom is a conquestalways partial, always precarious, always challenged.
GABRIEL MARCEL4

How Is Freedom Possible?


As self-conscious language users, humans have the capacity to become their own personsto be creators of themselves. However, this capacity is merely thata capacity, or a potentiality. In order for self-conscious agents to take more control over

SectIon 1 the PoSSIbIlIty oF Freedom

11

their own capacity for freedomand in so doing acquire a kind of second-order freedom (i.e., the freedom to maximize their own freedom)they must fully appreciate the mechanics that underpin the evolution of this qualitative difference in behavioral control.

Negative Freedom, or Self-Direction


The first step that is necessary in order to understand the dynamics of human freedom fully is to understand the default value of all animate behaviornamely, that it is controlled by the stimulus environment in which an animal moves. In behaviorist jargon, how an animal behaves is a function of the algebraic sum and relationship of appetitive stimuli that elicit approach behavior (such as food or a potential mate) and aversive stimuli that elicit avoidance behavior (such as fire, loud noises, or an enemy). Merely conscious entities are utterly at the mercy of the stimulus conglomeration of the surrounding environment. Humans, since they are animals, also respond to the stimulus conglomeration of the surrounding environment. How, then, do they gain control of their own behavior? How, precisely, do humans free themselves from the tyranny of the determining forces of the external environment? The human capacity for symbolic language, which enormously enhances the capacity of humans for self-consciousness, offers the answer. Because of their linguistically enhanced self-consciousness, humans can imagine themselves in far-flung future situations. These images, in turn, like the behavior-eliciting images of the external environment, also act as behavior-eliciting stimuli. It is these images that allow humans to gain control over their actions. That is, aside from reacting to the behavior-determining stimuli of the external environment, humans also respond to stimuli they themselves produce. These self-generated stimuli of imagined various future behaviors, actions, and interactions, which are imbued with varying degrees of positive and negative value, compete with the external5 behavior-guiding stimuli of the surrounding environment. A mundane example might help clarify this evolutionary miracle. A large and luscious piece of chocolate cake might exert as strong an appetitive pull on Joe as a steak does on Joes dog, Rover. However, unlike Rover, Joe has the capacity to imagine a future slim self (e.g., if he is overweight) or a future healthy self (e.g., if he is diabetic), and this imagined stimulus will serve as a competing stimulus force to the objective stimulus environment. The degree to which a projected imagined stimulus in fact alters Joes behavior presumably depends on the strength (i.e., the value) that Joe has assigned, consciously or otherwise, to that stimulus relative to the strength of the external stimulus environment. It is necessary to keep two points in mind in order to clearly understand this capacity for self-direction or self-control. First, self-control does not reside directly in action; rather, self-control resides in an agents capacity to think through an image of the future i.e., an agents capacity for self-consciousness. It is self-consciousnessi.e., the capacity to imagine oneself in various future scenarios (to think about and evaluate ends)that is a direct evolutionary product; self-control is simply a by-product. Second, self-control is not an all-or-nothing phenomenon; rather, it is always a matter of degree. In particular, it is a function of the strength of self-generated stimuli relative to the strength of the external stimulus environment in which an entity functions.

The possibility of self-control resides in the behavior-eliciting power of self-generated stimuli.

Self-control, which is always a matter of degree, is a function of an agents capacity to think into the future. You need to get control over your mind in order to control your behavior.

Positive Freedom, or Autonomy


If an image generated by a self-conscious agent overrides the strength of external stimuli, that individual can be described as having gained self-control. However, to say that this individual has gained the capacity for self-control is not to say that the individual has thereby become the author of his or her own actions. The dynamic thus far described merely depicts the movement of the locus of behavioral control from the external to the internal environment. As we are all too well aware, an individuals internal

12

Part I theory

Negative freedom (self-control): Ones acts are not determined by external forces. Positive freedom (autonomy): Ones judgments are not determined by external forces.

Self-control is the capacity to free ones actions from the determining influence of the external environment. Self-legislation is the capacity to free ones judgments from the determining influence of the external environment.

environment may nonetheless remain very much under the control of external influences. As we are all too well aware, what people think they should do in any given situation is very often a function of what others expect of them. What more is needed for a self-conscious agent to become truly autonomous? This question is the one that lies at the heart of Immanuel Kants moral philosophy. In attempting to answer this question, Kant emphasized the distinction between selfcontrol, which he referred to as negative freedom, and autonomy, which he referred to as positive freedom. Negative freedom, or self-control, can be characterized as the capacity to act in a way that is not determined by immediate external influences. In the preceding case, Joe can be described as free in the negative sense, because he freed himself from the appetitive pull of the slice of chocolate cake. Positive freedom, or autonomy, is different. Positive freedom, or what Kant also called self-legislation, is the capacity to free ones judgments (as opposed to merely ones actions) from the influence of external forces. In the preceding example, although Joes behavior was free in the negative sense, his decision may not have been autonomous at all. That is, Joes decision to forgo the cake may have been more a function of the wishes of his new love than a result of what he truly believed was a wise decision. Positive freedom, in other words, is the not just freedom from, it is freedom to. In particular, it is freedom to choose who it is you want to become. True freedom, that is, freedom in both the negative and the positive sense, is thus a two-step phenomenon. It requires that you free your behavior from the determining influence of external stimuli by responding to self-generated stimuli. However, it also requires that those self-generated stimuli be a genuine product of yourself, or your own thinking.

SELF-CONTROL IS NOT AUTONOMY (I)

SectIon 1 the PoSSIbIlIty oF Freedom

13

SELF-CONTROL IS NOT AUTONOMY (II)

The Dynamics of Value


Understanding the road to autonomy requires that you understand the role that value plays in animal and human life.6 Animals are originally set in motion and are kept in motion by value. Animals can be described as being set in motion by value in the sense that all animals begin life preprogrammed so that they respond to a range of objects or situations that are saturated with value. The shape of a hawk is saturated with a natural negative value for a gosling; a smile has a natural positive value for young humans. Through association, or learning, other objects and situations borrow value from the original sign stimuli. The resulting behavior is more complex, but the behavior nonetheless remains totally determined by percepts that are saturated with value, or what are more commonly referred to as external stimuli. This value/behavior dialectic can be illuminated through an analogy with color. Let us suppose that all animals are preprogrammed so that red is appetitive (i.e., red elicits an approach response) and blue is aversive (i.e., blue elicits an avoidance response). With association, red and blue rub off on various objects and situations so that, with extensive experience, an animals environment becomes a riot of color with many shades and variations of red, blue, and purple. Were we to have the appropriate metaphysical glasses, we would be able to predict an animals behavior merely by seeing the colors of its world. We would know, for instance, that a vibrant red would be extremely appetitive and a pale blue mildly aversive, while we would predict that a deep purple would elicit a highly ambivalent response. What is important to note with regard to the learning process is that value moves from object to object or from situation to situation as a result of association. What is
Behavior changes because of the perceived movement of value. Values move from one object or situation to another as a result of association.

14

Part I theory

also important to note with regard to the learning process is that this association can transpire either naturally or as a result of contrived pairing. The modification of the behavior of wild animals takes place entirely as a result of natural association, with the result being a fit between the animal and the environment that it inhabits. A domestic pet, on the other hand, is also subject to purposeful pairing, such as praise for urinating in the garden rather than in the house. A domestic pet, therefore, learns to fit into its social setting as well as its natural environment. The basic dynamic of behavioral control in both these situations, however, remains the same. The behavior of both wild and domestic animals is totally determined by values that adhere to external percepts.

With symbolic interaction, values can be moved in the imagination through reasoning.

LEARNING THROUGH LINGUISTICALLY ENHANCED IMAGINATION IS THE HUMAN TRUMP CARD

SectIon 1 the PoSSIbIlIty oF Freedom

15

With the emergence of symbolic language,7 the dynamics of value begin to change. Initially, the very young prelinguistic human, like a domestic pet, is subject to behavior modification both by natural association and by purposeful pairing. However, as the child matures cognitively and develops language, a qualitative change in the dynamic of value occurs. Now, instead of merely trying to repaint the values of a childs environment directly, a caregiver tries, through symbolic interactionspecifically, argumentation8to infuse values into the child (or into the childs inner imaginative world) in the hope that the child herself will project those values into relevant actual situations and in so doing be moved by them. Thus, with the development of language, the association processand hence, the movement of valuecan transpire in the imagination. Let us follow this change in the dynamic of value through an example. Let us suppose that a mother, Mrs. Smith, initially trains her child, Mary, through a typical association program: smiles and hugs for all the desired behaviors and negative nos for actions that she hopes to eliminate. With cognitive maturity and the development of language, however, Mrs. Smith begins to explain to Mary, that, for instance, generosity is a virtue that is highly regarded by others and that therefore Mary should always share her possessions with her playmates. In subsequent play situations, even if Mrs. Smith is absent, we can presume that although a new toy will serve as a strong behavior-eliciting stimulus for Mary, a projected image of a greatly admired child generously sharing her toys with her playmates will also exert an appetitive pull. What is happening here, in other words, is that a projected image of a self receiving praise from Mom serves as a behavior-eliciting stimulus that competes with the external9 behavioreliciting stimuli in the childs environment. If the self-projected stimulus overrides the external stimuli, Mary can be described as having gained self-control. From an evolutionary point of view, the emergence of self-control can be described as dramatic; from an everyday point of view, however, this drama is usually invisible. This is so because, on the surface, there is no obvious difference between, for example, Mary refraining from being selfish with her toys because it conflicts with her mothers worldview and a dog ceasing to chew on shoes because the behavior was paired with a wallop. However, the difference is dramatic, and that drama resides in the potentiality of the former that is absent from the latter. Clearly, the latteri.e., learning that results merely from physical associationis relatively crude and limited. On one hand, one can carry out only so many pairings; on the other, for an animal to be trained, it must first produce the behavior that one wants to positively or aversively reinforce. As well, the whole procedure must be done with great precision, so that one actually reinforces the desired behavior rather than a closely associated one. If Rover hesitates before coming, the subsequent treat may reinforce the hesitation rather than obedience. By comparison, values that emerge as a result of linguistic interaction are more malleable, complex, and precise, to say nothing of the virtually miraculous fact that human learning can transpire in the imagination rather than in actuality. Because selfconscious entities can imagine themselves in the future, they can imagine themselves doing both desired and undesired actions, and, through symbolic argumentation, others can pair those imagined behaviors with imagined positive or aversive reinforcement. Thus, in the preceding situation, if Mary had been a dog rather than a human child, the only way Mrs. Smith could have produced the desired sharing response would have been to somehow elicit a sharing response and then positively reinforce it (although, as we dog lovers know, eliciting a genuine sharing response from a dog is virtually impossible). However, since Mary is self-conscious and10 is capable of symbolic interaction, her mothers words can conjure up not only an image of Mary sharing her toys but also an image of her mother being mightily impressed by the fact that Mary is sharing her toys. Learning can thus take place in imagination. No actual behavior or actual reinforcement need take place. From a collective viewpoint, the evolutionary payoff of being able to undergo such complicated and precise learning without having to play it out in actuality is

With the emergence of language, the association process can transpire in the imagination.

Young self-conscious agents are often caught between the pull of external stimuli and the values of significant others.

Behold the emergence of social control on a vast scale. It is no wonder that these cooperative capacities have given humans a vast evolutionary advantage!

16

Part I theory

Because humans can be trained in their imagination, the studentteacher ratio for humans can be vastly increased (e.g., through schooling and propaganda), whereas the training of nonsymbolic animals usually requires a 1:1 ratio.

enormous. Such entities are able to coordinate their actions in a way that is far more complex and layered than entities that learn only through physical association. From an individual point of view, however, at least in terms of autonomy, entities seem to be going the wrong way. That is, precisely because language users are able to undergo such complicated and precise learning, language users appear to be subject to far more powerful and sophisticated external sources of behavior modification than non language users. Rather than move individuals toward autonomy, therefore, the emergence of linguistically enhanced self-consciousness appears to bring with it a kind of over-determination. Although linguistic interaction and imagined learning open the way for self-conscious entities to gain a high level of self-control, we need to keep firmly in mind that this so-called self-control is really behavior that results from projected values that have been introjected from others. It is precisely because of sym-

LANGUAGE TIGHTENS THE STRINGS OF SOCIAL CONTROL

SectIon 1 the PoSSIbIlIty oF Freedom

17

bolic interaction that Mary is far more finely tuned to the wishes of her mother than is their dog Rover. Self-direction does not result in the creation of a work of art; it is, rather, number painting with the design and color scheme having been predetermined. Self-directed individuals are not painting their own worlds; they are not in charge of their own practical judgments. The term self-control is thus a misnomer. The emergence of self-control is really the emergence of finely tuned social control. Unlike animals in the wild, which are controlled by the values that adhere to the natural environment, and unlike domestic pets, which are controlled by the natural environment and by contrived pairing, self-conscious language users are controlled by the natural environment and by contrived pairing and by introjected values that they themselves project into the environment. How can such entities reach for autonomy?

The emergence of selfcontrol is really the emergence of finely tuned social control.

MATERIALISM THREATENS FREEDOM

Without autonomy, we are just cogs in the social machine.

18

Part I theory

If the ultimate value for self-conscious entities is autonomy, it appears that there is irony in the route. In their pursuit of autonomy, it would seem that self-conscious entities must first go deeper into the quagmire of determination before rising out of it. This is so because, paradoxically, the capacity for self-consciousness, symbolic interaction, and value projectionall of which are necessary for autonomyfirst tightens the puppet strings. This is the risk of self-consciousness. If we lose ourselves in the journey, we are even farther from the ideal to which humans can aspire than we were at the beginning.

Getting Control of Your Own Values


It is value that moves behavior. Thus, if you want to control your own behavior, you must get control over your values. How is this done? The first step in taking control over your own behavior is understanding clearly how others have heretofore done so. Others have controlled your behavior either by literal purposeful pairing or by symbolic pairingi.e., deductive argumentation. Trying to take control over your own behavior by literal purposeful pairing is rare. For one thing, it is crude; for another, it is equivalent to treating yourself like a prelinguistic animal. However, in some instances, such literal purposeful pairing is appropriate. In an effort to conquer their addiction, for instance, some alcoholics resort to pairing highly aversive stimuli (e.g., drugs that produce severe nausea) with alcohol intake. The most effective way to take control over your own behavior, however, is to take control over your own practical reasoning. Others have plugged into your reasoning processes by pairing, through symbolic interaction,11 an imagined action with an imagined reinforcement, either positive or aversive. That reinforcement, or value, is what we call a reason. You should not do x because others will think ill of you, you should do y because it will help you flourish, and so on. The word because signals a reason. In theoretical reasoning, the word because signals a reference to a matter of fact; e.g., the bridge collapsed because the cable snapped. However, in practical reasoningthat is, reasoning about how you should and should not actthe word because ultimately signals a reference to a value. The reference may not be direct, but it will eventually get there. Thus, although the because in the claim you should contact your grandmother because she is dying directly refers to a matter of factnamely, that your grandmother is dyingit indirectly refers to the value that one should always try to make contact with people (or relatives) who are dying.12 Practical reasoning must eventually end with a reference to a value, because the only thing that ultimately moves behavior is value. Autonomy thus requires that you learn how to reason with yourself about practical issuesi.e., issues of value. However, to say that autonomy requires that you learn how to reason with yourself about practical issues is not to say that you must simply learn how to supply yourself with reasons. If autonomy is the goal, then there is a certain kind of reasoning that must be mastered, and that is the kind of reasoning that neutralizes outside influence, or bias. After all, to be autonomous means that you can make decisions by yourself, that your decisions are not primarily the result of outside pressures. Reasoning that is not influenced by outside pressure is impartial reasoning. Autonomy requires, in other words, that you learn to reason impartially. Autonomy requires that you follow reasons (as opposed to wishful thinking or preconceived ideas) where they lead. Autonomy requires that you become a reasonable individual.

Since it is value that moves behavior, you need to get control over your values in order to control your actions. With symbolic interaction, values can be moved in the imagination through reasoning.

The only way to get control over your values is to get control over your practical reasoning.

Autonomyi.e., freeing yourself from social controlrequires that you learn how to reason with yourself about practical issues in an impartial way.

Freedom through the Impartial Examination of Values


Before investigating how one goes about ensuring that ones thinking is impartial, let us summarize what we have covered thus far. It has been argued that self-conscious language users gain control over their actions by generating value-laden images of their future selves that compete with the value-laden stimuli of the external environment.

SectIon 1 the PoSSIbIlIty oF Freedom

19

Since the values that are enmeshed in those images may be values that have been introjected from others, the only way to strive toward becoming autonomous is to think through the values that guide your behavior in an unbiased way. It is only by neutralizing bias that you can be sure that the values that guide your behavior are your own. thinking through value issues impartially can thus be considered (at least according to this paradigm) the principle of freedom. It is essential that you clearly understand the dynamics of freedom, because you are being asked to adopt the habit of impartial reasoning. Reasoning impartially is hard work and often anxiety provoking, particularly in a world in which so many seek only your agreement rather than your council. In addition, impartial reasoning will often

FREEDOM REQUIRES COURAGE

20

Part I theory

result in judgments that may appear contrary to your own short-term best interests. That is why you must always be subliminally aware that when you engage in impartial practical reasoning, you are thinking your way to freedom and that the quality of your thought will reflect not just on who you are but that you are. It is only by becoming autonomous that you, as a unique reasoning individual, are fully born.

the eVolUtIon oF behaVIor


Correspondence with the natural world. Low-level correspondence with the social environment through concrete value manipulation. Higher-level correspondence with the social environment through practical reasoning. Correspondence with an ideal objective community13i.e., freedom from both physical and social imperatives through impartial practical reasoning.

SELF-CONSCIOUSNESS IS A DOUBLE-EDGED SWORD

(Suggested reading: Daniel Quinn, Ishmael [New York: Bantam, 1995].)

21

section 2
Impartial Thinking
SOCIAL DETERMINISM CAN BE INVISIBLE

24

ParT I Theory

Impartiality is necessary in order to ensure that the answers that you give to the value questions with which you are confronted are your answers and not answers that have been borrowedsubliminally or otherwisefrom your parents, peers, reference groups, or culture. Impartiality, in other words, requires that you learn how to eliminate your own biases. Thus, the questions that now require our attention are these: 1. How can we free our decision making from the undue influence of the external environment? 2. How can we know if we have brought that about? Since all thinking is, as it were, invisible, how can we ever tell whether any of our judgments are really our own? Immanuel Kant argued that one cannot know simply by analyzing the content of a judgment whether that judgment is ones owni.e., whether it is free from the determining influence of external factors. Let us take the example of Joe forgoing the large and luscious piece of chocolate cake. Kant would say of Joeor anyone else, for that matterthat we cannot know whether his decision to forgo the cake was or was not autonomous simply by examining what he decided to do. Kant argued that the only way to ensure that ones judgments are in fact ones own is to neutralize them through a particular reasoning process.14 This thinking insurance policy is necessary because the influence of external forces to which we are all exposed is so invisible. We are all constantly bombarded, silently or otherwise, with incentives to adjust our beliefs and opinions to conform to prevailing norms, the views of influential individuals, and pervasive media scripts. Because the power of these influences is so inconspicuous, we can never know for sure simply by inspecting the content of a belief or opinion whether it is actually our own or whether, in fact, it gained its original foothold or maintained life because of external factors. The only way to guarantee ownership of a judgment is to subject it to a process that neutralizes the potential force of external influence. Bias is the term we use to describe a preexisting way of viewing the world. Ones biases are not necessarily a product of external influence; one may have already spent a good deal of time and energy impartially thinking through an issue. However, one cannot rule out the possibility that external influence has nonetheless seeped in. The only way to know for sure that ones judgments are in fact ones owni.e., that they are the product of ones own thinking and not a result of silent external influence is to think through every issue and situation anew in a way that is as impartial, or objective, as possible. The resulting judgment may well be the same as the one embodied by ones original bias, but that is irrelevant. What is important is the process. It is only by embarking on a bias-neutralizing process that we have grounds for making the claim that our judgments are a product of our own thinking. It is only by embarking on a bias-neutralizing process that we become worthy of the dignity that Kant believed was the ultimate payoff of autonomy, or being ones own person. This was the brilliance of Kants insight. The fact that a judgment comes out of ones own mouth is not sufficient for claiming that that judgment is ones own. Although biases are in a sense internal, they may nonetheless be a product of external influences. As Kant pointed out, it is the form, not the content, of the judgment that is important. It is the process, not the product, that is crucial. An analogy may help underscore the importance of the process. Let us suppose that you are in charge of a medical procedure that requires the use of sterile instruments. Let us suppose that the instrument packs that are available have already been sterilized, but their sterilization limit dates have run out. You speculate that the enclosed instruments may well be still sterile. However, the only way to ensure that they are, in fact, still sterile is to subject them once again to a sterilization process. It is imperative, of course, that the process you use to sterilize the instrument packs does not subject them to new contamination. As with the instrument packs in a medical procedure, the only way to ensure that your judgments are sterilei.e., not contaminated by outside influenceis to subject

Your biases may not be your own.

Avoid contaminated judgments! Sterilize them!

SecTIon 2 ImParTIal ThInkIng

25

them to a sterilization, or bias-neutralizing, process. However, as with the instrument packs, it is imperative that you do not reintroduce contamination by thinking through the issue in a biased way. Merely thinking through an issue, in other words, is not sufficient to ensure ownership. The thinking process itself must be sterile, or impartial.

Bias Neutralization Is an Inter(Not Intra-) Subjective Process


Kant argued that the route to impartiality, and thus autonomy, was through rationality. Obviously, if one were able to be perfectly rational (e.g., the character Data from Star Trek), one could be sure that ones judgments were impartial or objective and therefore, from Kants perspective, moral. Kant offered the categorical imperative (act so that the maxim of your action can become a universal law) as a negative test of rationality. Since universality is a mark of rationality (e.g., two plus two equals four in all possible worlds), if ones maxims for action are not universalizable, then they are not contenders for objectivity and morality. Kants universalizability test is a kind of reverse roles or golden rule procedure. Using this procedure, a creditor, for example, can quickly see that his view that a debtor should be hung, drawn, and quartered could not be a candidate for rationality, or impartiality, as he himself (presumably) would not wish to be hung, drawn, and quartered if he were a debtor. Kants emphasis on the importance of rationality has lent powerful support to the glorification of the solitary thinker. Kants model suggests that good thinking, in and of itself, is the road to human excellence. After all, if autonomy resides in ones capacity to be rational, then apparently all one needs to do to become autonomousi.e., the best to which we humans can aspireis to rise up from ones lower, biased, particular point of view to the more ethereal, objective, universal point of view. All one need do to become autonomous is to travel upward within ones own mind. The difficulty with the process advocated by Kant is that the only check on the potential bias of ones own thinking is ones own biased self. I have reasoned it through, one may say, and I can assure you that I am not the least bit biased. It is a danger similar to the one inherent in postulating the possibility of a private language. As Ludwig Wittgenstein pointed out, this is like checking the accuracy of a newspaper by going out and buying another copy of the same edition. It also leads to the travesty of one apparently legitimately universalizing ones own biases. An example is Richard Hares fanatic Nazi,15 who argues that the extermination of the Jews is moral because he can universalize his maxim; i. e., he can will that, even if he were a Jew, he would still wish that all Jews of the world be exterminated. If we cannot go inward to find impartiality, what is the alternative? The goal is to view a given situation or issue as objectively as possible, to see it for what it really is rather than as a function of ones preconceived ideas. The optimum viewpoint, as Kant correctly pointed out, would be a universal, or Gods-eye, view. However, since striving for a universal view internally is problematic, surely the next best option is to go in the other directioni.e., to try to see the situation from as many actual points of view as possible, with the regulative ideal being an infinite number of viewpoints. Impartiality, in other words, is neither a state of being nor something that is best achieved through the play of ones own imagination or rationality. As American pragmatists Charles Peirce16 and John Dewey17 (using the community of scientific inquiry as a model) argued some time agoan argument that finds contemporary echo in Jrgen Habermass18 notion of communicative rationalityif impartiality is the goal, then one must actually communicate with others and genuinely try to perceive the issue from as many actual perspectives as possible. You cannot claim to be impartial or objective, in other words, unless you have

Impartiality requires that one strive toward universality by actually communicating with others and by genuinely trying to perceive issues from as many actual perspectives as is possible.

26

ParT I Theory

TO BE A CHOOSER REQUIRES GENUINE DIALOGUE WITH THOSE WHO THINK DIFFERENTLY

Autonomy requires access to opposing points of view.

made a genuine attempt to evaluate the legitimacy of opposing positions objectively.19 Impartial reasoning, in other words, is inevitably interactive.20 The difference between advocating an intersubjective process (a process that occurs between subjects) as a means of achieving impartiality and advocating a Kantian intrasubjective universalization process (a process that occurs within one subject) is highlighted by returning to the previous example. According to the intersubjective view of impartiality, the aforementioned Nazi cannot claim to be impartial simply because he can universalize his maximi.e., simply because he would wish to be exterminated if he were Jewish. He cannot claim to be impartial, because he has made no

SecTIon 2 ImParTIal ThInkIng

27

attempt to listen to and genuinely understand opposing viewpoints from the perspectives of those who hold them let alone objectively judge their merits relative to his own. Thus, listening to others, interestingly, turns out to be a necessary condition for autonomy. However, not just any kind of listening will do. You must keep in mind that the point of this endeavor is to view issues from as many perspectives as possible in order to estimate their relative adequacy (what is later referred to as their global sufficiency). This, in turn, requires that you understand clearly not only the viewpoints of others but also the reasoning that supports the positions. Ferreting out the reasoning behind the viewpoints of others can be a delicate and often arduous task. Thus, genuine

The measure of morality is global sufficiency rather than universalizability. Genuine communication requires that one learn to become an articulate listener.

YOU HAVE TO ENGAGE OTHERS IN ORDER TO BE FREE OF THEM

28

ParT I Theory

You need to learn to genuinely communicate with others in order to become yourself. Autonomy requires that one welcome opposing viewpoints. Impartiality requires that one judge as many actual points of view as possible.

listening, paradoxically, almost always requires dialogue. You need to speaki.e., consistently question and clarifyin order to listen well. You need to learn to become an articulate listener.21 You need to learn to genuinely communicate with others in order to become yourself. In summary, then, although pure rationality, or objectivity, or aspiring toward membership in Kants Kingdom of Ends is the regulative ideal, the path toward that ideal is concrete. Because the determining influence of others is most prevalent within our own minds, the only way to ensure that we have neutralized the internal determining influence of others is by forcing ourselves to test those perspectives against the viewpoints of actual others. Such interactive reasoning will preclude the possibility of a Pyrrhic victory, or what in critical thinking circles is referred to as an illegitimate strawperson maneuveri.e., the underestimation of the strength and wisdom of perspectives that are radically different from our own. It is only by forcing ourselves to test our judgments against the strongest possible actual oppositionor, in the words of Charles Peirce and John Dewey, it is only by continuously submitting our knowledge claims to the public test of a community of inquirersthat we can strive toward impartiality, and ultimately our own freedom.

Judging Quality: Estimating Truth through Falsification


Once one has assembled an array of viewpoints, how does one judge which ones are better than others?22 The term truth is common in science, but it is not an accurate turn of phrase. A more accurate way of speaking about claims that are held to be true would be to refer to them as claims that have not been proven to be false. As Karl Popper has pointed out,23 science does not progress on the backs of once and for all truths but rather on the backs of falsifications. A good scientist does not attempt to prove her or his claim to be true. Indeed, logically s/he cannot. This is so for two reasons. For one, the claim under investigation always refers to classes of thingsi.e., that something is true of all members of a class. A statement about all of a class can never be verified, because one can never access literally all of a class. However, an all statement can be falsified by finding one counterexample. The second reason that verification is not logically possible is that the kind of hypothetical reasoning that would be required for verification is fallacious. If theory x is true, then y should happen. Y happens. Therefore, theory x is true. In logic (as will become evident in Part II), this is referred to as the fallacy of affirming the consequent or affirming the necessary condition. On the other hand, the kind of hypothetical reasoning that is required for falsification is perfectly valid. Falsification starts with the null hypothesis: If theory x is not true, then y should not happen. Then, if y does happen, that shows that it is not the case that theory x is not true. This is a valid inference that, in logic, is referred to as modus tollens, or denying the necessary condition.24 A scientist, in other words, cannot prove his or her theories to be true; however, s/he can prove them to be false. Thus, what a good scientist does when faced with an intriguing hypothesis is to examine as much evidence as possible, in a manner that is as precise and as objective as possible, and only after the scientist has failed to prove the theory false (i.e., that it is not the case that theory x is not true) is s/he justified in proclaiming its truth, but even then only conditionally (i.e., on the condition that the theory remain open to reevaluation should new relevant evidence or reasoning emerge). Few would find the claim that a good scientist always remains open to new relevant reasoning and evidence with regard to truth claims contentious. However, if we reexamine our assumptions about truth, it is at least odd that we do not find such an assertion contentious. After all, Truth (with a capital T) carries with it the notion of infallibility. A claim that is later proven to be false was not once True. It was never

After getting an accurate picture of as many points of view as possible, one should test their adequacy through a falsification process.

SecTIon 2 ImParTIal ThInkIng

29

True; it was merely erroneously thought to be True, although in fact it was false all along. It is built into our concept of Truth that if a claim is True, it must be True for all time. Given that this is the case, does a scientist not contradict him- or herself by making the claim that a theory or claim is True while s/he nonetheless remains open to the possibility that it may one day be proven false? Surely, if someone believes that a theory or claim is open to further scrutiny, that person does not believe that theory or claim has been proven to be True for all time. However, if it is not True for all time, then it is not True at all. Something cannot be True now and false later. In actual fact, few scientists would claim to have discovered an eternal Truth i.e., a Truth with a capital T. When scientists use the word true, they are not referring to a product that is independent of the process from which it has emerged. When they use the word true, they are implicitly referring to a process: What is true is whatever survives the process. And since the process is not one of verification but one of falsification, when a scientist asserts that a theory or claim is true, what s/he means is that rigorous, objective, multifaceted, and public attempts to prove the claim or theory false have been futile. That is, strictly speaking, what a scientist means is not that a claim or theory is True but that, having been subjected to a rigorous public process of falsification, the claim or theory has not been proven to be false. The phrase having been subjected to a rigorous public process of falsification, has not been proven false is cumbersome. And since theoretically, the opposite of being false is being true, and since the claim in question is, as far as we know, not false, labeling it true with a lowercase t seems a fairly innocent move.

In saying that x is true, one should mean, having been subjected to a rigorous, objective, multifaceted, and public falsification process, x has not been found to be false or unacceptable.

Establishing Truth through a Falsification Process Is Not Possible, but Estimating truth Is
Calling a claim true when one means having been subjected to a rigorous public process of falsification, it has not been proven false may appear innocuous to those close to the process; it is, however, dangerous. It leads many to the erroneous assumption that truth and falsity, in the absolute sense, divide the world in two; that truth and falsity are mirror concepts; that if a claim is not one, it must be the other. However, truth and falsity are not mirror concepts. Although we must assume that, from a

Saying that it is not the case that x is not true is not equivalent to saying that x is true. In this case, not not x does not equal x. This is so because a falsification process, even theoretically, can never be assumed to be complete.

A GODS-EYE VIEW (a synthetic a priori assumption) False MOrTAl VIEW False False claims true Synthetic truths: These are claims that we have grounds for believing are True but that may nonetheless turn out to be false. The more rigorous, objective, and public the testing, the greater our justification in placing these claims to the right of this columni.e., closer to the Truth column. This category also includes claims that we believe to be true but that have not yet been subjected to rigorous testing. Such claims are more appropriately placed to the left of this column i.e., farther from the Truth column. True Analytic Truths: e.g., definitional Truths (bachelors are unmarried men) and mathematical Truths. These are Truths with a capital T. True

30

ParT I Theory

Gods-eye view, truth and falsity are mirror conceptsi.e., that either a claim is true or it is false in the absolute sensefrom a practical point of view, since we mortals can only falsify and not verify; and since, in principle, falsification can lead us to Truth in the absolute sense only in those situations in which we have falsified all possibilities but one; and since we mortals can never assume that we have in fact falsified all conceivable and actual possibilities but one,25 we mortals can never assume that we have reached the Truth in the absolute sense.26 However, if we mortals can never know that we have eliminated all possibilities but one, if we mortals can never know whether we have accessed Truth with a capital T, are we not back again to the claim that the whole notion of truth is vacuous?

TRUTH THROUGH FALSIFICATION


We move toward truth by throwing out what is false. An argument stays on the table unless you can falsify it. Note: You do not get to dismiss a position simply because you do not like it.

SecTIon 2 ImParTIal ThInkIng

31

The moral of the story is that although a rigorous public falsification process does not prove that a claim is True in the absolute sense, it increases the probability that the claim is True, and it is this increase in probability that we refer to as truewith a lowercase t. And as long as we know what we mean, or should mean, by truei.e., that a claim or theory has withstood a rigorous public test of falsification, e.g., that the claim does not contradict the facts (correspondence theory of truth), or that nothing contradicts the theorys predictions (pragmatic theory of truth), or that the theory does not contradict other apparently true theories (the coherence theory of truth)27 asserting that a claim or theory is true probably facilitates communication, particularly for those claims that have endured a long test of time. However, we must keep in mind that this notion of true is not the opposite of false. This is truth with a lowercase t, one for which the odds of its being True increase proportionately with the rigor of the falsification process but one for which the odds are never absolute.28

The probability that x is True in the absolute sense is proportional to the rigor of the public falsification process to which it has been subjected.

Truth Seeking in Ethics


What is true in theoretical reasoning is true in practical reasoning: What is true in science is also true in philosophy. When we say that we can make progress toward truth in ethics, what we are really saying is that since we can subject claims to a rigorous, objective, multifaceted, and public falsification process, we can increase the probability of these claims being True in the absolute sense. Philosophers have spent a great deal of energy trying to suggest ways to maximize objectivity in philosophical ethical inquiry. Immanuel Kant argued for universalizability,29 John Stuart Mill argued that one should attempt to identify what would produce the greatest good for the greatest number,30 John Rawls argued that one should imagine oneself behind a veil of ignorance,31 Hare argued for universal prescriptivism.32 As is evident even from this short list, controversy abounds in philosophy. However, this does not open the way to relativity. Simply because we cannot know for sure whether a or b is the best course of action in a given situation, this does not mean that we cannot know for sure that g, h, and i are unacceptable. In science, although we may not know for sure whether a radical mastectomy is better or worse than a lumpectomy in treating breast cancer, we can for sure rule out singing Gone with the Wind at noon every second week or eating peanuts for breakfast. Similarly, if you are the captain of a sinking lifeboat, it may be unclear whether the best ethical course of action is to throw some passengers overboard to save the others and, if so, how to decide who hits the water; however, we can for sure rule out throwing off more passengers than necessary, allowing bribes to determine who remains, and raping the women and children before you toss them over. For those who remain uncomfortable with predicating truth of an ethical judgment, the word objective or impartial or globally sufficient can be used in its stead; however, the meaning remains the same: that an individual has tried to eliminate personal and societal bias by attempting to view the situation from a multidimensional, truly interactive perspective in order to test, through a falsification process, whether, from those perspectives, the purported action still appears fair, just, or moral. There is no epistemological space between the process and the product.

The morality of an act is a function of the global sufficiency of the reasoning that backs it.

Practical Reasoning Is Inevitably a Two-Step Falsification Process


Although truth seeking in ethics is similar to truth seeking in science in the sense that both are grounded in falsification, practical reasoning is often more complicated than theoretical reasoning, since, in deciding a course of action, discarding all contenders is never an option. Doing nothing, after all, is just as much doing something as any other alternative. Thus, not only must each option be evaluated in and of itself; surviving options must be evaluated against one another. In practical reasoning, in other words, judging

SEEKING THE IMpOSSIBLE QUEST

32

SecTIon 2 ImParTIal ThInkIng

33

TRUTH SEEKING IS ALWAYS A TWO-STEp pROCESS

quality is inevitably a two-step process. One begins with a process of elimination i.e., throwing out any given option, suggestion, or judgment if it lacks local sufficiency, meaning that it has been shown to be inadequate because it is vulnerable to a powerful counterexample that shows that it does not fit the facts (correspondence theory of truth), or that it contradicts an agents other beliefs (coherence theory of truth), or that it contradicts concrete predictions (pragmatic theory of truth). The remaining contenders are then compared in order to estimate the least weak optioni.e., the option that is least weakened by a possible counterexample. This second step, which is referred to as global sufficiency,33 is particularly important, since, through it, an otherwise good option may turn out to be poor by comparison or a relatively weak option may turn out to be the best alternative available under the circumstances.

Impartiality requires that one test for (1) local sufficiency (falsification with the view to elimination) and (2) global sufficiency (falsification with the view to weaken by comparison).

34

ParT I Theory

NOT EVERYONES OpINIONS ARE EQUALLY GOOD

Talking to the Relativist


Engaging in intersubjective interactive inquiry will greatly facilitate the possibility that participants will capture an impartial or objective view of any issue. Nonetheless, unhappily, many remain reluctant to engage in such cooperative inquiry because they fail to see the point. That is, many remain genuinely perplexed about how it is possible to judge one persons value judgments as better or worse than anyone elses, whereas others dismiss the possibility of tracking truth in ethics altogether. Such skepticism leads

SecTIon 2 ImParTIal ThInkIng

35

WHAT IS OBJECTIVE IS FAULTY REASONING, NOT TRUTH

Autonomy requires that you continuously try to falsify your own opinions.

many to believe that seriously pondering the reasons for or against a given action is not worth the effort. If genuine dialogue is to continue in such situations, good thinkers need to remember, when confronted with skeptics, to turn the issue about truth on its head. You need to remember to concede that you are not claiming that you can know what is True when you see or hear it. What you are claiming is that you can know what is false or unacceptable by analyzing the adequacy of the reasons that support it (local sufficiency) and that you can judge whether one course of action is more or less problematic than another (global sufficiency). Remind the relativist that absolute Truth is

36

ParT I Theory

TAlKING TO THE rElATIVIST Although I cannot know for sure what the right course of action should be, I can know for sure what is unacceptable (local sufficiency), and I can also estimate whether one course of action is more or less problematic than another (global sufficiency).

like absolute cleanliness: For all practical purposes, there is no such thing. However, from that admission, it does not follow that we therefore should not engage in truth processing any more than it means that we should not wash. Nor does it follow that we cannot detect that one persons reasoning is more faulty than anothers any more than it follows that we cannot judge when someones clothes are considerably dirtier than someone elses.34 What we are trying to do when we attempt to make progress toward truth in ethics is to judge which of the competing options is the best under the circumstances. We are not trying to do that directly, by focusing on which option is better; rather, we are approaching the goal indirectly, by focusing on which are the least worthy candidates. Like a jet engine, we are attempting to move forward by putting behind us that which is faulty.35 (See the comic entitled Truth through Falsification on page 30.) We may end up with more than one plausible way to handle a situation. That may be because both options are, in fact, equally sound, or because we have been unable to detect the inherent weaknesses in one or both options, or because there is a better way to handle the situation than we have hitherto imagined. However, if we have tried our best to reason as impartially as possible with respect to as many options as possible, we should be comfortable with the fact that this is the best that a finite mind can do.

The Message
The values that you adhere to when you decide how to act in the short, medium, and long term will determine the kind of human being you become, the kind of life you live, and the kind of community that you help to create. To the degreeand only to the degreethat you seriously and impartially reflect on the values that guide your actions in the short, medium, and long term, you can describe yourself as the master of your own fate, as the creator of yourself, and only to that degree can you describe yourself as a wise individual and one worthy of respect. The concluding message, then, is this: You must remember that relativity is a bogus theory; that it is not the case that anyones opinion is as good as anyone elses. You must remember that if you wish your judgments to be respected as something more than sheer personal bias, whether in science, in ethics, or in the practical matters of everyday life, you must be prepared to follow a process that maximizes the possibility of objectivity. You must remember that if your claims are not subjected to a process of this sort, you have no grounds whatsoever for asserting that those claims should be viewed as the best of all competing positions, you have no grounds whatsoever for making the claim that they are anything more than personal whim, and you have no claim whatsoever to demand respect for those views. Whether we like it or not, and as Existentialists have so famously and forcefully reminded us, all self-conscious language users have the capacity for choice. Existentialists have argued that what we choose defines who we are, and for that reason they paint a rather pessimistic picture of this burden. However, choosing need not be perceived as a burden. Although it is true that we can never know that the decision we make in any given situation is the right decision, nonetheless we can know whether we have followed the only method to truth that is open to a finite mind. What we choose, in other words, is not as important as how we choose. And so the message continues: If your beliefs and opinions survive continuing impartial inquiry, they are worthy of being held and espoused, and you, in turn, are worthy of the dignity of being deemed an autonomous being. This is confidence both in yourself as a thinker and in the content of your thoughts that is deserved, not artificially endowed; this is self-respect that has been self-anchored and self-earned. This is the best that you can do. You should be at peace with that.

You need to think your way to freedom.

TRUTH SEEKING, NOT TRUTH, SHALL SET YOU FREE

37

THE STOp AND GO SIGNS TOWARD AUTONOMY

38

39

40

ParT I Theory

Postscript: Freedom Needs Determinism


Those of you who are interested in the theoretical conundrum of the apparent incompatibility of freedom and determinism should note that the paradigm presented here suggests that these two are not the antagonistic, mutually exclusive positions they are often portrayed to be. Indeed, what is being suggested here is that the possibility of any kind of freedom depends, both literally and conceptually, on the actuality of many layers of determinism. The fact that physical objects move according to physical/chemical laws is the foundational position of determinism, both concretely and conceptually. However, animate beings, precisely because they are also determined by the stimulus environment in which they move, areto a greater or lesser extentfreed from the universal determining power of physical/chemical laws (e.g., they can move uphill). On still another level, linguistically interacting self-conscious entities, precisely because they are also determined by the values they introject from linguistically interacting others, areto a greater or lesser extentfreed from the determining power of behavioral laws (e.g., they can learn to share what they are primed to keep). Finally, linguistically interacting self-conscious agents who strive for impartiality by submitting to the rules of practical reasoning outlined here free themselves from the determining power of social influence and, in so doing, make autonomy and its existential counterpart, individuality, possible.
InVITaTIon
The paradigm presented here echoes the foundational claim of American pragmatists Charles Peirce and John Dewey that assertions can be considered warranted only when they are subjected to public scrutiny by intelligent, courageous, open-minded inquirers who are conversant in the art of inquiry. For that reason, and in light of the fact that many of the issues presented here remain under considerable debate within the philosophical arena, once you have mastered the two-step falsification process, you, along with your classmates, are invitedindeed, encouragedto circle back and focus the glare of that process on any part of the theory contained herein. (See the comic entitled The Stop and Go Signs toward Autonomy on pages 3839.)

SecTIon 2 ImParTIal ThInkIng

41

rEVIEW QuESTIONS
1. Is there something essential that humans should learn? What and why? 2. What is the difference between negative and positive freedom, and how do they relate to autonomy? 3. How do children acquire self-control? 4. In what sense must self-conscious language users first go deeper into the quagmire of determinism before rising out of it? 5. The initial acquisition of self-control in humans does not appear to be substantially different from the behavior of a well-trained dog, but there is a substantial difference. What is it? 6. An individual who gains self-control becomes the author of her or his actions. True or false? Explain. 7. What is the difference between local sufficiency and global sufficiency? 8. Explain the evolution of behavior. 9. Reasoning with oneself about practical issues will lead to autonomy. True or false? Explain. 10. Once you have mastered impartial thinking, your autonomy will thereafter be secure. True or false? Explain. 11. The fact that a judgment comes out of your mouth is not sufficient to say that it is your own. True or false? Explain. 12. Bias neutralization is an intrasubjective process. True or false? Explain. 13. Autonomy requires that one continuously try to verify ones beliefs and opinions. True or false? Explain. 14. Listening to opposing points of view will ensure that you are able to judge your own opinions impartially. True or false? Explain. 15. One should not act on a judgment unless one is certain of its truth. True or false? Explain. 16. Why is your opposition your best friend? 17. How can you know for sure that your judgment is impartial? 18. How do you respond to the relativist who argues that when it comes to values or ethics, one persons judgment is as good as anyone elses? 19. How can you judge the adequacy of another persons claim? 20. According to the paradigm presented here, what is the principle of freedom?

42

ParT I Theory

ANSWErS TO rEVIEW QuESTIONS


1. Is there something essential that humans should learn? What and why? Yes, humans should learn to maximize their own freedom, because the capacity for freedom is unique to self-conscious language users. Thus, if humans do not learn how to maximize their own freedom, they throw away that which makes them unique. 2. What is the difference between negative and positive freedom, and how do they relate to autonomy? Negative freedom is the capacity to free ones behavior from the determining influence of the external environment by generating value-laden images of how one should act in any given situation. This self-control is not autonomy because, initially, the projected values are introjected or borrowed from others. Positive freedom, by contrast, emerges only when one gains the capacity to free ones judgments, beliefs, and opinions from the determining influence of the external environment by subjecting ones opinions to a bias-neutralization process. Autonomy requires both negative and positive freedom. 3. How do children acquire self-control? Children acquire self-control by introjecting values from others and then projecting value-laden images of how they think they should act in any given situation. However, this is not autonomy, as the introjected values that children project have not been subjected to a bias-neutralizing process. Therefore, these values cannot be described as the childrens own. 4. In what sense must self-conscious language users first go deeper into the quagmire of determinism before rising out of it? Self-conscious entities first go deeper into the quagmire of determinism because, unlike animals in the wild (which are controlled by the values that adhere to the natural environment) and unlike domestic pets (which are controlled by the natural environment and by contrived pairing), self-conscious language users are controlled by the natural environment and by contrived pairing and by introjected values that they themselves project into the environment. Thus, although linguistic interaction is collectively advantageous because it facilitates finely tuned group interaction, from the point of view of individual freedom, it propels humans in the wrong direction. 5. The initial acquisition of self-control in humans does not appear to be substantially different from the behavior of a well-trained dog, but there is a substantial difference. What is it? On the surface, there is no obvious difference between a child refraining from being selfish with her toys because it conflicts with her mothers worldview and a dog ceasing to chew on shoes because the behavior was paired with a wallop. However, the difference, which is dramatic, resides in the fact that because of language, the child, unlike the dog, can learn in her imagination. No actual behavior or actual reinforcement need take place. Thus, a child, for example, can learn to share her toys because her mothers words paint an imaginative picture of sharing being highly valued. Unlike a dog, the child does not have to be first induced to produce a sharing response that is only subsequently reinforced. What a child can learn through linguistic interaction, therefore, is infinitely more subtle and complex than what a dog can learn through direct experience. 6. An individual who gains self-control becomes the author of her or his actions. True or false? Explain. False. Humans initially gain self-control by introjecting values from others and then imagining value-laden images of how they should act in any given situation. If

SecTIon 2 ImParTIal ThInkIng

43

the value of these images outweighs the values enmeshed in the external environment, the agent will have gained self-control. However, that agent is nonetheless not the author of his or her actions, since s/he is not the author of her or his values. 7. What is the difference between local sufficiency and global sufficiency? You can test the local sufficiency of any position by subjecting the reasons that support it to a falsification process. You can test the global sufficiency of any position by assessing its merits relative to the merits of all other competing alternatives. In other words, a position wins only if it is the least weak of all contenders after all contenders have been tested for local sufficiency. 8. Explain the evolution of behavior. From a phylogenetic and ontogenetic point of view, behavior evolves through the following stages: (a) correspondence with the natural world, (b) low-level correspondence with the social environment through concrete value manipulation, (c) higher-level correspondence with the social environment through practical reasoning, and (d) correspondence with an ideal objective communityi.e., freedom from both physical and social imperativesthrough impartial, interactive, practical reasoning. 9. Reasoning with oneself about practical issues will lead to autonomy. True or false? Explain. False. This is an important question, as many people think that reasoning per se is sufficient to ensure autonomy and that therefore it is important merely to learn to reason efficiently. However, reasoning per se does not lead to autonomy. Only impartial reasoning (i.e., reasoning that eliminates bias) can lead to autonomy, and impartiality requires genuine interaction (i.e., it requires that one judiciously reflect on the relative merits of all relevant points of view). Hence, what is important is not merely that one learn to reason efficiently but that one learn to reason impartially by articulately listening to points of view that are radically different from ones own. 10. Once you have mastered impartial thinking, your autonomy will thereafter be secure. True or false? Explain. False. Autonomy is never absolutely secure, nor is autonomy a once-in-a-lifetime achievement. The self is a whirlwind of values that picks up value every time it touches groundi.e., every time an agent acts or is blind to the opportunity for action, adopts a belief or attitude, or gives in to an emotion. Autonomy is thus something that must be secured every day of your life in every judgment that you make. 11. The fact that a judgment comes out of your mouth is not sufficient to say that it is your own. True or false? Explain. True. Self-conscious agents introject values from significant others, reference groups, the media, and the general norms of the society in which they live. Your judgments therefore may be more a reflection of your environment than of who you are uniquely. The only way to ensure that what comes out of your mouth is truly a reflection of what you believe is to think through the issue in an unbiased way. 12. Bias neutralization is an intrasubjective process. True or false? Explain. False. You cannot be sure that you have eliminated all bias just by thinking through an issue on your own. The only way to ensure bias neutralization is to seek out those who disagree with you and then evaluate their opposing viewpoints in a judicious manner. In ethics, as in science, in order to move toward truth, one must be willing to engage in public inquiry.

44

ParT I Theory

13. Autonomy requires that one continuously try to verify ones beliefs and opinions. True or false? Explain. False. Verify means to show the truth of a claim. You can never show the Truth of a claim. All you can do is to show that a claim is false or unacceptable because it is supported by faulty reasoning. That is why autonomy requires that you continuously try to falsify your own opinions. 14. Listening to opposing points of view will ensure that you are able to judge your own opinions impartially. True or false? Explain. False. Merely listening to others is not sufficient to ensure that your own view is impartial. You must also be willing to (a) aspire to a deep understanding of what others believe by articulately listening to what they have to say and (b) fairly assess the adequacy of the reasoning that supports the viewpoints of others. It is important to note that an opposing viewpoint cannot be dismissed unless it is evident that the reasoning that supports it is faulty. 15. One should not act on a judgment unless one is certain of its truth. True or false? Explain. False. The only way to be absolutely certain that ones judgment is the right judgment in any given situation is (a) to imagine every possible course of action and (b) to accurately judge the sufficiency of the reasoning behind each option so that all possibilities but one are dismissed. Given your finite mind, neither a nor b is possible. The best that you can do is to test your judgment against as many opposing viewpoints as possible. Having done that, you should be comfortable in acting on that judgment, even though you can never be absolutely certain that this is the right course of action. 16. Why is your opposition your best friend? Your opposition is your best friend because only by testing your beliefs and opinions against the strongest opposition can you test whether your beliefs are worthy of being held by you. Only by testing your beliefs and opinions against the strongest opposition can you attempt to neutralize your beliefs of their bias and hence pursue your own autonomy. 17. How can you know for sure that your judgment is impartial? You can never know for sure that your judgment is impartial. However, you have grounds for believing your judgment is impartial if you have sincerely tried to view the issue from as many perspectives as possible and have attempted to judge the adequacy of each opposing viewpoint. 18. How do you respond to the relativist who argues that when it comes to values or ethics, one persons judgment is as good as anyone elses? This is the relativists central claim, and you must remember to turn it on its head. You can respond that you cannot know whether any judgment is true but that you can know whether any judgment is false, or unacceptable, by analyzing the adequacy of the reasons that support it (local sufficiency) and that you can judge whether one course of action is more or less problematic than another (global sufficiency). With regard to your own position, you can respond that you have thought through the reasons that support your position as judiciously as possible and that you have found no obvious flaw but that you would be happy to reevaluate your position if the relativist finds a flaw in your reasoning or can suggest a better alternative. (See the invitation on page 40.) 19. How can you judge the adequacy of another persons claim? No claim comes tagged as adequate or inadequate on its own. The only way to judge the adequacy of another persons claim is by judging the reasoning that supports it. Even when someone makes a claim that you believe to be false, or even

SecTIon 2 ImParTIal ThInkIng

45

ludicrous, you should never respond with immediate disagreement. You should first ask why the individual holds that particular belief. Only by analyzing the reasoning that supports a claim can you judge its adequacy. Remember that questioning is a more reliable sign of good thinking than pontificating about what one believes to be true. 20. According to the paradigm presented here, what is the principle of freedom? The principle of freedom dictates that freedom requires that one think through value issues in an impartial way.

Part II
PRACTICE
section 1
Learning the Intricacies of Practical Reasoning
which is necessary for

section 2

Thinking and Writing Your Way to Truth

48

PART II PRACTICE

It was argued in Part I that autonomy requires that one engage in impartial practical reasoning. For this to be true, obviously, the impartial reasoning must be logically adequate. Logically adequate impartial reasoning requires that one assemble as many potential candidates for truth as possible, that one test the adequacy of the reasoning behind each optionreferred to as establishing local sufficiencyand thereafter that one determine the least weak candidatereferred to as establishing global sufficiency. Section 1 of Part II focuses on local sufficiencyi.e., estimating the adequacy of individual arguments. Section 2 of Part II focuses on global sufficiencyi.e., estimating the adequacy of individual arguments relative to other contenders for truth. Clearly, there is a dynamic relationship between these two. For that reason, students are directed to move back and forth between Section 1 and Section 2. However, since Section 2 provides a template for writing any argumentative essay, Section 2 has been written as a self-sufficient unit (with only passing reference to Section 1) so that it can serve as a quick point of reference for future thinking and writing.

section 1
Learning the Intricacies of Practical Reasoning

50

PART II PRACTICE

SEEKING A GOOD YOU RATHER THAN GOOD TIMES

To be wise is to choose ones ends or values. To be intelligent is merely to choose the means to preestablished ends.

Intelligence can be described as the capacity to efficiently locate means that will lead to ends that are already predetermined. Wisdom, by contrast, can be defined as the capacity to choose a coherent set of ends, or values, that will lead to the creation of who it is you want to become. Treatises that focus on theoretical reasoning do so with the view to enhancing your intelligence; treatises such as this one that focus on practical (or value) reasoning do so with the view to enhancing your wisdom. Wisdom and autonomy can be viewed as flip sides of the same coin. The truth-seeking process that underpins wisdom demands that one acquire a number of specific reasoning skills. To begin with, it requires that one be awake to the value issues that saturate ones world. It also requires that one be able to vividly imagine alternative perspectives. It requires that one be able to accurately construct arguments

SECTIon 1 LEARnIng ThE InTRICACIES of PRACTICAL REASonIng

51

DOING NOTHING IS NOT AN OPTION: IT IS DOING SOMETHING

that support each alternative. It requires that one be able to analyze the potential flaws in argument structures that support each alternative. The kind of argument that underpins value reasoning is a deductive argument. Thus, since the goal of the present endeavor is to nurture autonomy (or wisdom), and since this in turn requires that one learn to think through value issues impartially, learning the basic mechanics of deductive argumentation is the major focus of what follows. Specifically, this section focuses on the following elements of logical reasoning in practice: 1. Learning to differentiate value from nonvalue (empirical) issues 2. Gaining the skill to formulate value issues in precise, manageable terms

You gain freedom through impartially thinking through value issues, which requires that you become adept at understanding the mechanics of deductive logic.

52

PART II PRACTICE

3. Becoming alert to the value issues that are woven into your own world 4. Becoming familiar with some common informal fallacies that can hinder your ability to judge the merits of any position 5. Understanding the basic format of deduction 6. Gaining skill in seeing the whole argument in which any reason is immersed by recognizing the hidden premise 7. Learning to evaluate reasons, or soundness (local sufficiency), by evaluating the vulnerability of premises to counterexamples 8. Learning to evaluate the local sufficiency of your own and your oppositions positions through parallel arguments 9. Learning to evaluate the global sufficiency of your own positions by writing mini-essays 10. Gaining practice in standardizing claims, thus ensuring that you know what counts as a legitimate counterexample in any instance 11. Learning to see hidden claims about sufficient and necessary conditions and to understand their significance 12. Learning to see the hidden premise in forced-choice situations 13. Learning to respond appropriately to incorrect counterexamples 14. Learning to differentiate between common valid and invalid deductive moves and learning how to respond appropriately 15. Reviewing Divas messages, which are woven throughout the text The autonomy-wisdom duet is not quickly or easily achieved; it is an ideal one must aspire and work toward every day of ones life. Even if you learn all that is to follow, success is not guaranteed. However, learning what is to follow will make the road substantially easier. In the long run, the degree to which you are successful in becoming your own person will be more a function of will than of skill, although skill is no small asset.

1. Knowing What to Look For: Reasons versus Evidence


Claims can be classified according to the method one would use to estimate their truth. Claims can be divided into two broad categories: those that are empirical and those that are nonempirical. The word empirical comes from the word experience and hence indicates the kind of support that empirical claims require. The claim that seat belts save lives is an empirical claim; it can be supported only by evidence gathered from experience. One has little justification for making such a claim unless, in reality, there is empirical evidence to support it. On the other hand, the claim that post-secondary education should be free is a nonempirical claim. Nothing in experience could directly support its truth. It receives its justification from reasons alone. This is not to say that no reference to facts is ever needed for the justification of a nonempirical claim. In the preceding example, one might refer to the positive experience of countries in which post-secondary education is free. Nonetheless, unlike empirical claims, the truth of this nonempirical claim is not solely a matter of fact. I can agree with your facts but still argue that for other reasons, postsecondary education should not be free. On the other hand, there are some nonempirical claims whose justification rests so heavily on facts that they appear to be neither strictly empirical nor strictly nonempirical. One should not smoke is an example. Although strictly speaking, this is a nonempirical claim, its justification is so tightly tied to the fact that smoking is harmful that if the facts turned out to be wrong, most people would retract the claim. Since the cardinal characteristic of an empirical claim is that its truth rests on matters of fact, it is in this sense that it resembles an empirical claim.

WISDOM VERSUS INTELLIGENCE


Wisdom is more will than skill.

53

54

PART II PRACTICE

WISDOM VERSUS INTELLIGENCE (continued)

Your response to an empirical claim should be What is your evidence? Your response to a nonempirical (value) claim should be What are your reasons?

It is important for you to know the difference between empirical and nonempirical claims because you need to know what kind of justification each requires. Empirical claims require empirical evidence. Thus, if someone makes an empirical claim, your immediate response should be What is your evidence? If someone says that murder is a common occurrence in baboon society, you should ask on what grounds that individual is making that claim. Do not be satisfied with the answer Studies have shown it to be the casean illegitimate appeal to authority (see section on Informal Fallacies). If the claim is highly contentious, you may want to withhold assent until you yourself have examined the study. By contrast, the justification of nonempirical claims rests on reasons rather than on empirical facts. Individuals who make nonempirical claims should always be prepared to lay bare the reasoning by which they make such claims. Nonempirical claims not backed by reasons are just so much noise pollution, just as empirical claims not backed by evidence are fantasy. Remember, all value claims are nonempirical, and only value claims can guide behavior.

FREEDOM IS ABOUT THE FUTURE

55

56

PART II PRACTICE

EXErCISE 1a
What kind of claims are these? Differentiating value (nonempirical) from empirical claims
In the following exercise, indicate whether the claims are value claims or empirical claims or a mixture. If the claim is a mixture, try to decide whether it is more of one or the other. Your response to empirical claims usually should be What is your evidence? The exceptions to this rule are empirical claims that are common knowledge. Most people would think you odd if you demanded evidence for the claim that water is made up of hydrogen and oxygen. Your response to value claims should be What are your reasons for saying that? Thus, in order to know what to look fori.e., whether you should be inquiring about evidence or reasonsyou must be able to distinguish empirical from value (nonempirical) claims. It is suggested that you copy the following exercise on a separate piece of paper. Then put an x in the appropriate column. The line below each has been left blank, so that you can make comments as necessary. Answers to the first five are in the back of the book.

SECTIon 1 LEARnIng ThE InTRICACIES of PRACTICAL REASonIng

57

Empirical: What Is Your Evidence? 1. exwithoutcommitmentiswrong. S 2. Photoradardoesntwork. 3. fyourfriendhasaseriousdrugaddiction,youshouldtell I hisorherparents. 4. Smokingcausescancer. 5. oddlerswhospendmostoftheirtimeindaycareendup T beingmoreindependentthantheirpeers. 6. idswhowatchalotofviolenceontelevisionendupbeing K moreviolentthantheirpeers. 7. facoupleplanstohavechildren,atleastoneofthem I shouldbepreparedtostayhomewiththeiroffspring. 8. hegovernmentshouldabolishitsaidtoforeigncountries T andspendmoremoneyonitsownpoor. 9. nimalsinzoosaredeprivedoftheirnaturalhabitats. A 10. tudentsinAsiaarebetteratmaththanstudentsinNorth S America. 11. hegovernmentshouldhavearesponsereadyincase T thereisapandemic. 12. apitalpunishmentshouldbebanned. C 13. xperimentingwithcocainewillleadtoaddiction. E 14. tiswrongtousecocaine,evenifitisnotobviously I destroyingoneslife. 15. eoplewhoarereligiousarelessself-confidentthanthose P whoarenotreligious.

Mixture

Nonempirical: What Are Your Reasons?

58

PART II PRACTICE

2. Pushing toward Precision


When people think, speak, and write, they often assume that if they think, speak, or write in complete sentences, they have articulated a complete thought. However, this is not necessarily true. If a thought is complete, it should be precise enough so that one could describe exactly what the world would look like if it were true. Thus, if somebody says that all drugs should be legal, we would know what s/he meant. However, if someone says that recreational drugs should be legal, we would not know what s/he would count as recreational without further clarification.

IMPRECISION IS INEFFICIENT IN ACTION GUIDING

SECTIon 1 LEARnIng ThE InTRICACIES of PRACTICAL REASonIng

59

If you want to be a good thinker, you must ensure that you yourself are not vague in your thinking, speaking, or writing and that you do not accept vagueness from others. If you let yourself get away with being vague, you are allowing yourself to skip along the surface of the issue. You are merely paying lip service to the problem at hand while ensuring that your thinking does not contribute to the solution. You are whining about a serious issue while leaving the hard work up to others. Life is too important for any of us to put up with impotent complaining. We need to either accept the imperfections of life or figure out ways in which it can be improved. Precise thinking is necessary for the latter.

EXErCISE 2a
Responding in a way that ensures precision
When someone speaks in vague terms, your job is to push toward precision. That is, it is important not only that you recognize vagueness but also that you know just the right question to ask in order to move toward maximum precision as efficiently as possible. In the following exercise, please rate the claims as either vague or precise enough. Then, in the line below, write the question that should be posed in order to move the discussion toward maximum precision. In your effort to push toward precision, be particularly aware of the following claims: There should be a law, the government should pay for (whatever), and educational courses should be required. All three can serve as a pretense for real thinking. They are often the road taken by the haughty, who do not have the integrity to be silent in the face of complex issues they themselves are unwilling or unable to genuinely tackle. If someone says that there should be a law, it is important to question how that individual envisions the implementation of that law. If someone suggests, for instance, that there should be a law against people swearing at other people, merely focusing on how such a law would be implemented would probably be sufficient to show that the suggestion is silly. The harm of having language police would far outweigh the harm of having to listen to the occasional obscenity. If someone suggests that the government should pay for (whatever), and the suggestion is not accompanied by a detailed account of where those funds should come from, the individual is under pie-in-the sky illusions. The government has no extra money! So there is no point in suggesting that it should pay for anything unless the suggestion is accompanied either by the admission that the individual is advocating a tax increase or by a specific plan for diverting funds from other uses. In the latter case, a fairly detailed knowledge of the finances in question is necessary. The third ruse, which lazy thinkers use to avoid depth, is to suggest that whatever the ill, we should throw education at it. In principle, of course, education is often a good idea. However, the hard part of this suggestion is to figure out what kind of education would produce the desired results, such as the elimination of drug addiction or juvenile delinquency. Without specifics, the suggestion is vacuous. It is suggested that you copy the following exercise on a separate piece of paper. Then put an x in the appropriate column. The line below each has been left blank so that you can write out how you would respond so as to push toward maximum precision. Answers to the first five are in the back of the book.

60

PART II PRACTICE

Vague 1. hildrenshouldtakemoreresponsibilityfordoingchoresathome. C Response: 2. llprospectiveparentsshouldberequiredtotakeparentingcourses. A Response: 3. Ourcityshouldhavemoreparks. Response: 4. hegovernmentshouldensurethatallpetsaretreatedhumanely. T Response: 5. urcityshouldbuildacomprehensivemasstransitsystem. O Response: 6. PeopleshouldwatchlessTV. Response: 7. Post-secondaryeducationshouldbefree. Response: 8. Weshouldmakepeoplerecycle. Response: 9. hepenaltyforyoungoffendersshouldbemuchmoresevere. T Response: 10. omenwhotakedrugswhilepregnantshouldbepunished. W Response: 11. Pornographyshouldbeillegal. Response: 12. Peopleshouldnotsupportprofessionalsports. Response: 13. Theaquariumshouldgetnewexhibits. Response: 14. incedriversbetweentheagesofsixteenandtwenty-onehavethehighest S accidentrate,thereshouldbemandatorydrivereducationcoursesinallhigh schools. Response: 15. Peopleshouldbelesscriticalofthemselves. Response:

Precise Enough

WITHOUT PRECISION, VALUE JUDGMENTS CANNOT ACT AS MAGNETS

61

62

PART II PRACTICE

WITHOUT PRECISION, VALUE JUDGMENTS CANNOT ACT AS MAGNETS (continued)

3. Taking a Look at Your Own Values (and Coming Up with Good Thesis Statements)
You need to be awake to your world.

You are constantly subjected to behavior-modifying stimuli that populate your world. To the extent that your behavior is governed by those stimuli, you cannot claim to be the author of your own actions. The only way you can gain control over your own behavior is by imaginatively creating your own bias-free behavior-modifying stimuli i.e., imagining how you would like to see yourself behaving in various situations. If those images are sufficiently specific, and if they are endowed with sufficient value, they may actually control your actions. To create a repertoire of specific images of your ideal behavior in the varying situations in which you will find yourself, you will have to think those situations through. Your autonomy, in other words, requires that you spend a considerable amount of time and energy now thinking about precisely how you should act in various specific situations in the future. Without such reflection, more often than not you will simply bow to the strongest external pressure. The impetus to engage in such autonomy-creating reflection stems in turn from the propensity to ask relevant and profound value questions about seemingly ordinary situations. Only by asking such questions can you piece together an image of the kind of person you want to become, the kinds of individuals with whom you should (or should not) interact, and the kind of society you would like to help create.

SECTIon 1 LEARnIng ThE InTRICACIES of PRACTICAL REASonIng

63

IF YOU DO NOT CARE, YOU ARE NOT IN CHARGE

CRITERIA FOR GOOD QUESTIONS If you ask yourself a practical question, the answer can have an impact on your behavior only if that question meets certain criteria. Thus, if self-legislation is your goal, you need to get into the habit of asking yourself good questions. In academia, answers to questions are called thesis statements. Thus, a good thesis statement can be described as the answer to a good question. The following are criteria for good thesis statements in response to practical questions. Ask yourself whether your thesis statement is About values (only value questions guide behavior) Clear and precise with respect to definition, detail, example, and quantification

64

PART II PRACTICE

Contentious (it is a waste of time to explore that which is unproblematic) relevant (if the answer to the question has no impact on your behavior, it cannot foster wisdom) tentative (you should be prepared from the outset to embrace its contrary) WHY PICK A THESIS STATEMENT THAT IS CONTENTIOUS? Many believe, particularly if there is something as important as grades at stake, that one should pick a thesis statement whose truth is relatively easy to prove. However, quite the reverse is the case. Apparently easy wins are no wins at all. If you pick a

THE MIND IS OPENED BY WHAT IS INTERESTING; WHAT IS INTERESTING IS CONTENTIOUS

SECTIon 1 LEARnIng ThE InTRICACIES of PRACTICAL REASonIng

65

EXAMPLES of ATTEMPTIng To PRoVE ThE oBVIoUS


problem:Thereisnoreasonableoppositionbywhichtotestthestrengthof yourclaim,andhencethewinisbydefault,ratherthanonthestrengthofyour reasoning.Theobviousnessoftheclaimisalsoinsultingtoyourreader.

Childabusersshouldbestopped. Minoritiesshouldnotbediscriminatedagainst. Menandwomenshouldgetequalpayforequalwork. Childrenshouldhaveadecenteducation. Animalsshouldnotbeunnecessarilyabused. rugcompaniesshouldensurethatproductsaresafebeforetheyareputon D themarket. Hospitalsshouldnotturnawaycriticallyillpeople. Kidsshouldntcarryguns. Rapistsshouldbeprosecuted. Peopleshouldnottrytosolvetheirfamilyproblemsbyusingviolence.

claim that has no believable opposition, that claim succeeds (if such a term is appropriate at all) by default, not on the strength of your reasoning. In principle, therefore, arguments in support of noncontentious claims cannot reflect the competence of the arguer, and hence they certainly cannot be the conveyor of grades. Such claims are also uninteresting, a waste of time, and insulting to your reader. Indeed, your choice of topic suggests that those for whom you write are so dim-witted that they need convincing of the otherwise obvious. This is a dangerous message to send to a reader who has power over your future well being! You cannot show that you are clever unless you tackle a topic that is contentious. CAN QUESTIONS ABOUT PRACTICAL ISSUES INEVITABLY BE SELF-DIRECTED? When you are thinking of how you would like the world to change, it is important that you think of changes that capture your own behavior. Focusing on how other people should act is often a waste of time, as it constitutes thinking that will do no real work. At its worst, it can be described as a kind of whining that serves to get you off the hook of doing anything about the issue. Thus, if you do not own a fur coat and have no real wish to do so, a vacuous thesis statement would be People should not wear fur coats. It is vacuous because it will have no modifying influence on your behavior and hence can do nothing to enhance your freedom. A far more fruitful claim would be If one believes that wearing fur coats is wrong, one should (or should not) say so when confronted by a friend (or relative or stranger) wearing a fur coat. The moral of the story is that if you ever start thinking that they should or should not do such and such, look inward and ask yourself, What should I do about it? Another important moral of the story is that questions with regard to what I should do can be vacuous and potentially hypocritical unless they are the sort that might be triggered by your own environment. Thus, for instance, it would be difficult to write honestly about what you should do in a situation in which your best friend is taking heroin or your spouse is a drunk if you have never been in these sorts of positions. As with they questions, it is all too easy to come up with pretty answers when you do not actually have to deal with the intricate and complex nuances of such situations. Hence, if you are going to try to imagine what you should do in potential environments, the closer they are to the possibility of reality, the better.

66

PART II PRACTICE

EXErCISE 3a
Categories for thesis statements
The goal remains that you get into the habit of seriously asking yourself good questions about your own life. However, for those who have heretofore been asleep at the wheel, this will be a huge challenge. For that reason, an array of value claims has been provided in an effort to prime the pump. review all of the following value claims. Circle at least one from each category that you find interestingone that you might like to explore further. NOTE: You can change the valence of many of the following claims by inserting or omitting not. However, note that this does not always work, as a change in valence sometimes renders the claim noncontentious.
1. Questions about What We Owe to Others

I am justified in not giving spare change to beggars. I think clear cutting is wrong, but I am justified in not engaging in civil disobedience in trying to prevent it (e.g., blocking a logging road), since I believe it would not be effective. It is not immoral to move to a country that cares less about its less fortunate than the one where I presently live (e.g., moving from a country that has a comprehensive social safety net to one that has a less comprehensive one). I am justified in not being informed about politics. It is more unethical for a sports star who is seen as a role model for teens to take serious drugs than it is for a person who is not a role model to do so.
2. Questions about Bodies

Women are not justified in looking down on other women who obtain breast implants. I am justified in believing that people who are out of shape and who have no underlying endocrine problem are pathetic. I am not justified in having a negative view of women who wear tight clothing. A spouse or mate is not justified in criticizing his or her partner for gaining weight.
3. Questions about Beliefs and Attitudes

People with a high level of post-secondary education are justified in feeling superior to people who have no post-secondary education and who work in menial jobsfor example, a lawyer versus a cleaning lady. It is perfectly acceptable for someone to listen to music with derogatory lyrics or violent and offensive language (insert a specific example). I am justified in believing that people who frequently use profane language (insert examples) are not pathetic. I am justified in thinking that men who wear earrings are pathetic. The use of non-gender-neutral terms is offensive to some. They are not justified in feeling this way. It is reasonable for people who are members of a group that has suffered discrimination (e.g., women, black people, First Nations) to harbor resentment toward those who are members of a group that perpetrated the discrimination. Looking down on people who dont speak the official language of your country is justified. A heterosexual is justified in being offended when called gay. I feel disgust when I hear of people who marry others twice (or half) their age. My disgust is justified.

LABELING RELEVANT BEHAVIOR CAN BE ACTION GUIDING

67

68

PART II PRACTICE

YOUR PRESENT VALUES ARE YOUR WINDOW INTO YOUR FUTURE

Aspiring to be filthy rich is justified. Women are justified in getting annoyed at their male partners who remain incompetent with regard to household chores.
4. Questions about Emotions

My father refused to let me have the car to go skiing. I was furious. I was justified in being furious. My friend and I both took part in a tennis competition. I ended up the champion. I was not only glad that I was the champion; I was glad that I beat my best friend. I was justified in feeling that way.

SECTIon 1 LEARnIng ThE InTRICACIES of PRACTICAL REASonIng

69

ARE YOUR EMOTIONS THOSE THAT YOU CHOOSE?

My friend and I both took part in a tennis competition. My friend ended up the champion. I was green with jealousy. I really wish that anybody else other than my friend had won. I was justified in feeling that way. Men make such a big deal about women having big boobs. I cant help but wish that I had big boobs too. I am justified in feeling that way. Last week, my girlfriend (boyfriend) broke up with me. I am feeling really sorry for myself. I am justified in feeling sorry for myself. My girlfriend (boyfriend) left me for another about 8 months ago. At the time, I was angry. I was justified in being angry.

70

PART II PRACTICE

People who have outrageous amounts of luxuries make me jealous. My jealousy is justified. I get annoyed when people drive slowly in front of me. My annoyance is justified. I got into an argument with my mother. She said . . . , I said . . . , she said . . . , and then I got really mad and said. . . . I was justified in my display of anger.
5. Questions about Behavior

Not talking with other people is considered rude for North Americans but not for Japanese. For example, in Japan, many people prefer silence to unnecessary small talk. If I were God and I could make only one society, I would make a Japanese one. Anyone who owns or aspires to own an unnecessarily expensive car (over $80,000) is not necessarily superficial. People who buy ridiculously expensive designer clothing (e.g., a $1,000 pair of pants) are superficial. People who watch a lot of time-wasting TV (e.g., more than one hour per day of sit-coms or reality TV) are pathetic. I am one of those people who watch TV all day long; its one of my favorite things to do. I think watching a lot of TV is perfectly acceptable. My friend copies his essays from the Internet. This enrages me, particularly when he gets better grades than I do. I should. . . . I am a full-time cashier at a grocery store. I am justified in frequently taking sick days even if I am not sick. I work in retail sales and often lie to a customer about how a garment looks on them in order to make a sale. I am justified in doing so. It is perfectly acceptable to have a passion for shopping and indulge it (e.g., go to the mall every week and buy unnecessary stuff). Spending a lot of time in chat rooms (e.g., two hours per day) on the Internet is acceptable. Young women who engage in BJ parties for men in order to be cool or who give men blow jobs in order to be invited to a party are acting in a way that is truly reprehensible. I am justified in lying to my younger siblings about my frequent use of marijuana, as I do not want to influence them negatively. They often ask me whether or not I smoke pot, and I tell them that I have tried it only a few times, although in reality I smoke up almost every day. I dont think that there is any problem if a young adult uses bad words such as sh*t or f*ck, but I do not think that it is acceptable for teenagers to use those terms. My beliefs are justified. I am justified in not taking any serious steps to stop my bulimia.
6. Questions about the Lack of Action

It is perfectly acceptable to make no attempt to keep current with world events. Even though doing so may save lives, it is perfectly acceptable for anyone over the age of eighteen not to sign up to be an organ donor (religious reasons aside). I am justified in not moralizing to my friends and family about not eating meat, even though I believe that they should not. No one is justified in making a comment such as I strongly disagree with you wearing fur to individuals who are not close friends or members of their immediate family. I am disgusted when my friend wears revealing clothes when we go out and completely changes her personality by constantly flirting with every guy she meets. Nonetheless, I am justified in not saying anything to her.

SECTIon 1 LEARnIng ThE InTRICACIES of PRACTICAL REASonIng

71

VALUES BECOME YOURS ONLY WHEN THEY INFORM YOUR ACTIONS

I am naturally shy. Doing nothing to overcome this natural shyness is perfectly acceptable. I do not push myself to start conversations with people when I start a new class. This behavior is perfectly acceptable. If you are a member of a group who once discriminated against others but you have not individually perpetrated that discrimination, you should nonetheless feel ethically required to right past wrongs (e.g., vote for restitution of tribal lands to First Nations people). When I am sitting in a vehicle that is going well over the speed limit and is being driven by a person without a criminal record, from a good home, who

72

PART II PRACTICE

gets good grades, I do not speak up, even though I wish the driver would slow down. My silence is justified. Not telling your friend that street racing is absolutely unacceptable is acceptable. I often hear my teammates, in a friendly situation, joke about racist remarks such as f*cking nigger. I am justified in not saying anything.
7. Questions about Intimate Relationships and Sex

My intent is to marry someone of the same cultural background as I (Chinese). For that reason, I date only individuals who are of the same background. This is justified. When it comes to picking partners, one should not be a critical thinker (i.e., one should not be open to what others might say about ones partner). Parents should try to dissuade their high school offspring from having sex. It is unethical to indulge in sexual relations (i.e., intercourse) with anyone until after the first few weeks of the relationship. Many women have sex with their partners solely because their partners want to have sex. Such women can nonetheless claim to have integrity. Given the appalling state of the heterosexual marriage institution, gay and lesbian couples are irrational in their pursuit of marriage (benefits aside). When it comes to my sexual orientation (which is gay), integrity does not necessarily require that I make my orientation as visible as possible. I am justified in believing that most men will do anything to manipulate a woman into bed and in thinking less of them for it. Casual sex (i.e., frequent sexual intercourse with a large variety of individuals with whom one has no emotional connection) is OK. Men who do not learn how to cook are passive-aggressively, attempting to ensure that gender roles are maintained, and so are immoral. Men should not feel concerned about the fact that their average income is higher than that of women. I am justified in feeling disgust for anyone who frequents strip bars. Indulging in soft porn is perfectly acceptable. A friend of mine asked me if it was OK to date my ex-girlfriend. I said he could, but felt anger afterward. My anger was justified. I have broken up with my girlfriend. To me, the relationship was one that was both meaningful and significant. I am justified in not acknowledging her presence if I see her. It is completely ethical to lie (either by commission or omission) about how one romantically feels about another person even in a committed relationship (e.g., saying, I love you even though you do not mean it). Cybersex should be avoided. White weddings with all the trappings are OK. The fact that Jack, who is a young man, does not want to see Jane for a period of time just because Jack wants some space and gives no other reason should not be perceived by Jane as offensive. I prefer women with large breasts, and I date mainly these women. I am not shallow for doing so. Women who dress in tight clothing and who use their bodies to get what they want out of men should not be perceived negatively. My boyfriend feels that the only way to be happy in a relationship is to be nonmonogamous. I do not agree with this. Still, since our relationship is otherwise terrific, I should try to accommodate him. I am a twenty-year-old man. I am justified in breaking up with a woman whom I otherwise like a lot if she believes that sex should only be undertaken in marriage.

SECTIon 1 LEARnIng ThE InTRICACIES of PRACTICAL REASonIng

73

YOU MAY BE BLIND TO THE PERSON YOU ARE BECOMING

I am justified in believing that guys who have sex with as many random women as possible are sexist. I am justified in continuing a friendship with my ex-girlfriend (i.e., talking to her frequently on the phone and seeing her for coffee from time to time) even though my new girlfriend would rather I didnt. I am justified in not expressing things that are bothering me in my relationship with my boyfriend of two years, such as putting me on hold during telephone conversations for extended periods of time, in order to avoid an argument.

74

PART II PRACTICE

I have a friend who is bisexual but does not tell his partners. I believe his silence is justified as long as he practices safe sex.
8. Questions about Addiction and Drugs

I am justified in believing that anyone who has a drug or alcohol problem is solely responsible for his or her condition. We are not justified in making alcoholics and drug addicts feel guilty about their addictions. Anyone who indulges in the use of cocaine or heroin is someone who is not worth helping. It is perfectly acceptable behavior to get drunk (or stoned, or whatever) every weekend. If ones partner gets drunk, or stoned, or whatever frequently, one should break up the relationship if s/he refuses to stop. I am morally opposed to the use of marijuana unless it is for medical purposes. However, I am not opposed to drinking alcohol, and I believe that my position is justified. Many would argue that the fact that I smoke shows that I am a weak individual. That belief is not justified.
9. Questions about Child-Parent Relationships

I am not a bad person if I decide to have children knowing that they will be cared for by nannies. Parents are not justified in forcing older siblings to stay home and babysit younger siblings. Parents are justified in giving their child a toy gun if the child wants it. If they are in a position to do so, parents should pay for the post-secondary education of their children. Students whose parents pay for their post-secondary education but who do not do their very best are grossly unethical. While I was growing up, my father, who is a Hindu, made it clear that I was not to marry a person of Muslim faith. Up until now, I avoided disagreeing with him in order to prevent an argument. However, even though I am not now in a serious relationship, I believe that I should tell my father that I dont agree with his opinion and that I would happily marry a Muslim should I fall in love with a Muslim. My father is obesea condition of which I disapprove, yet I say nothing to him. My silence is justified. My father has a number of ailments including alcoholism, colitis, and diabetes. He does not take care of himself and appears to be a burden on my mother. A few years ago, while intoxicated, he said he wished I had never been born. I ceased speaking to him for two years after his comment. I was justified in not speaking to my father for two years. Parents should not try to dissuade their children from tattooing themselves. Parents should never intervene when their children are trying to settle a dispute, provided that the method of dispute settling is nonviolent (give a specific example). It is hypocritical for parents to pass judgment on the use of marijuana by their children if they themselves once smoked it. I am justified in blaming my parents for my present state. (Be specific: What behavioral or attitudinal legacy do you think you have inherited from your parents?) Adults should not try to convince their children of the existence of such fictitious entities as Santa Claus or the tooth fairy.

SECTIon 1 LEARnIng ThE InTRICACIES of PRACTICAL REASonIng

75

Parents should not pressure children to participate in activities that are not essential to their welfare (e.g., playing piano). I am a nineteen-year-old man, and I believe that I should be able to come home late if I want to. Whenever I come home past 2:00 a.m., my parents become very angry with me. My parents are not justified in getting angry with me. I tell my mom that I am studying at the library when Im really over at my boyfriends house. I do this because she thinks that I spend too much time with my boyfriend, and I do not agree. I am justified in lying. Although my father is critical of my homosexuality, there have been rare moments where he has expressed admiration, pride, and genuine love for me. Nonetheless, I feel no love for him in return, and I believe that I am justified in not being at all concerned about this.
10. Questions about God, Death, and Religion

It is not foolish to believe in God as characterized by a specific religion. Those who have been at the wrong end of a gun or who have survived a shark attack often speak of the terror they felt. In situations such as these, I believe it is irrational to feel terror in the face of death. I am justified in believing that people who think that their religion is the only legitimate one are pathetic. It is irrational not to believe in life after death. My girlfriend believes there is an afterlife and that the life we have right now on Earth is just a small percentage of our ever-lasting life. I believe that is a stupid idea, and I am justified in trying to convince her of that. It is unethical for followers of religions such as Jehovah Witnesses to try to force their religious beliefs on others. It is perfectly acceptable for religious charities to make it clear to the recipients that the help comes in the name of their God or prophet. I am justifiably contemptuous of people who believe they have been saved by prayer. I am justified in being vigilant with regard to one part of my religion (i.e., as Muslim I do not eat pork) but not another seemingly more significant edict (e.g., I drink alcohol). I personally was forced to go to the Sikh temple when I was young. As a result, I have come to believe that parents are wrong to force their children to go to religious institutions. I believe this position is justified. My parents are Catholic, but I dont agree with many of the things the Church has to say. Nonetheless, I should continue to fool my parents into believing that I believe in Jesus Christ even though I do not.

EXErCISE 3B
Your list
review the value claims that you circled in Exercise 3a. Now, pick seven of those that you circled and create relevant thesis statements, or create seven original thesis statements that may have come to mind while you were doing Exercise 3a. Write out a list of seven thesis statements, and below each show how it is relevant by indicating how your behavior would change if you answered the question embedded in your thesis statement (1) in the affirmative or (2) in the negative. to ensure that your thesis statement is sufficiently contentious to be worth exploring, indicate what would count as the opposition.

76

PART II PRACTICE

EXAMPLE
Thesis Idonotkeepabreastofcurrenteventseitherbyreadingthenewspaperorby watchingthenewsonTV.Ithinkthispracticeisjustified. Relevance
affirm:Iwillcontinuetoavoidreadingnewspapers,andsoforth. deny:IwillmakeaseriousefforttoreadnewspapersorwatchthenewsonTV.

Opposition Thestrengthofademocracyisafunctionoftheinformationthatisnotonlyaccessiblebutalsoaccessedbyitscitizens.

Use the following checklist to determine the adequacy of your thesis statements.
Checklist for a good (practical) thesis statement

Is it about values? (Only values guide behavior.) Is it clear and precise? (What would the world look like if your thesis statement were true?) Is it contentious? (Can you quickly imagine a strong opposition?) Is it relevant? (How will this impact your behavior?) Is it tentative? (Are you genuinely prepared to embrace its contrary should the opposing viewpoint prove stronger?) NOTE: Above all, avoid Internet topicsi. e., familiar topics that you can find on the Internet, such as capital punishment, prostitution, the legalization of marijuana, and so forth. Since a lot of thinking has already been done on these topics, they tend not to elicit deep and novel ideas, and thus they preclude the kind of reflective practice that is necessary to become wise. In addition, because they have already been tackled in such depth, you run the risk of doing extremely poorly by comparison.

4. Common Informal Faults or Fallacies


A fallacy in argumentation will hinder your ability to judge the merits of any position and hence will prevent you from being able to estimate accurately whether your views are justified. For that reason, we begin with a brief overview of some of the most common informal fallacies. Fallacy means faulty argument. In subsequent sections, we study formal fallaciesi.e., illegitimate moves in logic. Here the focus is on informal fallaciesi.e., illegitimate moves in arguing. Such moves are considered illegitimate because they steer discourse away from rather than toward truth. The point of discourse among rational beings is to move toward truth. Thus, if you claim to be committed to truth, and if you wish to be respected as a rational being, then, as far as possible, you should avoid committing informal fallacies. The following are just a few of the most common informal fallacies. Study them, and then try the practice exercises.

1. Begging the Question


When you are constructing an argument, start with the assumption that your listener does not believe your conclusion; that is why you are going to the trouble of constructing the argument. Your job, in constructing a convincing argument, is thus not only

SECTIon 1 LEARnIng ThE InTRICACIES of PRACTICAL REASonIng

77

to search for reasons that lead to your conclusion but also to search for reasons that your listener believes. In practical reasoning, your conclusion asserts that a certain state of affairs should be perceived as having a certain value. Thus, what you need to do in constructing a convincing argument is to engage in what could be described as cognitive fishing: What you are trying to do is to hook onto a principle (or value/fact) that your opposition holds dear and then, on the strength of your reasons, paste that value onto a new fact. In begging the question, the so-called reason that one provides is often the conclusion, dressed up in different words. This is a fallacy because, if your listener does not believe your conclusion, obviously your listener is not going to believe your reasons. Reasons is put in quotes here because, in begging the question, you are not really

A REASON THAT IS NOT A REASON: BEGGING THE QUESTION (I)

78

PART II PRACTICE

giving a reason. You are merely stating a conclusion and then restating the conclusion as a reason; i.e., you are arguing in a circle.
Example of begging the question

It is obvious that God exists (conclusion) because it says so in the Bible, and the Bible is the word of God (reason).
If the conclusion is embedded in a reason, or if a reason implicitly assumes what the argument is trying to prove, it begs the question.

The preceding is an example of begging the question. Obviously, if your listener does not believe that God exists, then s/he will not believe that the Bible is the word of God. A common form of begging the question is an argument that makes a reference to a right. Certainly, citizens in a society governed by the rule of law have legal rights. However, a reference to any other kind of right must be backed by a reason why this so-called right should be recognized as a right. In the same vein, a reference to discrimination must be backed by reasons if one is to avoid simply assuming what one is trying to prove.
Example of begging the question

Post-secondary fees should remain frozen. Students have a right to a highquality education at an affordable price. Who says students have a right to a high-quality education at an affordable price? Is it written down somewhere? Anyone can say that anything is a right. However, merely saying that something is a right is not showing why it should be recognized as such. Saying that something is a right is assuming the truth of the very issue that is in question. That is why it is begging the question.
More examples of begging the question

abortion should be illegal because the fetus has a right to life. Physicians should never participate in an act of euthanasia because physicians should always do everything in their power to keep patients alive. In vitro fertilization should not be covered under Medicare because artificial reproduction should not be funded by other peoples tax money. Genetic testing should be allowed only when there are reasons to believe that the fetus is suffering from a severe highly debilitating disorder, such as tay-Sachs disease, because genetic testing should be used only to prevent pain and suffering; it should not be used to create the ideal child. Capital punishment should be banned because all humans have a right to life. Police officers should be allowed to wear visible religious symbols such as turbans while on the job, because if we do not allow people such as Sikhs to wear their turbans while working as police officers, then we are discriminating against them for their religious customs. Young offenders should be treated like any other criminal because all offenders should receive the same treatment under the law. Police officers should not be allowed to carry guns because no one has the right to carry a gun. targeted hiringthe practice of attempting to hire minorities who are equally qualifiedis wrong because it is reverse discrimination. Another common form of begging the question is the use of emotionally charged language, which tends to push the argument in a certain direction irrespective of the weight of the evidence or reasons. To say that abortion is wrong because it is murder is begging the question because the word murder carries with it the connotation that the killing is wrong, but that is the very question at issue.

SECTIon 1 LEARnIng ThE InTRICACIES of PRACTICAL REASonIng

79

A REASON THAT IS NOT A REASON: BEGGING THE QUESTION (II)

Another form of begging the question is housed in an actual question, and it is called a complex question. If I ask you whether or not you have stopped abusing your loved one, then, if you answer yes, you have implied that you had in fact been abusing your loved one, and if you answer no, you are also saying that you have been abusing your loved one. No matter how you answer, you cannot duck the assumption that you have been abusing your loved one. Thus, the question assumes the truth of a claimi.e., that you have abused your loved oneand is thus a complex question, or a form of begging the question. Begging the question is a tactic often resorted to by individuals who do not have the energy or the capacity to provide reasons or evidence in support of the conclusion.

80

PART II PRACTICE

It is also dishonest because, in begging the question, the arguer pretends to supply reasons in support of the truth of the conclusion but in fact merely assumes the truth of the conclusion from the beginning. Begging the question should be avoided at all costs. (For more examples of begging the question, see the description of support in Part II, Section 2.) RESPONSE TO BEGGING THE QUESTION It is important not only to recognize when one is faced with an informal fallacy but also to develop tactics to cope with it and, one hopes, get the discussion back to tracking truth. A helpful response is to name the fallacy and explain why it is a fallacy. Thus, when faced with begging the question, one might respond in the following way: What you are doing is begging the question. When we are discussing the issue of abortion, what we are doing is trying to establish whether or not it is true that fetuses have rights. It is hardly persuasive to simply proclaim that they have rights.

2. Ad Hominem/Ad Feminam Attack


A. Illegitimate ad hominem/ad feminam attack

The implicit rule of rational discourse is that participants judge arguments on their own merit. It does not matter whether that argument is written in a blue book or a brown book, or whether it is articulated by a man or a woman, by a Caucasian or a black, or by a five-year-old or an eighty-five-yearold; it is still the same argument. Hence an argument should be judged irrespective of its source. When you articulate an argument, think of it as a shield: It stands in front of you. When someone disagrees with your position, it, not you, should be the focus of the attack. If an individual tries to jump over the shield by attacking the source of the argument rather than the argument itself, this is called an illegitimate ad hominem attack (if the individual under attack is a male) or an illegitimate ad feminam attack (if the individual under attack is a female). There are many ways an ad hominem attacker can try to discredit the source of an argument. One obvious way is by saying nasty things about the individual who is presenting the argument. This is called an abusive ad hominem attack.
Example of an ad hominem attack (abusive)

the man is an utter ignoramus. Why should we believe anything he says? Another way to attack the source of the argument is to accuse the arguer of being a hypocritei.e., of acting in a way that contradicts the argument. This is sometimes referred to as tu quoque, which is Latin for you also. This is a seductive move, as we like to think that peoples actions should match their words. Nonetheless, although a listener may indeed deserve an explanation about why this is not the case, an argument should be judged on its own merits, irrespective of the actions of the arguer.
Example of an ad hominem attack (tu quoque)

Why should I listen to your arguments about not smoking? after all, you smoke! Yet another way to try to discredit the source is to suggest that the reason why the arguer is presenting the argument is because s/he has something to gain by it. However, the fact that someone might have something to gain by an argument has nothing whatsoever to do with whether or not an argument is or is not valid or sound. If it did, then, logically, we would never be able to argue our own case. Some refer to this move of attacking the arguer because the person has something to gain as a circumstantial ad hominem attack.

SECTIon 1 LEARnIng ThE InTRICACIES of PRACTICAL REASonIng

81

ATTACKING THE ARGUER RATHER THAN THE ARGUMENT MISSES THE POINT
Examine the counterargument. If it attacks the source rather than the argument, it is committing an illegitimate ad hominem/ ad feminam attack.

Example of an ad hominem attack (circumstantial)

the only reason the teachers are arguing for small class sizes is because it will make their jobs much easier. Yet another way to discredit the source is to associate the arguer with a group suffering from a negative reputation. This is sometimes referred to as poisoning the well.

82

PART II PRACTICE

Example of an ad hominem attack (poisoning the well)

Dont believe him. that is the kind of argument a Nazi would give. NOTE: Different philosophers tend to use ad hominem subheadings differently,36 so my suggestion is that you use the subheadings only insofar as you believe they help explain the illegitimate move of the attacker. The main point to remember is that the rules of rational discourse dictate that an argument must be judged on its own merit. If the source rather than the argument comes under scrutiny, then it is an illegitimate ad hominem/ad feminam attack. RESPONSE TO AN ILLEGITIMATE AD HOMINEM/ AD FEMINAM ATTACK If you present an argument and someone levels an ad hominem/ad feminam attack at you, you must grab the argument and pull it back in front of you. Do not allow the attacker to sidetrack you through personal abuse or unflattering innuendo. Sidetracking is precisely what the attacker is trying to do, probably because your argument appears too strong to be dealt with legitimately. So stand your ground, not by defending yourself against the accusations of the ad hominem/ad feminam attack but by pulling the argument back onto center-stage and demanding that it be dealt with as delivered. That is an illegitimate ad hominem attack. All arguments deserve to be judged on their own merits. Whether or not teachers themselves will benefit from smaller class sizes is irrelevant. They have argued that students will benefit enormously from smaller classes. Do you or do you not agree with that argument?
B. Legitimate ad hominem/ad feminam attack
Warning! If you give no reason or evidence in support of a claim, you are open to a legitimate ad hominem/ad feminam attack.

The point of rational discourse is to try and ferret out the truth. That is why a fundamental rule of rational discourse is that participants judge arguments on their own merit. It is reasons or evidence that points to truth, not people. Thus, since truth is the goal, if a speaker offers a claim without the backing of reason or evidence, the only way a listener can judge the truth of that claim is by judging the source. Is this source someone to be believed? Is this source someone who is in a position to verify the truth of this claim without reason or evidence? This examination of the source is a legitimate ad hominem/ad feminam attack, and it is legitimate precisely because the paramount goal of a rational individual is to seek truth. Without reasons or evidence, the only way to seek truth is to judge the source. It is of interest that it is this logic of truth seeking (not the stupidity or patriarchal bias of lawmakers, as some may believe) that creates problems in formulating fair practices for rape trials. Obviously, one does not want to further victimize the purported victim by subjecting her to character assassination on the stand. On the other hand, without corroborating evidence, it is impossible to tell whether the plaintiff or the defendant is telling the truth except by examining their respective characters. In light of this predicament, one finds that many legal systems try to walk the fine line of having such things as rape-shield laws (making it illegal to attack the purported victims background) that can, under certain circumstances, be lifted. In the final analysisas a result of logic, not the malice of menthe outcome is often determined by a character battle. RESPONSE TO A LEGITIMATE AD HOMINEM/ AD FEMINAM ATTACK Since attacking the source is legitimate in the absence of reasons or evidence, the best policy in this case is to avoid the possibility of such an attack by always supporting claims with reasons or evidence. Unhappily, there may be situations in which you have no proof. You can prepare for those instances only by building a track record of honesty and integrity. In such a situation, it is your character that will be under scrutiny.

SECTIon 1 LEARnIng ThE InTRICACIES of PRACTICAL REASonIng

83

3. Appeal to Authority
A. Illegitimate appeal to authority

Human societies have flourished because humans have been able to build on the knowledge of others. We do not have to reinvent the wheel with each new generation. Within societies, individuals can access truth based on the knowledge acquired by others. Thus, if my vet tells me that my old dog should be given cortisone, I can believe that it is true because I can appeal to the expertise of that authority. An appeal to authority is a substitute for an argument. Obviously, in our complex society, appeal to authority is a common move. It is so common, in fact, that we are often unaware when it is used illegitimately. Thus, it is helpful to keep in mind that certain criteria must be met if the appeal to authority is to be legitimate. First, in order for an appeal to an authority to be legitimate, the authority must be an authority in that field. Sports stars are not nutritionists and hence are not authorities on what cereals are good for you to eat. Second, there must be no disagreement in the field for an appeal to an authority to be legitimate. Remember that an appeal to an authority is a substitute for an argument. Clearly, omitting the arguments will not do if experts themselves disagree. When disagreement between experts exists, we must refer back to the respective arguments. Third, the arguer should be both willing and able to access the source upon request. Since an appeal to authority is used in place of an argument, if a listener is at all skeptical, s/he should be able to gain access to the original argument.
Example of an illegitimate appeal to authority

An appeal to authority is a substitute for an argument. It is illegitimate if the authority is not recognized as such, if there is disagreement among authorities, and if access to that authoritys argument when needed is not available.

One of the main causes of unemployment is laziness. Our governor said so on the television last night. The preceding appeal is illegitimate because there is no reason to believe that the governor is an expert sociologist, and hence there is no reason to believe that he is an authority on the causes of unemploymentto say nothing of the fact that, no doubt, disagreement would abound with regard to the precise causes of unemployment.
Example of an illegitimate appeal to authority

to get a more balanced view of abortion, a nationwide survey of doctors was conducted recently. the results of this survey show that an overwhelming majority of doctors believe that having an abortion is morally acceptable. The preceding appeal is illegitimate because doctors are experts in medicine, not ethics. Doctors are experts in carrying out abortions; they are not experts in judging whether or not such practices are or are not immoral. In addition, since ethics is a field in philosophy, and since, by definition, virtually every issue in philosophy is contentious, ethical issues can never be solved solely by an appeal to authority. RESPONSE TO AN ILLEGITIMATE APPEAL TO AUTHORITY You refer to Albert Einstein in making your case that it is unethical to stockpile nuclear weapons. That is an illegitimate appeal to authority. Whether or not we should stockpile nuclear weapons is an ethical or political question. Einstein was an expert neither in ethics nor in politics; he was an expert in physics. You would be correct in referring to Einstein if you wanted to make a bomb; however, you are out of order when you refer to him with regard to the question of whether or not we should stockpile them. Another common form of illegitimate appeal to authority is an appeal to the crowd. We tend to believe that if everyone thinks that something is true, then it must be true. However, at one time everyone believed the world was flat, and many believed that slavery was perfectly acceptable. People, no matter how many, do not point to

84

PART II PRACTICE

truth; only reasons and evidence do. We legitimately substitute an appeal to authority for an argument only if the authority in question is a recognized experti.e., recognized as one who would be able to supply the argument if asked. Crowds, no matter how large, do not qualify as an authority, so an appeal to the crowd is always illegitimate, even if what the crowd believes happens to be right!
Example of an illegitimate appeal to authority (appeal to the crowd)

Dad, there is nothing wrong with my going to a rave. Everyone else in my class is going. RESPONSE TO AN APPEAL TO THE CROWD You guessed it! It is precisely what, no doubt, your mother once said to you: So if everybody believed that jumping off the bridge was a great idea, would you jump off the bridge too? Another potential illegitimate appeal to authority is an appeal to tradition. Just because something has always been done a certain way does not necessarily mean that it should always be done in that way. On the other hand, we must keep in mind that tradition is the glue that keeps cultures intact, so referring to tradition per se is not illegitimate. However, it is illegitimate in the face of strong arguments against the practice, and it is illegitimate only because it is a substitute for an argument. If someone presents a strong argument against continuing a tradition, the onus lies with the traditionalist to support its continuing practice with argumentation; simply saying that that is the way it has always been done is not sufficient.
Example of an illegitimate appeal to authority (appeal to tradition)

An appeal to an authority is a substitute for an argument. Crowds are never legitimate substitutes; traditions can be.

It is perfectly acceptable to carry out clitoridectomies on young women. after all, we have been doing this in our culture for hundreds of years. RESPONSE TO AN ILLEGITIMATE APPEAL TO TRADITION To argue that a practice should be maintained because it is a tradition has some merit, on the face of it, as traditions help to maintain the cohesiveness of society. However, the argument ceases to be persuasive when the harm that results from maintaining that tradition outweighs the benefit. Surely you agree that it would be odd to argue that we should maintain slavery in the American South simply because that was once a traditional way of life. Thus, your argument that First Nations people should be able to resume whaling simply because it was once traditional is illegitimate because there is now ample evidence that whales, like humans, are self-conscious, and in our society, killing self-conscious entities is considered murder.
B. Legitimate appeal to authority
A legitimate appeal to authority requires (1) that the authority be an authority in that field, (2) that there be no disagreements between experts, and (3) that the source be verifiable.

Obviously, we cannot all be experts in all fields, so we must sometimes substitute an appeal to an authority in place of being able to supply the argument ourselves. Keep in mind, however, that an appeal to an authority in support of the truth of a claim is powerful only if it is presumed that the authority, if asked, could articulate the argument; it is to this argument that one is implicitly appealing. Thus, it is imperative that if one makes an appeal to an authority, one be able to provide a trail to that authority so that the skeptic can find his or her way to the actual argument in situations of doubt. That is why footnotes and endnotes are so crucial in academia. They provide a means whereby the original argument can be reexamined should questions arise about the authority to which one has appealed.

AN APPEAL TO TRADITION CAN SOMETIMES BE A REASON

85

86

PART II PRACTICE

4. Strawperson
Truth seeking requires that one estimate which of all possible contenders is the least weak. This process is destroyed if one of the options is made of straw.

One can never have direct access to truth. The only way to test the truth of a claim is to establish global sufficiencyi.e., test it against the strongest possible opposition. Unhappily, many individuals are more interested in appearing right than in seeking truth. (See the comic entitled It Is More Important to Be Right than to Appear Right on page 189.) Thus, they attempt to misrepresent the opposing position in an effort to make their own position appear stronger. This is a particularly effective move when the potential answers to a question appear to be bipolari.e., either A or not-A. To the degree that one is able to show that the other side is weak, ones own position becomes proportionately stronger. Thus, those who are pro-choice often misrepresent the concerns of those who are pro-life by stating, for example, that the only reason people are against abortion is because they want to keep women chained to the home, when clearly the real concern is the importance of valuing all human life. On the other hand, pro-lifers misrepresent pro-choicers by saying that they are unconcerned with the destruction of human life, while they ignore the real argument that since the fetus is not self-conscious, it is hypocritical to give its life more value than that of cows or chickens, which are routinely killed. This attempt to misrepresent or ignore the oppositions strongest argument is called a strawperson fallacy, and it is a fallacy because one has precluded the possibility of testing ones claim against the strongest possible opposition. One has thereby precluded the possibility of attempting to estimate whether or not ones claim is true.
Example of a strawperson argument

the only reason the government does not ban the sale of tobacco products is because the tobacco industry is a major provider of jobs and hence helps the government financially through taxation. A much stronger reason why the government does not ban tobacco is that it would be a highly paternalistic move whose justification would be inherently faulty as a result of a vulnerable major premisei.e., that all things that are harmful should be banned. What about fast food products that increase cholesterol levels, participation in dangerous sports, the building and driving of cars, the manufacture and use of pesticides, and so on?
Example of a strawperson argument

the only reason prostitution has not been legalized is because most people believe sex should take place only between people who love each other. A stronger opposition to the legalization of prostitution would be that since women enter prostitution only because they are desperate, legalization would not only symbolize that the government condoned the degradation of women but also serve as an excuse for the government not to rectify the desperate situation in which women too often find themselves. RESPONSE TO A STRAWPERSON I can understand why you would like to see tobacco banned. However, you commit a strawperson fallacy when you say that the only reason that the federal government refuses to ban tobacco is because it gets a lot of money from taxing cigarettes. Saying that misrepresents the governments position. On one hand, we can assume that the tax from cigarettes is more than used up in tobacco-related health care costs. More important, however, we should presume that the government does not attempt to ban tobacco because it would be perceived by the public as illegitimately paternalistic, just as it would if the government tried to ban any other dangerous activity, such as rock climbing.

SECTIon 1 LEARnIng ThE InTRICACIES of PRACTICAL REASonIng

87

REASONS ARE STRONG ONLY BY COMPARISON: BOWLING FOR TRUTH A STRAWPIN ARGUMENT

5. Slippery Slope
A, B. Legitimate and illegitimate slippery slopes

A slippery slope argument takes the following form. You cannot do a, not because there is anything wrong with a in and of itself but because a will lead to b, which will lead to c, which will ultimately lead to z, and it is z that is unacceptable. If we allow a, we will not be able to stop the slide to z. The line between a and z, in other words, creates a slippery slope. This argument form is also referred to as the wedge argument,

If you argue that you are not opposed to a, but a will lead to b, which will lead to c, which will lead to z, and it is z that is unacceptable, you have presented a slippery slope argument.

88

PART II PRACTICE

A slippery slope argument is legitimate if it explains why the slope is slippery. It is illegitimate if it does not.

although that phrase more accurately describes a possible response. If someone presents you with a slippery slope argument, one way to respond would be to suggest a way to stop the slidei.e., to suggest a way to put a wedge in the slope. A slippery slope argument is a legitimate argument so long as it demonstrates why the slope is slippery; a slippery slope argument is illegitimate if it does not show why the slope is slippery. In other words, a slippery slope argument is legitimate as long as reasons are presented; it is illegitimate if no reasons are presented. It should be kept in mind, however, that just because a slippery slope argument is legitimate does not mean that its conclusion should be accepted. To say that an argument is legitimate is simply to say that it is worthy of consideration; in the long run, opposing arguments may be stronger. To say that an argument is legitimate is simply to say that it has passed the test of local sufficiency; it may still fail the global sufficiency test.
Example of a legitimate slippery slope

Some argue against legalizing euthanasia on the basis that if euthanasia became widespread, ultimately our society might transform from one in which the default value is life (you have to argue strongly to be allowed to die) into one in which the default value is death (you have to argue strongly to be allowed to live). they argue that although there may be nothing wrong with allowing euthanasia in any particular instance, if euthanasia becomes quick, easy, and widely used, individuals who are an enormous burden to societye.g., those with catastrophic physical disabilitiesare going to be hard-pressed to justify their wish to stay alive in the face of (1) overwhelming financial and human energy costs and (2) the common assumption that there is an easy way out.37 RESPONSE TO AN ILLEGITIMATE SLIPPERY SLOPE ARGUMENT You argue that if you allowed girls to wear lipstick, the next thing you know they will be wearing all kinds of jewelry, then they will modify the school uniform, then they will refuse to wear the school uniform altogether, then they will be wearing short shorts and showing cleavage, and ultimately the reputation of this school will be in ruins. What you have presented is an illegitimate slippery slope argument. You have not given any reason why wearing lipstick should lead to anything else. Surely it would not be difficult to allow the girls to wear lipstick without changing anything else.

6. False Dilemma
You are one of a group of workers called to an emergency meeting. The president of the company describes, in detail after terrifying detail, how the company is on the verge of bankruptcy. He tells you that you have two choices. Either you all take a 20 percent cut in pay or the company will have to be shut down. Which is it? There is no time to lose. The decision has to be made now. Do you want a 20 percent cut in pay or no job at all? In the preceding situation, the president can be described as presenting you all with a false dilemma, sometimes referred to as false alternatives. That is, s/he is telling you that you have only two choices, a or b. In fact, however, there may be a number of other alternatives. Perhaps management could take a 40 percent cut in pay; perhaps the workers could buy the company; perhaps the president should be fired! Presenting a false dilemma is a particularly seductive move because its format is valid. That is, if there are only two alternatives, a and b, if you do not want a, then it is absolutely true that you must choose b. Or if you do not want b, then you must chose a. In logic, this is called a disjunctive syllogism. This is a valid argument, but it may not

SECTIon 1 LEARnIng ThE InTRICACIES of PRACTICAL REASonIng

89

REASONS CAN SOMETIMES CONNECT A TO Z: SLIPPERY SLOPE ARGUMENTS

be a sound argument.38 There may be a problem with the first premise, that is, there may be many more alternatives than two. Getting stuck in a false dilemma shows a failure of imagination. So, every time you hear, or say to yourself that there are only two choices, think again! RESPONSE TO A FALSE DILEMMA You have presented us with a false dilemma. There are many more alternatives than simply closely the factory or taking a 20 percent cut in pay. Why doesnt management take a 40 percent cut in pay or, God forbid, agree to take home the same salary as the front-line workers for a while?

A dilemma (or trilemma or quadrilemma) is false if it falsely limits the number of possible options. Do not trap yourself in a false dilemma. Always try to maximize the number of options that frame your choices.

90

PART II PRACTICE

WISDOM VARIES WITH THE OPTIONS CONSIDERED: FALSE DILEMMAS

7. Distinction without a Difference


Employing a distinction without a difference is a verbal tactic whereby one tries to dismiss the label that would ordinarily stick to a certain behavior or attitude.

If you consistently do not tell the truth, then you are a liar. If you believe that black people are subhuman, then you are a racist. In our society, like it or not, there are labels that accurately describe people who behave in certain ways. Labels, however, can be disconcerting, particularly if they are unflattering. Thus, if someone wants to continue to act in a certain way but avoid the discomfort of a label, that person might resort to using the fallacy referred to as a distinction without a difference. S/he might say, for example, It is not that I am a racist or anything; it is just that I dont think that black people should be allowed to join this club. This is referred to as a distinction without

SECTIon 1 LEARnIng ThE InTRICACIES of PRACTICAL REASonIng

91

a difference because the individual in question is trying to make a distinction (between the way s/he acts and the way a racist acts) when in fact there is no difference. The individual is trying to separate the label from the behavior that it accurately describes. Your response, when presented with a distinction without a difference, is to glue the label and the behavior back together again. In the preceding situation, for instance, you might say, But denying club membership solely on the basis of race is precisely the sort of action that defines what a racist is. Gluing the label and the behavior back together will, it is hoped, result in the happy consequence of a change in behavior. That is, the individual in question may feel so uncomfortable about having to bear the label that s/he dumps the behavior. Interestingly, however, there are some instances in which the point of exposing the use of a distinction without a difference would be to encourage an individual to embrace the label rather trying to put a stop to undesirable behavior. Thus, for example, young women, in contemporary society, who fear disapproval from their male colleagues, often say such things as I am not a feminist or anything, I just think that men and women should have equal access to the goods that any society has to offer. Believing that men and women should have equal access to the goods that any society has to offer, however, is the core belief of the ideology that defines someone as a feminist. Thus, in contrast to the previous example in which it was hoped that gluing the label and behavior back together would result in both being discarded, in this situation the goal would be that such gluing would result in the exaltation of boththat the individual would learn to stand up for what s/he believed. (See the comic entitled Labeling Relevant Behavior Can Be Action Guiding on page 67.)
Example of a distinction without a difference

Student to teacher: I dont want to be personal or anything; I was just wondering if you are married or involved with someone. The preceding example of a distinction without a difference shows it as a devious tactic. The student knows that asking the teacher personal questions is inappropriate but tries to get away with it by prefacing the question with the claim that this personal question is not, in fact, personal. RESPONSE TO A DISTINCTION WITHOUT A DIFFERENCE You say that you are not homophobic, but you believe that homosexuals should not be allowed in the teaching profession. That is a distinction without a difference. Believing the homosexuals should not be allowed in the teaching profession is precisely the sort of attitude that typifies homophobia. If that is what you believe, then you are homophobic, whether or not you admit it.

8. Post Hoc Fallacy


The term post hoc fallacy comes from the Latin, post hoc ergo propter hoc, which means after this, therefore because of this. Formally, the fallacy looks like this: y followed x; therefore, y must have been caused by x. This is a fallacy because the reasoning is utterly inadequate. To make a causal claim, one must invest a lot of careful reasoning. One cannot directly see causes; one can see only the constant conjunction of events, as David Hume pointed out. Thus, since events may be constantly conjoined for many other reasons than the first causing the second; a constant conjunction or a correlation, in and of itself, is no support for the claim that the first caused the second. To be justified in making a causal claim, one must eliminate all other possibilitiesi.e., the claim must pass the test of global sufficiency. The fact that x and y are constantly conjoined may be just an accident (e.g., the fact that something bad happened to someone was not because the person saw a black cat yesterday). It may be that x and y are constantly conjoined because y caused x (it is the rising sun that causes

92

PART II PRACTICE

OWN UP TO AN ACCURATE DESCRIPTION OF YOUR ATTITUDE: DISTINCTION WITHOUT A DIFFERENCE

To make a causal claim, one must first eliminate all other explanations. If one makes a causal claim merely on the basis of a temporal relation or correlation, one is committing a post hoc fallacy.

the cock to crow, not the other way around). The fact that x and y are constantly conjoined may be the result of some third element z (the fact that leeches and improved health are often correlated is not because leeches magically bring about good health but because of the resulting decrease in blood pressure). It takes a good deal of time, energy, and effort to be justified in making a causal claim. The bulk of scientists are employed in that endeavor. To make a causal claim solely on the basis of the fact that one event followed another is evidence of mental laziness.

SECTIon 1 LEARnIng ThE InTRICACIES of PRACTICAL REASonIng

93

Example of post hoc reasoning

Ever since Billy started hanging around with Joey his grades have dropped to Ds, and he is constantly breaking his curfew. Before Billy met Joey he got straight as, and he always listened to his parents. Joey must be responsible for Billys change in behavior. This is a post hoc fallacy because there might be all kinds of other reasons other than the presence of Joey that explain Billys change in behavior. In order to be justified in making a causal claim, one has to do a lot more than merely state a correlation.
Example of post hoc reasoning

In a recent newspaper article, Dr. Henry Morgenthaler, a prominent abortion advocate, argued that society at large has benefited from abortion because, ever since women have had easy access to abortion, there has been a dramatic drop in the crime rate. Specifically, he argued that unwanted and unloved children often fall into a life of crime. thus, since easy access to abortion ensures that fewer unloved and unwanted children are born, it ensures that there will be fewer criminals. a liberal abortion law, in other words, lowers the crime rate. This is a post hoc fallacy because there is no evidence to tie increased access to abortion to the drop in crime rate. The crime rate may have dropped because we are getting tougher on crime, or because we categorize fewer behaviors as criminal, or because we have better crime prevention programs. In order to be justified in making a causal claim, one has to do much more than merely state a correlation. Remember that the justification for believing that a claim is true is a function of the rigor with which one has estimated its merit relative to all other contenders. RESPONSE TO A POST HOC FALLACY You say that the Young Offenders Act should be rescinded because of the increase in juvenile crime since it was enacted. You are therefore assuming that the Young Offenders Act was the cause of the increase in juvenile crime. That is a post hoc fallacy. There may be all sorts of other reasons why there has been an increase in juvenile crimefor example, poor parenting or poor education. You need to provide a lot more evidence to make a causal claim than simply citing a correlation.

9. Analogies: Good and Faulty


An analogy can be described as an attempt to explain y (the primary subject) by likening it to x (the analogue).39 That is, in using an analogy, the arguer must assume that the listener is familiar with xi.e., the analogueand that by using an analogy, the listener will thereby come to have a better understanding of y, the primary subject. An analogy is good if it increases the general understanding of y (e.g., our unconscious is like the bottom of an iceberg). An analogy is faulty if it decreases the general understanding of y (e.g., sex is like food; you will wither away without it). There are also some analogies that are somewhere in between. Discussing the pros and cons of such analogies often leads to a richer understanding of the issue. Since faulty analogies decrease general understanding whereas good analogies increase understanding, it is imperative that we know how to effectively evaluate analogies. Effectively evaluating analogies requires that we recognize that there are two different kinds. The most obvious kind of analogy is one in which the author is trying to make a direct comparisonthat y really is substantially very much like x: The proposed American invasion of Iraq is similar to the American invasion of Vietnam, and you know where that got us. The effectiveness of this sort of analogy rests on the
An analogy is good or legitimate if it enhances our understanding of the primary subject, and it is poor or illegitimate if it does the opposite.

BAD REASONING CAN MISPLACE VALUES (AND ASSOCIATED RESPONSES)

94

SECTIon 1 LEARnIng ThE InTRICACIES of PRACTICAL REASonIng

95

degree to which x and y really are similar, and hence the evaluation of such analogies requires that one analyze the similarities and differences of the primary subject and the analogue. Trudy Govier refers to this kind of empirical analogy as inductive.40 The second kind of nonempirical analogy, which Govier refers to as a priori, are those in which the analogue x is substantially, often wildly, different from y. In fact, the effectiveness of such analogies often rests with the degree of dissimilarity. This is so because the intent of the author is often (although not always) to change the evaluative perception of the primary subject by borrowing it from the analogue. Thus, in his article, Sex with Love vs. Sex without Love,41 Russell Vannoy tries to support the claim that sex without love is better than no sex at all by using the analogy that although we may prefer filet mignon, we are happy to eat hamburger rather than go hungry. What Vannoy is trying to do with this analogy is borrow the neutral attitude that we have toward eating hamburger and attach it to our perception of sex without love. a priori analogies are challenging to evaluate. For one thing, it is always inappropriate to simply say of such analogies that they are faulty because they liken two entirely different entities (e.g., food and sex, or the unconscious and icebergs). As was pointed out earlier, such dissimilarities are often where the power of such analogies lies. The method of evaluation of such analogies thus is more a function of the intent of the comparison than its substance. Thus, if the intent of a particular analogy is to alter evaluative perception, then the evaluation of that analogy would seem to require a comparative analysis of the underlying structure of x and y that hold down, or should hold down, various attitudes. Thus, in response to the analogy between sex without love and eating hamburgers, one might argue that the analogy is faulty because eating hamburgers rarely results in harm, whereas women who consent to sex without love are often harmed because of the prevailing double standard that views sexually promiscuous men as jocks and sexually promiscuous women as sluts. A gendered evaluation of eating hamburger is virtually unheard of.
Example of an analogy

Life is rather like a bumpy bed in a bad hotel. at first you cant imagine how you can lie on it, much less sleep in it. But presently one finds the right position, and finally one is snoring away. By the time one is called, it seems a very good bed, and one is loath to leave it.42 This is a beautiful analogy that provides real insight into the journey of life.
Example of an analogy

It is necessary to force other people to accept our religious beliefs about an afterlife for their own good, just as force must be used to prevent a delirious person from leaping off a steep cliff. Most people would consider this a faulty analogy. After all, we know for sure that you will suffer terrible consequences if you jump off a steep cliff. We do not know for sure that you will suffer terrible consequences if you refuse to believe one set of religious beliefs over another or, indeed, none at all.
Example of an analogy

Government is like business; just as business must be sensitive primarily to the bottom line, so too must government. This is one of those in-between analogies. On one hand, the analogy seems appropriate because, like businesses, governments get into trouble when they get into debt. On the other hand, the prime concern of government is the promotion of the welfare of its citizens, whereas the prime concern of business is profit. A discussion of the pros and cons of this analogy might prove extremely fruitful.
Good analogies make ideas clearer. Faulty analogies mislead.

ANALOGIZING YOUR WAY TOWARD TRUTH

96

SECTIon 1 LEARnIng ThE InTRICACIES of PRACTICAL REASonIng

97

RESPONSE TO AN ILLEGITIMATE ANALOGY You think that we should excuse men their philandering because sex, like food, is a basic need. I think that that is an illegitimate analogy. Whereas we might excuse desperate behavior from those who are literally starving, no one ever died from lack of sex. So comparing food to sex, or the lack of food to the lack of sex, does not help us understand the latter.

98

PART II PRACTICE

10. Fallacy of the Golden Mean


Truth is a matter of evidence, not the arithmetic combination of what people say is true.

Sometimes, when confronted with individuals who strongly affirm two opposing viewpoints, a listener assumes that the answer must be somewhere in the middle. This is called the fallacy of the golden mean because it is fallacious to assume that simply because one is confronted with two extremes, the truth must be somewhere in the middle. Only reasons and evidence point to the location of truth, which may very well be at the extreme. The fact that one is confronted with an individual who strongly argues that slavery is wrong and another who argues equally strongly that slavery is perfectly legitimate in no way suggests that the truth must be somewhere in the middle.

TRUTH IS A FUNCTION OF REASON AND/OR EVIDENCE, NOT MATHEMATICS: FALLACY OF THE GOLDEN MEAN

SECTIon 1 LEARnIng ThE InTRICACIES of PRACTICAL REASonIng

99

It is sometimes hard to distinguish the fallacy of the golden mean from a legitimate attempt to find a social compromise. If you want to eat Chinese food tonight and I want to eat French food, a legitimate suggestion would be that we eat Chinese food tonight and French food tomorrow, or that we both eat Italian food. The difference is that in a social compromise, no truth claim is being made; by contrast, the fallacy of the golden mean, either overtly or covertly, refers to truth.
Example of the fallacy of the golden mean

In a recent divorce settlement a judge ruled that since the couple could not agree on who should keep the family dog, it would be best for the dog if it spent every other week with the husband and every other week with the wife. The preceding is a fallacy of the golden mean because it makes the assumption that the truth (what is best for the dog) lies in the middle solely on the basis that two extreme cases are being presented. However, one of the extremes may very well be right; i.e., the dog may be deliriously happy with one and miserable with the other. To find the truth about what is best for the dog, the judge should have taken the time to examine the evidence rather than jumping to the conclusion that the truth must be somewhere in the middle. RESPONSE TO A FALLACY OF THE GOLDEN MEAN Why should you assume that both children are to blame for the fight? That is a fallacy of the golden mean. Establishing blame is a matter of reason or evidence. It may very well be that the fault lies entirely on the shoulders of one individual.

EXErCISE 4a
Recognizing informal fallacies
respond to each of the following using the name of the fallacy (underlined), and show why the move is fallacious. Answers are in the back of the book. 1. Did you hear John arguing the other day that immigration levels should be curtailed in this country until unemployment levels are under control? I guess it is not surprising, coming from a person who is a member of the Ku Klux Klan. 2. If the government allows shooting galleries to be opened in the downtown East Side, it will be a step backward in the fight against drug abuse. Allowing addicts to indulge their habit in a warm and safe environment will only encourage them to continue their drug abuse. Giving addicts access to shooting galleries is like letting kids loose in a candy store. 3. Young offenders should be treated like any other criminal because all offenders should receive the same treatment under the law. 4. Your friend has been involved in a hit-and-run accident. You are trying to decide whether or not you should go to the police. You think that going to the police is something a friend would not do. On the other hand, you think that not reporting a crime is wrong. The situation is paralyzing you. 5. Sofia had a lot of blemishes on her face. She followed the instructions in her favorite magazine to wash her face in a mixture of lemon and Rindgolds Rosewater for ten days. Gradually, her blemishes disappeared. Sofia went out and bought a years supply of Rindgolds Rosewater.

100 PART II PRACTICE


6. Dont bother opening the door for me, you chauvinist pig! 7. It is not that I am sexist or anything, I just dont think woman have any place in the army. 8. Gay and lesbian couples should receive the same benefits as heterosexual couples. All adults should be considered equal under the law. 9. I think we can ignore the argument that Jennifer presented to the hospital panel that given the years of university training necessary to be a nurse, changing bedpans should not be part of a nurses job description. Jennifer has always hated changing bedpans. Who wouldnt? 10. It is perfectly acceptable for the First Nations people to resume whaling. After all, whaling was part of their tradition long before any white man set foot on this continent.

EXErCISE 4B
Recognizing informal fallacies
respond to each of the following using the name of the fallacy (underlined), and show why the move is fallacious. Answers are in the back of the book. 1. Teacher to student: If I let you break this rule, then I will have to let every student break this rule, and then I will have to let everyone break every rule, and then the next thing you know no rules at all will be obeyed. 2. You must follow all the rules. Either everyone follows all the rules or we shouldnt have any rules at all. 3. Who the hell are you to argue that women should stay home and look after their children in order to ensure that they have quality bonding? What a chauvinist pig! 4. Abortion should be legal. Women should have the right to decide what happens to their own bodies. 5. Just because. Is that what you are telling me, Joanthat abortion should be legal just because? That is kind of dumb, isnt it? 6. Raymond: I dont think condom machines should be put in school bathrooms. They promote promiscuity by sending the message that sex is cool. Marco: You are just saying that because nobody wants to have sex with you. What a loser! 7. What is taught at university should depend entirely on what students are interested in. After all, consuming knowledge is like consuming anything else in our society. The teacher is the seller and the student is the buyer. Buyers determine what they want to buy, so students should determine what they want to learn. 8. Teenager to father: You are telling me that I shouldnt be smoking pot because it may interfere with my focus at university! Why should I listen to you? You have a martini every night. 9. Foreign students should not have to pay higher fees than students who are citizens of the country do. All students should be treated equally. 10. It has always been our tradition to sacrifice three elephants per year to please the gods of the sky. Not allowing us to do this would be infringing on our cultural traditions.

SECTIon 1 LEARnIng ThE InTRICACIES of PRACTICAL REASonIng

101

EXErCISE 4C
Recognizing informal fallacies
respond to each of the following using the name of the fallacy (underlined), and show why the move is fallacious. Answers are in the back of the book. 1. Yes, of course, I drive fast. If I didnt drive fast, I would never get anywhere on time. 2. There is nothing wrong with sleeping around. Ask anyone my age. Everyone my age sleeps around. 3. The United States should destroy its nuclear weapons. It is immoral to stockpile them. Even Einstein, who can be considered the father of the nuclear age, argued against the use of nuclear weapons. 4. The only reason why the government doesnt ban tobacco altogether is because it gets so much money from taxing cigarettes. 5. I am not a racist. I just dont want my daughter to marry a black man. 6. Mother to teenage son: I dont believe you when you tell me that you were not with your friends when they broke into that store last night. Ever since you started hanging around with those kids you have been in constant trouble, to say nothing of the fact that you seem to have gotten into the habit of lying. So let me tell you, I dont believe you now. 7. Im never going to eat at Eds Shack again. We went there to have something to eat after the party last night and I have been sick ever since. 8. I am sick and tired of these high college fees. The government should make education free for students. Students have an inherent right to high-quality, no-cost education. 9. Did you hear that John and Jane are getting a divorce? Jane said that John is extremely abusive. John said that Jane is a nonstop nag. I guess both of them are pretty impossible to live with. 10. The only reason that critical thinking courses have not been adopted in every classroom in every school in this country is because the school system doesnt want kids to learn how to think.

EXErCISE 4D
Recognizing informal fallacies
respond to each of the following using the name of the fallacy (underlined), and show why the move is fallacious. Answers are in the back of the book. 1. All this chatter about global warming is nonsense. The only reason that those environmental extremists keep harping about global warming is because they are worried that if people dont cut down on the use of gas and oil, the prices of gas and oil will go through the roof, and they have an irrational fear that the big oil companies will make too much money and hence have too much power. 2. I have absolute proof that St. Laura keeps me safe. I was wearing my St. Laura medal when I was in a car crash last year, and I was the only one in the car who didnt get injured.

102 PART II PRACTICE


3. If our kids start listening to Marilyn Manson, theyll start dressing differently and hanging out with the wrong crowd. They will get themselves into drugs or violence, and soon enough they will start acting like freaks, just like Marilyn Manson. 4. People should not engage in sex unless they are in a committed relationship because sex is a serious matter and should be engaged in only by those who genuinely care for one another. 5. We either had to trust the word of a mad man and believe Hussein when he said that there were no weapons of mass destruction or engage in a preemptive strike in order to ensure the safety of the United States. That is why we attacked Iraq. 6. I am sick and tired of you two fighting. You are ten-year-olds, for heavens sake. No, I dont want to hear any lame excuses from either of you. You are both at fault. No television for a week for either of you. 7. I am so tired. What a boring day. I hardly did anything. It is not like I am lazy or anything, but the only real jobs to do were mucking out the stalls or cleaning tack. I hate mucking out stalls even more than I hate cleaning tack, so I kind of hung around and did nothing. 8. My name is Todd Bertuzzi, my team is the Vancouver Canucks, my number is 44, and my car is the Grand Panther. If you want to have the best car in town, take it from me. You cant do any better than the Grand Panther. 9. Mary: You should stop eating at KFC because the chickens that they use at KFC are farmed in the most appalling conditions. Supporting the continuation of the avoidable suffering of living things that have the capacity to feel pain is immoral. John: Why should I listen to you, you whining left-winger? What do you want me to eat, tofu burgers with lettuce? 10. I dont know why you are going on about how wrong it is to keep whales in captivity. After all, you have a dog and you keep it in captivity. It seems to me that as long as you are good to the animal, keeping it in captivity is perfectly acceptable.

5. What Kind of Argument Is It?


Now that you have become familiar with some of the more common informal fallacies that can sidetrack you from the get-go, the next move is to analyze the structure of reasoning more closely. Being able to reason gives reasoning beings the capacity to access many more truths than they otherwise could. Reasoning beings do not always have to literally bump up against a truth in order to know it. We can reason our way to truth. I can know, for instance, that you have a beating heart without ever meeting you. I know that you have a beating heart because I know that all humans who can read this text are alive, and all humans that are alive have beating hearts. In more technical terms, the claims that we reason our way to are referred to as conclusions; the reasons on which these conclusions are based are referred to as premises. Premises and conclusions together make up an argument. The point of an argument, in other words, is to support the truth of one claim or statement (the conclusion) by reference to at least two43 other claims or statements (referred to as premises or reasons). Keep in mind that claims do not change simply because they are called premises and conclusions. These statements simply adopt different names that serve as tags, so that we know how they are being used in an argument. Broadly speaking, arguments can be divided into two kinds, deductive and inductive. The most common inductive argument is the inductive generalization,44 whereby one makes an assumption about an entire class based on the characteristics

An argument consists of a conclusion supported by two or more premises.

SECTIon 1 LEARnIng ThE InTRICACIES of PRACTICAL REASonIng

103

of a sample. Thus, on perceiving that x number of robins have red breasts, you might induce that all robins have red breasts. The art of such induction is to ensure that your sample size is sufficiently large and sufficiently representative that your induction is warranted. Even if your sample is optimal, however, because the argument moves from some of the class to all of the class, you can never be absolutely sure that the sample accurately reflects the entire class. Thus, the definition of an inductive argument is, if the premises are true, the conclusion is always only probable. The art of induction is to maximize that probability. Induction is the most common argument used in the natural and social sciences. By contrast, the definition of a deductive argument is that if the premises are true, the conclusion must be trueat least according to the arguers intent (with the preceding qualifier being necessary in order to allow the distinction between valid and invalid deductive moves).45 The reason that the conclusion in a deductive argument must be true is because the argument moves from what is true of the whole class to what is true of part of the class. That is, the argument moves from all to some. If you say that intentionally deceiving another for ones own benefit is always wrong and John did that, you can correctly deduce that what John did was wrong. Arguments about valuesi.e., practical reasoningare ultimately based on deductive reasoning. Hence, if you want to be able to evaluate the supporting strength of the values that you presently hold, and if you want to be able to move values around in your head accurately and efficiently, you need to get comfortable with deductive arguments. GETTING COMFORTABLE WITH DEDUCTION The definition of a valid deductive argument lacks the qualifier of the arguers intent and therefore is simply this: If the premises are true, the conclusion must be true. Validity specifically refers to the logical relation of the premises to the conclusion in a deductive argument. If I argue that all mammals have lungs and that sharks are mammals, therefore sharks have lungs, my argument is valid even though one premise is false, because if all the premises were true, the conclusion would also necessarily be true (the definition of a valid deductive argument). It is important to remember this definition of validity because it is a necessary condition of gaining reasonable flexibility, specifically to be able to grant validity, without granting soundness. The definition of soundness is a valid argument whose premises are true. When evaluating deductive arguments, your first job is always to investigate validity, because if an argument is not valid, then you need not embark on the often harder and lengthier process of attempting to determine whether or not the premises are true. However, if an argument is valid, you should grant validity, without necessarily granting soundness. Thus, to the preceding argument that all mammals have lungs and that sharks are mammals, therefore sharks have lungs, you could respond, Your argument is certainly valid; however, I disagree with your second premise.

A valid deductive argument is one such that if the premises are true, then the conclusion must be true.

A sound argument is a valid argument whose premises are true.

DEfInITIonS
In(valid)deductive arguments,ifthepremisesaretrue,theconclusionmustbe true. Ininductive arguments,ifthepremisesaretrue,theconclusionisalwaysonly probable. Thereasonsforthisareasfollows: Adeductiveargumentmovesfromalltosome. Aninductiveargumentmovesfromsometoall.

104 PART II PRACTICE


EXErCISE 5a
Is it inductive (and hence strong or weak), or is it deductive (and hence valid or sound)?
Describe the following arguments. The answers to the first five arguments are in the back of the book. 1. All dogs are birds. Rover is a dog. Therefore, Rover is a bird. 2. All humans who can read are alive. All humans who are alive have beating hearts. You are reading this right now. Therefore, you have a beating heart. 3. All water is a chemical compound. All chemical compounds are made up of hydrogen and oxygen. Therefore, water is made up of hydrogen and oxygen. 4. Sixty percent of this sample of students has tried marijuana. This sample is representative of the student body as a whole. Therefore, 60 percent of the entire student body has tried marijuana. 5. All people who grow up in alcoholic households have a low tolerance for unstructured environments. John grew up in an alcoholic household. Therefore, John has a low tolerance for unstructured environments. 6. All mammals have wings. Bats are mammals. Therefore, bats have wings. 7. All of the men that I have been out with have been selfish. Those men are representative of all men. Therefore, all men are selfish.

EXErCISE 5B
The features of a deductive argument
answer the following questions. The answers to the first five questions are in the back of the book. 1. In a valid deductive argument, if the premises are true, what do you know about the conclusion? 2. In an inductive argument, if the premises are true, what do you know about the conclusion? 3. In a deductive argument, if the argument is valid, what do you know about the premises? 4. In a deductive argument, if the argument is sound, what do you know about the premises? 5. In a deductive argument, if the argument is valid, what do you know about the conclusion?

SECTIon 1 LEARnIng ThE InTRICACIES of PRACTICAL REASonIng

105

6. In a deductive argument, if the argument is sound, what do you know about the conclusion? 7. In a valid deductive argument, if the conclusion is true, what do you know about the premises? 8. In an inductive argument, if the conclusion is true, what do you know about the premises? 9. In a valid deductive argument, if the conclusion is false, what do you know about the premises? 10. In a valid deductive argument that has multiple premises, if all but one of the premises are true, what do you know about the conclusion?

6. Seeing the Whole Argument: A Valid Deductive Argument Is a Necessary Condition for Evaluating Reasons (Soundness)
Another name for premise is reason. Thus, a claim is called a reason when it provides grounds for believing another claim. That is, claim b becomes a reason when it provides grounds for believing claim a. However, claim b cannot work alone. Claim b can become a reason only when it is embedded in a valid deductive argument. Remember that an argument consists of two or more reasons (or premises) and a conclusion. The tricky part is that one (or more) of the premises of an argument may be hidden (or assumed). However, to evaluate how good any reason (or premise) is, you must evaluate the entire argument, including the hidden premises. Let us suppose that someone says that the reason s/he believes that violent pornography is unethical is because s/he believes that violent pornography promotes violence against women. To evaluate how good that reason is, you must first reconstruct the entire argument, including the hidden premise, so that the result is a valid deductive argument. The entire argument looks like this (P1 = major premise; P2 = minor premise; C = conclusion): (P1) All things that promote violence against women are unethical. (P2) Violent pornography promotes violence against women. (C) Therefore, violent pornography is unethical. This is a valid deductive argument. (Remember, the definition of a valid deductive argument is that if the premises are true, the conclusion must be true. Thus, the hidden premise that you come up with must result in an argument that conforms to this definition.) It is important to note again that for a claim to become a reason, not only must it be embedded in an argument; it must be embedded in a valid deductive argument. Thus, if an individual is claiming to give a reason for believing another claim, that individual has little control over the argument that implicitly accompanies his or her reasoning: The hidden premise must be such that the stated reason is embedded in a valid deductive argument because validity is a necessary condition of soundness. Knowing the hidden premise is particularly important in practical reasoning i.e., reasoning that guides action. Remember, in practical reasoning, what you are doing is pairing an imagined action with an imagined reinforcement or value. However, in order to add value to an act, you must borrow that value from somewhere else. Where you have borrowed it from becomes evident in the hidden premise. Thus, if you say (conclusion or thesis), buying or selling violent and degrading pornography is unethical because (reason) it promotes violence against and the degradation of women, what you are trying to do is pair the negative value of unethical with the action of buying and selling pornography. However, you need to know where the value of unethical comes from. In this particular instance, it was
A claim becomes a reason only when, in combination with another claim, it is purported to support a conclusion.

A reason is only as good as the entire argument in which it is embedded.

In practical reasoning, you are trying to attach a value to an act, a thing, or a situation. In order to do that, you must borrow the value from somewhere else. The major premise shows you where you got the value.

REASONING CHANGES BEHAVIOR BY MOVING VALUE

106

SECTIon 1 LEARnIng ThE InTRICACIES of PRACTICAL REASonIng

107

borrowed from the fact-value attachment that is reflected in the hidden premise, namely, all things46 that promote violence against and the degradation of woman are unethical. Thus, in practical reasoning, if you do not know the hidden premise, you cannot know where the value that you are trying to attach to an act came from, and hence you cannot judge whether the movement of value from its old attachment to the new suggested attachment is valid.

Arguing about How Classes of Things Relate


In order for you to see the hidden premise in your own and other peoples arguments, it may be helpful if you understand the underlying logic. What you are doing when you are providing reasons is making the claim that various classes fall within other classes. Let us take the preceding argument reworded as follows: (P1) All [things that promote violence against women] are things that are unethical. (P2) (Violent pornography) is [something that promotes violence against women]. (C) Therefore, (violent pornography) is something that is unethical. In the previous argument, the smallest class (S) is in round brackets, the largest class, Lthe value classis underlined, and the medium-sized class [M] is contained in square brackets. The conclusion makes the claim that the smallest class is contained within the largest class. The reasoning says this is so because the smallest class is contained in the medium-sized class. For this to be a good reason, obviously the hidden premise must make the claim that the medium-sized class is contained in the largest class. The reasoning, in other words, looks like this: (Class S) is contained in class L, because (class S) is contained in [class M]. In order for this reasoning to make sense, we must assume that the arguer is also making the claim that [class M] is contained in class L. In more formal terms, the argument looks like this: Premise 1: [M] is L. Premise 2: (S) is [M]. Conclusion: (S) is L. Without the hidden premise, the argument would look like this. (Class S) is contained in class L because (class S) is contained in [class M]. It does not make much sense, does it?

108 PART II PRACTICE


A TRICK foR fInDIng ThE hIDDEn PREMISE
Hereisatrickforfindingthehiddenpremise.Letstaketheargumentthat violentpornographyisunethicalbecauseitpromotesviolenceagainstwomen. Startwiththeconclusion(C)i.e.,startfromthebottom.Putroundbrackets aroundthetopicandunderlinethevalue. (C)(Violentpornography)isunethical. Nowstartwritingtheminorpremise(P2)bywritingoutthetopicasecondtime. (P2)(Violentpornography) (C)(Violentpornography)isunethical. Nowwriteoutthereason,theendof(P2),withsquarebracketsaroundthe reason. (P2)(Violentpornography)[promotesviolenceagainstwomen]. (C)(Violentpornography)isunethical. Nowwriteoutthemajorpremisebyputtingwhatisinthesquarebracketsfirst, followedbywhatisunderlined,precededbythewordallandanywordsthat youneedtomaketheclaimgrammaticallycorrect.Nowyouhaveyourhidden premise! (P1)All[thingsthatpromoteviolenceagainstwomen]arethingsthatare unethical. (P2)(Violentpornography)[promotesviolenceagainstwomen]. (C)(Violentpornography)isunethical. NOTE:SomestudentsfinditeasiertowriteoutP2first,thenwriteouttheconclusion,andthenmoveontoP1.Trybothmethods;thendowhatworksbest foryou.

Note that the topic of the argument (i.e., the small class) drops out in the major premise. This is so because otherwise you would be begging the question. In practical reasoning, your conclusion adds a value to the small class. If you already assumed that the value was added to the small class in the major premise, you would be assuming what you are trying to prove, which is begging the question. PRACTICAL REASONING IS LIKE PAVLOVIAN CONDITIONING IN YOUR HEAD It may be interesting for you to note that when you engage in practical reasoning, you are actually engaged in a kind of Pavlovian conditioning of yourself. What you are doing is trying to borrow value from a stable fact-value combination and then transfer that value to another fact or state of affairs by showing that you believe that the two facts or states of affairs are linked (associated), so that value can move from one to the other. Suppose that you argue that driving more than ten miles per hour over the speed limit is wrong because it seriously jeopardizes the lives of others. What you have done here is to take the value of wrong from a stable fact-value combinationi.e., that [seriously jeopardizing the lives of others] is wrong (P1)and transfer the value to the fact of (driving more than ten miles per hour over the speed limit) by making the claim that the two facts, namely (driving more than ten miles per hour over the speed limit) and [seriously jeopardizing the lives of others] are inevitably linked (P2). The argument looks like this:

SECTIon 1 LEARnIng ThE InTRICACIES of PRACTICAL REASonIng

109

(P1) All [acts that seriously jeopardize the lives of others] are acts that are wrong. (P2) (Driving more than ten miles per hour over the speed limit) is [an act that seriously jeopardizes the lives of others.] (C) Therefore, (driving more than ten miles per hour over the speed limit) is an act that is wrong. Or in more abstract terms, it looks like this: All [states of affairs 1] have value 1. All (states of affairs 2) are linked to [states of affairs 1]. Therefore, (all states of affairs 2) have value 1. As will become evident in the next section, the only way to falsify the preceding reasoning is either by showing (1) that it is not the case that [all states of affairs 1] have value 1 or (2) by showing that (all states of affairs 2) are not necessarily linked to [states of affairs 1]. Note the similarity to Pavlovian conditioning: All [serving of food] elicits salivation from the dog. All (ringing bells) are linked to [the serving of food]. Therefore, all (ringing bells) elicit salivation from the dog. In the preceding dog-conditioning case, what has been done in reality is exactly the same as what happens imaginatively in linguistically mediated practical reasoning. In the dog-conditioning case, the positive value that is connected to the fact of serving food has been transferred to the new fact, namely, the ringing of the bell, by literal association, rather than by imaginative association, as in the case of practical reasoning. Interestingly, the process required to decondition the dog is the same as the process required to falsify practical reasoning. You can either (1) break the link between [states of affairs 1] and value 1 (i.e., serve food that does not elicit salivation) or (2) sever the link between (states of affairs 1) and [states of affairs 2] (i.e., ring the bell at times that are not associated with food). First, it is useful to remember the similarities between practical reasoning and conditioning an animal, for several reasons. The similarity will help dissipate the mysteriousness of practical reasoning, which otherwise may seem utterly disconnected from your basic animal nature that is a product of your evolutionary history. Practical reasoning is merely linguistically mediated Pavlovian conditioning in the emergent imaginary space created by linguistic self-consciousness. Second, recognizing the Pavlovian nature of practical reasoning will help you remember the importance of the hidden premise. Third, recognizing the Pavlovian nature of practical reasoning should help you understand that whether or not you like it, if you are claiming that value 1 gets transferred to fact 2 because of its association with fact 1, you must accept that anything else associated with fact 1 will likewise be infected with value 1. This is another way of saying that the precise characteristic of the smallest class, referred to in logic as the minor term47(i.e., the ringing of the bell), is relatively unimportant. Playing a flute or a violin or singing the Lords Prayer would be just as effective in eliciting salivation. Similarly, if anything jeopardizes the lives of othersnot just driving ten miles per hour over the speed limitit too is wrong. (See the comics entitled I Am Talking about Rapists, Not Enron Executives and I Am Talking about Homosexuals, Not Glasses on pages 116 and 118.)

BREAKING THE MAJOR PREMISE

110

SECTIon 1 LEARnIng ThE InTRICACIES of PRACTICAL REASonIng

111

EXErCISE 6a
Finding the hidden premise
Write out the entire argument for each of the following. NOTE: Always start at the bottom and work up, or start with P2 and then zigzag up. (P1) All [ ] are ______. (P2) ( ) are [ ]. (C) ( ) are ______. Answers are in the back of the book. 1. Abortion is wrong because it takes a life. 2. Outlawing abortion is wrong because it denies women the right to choose. 3. Nudity should be outlawed because seeing naked bodies is offensive to many. 4. Because we cannot guarantee that we will not execute an innocent man, capital punishment should be outlawed. 5. Rape forever harms a woman. For that reason, rapists should be castrated.
InDICAToR WoRDS
Indicator Words for Reasons: Because Since As Inviewof Indicator Words for Conclusions: Therefore Then Thus Forthatreason Hence

112 PART II PRACTICE


EXErCISE 6B
Finding the hidden premise
Write out the entire argument for each of the following. NOTE: Always start at the bottom and work up, or start with P2 and then zigzag up. (P1) All [ ] are ______. (P2) ( ) are [ ]. (C) ( ) are ______. 1. Because clear-cutting forever destroys the integrity of an ecosystem, it should be banned. 2. Adult video pornography should not be banned, as any kind of censorship always deteriorates into a situation in which one group of people think they have the wisdom to decide what another group of people should and should not see and read. 3. An enormous amount of evidence suggests that marine mammals and great apes are self-conscious. Therefore, marine mammals and great apes are persons. 4. It is not unethical to kill a cow, because a cow is not self-conscious. 5. We kill cows but not humans because the latter are self-conscious but the former are not. Since whales are self-conscious, killing a whale is equivalent to killing a human. Its murder. 6. It is unethical to make a cow suffer, because cows, like any other animal, have the capacity to feel pain. 7. Since it promotes the objectification of women, women should not buy fashion magazines. 8. We should not give money to those who beg because, in doing so, we promote harmful drug addictions. 9. We should not condemn the relatives of the victims of the Oklahoma bombing for wishing to view the execution of Timothy McVeigh. It may be the only thing that will help them heal. 10. Human beings have the right to rule all animals who are below them on the phylogenetic scale because humans are smarter and stronger.

7. Evaluating Reasons or Soundness (Local Sufficiency)


After recognizing a claim as a reason by placing it in a valid argument, the next step is to decide whether or not it is a candidate for being a good reason. You need to remember that the process of evaluating reasons is primarily negative. That is, the aim in evaluating reasons is not, strictly speaking, to try to figure out whether or not the stated reason is a good one but rather whether or not it is a poor one. This in no way indicates a negative attitude of the listener. This is a function of the entire truth-seeking processi.e., that it is one of falsification, not verification. As individuals with finite minds, we can only eliminate illegitimate contenders to truth. We can never attest to the absolute truth of any claim. Since the goal is falsification, or trying to discover weaknesses in an argument, the next step in evaluating reasons (after recognizing a claim as a reason by placing it in a valid argument form) is to evaluate the soundness of the entire argument. A sound argument is a valid argument whose premises are true (i.e., not obviously false).

SECTIon 1 LEARnIng ThE InTRICACIES of PRACTICAL REASonIng

113

Let us say that someone says that homosexuality is immoral because it is unnatural. The whole argument would look like this: (P1) All [unnatural practices] are practices that are immoral. (P2) (Homosexuality) is [an unnatural practice]. (C) Therefore, (homosexuality) is a practice that is immoral. Now, in order to decide whether the reason given is a good one, we must decide if the entire argument is sound. Remember, an argument is only as strong as its weakest link; if any of the premises is weak, the entire reasoning process is weak.

A reason is only as good as the entire argument in which it is embedded.

Testing for Local Sufficiency (Soundness) through Counterexamples


Once you have before your minds eye the entire argument, you should check the strength of each premise by trying to imagine a counterexample. In the preceding argument, it is clear that one of the premises is extremely weak, namely, the premise that all unnatural practices are immoral. After all, chewing gum is unnatural, as is wearing clothes and using any kind of machinery, but few would consider these kinds of activities immoral. However, the argument stands or falls altogether. Thus, to offer as a reason in support of the claim that homosexuality is immoral that it is unnatural is equally weak. If you are dealing with someone elses argument, your job is to show wherein the fault lies. In this particular instance, your first response should be Are you telling me that you believe that everything that is unnatural is immoral, like chewing gum? If the person is not telling you that, then that person is withdrawing the reason, and the claim, which it is purported to support, along with it. Providing feedback with regard to both the hidden premise and a plausible counterexample allows you, and those with whom you communicate, to dispose of ineffective reasoning in an efficient manner. It also, interestingly, allows you to explore topics with ever-greater degrees of precision so that you can see whether the path on which you are traveling has the possibility of bearing fruit of any kind. That is, if you find yourself uncomfortable with the implied major premise (e.g., everything that is unnatural is immoral), you can try out a more confined class (e.g., all sexual acts that are unnatural are immoral) to see whether or not this results in an argument that you would accept as sound. In this case, you may be happy to throw out the entire line of reasoning, but in others, the process of trying to find the right major premise that captures just the right kinds of cases may prove to be extremely fruitful. You might decide, for example, that murder is immoral not because all killing is immoral (due to the counterexample of mosquitoes), nor because all killing of human life is immoral (due to the counterexample of war and self-defense), nor because all killing of innocent human life is immoral (due to the counterexample of euthanasia), nor because all nonconsensual killing of innocent human life is immoral (due to the counterexample of abortion), but because all nonconsensual killing of innocent humans who are self-conscious is immoral. The preceding example is particularly helpful in demonstrating the imaginative procedure that you should undertake when you are seeking to find the right hidden premise. As has already been stated, what you are doing when you are attempting to locate the right hidden premise is trying to discover which medium-sized fact class (which is value laden) the smaller fact class in question most appropriately fits. In the preceding, you are trying to decide whether the act of murder most aptly falls within the class of killing all human life, the class of killing innocent human life, and so on. Keep in mind that all deductive arguments in practical reasoning must have premises with the quantifier all. Thus, an individual making the preceding argument that homosexuality is immoral because it is unnatural might try to hold on to her or his reasoning while distancing her- or himself from the implicit major premise. That is,

THE WONDERS OF FAULT FINDING

114

SECTIon 1 LEARnIng ThE InTRICACIES of PRACTICAL REASonIng

115

s/he might respond to your inquiry about whether s/he believes that all things that are unnatural are immoral by saying that s/he does not believe that this is the case. S/he believes, rather, that some things that are unnatural are immoral, and homosexuality is one of them. However, if s/he makes this move (i.e., from all to some), s/he annihilates the validity of the implicit argument because it is no longer the case that if the premises are true, the conclusion must be true. Thus, the onus now rests with her or him to construct a new valid argument. That is, if the arguer assumes that only some things that are unnatural are immoral, the fact that homosexuality is unnatural no longer serves as a reason for saying that homosexuality is immoral (see the sequence that follows). A valid argument is needed in order for a reason to be a reason. (P1) Some [things that are unnatural] are things that are immoral. (P2) (Homosexuality) is [something that is unnatural]. (C) Therefore, (homosexuality) is something that is immoral. This is not a valid argument because of the quantifier some in the first premise.

I AM TALKING ABOUT RAPISTS, NOT ENRON EXECUTIVES

116

117

I AM TALKING ABOUT HOMOSEXUALS, NOT GLASSES

118

SECTIon 1 LEARnIng ThE InTRICACIES of PRACTICAL REASonIng

119

Local Sufficiency and Theories of Truth in Practical Reasoning


Among the theories of truth that abound in philosophy, the two most common are the correspondence theory of truth and the coherence theory of truth: The first looks at the correspondence between claims and the world, whereas the second looks at the coherence of any body of beliefs. (Note: Although they are generally called theories of truth, it would be more accurate to call them theories of falsityi.e., theories about what would count as making a claim false.) When engaged in practical reasoning, it may be helpful to keep in mind that premise 1 and the conclusion focus on coherence. What you are trying to do in practical reasoning is to show your listener that s/he cannot coherently believe premise 1 without also believing the conclusione.g., s/he cannot coherently believe that it is wrong for women to be subjected to violent and degrading behavior and, at the same time, reject the thesis that violent and degrading pornography would be illegal. Understanding that coherence is the focus of the relationship between premise 1 and the conclusion will serve to remind you that you must offer your listener a major premise that s/he already accepts if you are going to reel him or her in. Understanding that coherence is the focus of the relationship between premise 1 and the conclusion will also serve to remind you that if that coherence shifts to virtual identity, you have begged the question (i.e., your conclusion is buried premise 1). Premise 2, by contrast, focuses on correspondencee.g., whether pornography in fact encourages violence against women or whether homosexuality in fact is something that rarely occurs in nature. Since premise 2 focuses on the relation between facts, it is crucial to keep in mind that premise 2 must be robustly falsifiable (i.e., you must be able to state what would count, in fact, as showing this claim to be false). Thus, in the following argument, P2 is inherently unfalsifiable because it is true by definition. The argument thus also begs the question. (See the comic entitled What Would Count as Falsifying Your Belief? on page 120.) (P1) All [acts that use other peoples tax money to fund artificial insemination] are acts that should not be done. (P2) (Covering in vitro fertilization under Medicare) is [an act that uses other peoples tax money to fund artificial insemination]. (C) (Covering in vitro fertilization under Medicare) is an act that should not be done. Remembering the importance of the falsifiablity of the second premise will be particularly helpful when you are faced with apparently intractable racism. Thus, if someone says that all Jews (or Palestinians or Serbs or Croats) should be shot because their very existence makes life miserable, you need to point out that this attitude cannot be justified unless it is founded on truth and that in order to estimate the truth of this claim, one would have to be prepared to articulate what in fact would count as falsifying (not verifying) the claim that their existence make life miserable. If one is not so prepared, then one is merely creating ones own definitions and hence is out of the truth game altogether.

Local Sufficiency Does Not Guarantee Global Sufficiency


It is important to keep in mind that the present focus is on local sufficiency, not global sufficiency. An attempt to establish local sufficiency is an attempt to establish the adequacy of one side of an argument only. An attempt to establish global sufficiency is an attempt to evaluate the adequacy of any argument relative to competing arguments. A necessary condition of estimating the global sufficiency of any claim is to first estimate the local sufficiency of contending options.

To evaluate a reason is to attempt to establish local sufficiency. Establishing local sufficiency is a necessary but not a sufficient condition for establishing global sufficiency.

WHAT WOULD COUNT AS FALSIFYING YOUR BELIEF?

120

SECTIon 1 LEARnIng ThE InTRICACIES of PRACTICAL REASonIng

121

EXErCISE 7a
Evaluating reasons: Finding the hidden premise and looking for counterexamples
Write out the entire argument form for each of the arguments presented. after each argument, rate each premise on a five-point scale, according to the strength and plausibility of a potential counterexample. remember that the lowest value received by any premise is the value of the entire reasoning process. (++) very good (note: a premise is rated ++ unless you can think of a counterexample) (+) good () so-so () poor () very poor (the counterexample shows that the reason is utterly inadequate)

EXAMPLE
Homosexuality Is Immoral because It Is Unnatural.
the argument form is this:

(P1)All[thingsthatareunnatural]arethingsthatareimmoral. (P2)(Homosexuality)is[somethingthatisunnatural]. (C)Therefore(homosexuality)issomethingthatisimmoral.


premise 1 counterexample:Chewinggum,wearingclothes,usinganykindof machinery.

Rating:
premise 2 counterexample:Thereisempiricalevidencetosuggestthatmanyspeciespracticehomosexuality.

Rating: (Note:Iftheultimaterejectionofthisargumentdependedonthepoorrating ofpremise2,specificreferencetoempiricalfactswouldbeneeded.However,sincethereasoningcanbedismissedonthepoorratingofpremise1, empiricalreferencingisnotneeded.) Overallrating: (Rememberthattheoverallratingcanonlybeasstrongasitsweakestlink)


question:Areyoutellingmethatallthingsthatareunnaturalareimmoral(e.g.,

wearingglasses)?

122 PART II PRACTICE


The Form of Your Answers Should Look Like This:
argument form

(P1)All[]are______. (P2)()are[]. (C)()are______.


premise

1 counterexample 2 counterexample

Rating:
premise

Rating: Overallrating:
question

1. 2.

3.

4. 5. 6. 7.

8.

9. 10.

Answers are in the back of the book. Our city should provide paved bicycle lanes on most of our busy streets because it would increase the safety of cyclists and make driving easier for motorists. Using animals in medical research increases the chances of finding cures for many of the deadly diseases from which many humans suffer. For that reason, it is not unethical and should be continued. An individual who is significantly overweight and who has no underlying endocrine problem should be ashamed of his or her condition because it indicates poor self-control. If you see a friend doing something like cheating on an exam, you should not report it because doing so would threaten your friendship with that person. On a first date, both individuals should insist on paying half, as a first date should be considered a date between friends. Parents should not consider the use of physical punishment unethical, as physical punishment can be very effective in changing a childs behavior. Since it will clearly send the message to their children that the use of violence is an acceptable way to solve problems, parents should consider the use of physical punishment unethical. Relationships are built on trust, and that trust should never be broken. For that reason, if two people are in a committed monogamous relationship and one cheats on the other, the non-cheating partner should end the relationship. It is perfectly justified to buy really extravagantly expensive clothing because all you are doing is buying what you really want with your own money. As far as possible, when referring to others, one should endeavor to use politically correct language, since many find the use of politically incorrect language (e.g., using the term men when referring to both men and women, or using the term nigger when referring to African Americans) personally insulting.

THE POSITIVITY OF NEGATIVITY

123

ME ONLY BECAUSE OF WE

124

SECTIon 1 LEARnIng ThE InTRICACIES of PRACTICAL REASonIng

125

EXErCISE 7B
Evaluating reasons: Finding the hidden premise and looking for counterexamples
Write out the entire argument form for each of the arguments presented. after each argument, rate each premise on a five-point scale, according to the strength and plausibility of a potential counterexample. remember that the lowest value received by any premise is the value of the entire reasoning process 1. All college students should be required to take a critical thinking course because it improves the way students think. 2. All college students should be required to take a critical thinking course because, without it, students rarely or never learn how to think well. 3. Since having children is one of the greatest joys that life has to offer, Chinas one child per family law should be abolished. 4. If someone does not attempt to eat organic vegetables, s/he cannot claim to be concerned about the environment, because eating nonorganic vegetables promotes an industry that uses large amounts of environmentally destructive pesticides. 5. Cosmetic surgerye.g., to change ones nose or breastsis immoral because we should all consider whatever nature gave us as beautiful. 6. In vitro fertilization should not be covered under Medicare because artificial reproduction should not be funded with other peoples tax money. 7. Prostitution is the oldest profession in the world; it is never going to go away. We might as well legalize it. 8. Books about same-sex couples should be part of the elementary school curriculum. After all, same-sex couples exist in real life. Children might as well hear about them from people they trust. 9. If homosexuality is against their religious beliefs, teachers should not be required to teach about same-sex couples even if the topic is part of the curriculum. 10. Genetic testing should be allowed only when there are reasons to believe that the fetus is suffering from a severe highly debilitating disorder, such as TaySachs disease, because genetic testing should be used only to prevent pain and suffering; it should not be used to create the ideal child. NOTE: For added practice, evaluate the argument forms found in the answers to Exercise 6A. HOW THREE NEGATIVES CAN MAKE ONE POSITIVE: A SHOTGUN ARGUMENT Being able to test the adequacy of an argument that contains only one reason is fairly simple. One finds the hidden premise and then tries to imagine whether or not one can find a convincing counterexample to either of the premises. If one can find a convincing opposition, then the reason is not a good one. Being able to test the adequacy of an argument that contains multiple linked reasonsoften referred to as a shotgun argumentis more complex. Let us suppose that you are presented with the following argument: Drugs should be legalized because legalization will decrease crime and increase government revenues through taxation, and it will be beneficial to the health of many addicts. The argument looks like a combination of three arguments with three different hidden premises: 1. All acts that decrease crimes should be done. Legalizing drugs will decrease crime. Therefore, legalizing drugs should be done.

126 PART II PRACTICE


2. All acts that increase government revenues should be done. Legalizing drugs will increase government revenues. Therefore, legalizing drugs should be done. 3. All acts that are beneficial to the health of many addicts should be done. Legalizing drugs will be beneficial to the health of many addicts. Therefore, legalizing drugs should be done. If you examine the preceding three arguments, you will see that it is fairly easy to think up a counterexample to each of the major premises. In the first case, boiling people in hot oil or living under martial law would probably decrease crime. In the second example, increasing taxes by a huge margin or selling off government-owned land would increase government revenues. In the third example, sending people addicted to drugs to Club Med for as long as it suited them presumably would be beneficial to their health. It would appear, then, that each reason offered on its own is a poor one. How, then, can three poor reasons combine to make a good reason? The answer lies in the fact that when someone presents a shotgun argument, s/he is saying that something is the case not because it lies in three separate classes but because it lies in three overlapping classes. That is, a reason becomes good because it falls within several classes at the same time. Thus, a shotgun argument should not be analyzed as separate arguments but as one. Thus, the preceding argument should be analyzed as follows: 1. All acts that decrease crime and increase government revenues and are beneficial to the health of addicts are acts that should be done. 2. Legalizing drugs is an act that would decrease crime and increase government revenues and would be beneficial to the health of addicts. 3. Legalizing drugs should be done. The fact that shotgun arguments should be analyzed as a unit rather than severally becomes evident if one looks at an argument about matters of fact. Let us take the following: He died because he was hemophiliac and he suffered a severe cut and he had no access to medical care. In the hemophiliac example, if you try to analyze each of the reasons separately, it is clear that none carries much weight. People do not die simply because they belong to the class of hemophiliacs, or because they belong to the class of people who suffer severe cuts, or because they have no access to medical care. People die when they belong to all three classes at the same time. So with regard to the falsification of the major premise of a shotgun argument, one would need to show, in this example, that there is at least one instance in which a hemophiliac had cut herself severely and had no access to medial care but did not die. The major premise of a shotgun argument is usually strong because of the combining reasons; however, the minor premise, by contrast, is proportionately potentially weak because of the combination. That is, the minor premise says of the topic in question that it is x and y and z. Thus, one need only falsify one of those claims to falsify the combination. In the preceding, for example, one need only show that the individual was not a hemophiliac, or that she was not cut severely, or that she had access to good medical care, to blow the whole thing out of the water. The moral of the story, in other words, is that compared to a simple argument, a shotgun argument usually has a major premise that is stronger but a minor premise that is weaker.

In a shotgun argument, the strength of the major premise is more than the sum of its parts. In a shotgun argument, the strength of the minor premise is only as good as its weakest link.

SECTIon 1 LEARnIng ThE InTRICACIES of PRACTICAL REASonIng

127

EXErCISE 7C
Shotgun arguments
Write out the entire argument form for each of the arguments presented. after each argument, rate each premise on a five-point scale, according to the strength and plausibility of a potential counterexample. remember that the lowest value received by any premise is the value of the entire reasoning process. The answers to the first two are in the back of the book. 1. Parents should encourage their children to play computer games because computer games help develop eye-hand coordination and nurture memory skills. They also ensure that children become comfortable with the technology of the future. 2. Animals should not be used in scientific research because, like humans, animals can feel pain. In addition, a lot of the knowledge acquired from research done on animals cannot be safely transferred to humans. And in any case, we are at a point of diminishing returns in medicine: If we find a cure for one disease, we will simply die of another. 3. You should exercise at least three times per week because you will lose weight, it will get your cardiovascular system into shape, and it will increase your ascorbic acid levels. 4. A woman should never fake an orgasm because doing so perpetuates the situation and virtually ensures she will never be sexually satisfied. Not only that; she belittles herself by refusing to muster up the courage to be honest. She also dishonors her mate by assuming he is so caught up in his macho image that he would rather live in a false fantasy than in honest reality. 5. Since people often swear when they cannot think of what else to say, swearing is a sign of intellectual poverty. In addition, since swearing conjures up less than aesthetically pleasing images, it is a sign of bad taste. It seems to me, therefore, that as far as possible, one should not swear.

8. Evaluating the Local Sufficiency of Your Own and Your Oppositions Positions
With your newly acquired skill for establishing local sufficiency, you are now in a position to test the soundness of your own positions, as well as the positions of those with whom you disagree. Keep in mind that you are not yet in a position to establish the global sufficiency of your reasoning. Remember that global sufficiency requires first that one establish the local sufficiency of all the competing contenders for the truth; only thereafter is one is a position to pick the least weak of all available options. However, a more efficient way of establishing the global sufficiency is simply to imagine the two extremesi.e., the local sufficiency of the strongest reason for a position and the local sufficiency of the strongest reason against that position. That is what you are asked to do here: to establish the local sufficiency of the extremes both for and against a position, referred to as creating parallel arguments.
Global sufficiency first requires that one establish local sufficiency of all the competing contenders.

GETTING TO GLOBAL SUFFICIENCY BY TESTING LOCAL SUFFICIENCY THROUGH COUNTEREXAMPLES

128

SECTIon 1 LEARnIng ThE InTRICACIES of PRACTICAL REASonIng

129

EXErCISE 8a
Parallel arguments using one of your own thesis statements
take one of your thesis statements and construct an argument in its favor by articulating a strong reason for supporting it, find the hidden premise, and then test the soundness of your own argument by subjecting the two premises to as many counterexamples as you can. Obviously, if the rating for your argument is negative, you need to go back to the drawing board to try to think of a more robust reason. On the other hand, if the rating is consistently ++, that may indicate that you are not trying hard enough to find a counterexample. Now do the same for your opposition. the thesis statement for your opposition is usually the same as your support except that it contains a not. NOTE: Since this serves as a template for your written mini-essay, you may omit the ending question, whose function is largely for oral discourse.
EXAMPLE
Thefollowingisanexampleofparallelargumentsforthefirstofthesample mini-essayspresentedinAppendixIII.
thesis

Parallel arguments can be defined as establishing the local sufficiency (i.e., soundness) of the strongest arguments both for and against a position.

AnAIDSmemorialwallshouldnotbeconstructedinStanleyPark. (support) (P1)All[thingsthatwilldolittletostopthespreadofAIDS,destroytheoriginalpurposeofStanleyPark,whichisescapefromreality,andopentheway forotherstowantothermemorials]arethingsthatshouldnotbepursued. (P2)(ConstructinganAIDSmemorialwallinStanleyPark)is[somethingthat willdolittletostopthespreadofAIDS,willdestroytheoriginalpurposeof StanleyPark,whichisescapefromreality,andwillopenthewayforothers towantothermemorials]. (C)(ConstructinganAIDSmemorialwallinStanleyPark)issomethingthat shouldnotbepursued.
premise

1 counterexample

Rating:++ 2 counterexample Itwillnotcompletelydestroythepark,asthememorialisnotthatbig.Also, StanleyParkisprettybig;whynothaveothermemorials?


premise

Rating:+ Overallrating:+ *
counterthesis

AnAIDSmemorialwallshouldbeconstructedinStanleyPark. (opposition) (P1)All[thingsthatincreaseourawarenessofadeadlydiseasewhileatthe sametimeshowingoursympathyforthoseinourmidstwhohavedied]are thingsthatshouldbepursued. (P2)(ConstructinganAIDSmemorialinStanleyPark)is[somethingthatwill increaseourawarenessofadeadlydiseasewhileatthesametimeshowingour sympathyforthoseinourmidstwhohavedied]. (C)(ConstructinganAIDSmemorialinStanleyPark)issomethingthatshouldbe pursued.

130 PART II PRACTICE


1 counterexample Hugesignsoneverybuildinginthedowntowncorewouldincreaseour awareness.
premise

Rating:+
premise

Rating:++ Overallrating:+

Note that both the support and the opposition receive an overall rating of +, which shows this is not a strawperson argument. A strawperson argument is one that appears to be strong but is, in fact, a cheat because the opposition that is presented is illegitimately weak. If the ideal is truth, the only way to that goal is by testing your claim against the strongest possible opposition. Note also that since both the support and the opposition are strong, the outcome of this argument lies in the response to the opposition (when you write your miniessay), which decreases its strength. The fact that the opposition is somewhat easier to respond to (i.e., has a more prominent counterexample) supports the truth of the original claim.
ChECKLIST foR PARALLEL ARgUMEnTS
Thewholetopicshouldbecontainedintheroundbrackets. Allfourroundbracketsshouldbethesame. Allfouritemsthatareunderlinedshouldrefertoavalue. llfouritemsthatareunderlinedshouldbethesameexceptfortheword A notononeside. Allsixsentencesshouldbegrammaticallycorrectandperfectlyintelligible. W hatiscontainedinthesquarebracketsononesideshouldbeentirelydifferentfromwhatiscontainedinthesquarebracketsontheother. remise2,inboththesupportandopposition,shouldberobustlyfalsifiable; P otherwiseitisbeggingthequestion. remise1andtheconclusion,inboththesupportandopposition,shouldnot P bevirtuallyidentical;otherwiseitisbeggingthequestion. hatiscontainedinsquarebracketsshouldseemsufficienttoconvinceanyW oneoftheconclusion. fyourreasoningdoesnotrateontheplussideforeitheryoursupportoropI position,eitherstrengthenyourreasoningorthrowitout. Themorelethalthecounterexample,thelowertherating.

SECTIon 1 LEARnIng ThE InTRICACIES of PRACTICAL REASonIng

131

9. Evaluating the Global Sufficiency of Your Own Position EXErCISE 9a


Writing your mini-essays
Once you have successfully produced a set of strong parallel arguments using one of your thesis statements, you are in a position to engage in a bias-neutralization process by writing multiple five-part mini-essays. Your five-part mini-essay should consist of the following: 1. a precise contentious thesis statement Remember that whether your thesis statement remains as originally stated or whether you instead need to defend its contradictory depends on whether the counterexample to your opposition does more harm than the counterexample to your support. In other words, you need to go all the way to your response before you can determine whether, in the final analysis, your original thesis statement or its contradictory is the least weak position. 2. Support The reasoning of your support has already been captured in your parallel argument. 3. Opposition The reasoning of your opposition has already been captured in your parallel argument. 4. response This is the crucial part of the argument. This is where you must show that the weakness of your opposition results in the win for your support. If the reasoning is strong in your parallel arguments, often the suggested counterexample can be incorporated in your response. (Note: This does not work if your original reasoning is weak.) Remember that your response only talks to the opposition; it does not reiterate or add anything new to the support. 5. Conclusion Since these are the last words that your reader will remember, they should be memorable. They should encapsulate your argument by offering up the principle of action that you have learned in your voyage through this bias-neutralization process. NOTE: Section 2 of Part II contains a more thorough explanation about writing a mini-essay. NOTE: Appendix II contains ten mini-essays with their corresponding evaluation. Use these to enhance your capacity to judge the good from the not so good. NOTE: Appendix III contains a number of good mini-essays.

132 PART II PRACTICE


A SET OF PARALLEL ARGUMENTS WITH MATCHING MINI-ESSAY A SET of PARALLEL ARgUMEnTS
Example 1
thesis

BackinIran,Iusedtogotoaprivateschool.Once,inclass,myteachercalled peoplefromreligionsotherthanIslamunaware.AsaZoroastrian,Ihadthe opportunitytoargue,tostanduptohim,oreventoleave.HoweverIstayedin classquietly.IusedtobelievethatIwaswrongnottohavespokenup;nowI donot.


support

(P1)All[instancesinwhichareactioncouldgetsomeoneinbigtrouble, includingphysicalharm]areinstancesinwhichsomeoneisjustifiedinkeepingquiet. (P2)(Myteacherscallingpeopleofmyreligionunaware)is[aninstancein whichareactioncouldgetmeinbigtrouble,includingphysicalharm]. (C)(Myteacherscallingpeopleofmyreligionunaware)isaninstance whenIwasjustifiedinkeepingquiet. 1 counterexample Whatifeveryonedidthat?Whowouldstandupagainstthetyrants?


premise

Rating:+
premise

2 counterexample

Rating:++ Overallrating:+ *
opposition

(P1)All[instancesinwhichanindividualcanbearolemodelforintegrityand couragebystandingupagainstsomeonewhoinsultedandridiculedsome ofmymostcherishedbeliefs]areinstancesinwhichthatindividualisnot justifiedinkeepingquiet. (P1)(Myteacherscallingpeopleofmyreligionunaware)is[aninstancein whichIcouldhavebeenarolemodelforintegrityandcouragebystanding upagainstsomeonewhoinsultedandridiculedsomeofmymostcherished beliefs]. (C)(Myteacherscallingpeopleofmyreligionunaware)isaninstance whenIwasnotjustifiedinkeepingquiet. 1 counterexample Supposingyourbosssayssomethingthatyoudonotapproveof,butspeaking outmightgetyoufired?Supposingthatspeakingoutmightputyouoryourfamilyingreatdanger?Supposingyouwerekilled?
premise

Rating:+
premise

2 counterexample

Rating:++ Overallrating+

SECTIon 1 LEARnIng ThE InTRICACIES of PRACTICAL REASonIng

133

A MInI-ESSAY
Example 1
thesis

BackinIran,Iusedtogotoaprivateschool.Once,inclass,myteachercalled peoplefromreligionsotherthanIslamunaware.AsaZoroastrian,Ihadthe opportunitytoargue,tostanduptohim,oreventoleave.However,Istayedin classandkeptquiet.IusedtobelievethatIwaswrongnottohavespokenup. NowIdonot.


support

Atthattime,althoughspeakingoutinIrandidnotinevitablyresultinbeing harmed,therewasnonethelessadistinctpossibilitythatIcouldhavesufferedsevererepercussions,eitherintheimmediatefutureorinthelongterm. Besides,theclasswasoverwhelminglyMuslim,somytinyvoicewouldhave hadverylittleimpactinservingasarolemodeltootherswhomighthavefelt marginalized.


opposition

Thereareonlysomanyopportunitiesinalifetimeforonetodemonstrate genuinecouragebystandingupforonesbeliefsinthefaceofopposition.Since thisincidentoccurredinaprivateschoolandthechancesofrealharmwerenot significant,Ishouldhavebeenpreparedtoactasarolemodeltootherstudents byshowingthatIhadtheintegritytostandupforwhatIbelievedin.


response

Therearetimeswhenoneneedstoretreatinordertolivetofightanotherday. Althoughformyteachertocallmembersofmyreligionunawarewasinsulting,itwasnotthesortofverbalonslaughtthatwouldhavemany,orevenany, socialrepercussions.Indeed,itseemedtomethathiscommentsfellondeaf ears.Weallneedtolearnhowtopickourbattles.Thisignorant,biasedremark wasnotworthputtingmyacademiccareerinjeopardy,letalonemysafetyand thepotentialsafetyofmyfamilyeveniftheriskwasrelativelysmall.


conclusion

Probablynotadaygoesbywhenwedonothearcommentsthatwefindabhorrent.Ifwerosetotheoccasionanddidbattlewitheverysmall-minded,biased personweencountered,wewouldcollapseinaheapofimpotentexhaustion. Althoughitisabsolutelyimperativethatwestandupforwhatwebelieve,itis equallyimperativethatweadoptastrategythatmightbesuccessfulinactually givinglifetoourbeliefs.Whenconfrontedwithmindsencasedinthehardamber ofbias,andwhenouronlyweaponsarewordsthatwilldolittleotherthanimpressthespeaker,silencemaybenotonlyjustifiedbut,indeed,thebestpolicy.

134 PART II PRACTICE


A SET OF PARALLEL ARGUMENTS WITH MATCHING MINI-ESSAY

A SET of PARALLEL ARgUMEnTS


Example 2
thesis (belonging to the author) IrecentlytoldmycriticalthinkingclassthatIwasangryintheextremebecause somanyfailedtocompletetheirhomework,eventhoughwhatwasrequired wasclearlyindicatedonthecourseoutlineandeventhoughtheywerereminded theweekbefore.Itoldthemthatifhomeworkwasnotdoneinthefuture,they shouldnotbothercomingtoclass.IbelievethatthisdisplayofangerwasOK. support

All[authenticemotionalreactionsthatmayspurotherstodobetterinthe future]arereactionsthatareOK. (Ateacherdisplayingangerwithaclasswhenthemajoritydidnotcomplete thehomeworkassigned)was[anauthenticemotionalreactionthatmayspur otherstodobetterinthefuture]. (Ateacherdisplayingangerwithaclasswhenthemajoritydidnotcomplete thehomeworkassigned)wasareactionthatwasOK. 1 counterexample Whippingstudentsmayspursometodobetter.


premise

Rating:+ 2 counterexample Somemaybeannoyedatthebitchteacherandhencemayactuallybeless inclinedtocompletework.


premise

Rating:+ Overallrating:+ *
opposition

All[emotionaldisplaysthatmayleadsomestudentsparticularlythosewho hadcompletedsomeofthehomeworktobelievethattheywereunjustly chastised,whichinturncouldsourthelearningatmospherebycreating animageofaconstantlydissatisfiedbitchteacherwhofavorsonlya fewnamely,thegoodietwoshoesintheclassattheexpenseofthe many]arereactionsthatarenotOK. (Ateacherdisplayingangerwithaclasswhenthemajoritydidnotcomplete thehomeworkassigned)is[anemotionaldisplaythatmayleadsomestudentsparticularlythosewhohadcompletedsomeofthehomeworkto believethattheywereunjustlychastised,whichinturncouldsourthe learningatmospherebycreatinganimageofaconstantlydissatisfiedbitch teacherwhofavorsonlyafewnamelythegoodietwoshoesinthe classattheexpenseofthemany]. (Ateacherdisplayingangerwithaclasswhenthemajoritydidnotcomplete thehomeworkassigned)wasareactionthatwasnotOK. 1 counterexample Eventhoughahappyatmosphereintheclassroomisoptimal,itmaynot alwaysbeconducivetomaximizinglearning.
premise

Rating:+

SECTIon 1 LEARnIng ThE InTRICACIES of PRACTICAL REASonIng

135

2 counterexample Someofthestudentswhodidtheworkmayactuallywelcomethefactthat thosewhoarenotcarryingtheirweightarebeingchastised.


premise

Rating:+ Overallrating:+

A MInI-ESSAY
Example 2
thesis

IrecentlytoldmycriticalthinkingclassthatIwasangryintheextremebecause somanyfailedtocompletetheirhomework,eventhoughwhatwasrequired wasclearlyindicatedonthecourseoutlineandeventhoughtheywerereminded theweekbefore.Itoldthemthatifhomeworkwasnotdoneinthefuture,they shouldnotbothercomingtoclass.IbelievethatthisdisplayofangerwasOK.


support

Althoughsomemaybelievethatanydisplayofemotionisinappropriateina professionalsetting,asitcreatesapersonalcrackinwhatshouldbeapolishedveneer,itwasnonetheless,inthisinstance,utterlyauthentic.SinceIcare deeplythateachstudentrisetohisorherpotential,Ihavealreadyrelinquished theimpersonalmodeofteachingandtheveryauthenticityofmyresponse mayspursometodobetterinthefutureinordertoavoidmywrath.


opposition

Althoughmanyfailedtodoallthehomework,someatleastdidsomeofthe homework,asevidencedbythefactthatmanydidwellonthequiz.Focusing primarilyonimperfectionsratherthanaccomplishmentsmaycreateanabidingresentmentthatwillactuallydecreasemotivationinthefuture.Mydisplayofanger canbelikenedtothatofamotherwhogetsmadatherchildforcleaningonlyone roomwhenshehadbeenaskedtocleantwo,whilelavishingpraiseonherother childwhocleanedboth.Thisapparentfavoringofonechildoveranothercan createachronicsenseofinjustice,whichinturncanleadthevictimizedchild tosimplytuneouttheadmonitionsofwhatisnowperceivedasabitchmother.


response

Likeningtheroleofapost-secondaryprofessorofphilosophytoamotherof childrenwhohasalifetimetoquietlyandkindlyinstillvaluesisafaultyanalogy. AmorehelpfulcomparisonwouldbetothatofaMarinegeneralinbootcamp. Timeisoftheessence.Ihaveonechance,andonechanceonly,tofortifymy studentsagainsttheautonomy-destroyingseductionsoftherealworld.Although creatingafunenvironmentwouldbeoptimal,itcannotbedoneattheexpense ofthegoal,andalthoughdifferentialtreatmentmayseemunfairtothosewho havefallenbehind,non-differentialtreatmentwouldseemoutrageousandhighly discouragingtothosewhohavegivenittheirall.


conclusion

Gettingstudentstoembraceimpartialthinkingisliketryingtogetthemtoappreciatefinewineorgoodliterature.Youfirsthavetogetthemtodrinkgoodwineand readgoodbooksbeforetheycanappreciatetheinherentvalueofboth.Itseems paradoxical,butinthisbusyworldofours,forcemaybenecessarytoplantthe seedsofautonomy.Thereisnodoubtthatsomeoftheseseedswillfallonfallow ground.However,thatisnoexcusefornotusingeverypedagogicaltoolpossible inanefforttogeteveryseedtoflourish,evenifitseriouslydamagesthelikability quotientoftheGardner.Toughlovetacticsthatareinherentlyauthenticare notlesslovingsimplybecausethoseonthereceivingenddonotlikethem.

136 PART II PRACTICE


Remember, you cannot make true what is not true, so be prepared to flip your thesis statement if your opposition is stronger than your support.

EXErCISE 9B
Writing more mini-essays
Throughout the remainder of the course, you should continue to write mini-essays as directed by your instructor.

10. Avoiding Rottweiler Flips: Getting Your Counterexamples Straight


Now that you have a macro overview of how to estimate the value of reasons and arguments, we need to take another pass at the process, with a view to hooking into the logic of your language in order to maximize your capacity for precise communicative thinking. When you speak, what you are actually doing is making a claim about the relationship between classes of things. This is how you acquired language in the first place. You could not have learned the concept of red, for example, had your mom referred to only your toy fire engine because, in such a situation, you would have been unable to discern whether the sound red was referring to toy, size, function, or color. The only reason that you learned that red referred to color was because you were exposed to a class of entities that were all different except for one property, namely, the color red. Given that our concepts refer to classes (with a few exceptions such as proper names), it follows that with regard to the underlying logic of classes, we can only ever say one of four things. We can say that one class is entirely contained within another (all S is P) or that there is no relationship between two classes (no S is P)with all and no being referred to as universal quantifiers. Or we can say that that some classes overlap (some S is P) and (some S is not P)with some being referred to as a particular quantifier. In categorical logic, S represents the subject term, and P represents the predicate term. These standard forms form a pattern of contradictories that is referred to as aristotles square of opposition.

ARISToTLES SQUARE of oPPoSITIon


ALLSisP NOSisP

If you make a claim that contains a universal quantifier, you will have addressed your opposition if you address its contradictory. If you make a claim that contains a particular quantifier, in real life, you have to address both opposing universal quantifiers.

SOMESisP

SOMESisnotP

Note the real-life exception: If one were to argue for the legalization of some drugs (e.g., marijuana), although Aristotles square suggests that one has only one opposition, namely, that no drugs should be legal, in real life one actually has two oppositions to address. One must not only speak to those who support total abstinence (the no side) but also show why the reasoning supporting the legalization of some drugs does not apply to all drugsi.e., what the significant difference is between this set of drugs and the rest. In other words, if you make a claim that contains a quantifier that comes from the bottom line of Aristotles square of opposition, in real life, you have to address both claims on the top.

SECTIon 1 LEARnIng ThE InTRICACIES of PRACTICAL REASonIng

137

Since truth seeking requires that claims undergo a process of falsification, Aristotles square of opposition is helpful in directing one toward finding relevant counterexamples. Thus, if someone says that all S is P, the way to oppose that position would be to show that some S is not P, and vice versa. Similarly, the way to oppose a claim that no S is P is to show that some S is P and vice versa. All and some are not claims are referred to as contradictories, as are no and some are claims. Two propositions are contradictory if the truth of one implies the falsity of the other, and vice versa. It is evident from looking at Aristotles square that the S, P order of all claims is crucial. Not only is there an obvious difference between saying that all Rottweilers are dogs and all dogs are Rottweilers but also if one does not get the order straight, the counterexample will be incorrect. In the preceding example, a Rottweiler that is not dog is the counterexample to the first claim, whereas a dog that is not a Rottweiler is the counterexample to the second. Getting the order mixed up, from here on in, is referred to as a rottweiler flip. Although it may not be initially apparent, getting the S, P order straight in all claims can be extremely tricky because all comes buried in all kinds of forms (see box entitled The Hidden All). Thus, the assertion that your mother will only live if she has this operation is an all claim, but it is entirely different from the assertion that if your mother has this operation she will live, which is also an all claim. The fact that your mother lived even though she did not have the operation would show the first claim to be false, whereas the fact that your mother died even though she had the operation would show the second claim to be false.
ThE hIDDEn ALL
ThefollowingsentencesareequivalenttoALLRottweilersaredogs: ALL NON-dogsareNON-Rottweilers. (Note:Youcanswitchthenounphrasesbyswitchingthesigns.) ARottweilerisadog. Rottweilersaredogs. WHATEVER (WHOEVER) isaRottweilerisadog. ItisaRottweilerONLY IFitisadog. ONLYdogsareRottweilers. (Note:Onlyistheflipofall.) ItisONLYaRottweilerifitisadog. (Watchthepreceding:Itisasplitonlyif.Itworksexactlyliketheonlyifbut isthereverseofonlyalone.) IfitisaRottweiler,itisNECESSARILYadog. IN ORDER tobeRottweiler,itMUST beadog. IT IS ESSENTIAL thatitbeadog,IN ORDER forittobeaRottweiler. IFitisaRottweiler,THENitisadog. IF itisNOT adog,THEN itisNOT aRottweiler. (Note:Whenthesignschange,thenounphrasesaretransposed.) ItisNOT aRottweilerUNLESS itisadog. (Note:Thenotthataccompaniesunlessdropsout.) ItCANNOTbeaRottweilerWITHOUT beingadog. (Note:Thenotthataccompanieswithoutdropsout.) ItisadogBECAUSE itisaRottweiler. SinceitisaRottweiler,itmustbeadog.

The counterexample to all Rottweilers are dogs is a Rottweiler that is not a dog, not a dog that is not a Rottweiler.

138 PART II PRACTICE


Standardization is necessary for picking out the correct counterexample. Picking out the correct counterexample is necessary for falsification.

STANDARDIZATION Learning how to standardize an ordinary-language sentence is crucial for determining meaning and for ensuring that you know what counts as a relevant counterexample. It can also help you determine necessary and sufficient conditions (see the next section). The following box shows the formula for standardizing ordinary-language sentences. Note that the noun phrases (in brackets) denote the classes that are being compared. The quantifier merely states how much of the first class is contained in the second.

ThE foRMULA foR STAnDARDIZATIon


Quantifier (a noun phrase) the verb to be (a noun phrase): 1. 2. 3. 4. Examples of standardizing ordinary-language sentences Lemonsareyellow= All(lemons)are(thingsthatareyellow). Thetwowomen,whowenttotheconference,cameintotheroom= Some(womenwhowenttotheconference)are(thingsorpeoplewhocame intotheroom). Whalesarenotfish= No(whales)are(fish). Dinosaurswereextinctbeforetherewereanymodernmammalslivinginthe world=All(dinosaurs)are(thingsthatbecameextinctbeforetherewereany modernmammalslivingintheworld).

There are several things to remember about standardization. The first is that standardized claims always begin with a quantifier (1), which states how much of the first class is contained in the second. First and foremost, we must get straight whether we are talking about all, no, or some of the entities in question. The next issue of note about the preceding examples is that a class can be referred to only by a noun or a noun phrase (2, 4)e.g., lemons or women who went to the conference. A noun phrase can be indefinitely long, with its length usually varying inversely with the size of the class to which it referse.g., the class of Rottweilers versus the class of black-and-gold, 106-pound, female Rottweilers with small, white chest marks that belong to philosophy professors. For those of you who are relatively unfamiliar with noun phrases, a good habit to get into, to ensure that the phrase is indeed a noun phrase, is to begin the phrase with a noune.g., things that are yellow rather than yellow things. Keep in mind that adjectives are not nouns; they do not refer to classes. Yellow does not refer to a class; things that are yellow does. If you reflect, once again, on how we learned adjectivesi.e., attached to thingsthe need for this translation becomes apparent. The last point to note in the preceding examples is that the verb to be is always used to compare classes, and it is always used in the present tense. This is so because you are in the present, and you are now making a claim about classes of things; that all S is P, that no S is P, that some S is P, that some S is not P. If you now want to say something about the past (or the future), as in the last of the examples, tensed terms can be contained in any of the noun phrases, for example, all (people who took part in the French Revolution) are (people who died over two hundred years ago).

SECTIon 1 LEARnIng ThE InTRICACIES of PRACTICAL REASonIng

139

YoUR fRIEnD ThE RoTTWEILER hELPS YoU To DETERMInE oRDER


Aneasywaytofigureoutwhichnounphrasegoesfirstandwhichgoessecond istopluginthewordsRottweileranddogineachoftheslots.Onceyou haveatrueclaim,thenwhichevernounphraseisintheRottweilerposition goesfirstandtheoneinthedogpositiongoessecond. Example: Only philosophers are wise. Now,whichofthefollowingistrue? 1. OnlyRottweilersaredogs. or 2. OnlydogsareRottweilers. Onlyno.2istrue. Thus,philosophersgoesinthedogpositionandpeoplewhoarewise goesintheRottweilerposition. ThestandardizedformoftheoriginalclaimisthusAll people who are wise are philosophers. Toshowthatthisclaimisfalse,youwouldhavetofindawisepersonwhowas notaphilosopher,notaphilosopherwhowasnotwise.

NOTE: The advantage of using the Rottweiler/dog tactic is that you can determine order without memorizing the cheat sheet. However, for those of you who find this tactic problematic, you might benefit from moving directly to subsection 11, despite the fact that it requires the memorization of a cheat sheet. Subsection 11 is similar to subsection 10 in that the goal is to determine the correct order. However here, in subsection 10, because we are translating everything back to all claims (in order to determine the counterexample), we are working within the territory of categorical logic (which examines the relationship between classes). In subsection 11, where the focus is on if . . . , then or conditional statements (which are direct translations of all claims), we will be moving into the territory of propositional logic (which examines the relationship between propositions). Some find the latter to be easier.

140 PART II PRACTICE


EXErCISE 10a
Standardize, and state the counterexample
Standardize the following sentences, and below each write out a counterexample. NOTE: No translates directly into all. Avoid using the quantifier no, as it masks information, as will become evident in the next section.
Quantifier (a noun phrase) the verb to be (a noun phrase). 1. 2. 3. EXAMPLE
Youcannotfoolothersifyouarefoolingyourself. All(peoplewhofoolthemselves)are(peoplewhocannotfoolothers)
counterexample

4.

Apersonwhofoolshim-orherselfandwhoalsofoolsothers.

1. 2. 3. 4. 5. 6. 7. 8. 9. 10. 11. 12. 13. 14. 15.

Answers are in the back of the book. Any fool knows that. It is a mammal because it is a whale. It is not acceptable unless it is typed. Only girls are allowed in this club. If you pay the admission, you will get in. You are eligible to serve on a jury only if you are eligible to vote. Only Democrats will vote for the abortion act. It is not true that only Republicans are conservatives. You will only live if you have this operation. You will live if you have this operation. You cant get any good jobs without experience. More than half the class was home last week with the flu. Attending weekly meetings is a condition of your employment. Unless you know someone of influence, you can never get a good job. You will be successful because you work hard.

EXErCISE 10B
Standardize, and state the counterexample
Standardize the following sentences, and below each write out a counterexample. Answers are in the back of the book. 1. Suicide is always wrong. 2. Most suicides are wrong. 3. Last year, no one under the age of ten committed suicide. 4. Committing suicide is wrong because it hurts the people who love you.

SECTIon 1 LEARnIng ThE InTRICACIES of PRACTICAL REASonIng

141

5. 6. 7. 8. 9. 10.

If someone commits suicide, that shows s/he was not sane at the time. You cant commit suicide very easily unless you have some help from others. Crops only grow if it rains. If you like existentialist theory, you will like Albert Camuss The Plague. Only pessimists like existentialist theory. Existentialism is not real philosophy.

EXErCISE 10C
Standardize, and state the counterexample
Standardize the following sentences, and below each write out a counterexample. Answers are in the back of the book. 1. You can only be autonomous if you learn to think impartially. 2. An entity can learn a symbolic language if and only if that entity is self-conscious. 3. You must learn to articulately listen to others if you are going to master the art of thinking impartially. 4. You are a good thinker because you ask lots of questions. 5. Your autonomy is jeopardized every time you give in to groupthink.

EXErCISE 10D
Standardize, and state the counterexample
Standardize the following sentences, and below each write out a counterexample. 1. You will never learn to think impartially if you do not learn to listen to opposing points of view. 2. You must learn to think through value issues in order for your thinking to actually govern your behavior. 3. You will only follow reasons where they lead if you learn not to be humiliated by backing down. 4. Only self-conscious animals can be more or less wise. 5. You cannot estimate the truth of a claim unless you subject it to a falsification process.

EXErCISE 10E
Standardize, and state the counterexample
Standardize the following sentences, and below each write out a counterexample. Answers are in the back of the book. 1. Horses are not self-conscious. 2. Only an Arabian horse can run like the wind. 3. If you want your horse to be healthy, you must give it high-quality food. 4. Dont walk your horse on pavement without shoes. 5. You should always pick your horses hooves before riding.

142 PART II PRACTICE


EXErCISE 10F
Standardize, and state the counterexample
Standardize the following sentences, and below each write out a counterexample. 1. 2. 3. 4. 5. 6. 7. 8. 9. 10. Put a little oil in your horses food if you want it to have a shiny coat. If you put a little oil in your horses food, it will have a shiny coat. Do not yank on your horses mouth unless it is being really naughty. You can only take your horse across the border if it has a passport. It is not true that all animals can be domesticated. Every human should try to form a connection with an animal because it helps to get human priorities straight. To say that humans are better than animals is false. You can only be at peace with yourself and the world if you are kind to animals. Unless you are prepared to make some significant sacrifices to make your animal friend happy, you should not own a pet. Animals are people, too.

11. Are You Making a Claim about a Sufficient or a Necessary Condition?


Every time you make an all claim you are making a claim about sufficient and necessary conditions.

In a standardized all sentence, the first noun phrase is always a sufficient condition of the second, and the second noun phrase is always a necessary condition for the first.

If you claim that all S is P, you are saying that S is sufficient but not necessary for P and that P is necessary but not sufficient for S.

Every time you use an all statement, you are making a claim about sufficient and necessary conditions. You have no choice: It is built into the meaning of all. If you make the claim that a cobra bite kills (i.e., all cobra bites are things that kill), you are saying that knowing that someone has been bitten by a cobra is a sufficient condition to know that s/he will die and that dying will necessarily follow being bitten by a cobra (i.e., if the individual does not die, then we would know that s/he was not bitten by a cobra). Determining which class is the sufficient condition and which is the necessary condition in an all statement is easy. In a standardized all sentence, the sufficient condition always comes first, and the necessary condition always comes second; this is one of the reasons why order is so important. Because I know that all Rottweilers are dogs, if you tell me you have a Rottweiler, that is a sufficient condition for me to know that you have a dog. Knowing that you have a dog, of course, is not sufficient for me to know that you have a Rottweiler. You may very well have a Yorkshire Terrier. On the other hand, if you tell me that you do not have a dog, I thereby know that you do not have a Rottweiler, because having a dog is a necessary condition for having a Rottweiler. One way to become comfortable with the claim that necessary and sufficient conditions are built into the word all is to focus on the if . . . , then translation of all. That is, all translates directly into if . . . , then. If I say that all Rottweilers are dogs, then I am making the conditional claim that if it is a Rottweiler, then it is a dog. If you think about the words if . . . , then, they seem to translate directly into making a claim about sufficient and necessary conditions. If I say that if you shoot the man in the heart, then he will die, it is very close to directly saying that shooting is a sufficient condition for death and that if I shoot him, death will necessarily follow. Again, the order is crucial. If you have an if . . . , then sentence, the if portion, also called the antecedent, always names the sufficient condition, and the then portion, also called the consequent, always names the necessary condition.

SECTIon 1 LEARnIng ThE InTRICACIES of PRACTICAL REASonIng

143

Becoming comfortable with sufficient and necessary conditions can greatly facilitate communication and clear thinking. Once you have established that the claim in question really is an all statement, in order to ensure that the order is right, as well as just to get a different look at the claim that is being made, it is helpful to question whether S really is a sufficient condition for P, or perhaps it is the other way around, or neitheri.e., perhaps this is not an all claim after all. The following exercises provide practice in responding to hidden all claims to ensure that meaning is correctly understood. The following cheat sheet should be committed to memory unless you are completely comfortable with using the Rottweiler/dog tactic suggested in the last section. Remember that in real life, you will not have a cheat sheet with you. However, in real life it is imperative that you be able to recognize the difference between being told that you will live if you have this operation and you will live only if you have this operation. In the first situation, you should be able to respond, Are you telling me that this operation is sufficient to ensure my life but may not be necessary? and in the second, Are you telling me that this operation is necessary to ensure my life but may not be sufficient?
ChEAT ShEET foR nECESSARY AnD SUffICIEnT ConDITIonS
all=sufficient anyone=sufficient whoever=sufficient if=sufficient inorderfor=sufficient because=sufficient since=sufficient if...,then=sufficient/necessary only=necessary onlyif=necessary only...if=necessary(beware the only if split ) not/unless=necessary(with the not dropping out ) cant/without=necessary must(have)=necessary onconditionthat=necessary isessentialfor=necessary ifandonlyif=biconditional providedthat=biconditional

In an if . . . , then sentence, the first clause, or the antecedent, is always a sufficient condition of the second; the second clause, or the consequent, is always a necessary condition of the first.

A NOTE ABOUT ONLY Only can be used as an adjective as well as a logical connective. If you can substitute sole, exclusive, or alone, you will know that this is an adjective and does not signal a necessary conditione.g., she was the only survivor of the crash. Examine the difference in the following two claims: 1. If a dump truck crashes into a Volkswagen, the Volkswagen will be crushed. All (times that a dump truck crashes into a Volkswagen) are (times that the Volkswagen is crushed).

144 PART II PRACTICE


2. If a dump truck crashes into a Volkswagen, only the Volkswagen will be crushed. All (times that a dump truck crashes into a Volkswagen) are (times that only the Volkswagen is crushed). In addition, you should note that if only usually signals that only is being used as an adjective and not as a logical connective: 1. If only you loved me, my life would be perfect. 2. If only that country would cease its aggressive tactics, peace would have a better chance. NOTE THAT REASONS ARE ALWAYS SUFFICIENT CONDITIONS Whenever you give a reason, you are implicitly making the claim that that reason is sufficient to prove the truth of the conclusion. You are also making the claim that the conclusion necessarily follows from the reason. Thus, whenever you have a claim that contains an indictor either for a reason (e.g., because) or for a conclusion (e.g., therefore), you know that the reason holds down the sufficient slot, whereas the conclusion holds down the necessary spot. Thus, if someone says to you that people are beautiful because they have good genes, you can accurately mirror what that person has said by saying, So you are telling me that having good genes is a sufficient condition for ensuring that someone is beautiful?
REASonS ARE In ThE SUffICIEnT SLoT; ConCLUSIonS ARE In ThE nECESSARY SLoT
ItisadogbecauseitisaRottweiler. SinceitisaRottweiler,itmustbeadog. ItisaRottweiler,soitisadog. ItisaRottweiler;therefore,itisadog. ItisaRottweiler;hence,itisadog.

NOTE ABOUT A DOUBLE NEGATIVE You are beautiful because you have good genes. (Genes are a sufficient condition.) You are not beautiful because you do not have good genes. (Genes are a necessary condition.)

NOTE ABOUT A DOUBLE NEGATIVE As already noted, all reasons make claims about sufficient conditions. If someone says, (1) He got into medical school because he had first-class grades, that person is making the claim that having first-class grades is a sufficient condition for getting into medical school. However, if someone says, (2) He did not get into medical school because he did not have first-class grades, that person is making the claim that not having first-class grades is a sufficient condition for not getting into medical school. Note that this is a double negative. Remember that placeholders can be switched if the signs are changed. Thus, the second sentence not only means that not having first-class grades is a sufficient condition for not getting into medical school, it also means that having first-class grades is a necessary condition for getting into medical school. HELPFUL HINTS ABOUT TRANSLATIONS In straightforward translations, all S is P can be translated into are you telling me that S is a sufficient condition for P? and are you telling me that P is a necessary condition for S? Thus, the ordinary-language sentence a whale is a mammal can be translated as being a whale is a sufficient condition for being a mammal or being a mammal is a necessary condition for being a whale.(Note: In the preceding, beginning the claims with a gerund (i.e., a verbal noun), which ends in -ing, is helpful.) However, sometimes the straightforward translation does not make sense. The ordinary-language claim that only people who are depressed commit suicide is standardized as all people who commit suicide are people who are depressed. However,

SECTIon 1 LEARnIng ThE InTRICACIES of PRACTICAL REASonIng

145

saying that committing suicide is a sufficient condition for being depressed does not make sense. A more appropriate translation is if you know someone committed suicide, that is a sufficient condition to know that that person was depressed. On the other hand, there is no problem with saying that being depressed is a necessary condition for committing suicide. There are also instances in which problems may arise when translating necessary conditions. These problems usually arise as a function of the temporal relationship between the two classes. Sometimes the class in the second slot of a standardized sentence is a necessary precondition of the first, sometimes it is a necessary accompaniment, and sometimes it necessarily follows. Thus, if someone says that you will get drunk if you drink five martinis, the meaning is not preserved if you translate it by saying that getting drunk is a necessary condition of having five martinis. A more appropriate translation is if you have five martinis, getting drunk will necessarily follow. You should also keep in mind that although all all statements can be translated into a claim about sufficient conditions and necessary conditions, in the real world, usually the context will determine which translation is appropriate. Lets take the claim that mammals need oxygen to live. If one found mammalian bones on a far-off planet, one might appropriately say that the finding of mammalian bones is sufficient to show that there must have been oxygen on this planet at one time. On the other hand, if one discovered evidence that a certain planet could never have had oxygen, one could appropriately say that since oxygen is necessary for life, it follows that no mammals could ever have lived on that planet. The formal rule with regard to which translation is appropriate is the following. The presence in the real world of a sufficient condition (of a pair of classes that are conditionally relatede.g., mammals and oxygen) signals that a translation about sufficient conditions would be appropriate. The absence in the real world of a necessary condition (of a pair of classes that are conditionally related) signals that a translation about necessary conditions would be appropriate. The validity of this formal rule becomes evident in the next section on valid and invalid deductions.
TRAnSLATIon RULES
AllSisPisthesameassayingthatifS,thenP. Theprecedingclaimcanbetranslatedaseither 1. SisasufficientconditionforP. or 2. PisanecessaryconditionforS. IfSisthefocusofthediscussion,use(1),thesufficienttranslation. IfPisthefocusofthediscussion,use(2),thenecessarytranslation.

It is of interest to note that the all to if . . . , then translation moves us from territory referred to as categorical logic (which examines the relationship between classes) to territory referred to as propositional logic (which examines the relationship between propositions). This is evident by looking at the difference between all Rottweilers are dogs, which indicates a relationship between classes, and if it is a Rottweiler, then it is a dog, which indicates a relationship between two propositions or statements. As we move into the territory of propositional logic, we will need symbols that indicate the relationship between the statements (see the box). Note that it is traditional in propositional logic to use uppercase letters when symbolizing specific simple statementsfor example, using R as an abbreviation for it is a Rottweiller, while unspecified statements are represented by the lower case letters, p, q, r, s, . . . So the translation rule in the preceding box might be reworded as follows: All S is P is the same as saying that if p, then q.

146 PART II PRACTICE


LogIC SYMBoLS
IF...,THEN:or NOT:~ AND:or& OR:v NOTE: Inanif...,thenproposition,whateveristotheleftofisasufficient condition;whateveristotherightofisanecessarycondition.

ALL ROTTWEILERS ARE DOGS I.E., BEING A ROTTWEILER IS A SUFFICIENT CONDITION FOR BEING A DOG

A ROTTWEILER FLIP: MISTAKING A NECESSARY FOR A SUFFICIENT CONDITION

147

148 PART II PRACTICE


EXErCISE 11a
Sufficient and necessary conditions
In the following exercise, (1) standardize the statements, and (2) if the statement is an all claim, respond with two questions about necessary and sufficient conditions. Be sure that your responses are in ordinary language. remember that the point of this exercise is to rephrase the claim in different, more precise logical terms so that agreement on meaning can be ensured. Obviously, this can happen only if the person to whom you are speaking can understand you!

Example Inorderfortheretobehumanlife,theremustbeoxygen. Alllivinghumansarethingsthatneedoxygen. 1. reyoutellingmethatifhumanlifeispresent,thatissufficientforustoknowthereis A oxygenpresent? 2. Areyoutellingmethatoxygenisnecessaryforlife?

NOTE: If your statement is not well phrased, it is inadequate. Remember that the point of this exercise is to learn to reflect on necessary and sufficient conditions in everyday conversations. In the preceding situation, if you respond, Are you telling me that human life is sufficient for oxygen? no one would know what you are talking about. Answers are in the back of the book. 1. If we had made a left turn three kilometers back, we wouldnt be stuck in this awful swamp. 2. Unless you register, you are not allowed to vote. 3. Only the rich and famous can get past the doorman and into the club. 4. You should stay off the ride called the Howlin Screamer because youve just eaten. 5. Only geeks wear ties to school. 6. If you could see me now, youd be very proud of me. 7. I wont be joining you on the hike on Saturday unless you can lend me some rain gear. 8. You should feel compelled to commit the cheat sheet to memory unless you are completely comfortable with using the Rottweiler/dog tactic. 9. The sign on the elevator says, Bank employees only. 10. You can go out provided that you have cleaned your room.

EXErCISE 11B
Sufficient and necessary conditions
In the following exercise, (1) standardize the statements, and (2) if the statement is an all claim, respond with two questions about necessary and sufficient conditions. Be sure that your responses are well phrased. Answers are in the back of the book.

SECTIon 1 LEARnIng ThE InTRICACIES of PRACTICAL REASonIng

149

1. If you love me, I will follow you anywhere. 2. Houston, this is Orbiter One. Unless we fire retro rockets now, were going to miss our reentry window. 3. It is OK, Orbiter One. If you fire the retro rockets in three minutes, you will be able to reenter Earths atmosphere. 4. Unless you are independently wealthy, you should strive to create a career that you can commit to for the rest of your life. 5. If roses are red and violets are blue, then writing a short poem is easy to do. 6. I will come up with more sentences only if you really think you need them. 7. The refugee situation will be relieved provided that immigration standards are not too strict. 8. We know that complex technology has unpredictable effects because complex technology leads to occasional bizarre accidents. 9. Having a computer is essential for doing well in academia. 10. You cannot think well unless you are able to articulately listen to the point of view of others.

EXErCISE 11C
Sufficient and necessary conditions (fill in the blanks)
State whether the bold, italicized phrase in each of the following sentences states a necessary condition, a sufficient condition, neither, or both. Answers are in the back of the book. 1. I am not going out with you if you dress like that. 2. any student who purposefully plagiarizes in this course will be given a zero. 3. To be admitted to graduate school, applicants must have a four-year honors degree, with at least a 75 percent average on all courses taken in the last two years. 4. You cannot get an A in this course unless you are prepared to work consistently hard throughout the term. 5. Essays will be returned to the student without being graded if the English structure is not up to college standards. 6. You cannot expect to have a successful life unless you can think well. 7. For the business community to regain confidence in the economy, interest rates must come down. 8. You have to be in good physical condition to be a good skier. 9. a prize will be given to the group that has the highest score. 10. No one can become a philosophy professor these days unless s/he has a Ph.D. 11. If you think positively, you will be a success; if you dont think positively, you wont amount to anything. 12. Whoever has the drugs must be the drug smuggler. 13. People who smoke cigarettes are much more likely to develop lung cancer than those who do not smoke. On the other hand, some nonsmokers also develop lung cancer, whereas some people who smoke all their lives never get it. 14. If the hill is icy, Bill will win the race. 15. Bill will not win the race unless the hill is icy.

150 PART II PRACTICE


EXErCISE 11D
Standardization, sufficient and necessary conditions, and counterexamples
In the following exercise, (1) standardize the statements, and (2) if the statement is an all claim, respond with two questions about necessary and sufficient conditions; finally, (3) write out what would count as a counterexample. Answers to the first eight are in the back of the book. 1. You cannot have faith without hope. 2. Only love can bring you faith. 3. You cannot truly love unless you are kind. 4. Because you have faith, you will have hope. 5. Kind people are loving people. 6. You can only have faith if you have charity. 7. You can have genuine hope if and only if you have faith. 8. Anyone with integrity will value honor. 9. Love will come to those who have faith. 10. You can only have integrity if you are kind. 11. He is kind because he is loving. 12. You can have integrity if you are kind. 13. Hope follows love. 14. Unless you have faith, you cannot have hope. 15. Faith and hope always go together.

EXErCISE 11E
Standardization, sufficient and necessary conditions, and counterexamples
In the following exercise, (1) standardize the statements, and (2) if the statement is an all claim, respond with two questions about necessary and sufficient conditions; finally, (3) write out what would count as a counterexample. Answers to the first five are in the back of the book. 1. Without impartiality, there is no autonomy. 2. Your autonomy is threatened if you spend a lot of time with nonautonomous individuals. 3. Hatred precludes autonomy. 4. You cannot be autonomous unless you are prepared to stand up and be counted. 5. Only those who can genuinely care about at least some others can be autonomous. 6. You can only be autonomous if you do whatever is in your power to do. 7. Unless we cease to blame others, autonomy is impossible.

SECTIon 1 LEARnIng ThE InTRICACIES of PRACTICAL REASonIng

151

8. If autonomy requires perspective, then autonomy requires that we genuinely get to know others. 9. Understanding the perspective of others is sometimes nothing more than bumping into bias. 10. Power over yourself is stronger than power over others.

EXErCISE 11F
Working quickly with reasons
Fill in the following blanks with sufficient or necessary. Answers to the first four questions are in the back of the book. 1. If someone says, You wont get into medical school because you didnt get firstclass grades in college, that person is assuming that getting first-class grades is a _____________ condition for getting into medical school. 2. If someone says, We gave the patient ten grains of aspirin so his fever will come down, that person is assuming that ten grains of aspirin is a _____________ condition for bringing down a fever. 3. If someone says, She is going to be as drunk as a skunk: She polished off three martinis in the last half-hour, that person is assuming that drinking three martinis is a _____________ condition for getting as drunk as a skunk. 4. If someone says, We have run out of flour, so we cant make bread, that person is assuming that flour is a _____________ condition for making bread. 5. If someone says, Children should have rights because children are people too, that person is assuming that children being people is a _____________ condition for having rights. 6. If someone says, Women generally dont make good executives because most women are not very aggressive, that person is assuming that being aggressive is a _____________ condition for being a good executive. 7. If someone says, There is an enormous amount of evidence that suggests that the marine mammals and great apes are self-conscious. Therefore, marine mammals and great apes are persons, that person is assuming that being self-conscious is a _____________ condition for being a person. 8. If someone says, It is not unethical to kill a cow because a cow is not selfconscious, that person is assuming that being self-conscious is a _____________ condition for making it unethical to kill that thing. 9. If someone says, It is unethical to make a cow suffer because cows, like any other animal, have the capacity to feel pain, that person is assuming that having the capacity to feel pain is a _____________ condition for making it unethical to cause suffering. 10. If someone says, Since he didnt intend to kill her, he cannot be charged with first-degree murder, that person is assuming that having an intent to kill is a _____________ condition for being charged with first-degree murder. NOTE: If you look carefully at the answers to Exercise 11F, you should be able to deduce the following rule: A positive reason is always a sufficient condition. A negative reason (i.e., one with a not in it) is always a necessary condition.

WORKING QUICKLY WITH REASONS If you deduce a conclusion from the fact that something is present, then you must be assuming that it is a sufficient condition. If you deduce a conclusion from the fact that something is absent, then you must be assuming that it is a necessary condition.

152 PART II PRACTICE


EXErCISE 11G
Sufficient/necessary conditions and counterexample (multiple-choice questions)
Answers are in the back of the book. If you assume that each of the given statements is true, what follows? 1. If roses are red, then violets are blue. a. The fact that violets grow blue in the spring before the roses bloom shows that this claim is false. b. The fact that roses grow to a lovely red rich color long after violets are dead shows that this claim is false. c. Both a and b. 2. It is not true that only happy people are generous. a. All happy people are generous. b. All generous people are happy. c. Neither a nor b. 3. There can be no peace because there is no joy. a. There can be no joy unless there is peace. b. The fact that many people are miserable in situations of real peace shows that this claim is false. c. All joyful times are peaceful times. 4. You cannot be fair if you are not intelligent. a. Well, Hitler was certainly intelligent, but he was hardly fair. That shows that this statement is false. b. Are you really telling me that only intelligent people are fair? c. Neither a nor b. 5. Dull women have immaculate houses. a. That is terrific. My house is always a mess. I guess that means I am a very interesting person. b. Oh, no. My house is always clean. I guess that means I am dull. c. Both a and b. 6. You should hug your co-workers in order to elevate office spirit. a. Hugging will raise office spirits. b. Everybody in the office is miserable. That shows there was not much hugging going on. c. Neither a nor b. 7. It is not true that only empathetic people are kind. a. The fact that some empathetic people are kind shows that this statement is false. b. The fact that you can only be empathetic if you are kind shows that this statement is false. c. The fact that all kind people are empathetic shows that this statement is false. 8. Only citizens can serve on a jury. a. You can serve on a jury if you are a citizen. b. All citizens have the right to serve on a jury. c. Neither a nor b. 9. Hope follows love. a. The fact that Jane has been hopelessly in love for years shows that this claim is false. b. The fact that there are many instances in which people bravely hope even though they may feel loveless shows that this claim is false. c. Both a and b.

SECTIon 1 LEARnIng ThE InTRICACIES of PRACTICAL REASonIng

153

10. Save the planet. It is the only one where chocolate grows. a. Are you telling me that the only reason that we should try to save this planet is because your fat gut cant go a day without chocolate? b. Are you telling me that everyone who loves chocolate should be an environmentalist? c. Neither a nor b.

EXErCISE 11H
Sufficient/necessary conditions and counterexample (multiple-choice questions)
Answers are in the back of the book. If you assume that each of the given statements is true, what follows? 1. People are greedy because they are ignorant. a. All greedy people are ignorant. b. The fact that Scrooge was greedy but not ignorant shows that this claim is false. c. Neither a nor b. 2. Courage is essential for honesty. a. The fact that John is dead honest but nonetheless a coward shows that this claim is false. b. You cannot be courageous unless you are honest. c. Alexander the Great must have been an honest man. We know for sure that he was courageous. 3. Life is not worth living without chocolate. a. Jean is bound to feel better soon. I bought her the biggest box of chocolates in the store. b. They say that the cacao bean is becoming endangered. That is bad news for humankind. No chocolate will mean mass human misery. c. Neither a nor b. 4. You should eat red meat if you want to have a good blood count. a. No vegans have good blood counts. b. Everyone who eats meat has a good blood count. c. Neither a nor b. 5. You cannot fail unless you are lazy. a. Jane didnt fail. That proves that she is not lazy. b. Are you telling me that every single person who has ever failed is a person who is lazy? c. John is the laziest person I know. He is bound to fail. 6. You can only consider yourself beautiful if you have lots of dates. a. I guess that means that Jane is an ugly duckling. She never goes out. b. Beautiful people always have lots of dates. c. Both a and b. 7. Lonely people make good poets. a. You can only become a good poet if you experience loneliness. b. John has no friends. He is bound to win the poetry prize. c. Both a and b. 8. Religion is the breeding ground for arrogance. a. The fact that Buddha was not arrogant shows that this claim is false. b. The fact that there are lots of arrogant people in this world who are not religious shows that this claim is false. c. The fact that the world is full of people who are neither religious nor arrogant shows that this claim is false.

154 PART II PRACTICE


9. Unless you are really careful, you will not be able to cope. a. Dan is hopelessly careless. That means he wont be able to cope. b. If you are coping, that shows that you are a careful person. c. Both a and b. 10. You should be kind to her because she is sad. a. Are you telling me that I should be kind to every single sad person I meet? b. The fact that I shouldnt be kind to the local serial killer who is sad shows that this is a stupid thing to say (i.e., that it is false). c. Both a and b.

EXErCISE 11I
Sufficient/necessary conditions and counterexample (multiple-choice questions)
If you assume that each of the given statements is true, what follows? 1. It is not true that only conservatives will vote for the environmental bill. a. Some liberals will vote for the environmental bill. b. No conservatives will vote for the environmental bill. c. Some conservatives will vote against the environmental bill. 2. We can do no great things in this world without love. a. That is a stupid statement. Romeo was a great lover, but he never did great things. b. That is a stupid statement. Shakespeare did great things, but it is not at all clear that he was a great lover. c. Neither a nor b. 3. If you can create your own personal style, that means that you have great inner confidence. a. People with confidence are able to create their own personal style. b. The fact that confident Charlie has no style shows that this claim is false. c. You can have style only if you have confidence. 4. I love daisies. I think they are truly beautiful. However, I do not think anybody likes the smell of daisies. a. This serves as a counterexample to the claim that only sweet-smelling flowers are beautiful. b. This serves as a counterexample to the claim that flowers can only smell good if they are beautiful c. Both a and b. 5. It is not true that only people who have faith have hope. a. Some people with no faith have hope. b. All people with hope are people who have faith. c. Some hopeful people have faith. 6. Life is only possible if you have courage. a. Cowards are sure to die. b. A dead coward would show that this claim is false. c. Both a and b. 7. Only loving people are honest. a. All nonloving people are dishonest. b. All loving people are honest. c. You can only be loving if you are honest.

SECTIon 1 LEARnIng ThE InTRICACIES of PRACTICAL REASonIng

155

8. She was unhappy because she was a failure. a. All people who are unhappy are failures. b. Some people who are unhappy are failures. c. All happy people are people who are successful. 9. The movie cannot be a success because it does not have a happy ending. a. If a movie has a happy ending, that will guarantee its success. b. All successful movies have happy endings. c. Only successful movies have happy endings. 10. You cannot succeed unless you are self-confident. a. Only successful people have self-confidence. b. All people who are failures are people who lack self-confidence. c. If you are a success, that shows that you must have self-confidence.

EXErCISE 11J
Sufficient/necessary conditions and counterexample (multiple-choice questions)
If you assume that each of the given statements is true, what follows? 1. That pit bull is a very gentle animal. a. This serves as counterexample to the claim that pit bulls are vicious. b. This serves as a counterexample to the claim that only pit bulls are vicious. c. Both a and b. 2. If you have wisdom, then you will know when to push for change and when to leave well enough alone. a. That leaves John out of the wisdom camp. He hates change. b. The fact that wise Willie failed to know when to act shows that this claim is false. c. Both a and b. 3. Since there is a lack of understanding, there is bound to be war. a. The different communities in Switzerland understand each other well. That means that they will never wage war against one another. b. The Protestant and Catholic Irish have been warring for centuries. That shows that they do not understand one another. c. We have to get the Israelis and Palestinians to understand one another because there can only be peace if there is understanding. 4. A lack of courage breeds unhappiness. a. Courage is a necessary condition for happiness. b. If someone is unhappy, that is a sufficient condition to know that that person is a coward. c. Both a and b. 5. You are wise because you have suffered greatly. a. Are you telling me that since we know that Gandhi was a wise individual, it follows that he must have endured great suffering? b. Are you telling me that Winston Churchill is not wise because there is no evidence that he ever had to endure real suffering? c. Are you telling me that everyone who suffers greatlylike the people who are put in prisonis wise? 6. Misunderstanding breeds hatred. a. All people who love are people who understand. b. Love is a necessary condition of understanding. c. Understanding breeds love.

156 PART II PRACTICE


7. Joe is unhappy but kind. a. This is a counterexample to the claim that all happy people are kind. b. This is a counterexample to the claim that you will be happy because you are kind. c. This is a counterexample to the claim that you can be happy only if you are kind. 8. It is not fun unless it is challenging. a. Critical thinking is sure challenging. No wonder it is fun. b. The fact that a lot of people think it is fun to passively watch life go by shows that this claim is false. c. Both a and b. 9. Marilyn Monroe was a very unhappy individual. a. She thus serves as a counterexample to the claim that happy people tend to be rich. b. She thus serves as a counterexample to the claim that only rich people are happy. c. She thus serves as a counterexample to the claim that only poor people are unhappy. 10. If only Jane will love me, my whole world will be perfect. a. John is saying that Jane loving him is a necessary condition for having a perfect world. b. John is saying that Jane loving him is a sufficient condition for having a perfect world. c. If Johns world ends up being perfect even if Jane does not love him, that would show that this claim is false.

12. Back to Seeing the Whole Argument: Finding the Hidden Premise in Forced-Choice Situations
Since you are now able to see sufficient and necessary conditions as well as standardized forms, you are in a position to see the hidden premise in all kinds of ordinarylanguage claims. The following multiple-choice questions provide practice in perfecting that skill (and help for taking many standardized academic tests, such as the LSATs).

EXErCISE 12a
Finding the hidden premise (assumption questions)
1.
48

2.

Answers to the first five are in the back of the book. Karen was selected for the committee. She must be a sophomore. the preceding statement makes the following assumption: a. Only Karen can be selected for the committee. b. Only sophomores can be selected for the committee. c. Some sophomores must be selected for the committee. d. Some sophomores might not be selected for the committee. e. Karen did not refuse to serve on the committee. The Republican candidate for governor of State X will get the education vote. More than $200,000 was donated to her campaign by the state teachers union.

SECTIon 1 LEARnIng ThE InTRICACIES of PRACTICAL REASonIng

157

3.

4.

5.

6.

The same union donated only half that amount to the Democratic candidates campaign. Choose the statement that best reveals an assumption underlying the preceding argument. a. The Republican candidate is a former teacher. b. The Democratic candidate will lose the election. c. A donation usually indicates approval of a candidate. d. Most teachers have joined the union. e. Unions endorse candidates in each election. Officials reviewing conditions of a local police station are considering whether major structural renovations are required. The chances are good that a police officer will be killed or injured by a prisoner because of the cramped space and poor design of the holding cells, one official concluded. the officials conclusion is based most on which of the following assumptions? a. All criminals who are brought to this particular holding cell are violently dangerous and present a serious threat to safety. b. Storage of and access to police files will be more efficient if the planned renovations are accomplished. c. Police officers will be able to perform their public duties more effectively if they are provided with new office spaces. d. Holding prisoners in small, cramped holding cells is unconstitutional because it constitutes cruel and inhuman punishment. e. In crowded conditions, dangerous criminals will have more access to weapons and closer contact. Frederic Chopin is the greatest piano composer who ever lived. After all, more different musical variations have been based on the Prelude in A, Chopins most famous piano theme, than in any other musical theme ever written. Which of the following assumptions is indicated in the preceding argument? a. Only musical preludes are used as the basis for other composers musical variations. b. Only composers whose compositions become the subject for musical variations can be considered great. c. Chopins Prelude in A is a more famous musical composition than George Handels Theme in G. d. No great musical compositions have ever been written by Spanish composers. e. Frederic Chopin wrote music because he wanted to become famous. Marvin is a senior, so he must have gone to the senior prom. the statement assumes that a. Marvin is the only senior who went to the prom. b. All seniors went to the prom. c. Only seniors went to the prom. d. Some seniors did not go to the prom. e. No seniors went to the prom. It is hardly surprising that Jennifer Gates got into Harvard. Did you know that she is one of Bills relatives? the person presenting this argument must assume that a. Only people who are computer geeks get into Harvard. b. You cannot get into Harvard unless you are really wealthy. c. All people who get into Harvard are wealthy. d. All people who are well connected get into Harvard. e. No poor people get into Harvard.

158 PART II PRACTICE


7. Globalization, as a guiding economic theory, has been a disaster. It has resulted in an increasing disparity between the rich and the poor all over the world. the person presenting this argument must assume that a. An economic theory is not a disaster if it does not result in an increase in the disparity between the rich and the poor. b. An economic theory is only a disaster if it results in an increase in the disparity between the rich and the poor. c. An economic theory is a disaster if it results in an increase in the disparity between the rich and the poor. d. If the poor get richer and the rich do not, that means that the economic theory was a success. e. All economic theories are inevitably disastrous. Using animals in medical research increases the chances of finding cures for many of the deadly diseases from which many humans suffer. For that reason, it should be considered ethically acceptable. the person presenting this argument must assume that a. Research on unwilling prisoners is ethically acceptable because it would increase the chances of finding cures for many of the deadly diseases from which many humans suffer. b. A research protocol is not ethically acceptable unless it increases the chances of finding cures for many of the deadly diseases from which many humans suffer. c. If we do not use animals in research, then we will not be able to increase the chances of finding cures for many of the deadly diseases from which many humans suffer. d. Using animals in medical research will not cause the animal unacceptable levels of pain and suffering. e. If animals were subjected to unacceptable levels of pain and suffering, then the research should not be carried out. I think all convicted rapists should automatically be castrated, because if we did that, the crime of rape would dramatically decrease. the person presenting this argument must assume that a. Only castration will dramatically decrease the crime of rape. b. If statistics showed that there was a country in which there was no rape, this would show that that country must have instituted dramatic punishments, such as castration. c. Unless we get really tough on rapists, rape statistics will never come down. d. Since we do not have dramatic punishments such as castration for rape, that shows that we do not really care about rape victims. e. We should be prepared to institute any punishment that would result in a dramatic decrease of rape. If your friend is buying essays on the Internet and refuses to stop, even though you have told him you think it is wrong, you should nonetheless not report him because doing so will seriously harm your friend and therefore your friendship. the person presenting this argument must assume that a. You should not report your friend to authorities even if your friend was a rapist. b. It would be perfectly legitimate for you to decide to buy essays on the Internet because doing so would not harm any of your friends. c. If an act is acceptable, that means that it does not harm any of your friends. d. a, b, and c. e. a and c.

8.

9.

10.

SECTIon 1 LEARnIng ThE InTRICACIES of PRACTICAL REASonIng

159

13. Responding to Incorrect Counterexamples


You hear a fellow student say the following: The teacher told me that I could only pass the course if I passed the exam. Well, I passed the exam, and now the teacher tells me that I am not going to pass anyway because my written work is weak. That is so typical of teacherschanging the rules whenever they please. Well, I am going to go to the president of the college. I am going to get this teacher fired. Clearly the student in the preceding example is not able to see the logical form of what the teacher saidi.e., the student has committed a rottweiler flip. Because the

A YORKIE FLIP

160 PART II PRACTICE


student is not able to standardize, the student has not picked a valid counterexample; i.e., the counterexample does not work to disprove what the teacher said. What has happened in the preceding example is that the student believes that s/he is an exception, or a counterexample, to the teachers own rule. She believes that this is the case because she believes that the logical form of what this teacher said is that all people who pass the exam are people who pass the course. However, what the teacher actually said was that a student could only pass the course if s/he passed the exami.e., that all people who pass this course are people who pass the exam. The only valid counterexample to what the teacher actually said would be a student who passed the course but who did not pass the exam, not a student who passed the exam but did not pass the course. When faced with such misunderstandings, you need to be able to clarify the situation and show the individual in question why the counterexample used is not legitimate.

EXErCISE 13a
Responding to misunderstandings
the following are all examples of mishearing what a speaker has said. respond in a way that clears up the misunderstanding. In particular, (1) show that there has been a flip between necessary and sufficient conditions, (2) state what the speaker said in standardized form, and (3) articulate what would be a legitimate counterexample to what the speaker said. (See the comic entitled a rottweiler Flip: Mistaking a Necessary for a Sufficient Condition on page 147.)

Example of clearing up a misunderstanding


argument

Mypsychologyteachersaidthatallsuper-intelligentpeopleareneurotic.Butthat cantberight.Myauntisasnuttyasafruitcake,butsheisadumbasapancake.
response

Youmisunderstoodyourteacher.Yourteachersaidthatbeingsuper-intelligent wasasufficientconditionforbeingneurotic,notanecessarycondition. Whatyourteachersaidisthatallsuper-intelligentpeopleareneurotic. Theonlycounterexamplethatwouldshowthatwhatyourteachersaidwas faultywouldbeasuper-intelligentpersonwhowasnotneurotic,notaneurotic individualwhowasdumb.

1. The teacher told me that I could only pass the course if I passed the exam. Well, I passed the exam, and now the teacher tells me that I am not going to pass anyway because my written work is weak. That is so typical of teacherschanging the rules whenever they please. 2. The chief examiner told me that I would qualify to get in if I got straight As on my exams. Well, I got straight As on my exams, and I got in. Then I started to talk to some of the other people who were admitted, and I found out that a lot of them had only a B average. What am I, chopped liver? How come I had to have straight As and no one else did? 3. What a crummy magazine. The so-called love expert wrote that passion is essential for a long-lasting relationship. Well, no two people could have had more passion in their relationship than John and I had, and he dumped me last

SECTIon 1 LEARnIng ThE InTRICACIES of PRACTICAL REASonIng

161

week after only a three-month relationship. I hardly call three months long term. That just goes to show you what that so-called love expert knows. 4. I heard the minister say that whoever is really religious will show their love of God through great works of charity. That is so typical of that sort of exclusive thinking that religion perpetuates. I know a lot of people who devote themselves to the welfare of others but who are not religious. Some are even atheists. 5. You have to have a size 40 bust to be able to get an interview to be the pin-up girl for Playboy. That is exactly what it said in the ad. Jennifer showed it to me. Well, Jennifer sent in a picture of herself along with her measurements, and I can assure you that she has at least a size 40 bust. But do you know what they told her? They told her that they would call her and not to bother calling them. I dont know why they put ads like that in their magazines. It is so dishonest.

EXErCISE 13B
Multiple-choice questions for misunderstandings
Instructions for the first three questions are provided; for the last seven questions, indicate the most appropriate response. Answers to the first six questions are in the back of the book. 1. Dr. Burns: If the medicine arrives today, the baby will be saved. Dr. Mills: No, if the medicine arrives tomorrow, it will be just as good for the baby. Dr. Millss reaction to Dr. Burnss assertion indicates that she understood Dr. Burns to mean that a. Nothing but the medicine will save the baby. b. If the medicine arrives today, it will not arrive tomorrow. c. Today is the only day that the medicine can arrive. d. Medicine arriving tomorrow will be of no use. e. Medicine will save the baby only if it comes today. 2. Vogel: Every painting done by Pablo Picasso is a masterpiece. ernst: Thats not true. Several masterpieces by Jacques-Louis David and Eugne Delacroix hang in the Louvre in Paris. Ernst apparently assumes that Vogel said: a. Picasso only painted masterpieces. b. All masterpieces were painted by either David or Delacroix. c. Only Picasso painted masterpieces. d. Only masterpieces by David and Delacroix hang in the Louvre. e. Some of Picassos masterpieces do not hang in the Louvre. 3. Jorge: To be a good carpenter, one must have patience. gloria: That is not so. A good carpenter must also have the right tools. Gloria has misunderstood Jorges statement to mean that a. If a person is a good carpenter, she will have the right tools. b. If a person is a good carpenter, she will have patience. c. If a person has patience, she will be a good carpenter. d. If a person has the right tools, she will be a good carpenter. e. If a person does not have the right tools, she cannot be a good carpenter. 4. My teacher said that you can only have faith if you have hope. But my grandmother proves him wrong. Shes a very optimistic person. She always hopes for the best and she doesnt have a religious bone in her body.

162 PART II PRACTICE


a. Your teacher is an idiot. b. You misunderstood your teacher. Your teacher said that having hope was a sufficient condition for having faith, not a necessary condition. c. You misunderstood your teacher. Your teacher said that having hope was a necessary condition for having faith, not a sufficient condition. My psychology teacher said that all super-intelligent people are neurotic. But that cant be right. My aunt is a highly intelligent person and she is not neurotic. a. Well, that certainly seems to show that what your teacher said was wrong. b. You misunderstood your teacher. Your teacher said that being super-intelligent was a sufficient condition for being neurotic, not a necessary condition. c. You misunderstood your teacher. The only counterexample that would prove your teacher wrong would be a really dumb person who was neurotic. I know about the philosopher who said that people can only be truly courageous if they are wise. But that just doesnt make any sense to me. I was reading about the history of Athens, and one of the pre-Socratics, a very wise fellow, recanted all his beliefs in order to avert the fate that eventually befell Socrates. It is true that drinking hemlock is no party, but it seems to me that this guy was a real coward. a. Yes, youre right. I guess that just goes to show that philosophers dont know everything. b. You misunderstood the philosopher. The philosopher said that being wise is a necessary condition for being courageous, not a sufficient condition. c. You misunderstood the philosopher. The philosopher said that being wise is a sufficient condition for being courageous, not a necessary condition. My teacher said you can only have genuine faith if you have integrity. However, I know lots of people who are really religious and who seem to have amazing faith yet who have very little integrity. Indeed, one might even want to include a certain president of the United States in that category. a. That just goes to show that what your teacher said was obviously false. b. You misunderstood your teacher. Your teacher said that having faith was a necessary condition for having integrity, not a sufficient condition. c. You misunderstood your teacher. Your teacher said that unless you have faith, you cannot have integritynot the other way around. Matt was saying the other day that wherever you find moose, you also find mosquitoes, because both mosquitoes and moose love swamps. But there must be something wrong with his reasoning because there are plenty of mosquitoes but not a single moose in Nova Scotia. a. You are absolutely right. There is something wrong with Matts reasoning. b. You misunderstood Matt. Matt said that the presence of mosquitoes is a sufficient condition for ensuring that moose are present, not a necessary condition. c. You misunderstood Matt. Matt said that the presence of mosquitoes is a necessary condition of there being moose in the area, not a sufficient condition. My teacher said that you can get into graduate school if you get a 4.0 grade point average. I do not know what she is talking about. I know lots of people who have gotten into graduate school who did not get a 4.0 grade point average. Maybe she is just trying to scare us into working harder. a. You misunderstood your teacher. She said that getting a 4.0 grade point average was a sufficient condition for getting into graduate school, not a necessary condition. b. You misunderstood your teacher. She said that getting a 4.0 grade point average was a necessary condition for getting into graduate school, not a sufficient condition.

5.

6.

7.

8.

9.

SECTIon 1 LEARnIng ThE InTRICACIES of PRACTICAL REASonIng

163

c. You are right. I guess you cannot blame her for telling a little white lie if she thinks that doing so will make us work harder. 10. It said in the sociology book that I was reading the other day that, more often than not, criminals come from highly dysfunctional families. But that cant be right. Most of the people I know come from dysfunctional families and they are not criminals. In fact, I came from a dysfunctional family, and I am not a criminal. a. You must read very carefully. Good for you for picking up on that silly generalization. b. You misunderstood what you read. The claim was that a dysfunctional family is, more often than not, a necessary condition for eventually resorting to a life of crime, not a sufficient condition. c. You misunderstood what you read. The claim was that a dysfunctional family is, more often than not, a sufficient condition for ensuring that one will eventually resort to a life of crime, not a necessary condition.

14. Deducing from Conditional or All Claims: Valid and Invalid Moves
One of the reasons why Homo sapiens are so successful as a species is our exceptional ability to deduce. If I see a child at the bottom of a swimming pool, I do not need to get into a long discussion about whether or not that is a good place for the child to be. Because I know that all humans need oxygen in order to live, I will jump into the pool on the basis of the deduction that this human needs oxygen in order to live. The point of the following exercises is not to teach you how to deduce correctly. You already know how to do that. The point is to try to stop you from deducing incorrectly. Unhappily, but not surprisingly, we are all prone to frequent incorrect deductions. This is so because a lot of our knowledge is encoded through associatione.g., oxygen is associated with life. Because of this, we tend to assume that when one of a pair of associated items is present, the other is present, and when one is absent, the other is absent. This is correct50 percent of the time. However, this is also incorrect 50 percent of the time. Thus, the point of the following exercises is to alert you to the fact that since there is a fifty-fifty chance that you might get it wrong, it is worth a moments reflection whenever you, or anyone else, make a deduction. The rule for getting it straight is very simple. A deduction is legitimate, or valid, when a sufficient condition (the antecedent of an if . . . , then or conditional statement) is present (or affirmed). A deduction is not legitimate when a sufficient condition is absent (or denied). Conversely, you may not make a deduction when a necessary condition (the consequent of an if . . . , then statement) is present, but you may make a legitimate deduction when a necessary condition is absent. If you think carefully about the meaning of sufficient, it becomes obvious that if p is sufficient for q, then if p is present, q must be present; otherwise p would not be sufficient for ensuring q. On the other hand, simply because p is sufficient for q, it does not follow that because p is absent, q will also be absent. After all, just because shooting someone in the heart is a sufficient condition for killing someone, it does not follow that no one was killed before guns were invented. Chopping off someones head is also sufficient, as is poisoning the person or burying someone alive. If you think carefully about the meaning of necessary, it also becomes obvious that if q is really necessary for p, then if q is absent, it follows that p will also be absent; if that were not the case, then q would not be necessary for p. On the other hand, simply because q is present, it does not follow that p will also be present. Even though chocolate is a necessary condition for making chocolate ice cream, it does not follow that simply because you have chocolate, you will be able to make ice cream.

You already know how to deduce what you need to know. What you need to learn is how not to deduce incorrectly.

If a sufficient condition is present, that for which it is sufficient must also be present.

If a necessary condition is absent, that for which it is necessary must also be absent.

164 PART II PRACTICE


Lets try to put the preceding rules into formulas. The box shows some symbols.
LogIC SYMBoLS
IF...,THEN:or NOT:~ AND:or& OR:v

Lets start by first taking the idea that life needs oxygen (for the moment, lets ignore anaerobic life). The formula for this could be written in the following way: L O (which translates literally as if there is life, then there is oxygen). Given that you carry this association around in your head, you can meet four possibilities in the real world that relate to this bit of knowledge: (1) the presence of life, (2) the absence of life, (3) the presence of oxygen, and (4) the absence of oxygen. In each of these cases, you have to figure out whether or not you can correctly deduce the presence or absence of its associate. Lets put each of these possibilities into a full argument. Remember the following two rules: If a sufficient condition is present, that for which it is sufficient must also be present. If a necessary condition is absent, that for which it is necessary must also be absent. All other deductions from all, or conditional, sentences are invalid. Since you already understand standardization, you know that in the formula L O, L is the sufficient condition and O is the necessary condition.
ChEAT ShEET foR VALID AnD InVALID DEDUCTIonS fRoM ALL oR ConDITIonAL SEnTEnCES
1. 2. 3. 4.

BitofknowledgeLOLO LO Trigger L ~L _________ _________ ThedeductionTherefore,O Therefore,~O

LO O ~O _________ _________ Therefore,LTherefore,~L

Affirmingthe sufficient condition (modus ponens)

Denyingthe sufficient condition

AffirmingtheDenyingthe necessarynecessary conditioncondition (modus tollens)

valid

invalid

invalid valid

NOTE:Itiscommontouseahorizontaldividinglinetoindicatethattheconclusionappearsbelow.

SECTIon 1 LEARnIng ThE InTRICACIES of PRACTICAL REASonIng

165

The name for each of the preceding arguments is provided by what triggered the deduction (the triggers are in bold). You can think of the trigger as what started, or triggered, the thought process. You can think of the major premise as the bits of knowledge to which you have access. In argument 1, since the presence of life is the trigger, and since the presence of life stands in the sufficient position in L O, this argument is called affirming the sufficient condition or affirming the antecedent. The Latin name for this argument is modus ponens. In argument 2, the trigger is the absence of life. Since the presence of life is sufficient condition in the conditional L O, and it is the absence of life that triggers the argument, this argument is called denying the sufficient condition or denying the antecedent. In argument 3, the trigger is the presence of oxygen. Since the presence of oxygen is the necessary condition in the conditional L O, this argument is called affirming the necessary condition or affirming the consequent. In argument 4, the trigger is the absence of oxygen. Since oxygen is the necessary condition in the conditional L O, this argument is call denying the necessary condition or denying the consequent. The Latin name for this argument is modus tollens. NOTE: There are all kinds of other deductions that have equally fancy Latin names. For our purposes, however, we stick mainly to those described previously, although we also tangentially refer to the disjunctive syllogism.
DISJUnCTIVE SYLLogISM
SomeonesaystoyouthatJaneonlydatesguyswhoareeitherwealthyorgood looking.YouhappentoseeJaneattherestaurantwithadistinctlyuglyman.Ifyou believewhatthespeakersaysaboutJane,youcancorrectlydeducethattheman atthedinnertablewithJanewaswealthy.Thedisjunctivesyllogismlookslikethis. Ifyoudenyoneelementofadisjunct,thenitfollowsthattheotheristrue. If you deny one element of a disjunct, then it follows that the other is true.

pvq ~p ______ q

pvq ~q ______ p

RESPONDING TO DEDUCTIVE ARGUMENTS When someone presents you with a deductive argument, your first job is to figure out what kind of deductive argument it isi.e., you need to name it. you name an argument by looking at the trigger: Is it affirming the sufficient condition, denying the necessary condition, and so forth? Only by naming the argument will you know whether or not it is valid or invalid. For conditional syllogisms, remember the simple rule that of the four possibilities discussed, the two arguments that are valid are affirming the sufficient condition and denying the necessary condition. Simply saying that a sufficient condition is present or a necessary condition is absent should make it obvious that these are the two conditions under which a deduction from a conditional claim is valid. After you know whether or not the argument is valid or invalid, you are in a position to respond to the individual presenting the argument. Your response will differ depending on whether or not the argument is valid or invalid. Let us deal with the former first.

Validity is a necessary but not sufficient condition for soundness.

BAD DEDUCTIVE REASONING CAN LEAD TO BAD DECISIONS (I)

166

BAD DEDUCTIVE REASONING CAN LEAD TO BAD DECISIONS (II)

167

168 PART II PRACTICE


RESPONDING TO VALID DEDUCTIVE MOVES: GRANTING VALIDITY BUT NOT SOUNDNESS If the argument with which you are presented is valid, you should grant validity, yet withhold, at least temporarily, granting soundness. It is important that you grant validity promptly. If you do not, you may find that the individual will endlessly repeat the argument until you recognize its validity. However, you should withhold granting soundness until you have had the opportunity to investigate and/or discuss the truth of the premises. (Remember that the definition of a valid deductive argument is one such that if the premises are true, the conclusion must be true. The definition of a sound argument is a valid argument whose premises are true.)
In responding to a valid deduction from a conditional claim, you should grant validity without granting soundness: Your argument is certainly valid. If it is true that (premise 1), and if it is true that (premise 2), and if it is true that (premise 3), then it certainly follows that (the conclusion). However, I am concerned about (the truth of a particular premise).

Example of a Response to a Valid Deductive Argument


argument

Alllifeneedsoxygen.Thus,sincethereisnooxygenonMars,therecanbeno lifeonMars. Theformulafortheprecedingargumentlookslikethis: LO ~O ~L Thenameoftheargumentisdenyingthenecessarycondition. Thisargumentisvalid.


response

Yourargumentiscertainlyvalid.Ifitistruethatalllifeneedsoxygen,andifitis truethatthereisnooxygenonMars,thenitcertainlyfollowsthatthereisnolife onMars(butIamconcernedaboutyourfirstpremise,andsoforth).

RESPONDING TO INVALID DEDUCTIVE MOVES: EXPOSING THE ROTTWEILER FLIP If the argument is invalid, you need to do more than simply say that. The goal, after all, is to facilitate communication so that you, and those with whom you communicate, can move toward truth. Thus, if you are presented with an invalid argument, your job is to show how it is invalid. One of the most common mistakes in deductive arguing is to switch necessary and sufficient conditions (a Rottweiler flip). If that happens, your response should point that out. In order to know how to respond, you need to refer once again to Exercise 11F. After filling in the blanks, you should have been able to deduce the following two rules. If someone deduces something from the fact that x is present, s/he must be assuming that x is a sufficient condition. If someone deduces something from the fact that x is absent, s/he must be assuming that x is a necessary condition. This rule helps you expose the Rottweiler flip.

SECTIon 1 LEARnIng ThE InTRICACIES of PRACTICAL REASonIng

169

Example of a response to an invalid deductive argument


argument

ThemedicalschooltoldJohnthathewouldhavetogetstraightAsinorderto getin.Iamsohappyforhim.JohnworkedreallyhardandhegotstraightAs.So thatmeansthathisentranceintomedicalschoolisadefinite. Theformulafortheprecedingargumentlookslikethis. MA A M Thenameofthisargumentisaffirmingthenecessarycondition.


response

Yourargumentisinvalid.FirstyousaythatgettingstraightAsisanecessary conditionforgettingintomedicalschool,andthenyouimplythatitissufficient. Whichisit? Noteintheprecedingexamplethatyoumustsay,Youimplythatgetting straightAsisasufficientcondition,becausethepersondidnotactuallysay it. ThereasonthepersonimpliedthatgettingstraightAswasasufficientcondition forgettingintomedicalschoolisbecauses/hededucedthatsomethingfollowed(goingtomedicalschool)fromthefactthatsomethingwaspresenti.e., straightAs(seeExercise11F).

In responding to an invalid deduction from a conditional claim, you should explain the source of the invalidity i.e., the Rottweiler flip: Your argument is invalid. First you say that p is a sufficient (or a necessary) condition for q, and then you imply that it is a necessary (or sufficient) condition.

EXErCISE 14a
Argument forms
1. iagram the following arguments: D p q p _____ q 2. Name theme.g., affirming the necessary condition (or the consequent). 3. ndicate, in a well-phrased sentence, whether the argument is valid or invalid I and why. For example: Your argument is certainly valid. If it is true that . . . , and if it is true that . . . , then it certainly follows that. . . . Or: Your argument is invalid. First you said that q was a necessary condition for p, and then you implied (by deducing from its presence) that it is a sufficient condition, or the reverse. Answers are in the back of the book. 1. Everyone knows that if you think positively, you will be a success. I guess we can predict that Johnny will never amount to much of anything; he is always so negative. 2. Our real estate agent told us that we would be able to sell our house only if there is a good real estate market. Well, the real estate market is booming, so I guess we will be successful.

170 PART II PRACTICE


BAD DEDUCTIVE REASONING CAN LEAD TO BAD DECISIONS (III)

3. Have you heard that a strike has been called? I guess that means that the plant is in real trouble. The Chairman of the Board told us the other day that unless workers agreed not to go on strike, prospects for recovery of the plant are dim. 4. Anyone with any courage would speak out against such practices. I guess we can be sure that Judith wont say anything; she is such a coward. 5. Steve must have had a terrible education. He hates to learn anything new, and we all know that people retain their love of learning if they have a good education. 6. Our teacher told us that only the good die young. That just goes to show how much he knows; some of the noblest people ever known have lived to a ripe old age. Socrates is a prime example.

SECTIon 1 LEARnIng ThE InTRICACIES of PRACTICAL REASonIng

171

7. John cant be the murderer; he has no jewels, and Sherlock said that whoever has the jewels must be the murderer. 8. Teenager: Mom and Dad told you that I could go out only if it was safe. I heard them saying so. Well, three of my friends have arrived to go to the movies with me, so you can see that I will be safe. Babysitter: That is not the point, you havent done your homework. 9. I heard that your cousin is a model. She must be really beautiful. Everyone knows that you cant be a model unless you are beautiful. 10. I dont see how you can say that cyclists should be required to wear helmets. Skiing is just as dangerous, perhaps even more dangerous. Why shouldnt skiers be required to wear helmets?

EXErCISE 14B
Argument forms
1. Diagram the following arguments: p q p _____ q 2. Name theme.g., affirming the necessary condition (or the consequent). 3. Indicate, in a well-phrased sentence, whether the argument is valid or invalid and why. For example: Your argument is certainly valid. If it is true that . . . , and if it is true that . . . , then it certainly follows that. . . . Or: Your argument is invalid. First you said that y was a necessary condition for x, and then you implied (by deducing from its presence) that it is a sufficient condition, or the reverse. The answers are in the back of the book. 1. You will not live unless you get these drugs. However, these drugs are so expensive, that you will not be able to afford them unless you make some extra money. You will be able to make extra money only if you are able to work overtime. However, this disease will make you so tired that you wont be able to work overtime. It looks like you are toast, kid. 2. Our teacher said that you can get into graduate school only if you get first-class grades. She also said that anyone who puts in two hours of homework for every hour in class is bound to get first-class grades. Ben is taking a full course load and he puts in forty hours of homework per week. He is sure to get into graduate school. 3. Unless the workers agree not to strike within the next decade, prospects for the recovery of the plant are dim. However, unless management agrees to forgo special parking and washroom privileges, workers will not agree not to strike. So there can be no recovery in the plant because the stupid management insists on retaining their special parking privileges. 4. If you are beautiful, then you are loved. You are not beautiful. Therefore, you are not loved. 5. Picassos Guernica has merit only if its appreciated by most people, which it is. Hence, I think we can conclude that it does have artistic merit. 6. Denise must not be home. She said that if the light was on when we came by, we could be assured that she had arrived home safely. However, the light is not on.

172 PART II PRACTICE


BAD DEDUCTIVE REASONING CAN LEAD TO BAD DECISIONS (IV)

7. Cynthia told me that if she failed Philosophy 110, she would quit college. Well, she hasnt shown up this term, so I guess she must have failed. 8. If capital punishment actually deterred people from committing capital crimes, then it would be justified. However, since it does not have that deterrent effect, it is not a justifiable practice. 9. You are either with us or against us. If you prevent us from sending troops into Iraq, then the country will destabilize. If Iraq is destabilized, its oil production will be interrupted. If its oil production is interrupted, that will wreck our economy. Clearly, if you dont care whether our economy is wrecked, then you can hardly claim to be with us. Thus, the fact that you are trying to prevent us from sending troops into Iraq shows that you are against us.

SECTIon 1 LEARnIng ThE InTRICACIES of PRACTICAL REASonIng

173

10. The murderer was either Mr. Plum or Mrs. White. We would know that the murderer was Mr. Plum if there were large footprints detected. We would know it could not have been Mrs. White unless there was the smell of perfume. Footprints would only have been detected if it was not raining last night. However, it did rain last night, so there were no footprints. There was no smell of perfume, so the murderer must have been Mr. Plum.

EXErCISE 14C
Argument forms
1. Diagram the following arguments: p q p _____ q 2. Name theme.g., affirming the necessary condition (or the consequent). 3. Indicate, in a well-phrased sentence, whether the argument is valid or invalid and why. For example: Your argument is certainly valid. If it is true that . . . , and if it is true that . . . , then it certainly follows that. . . . Or: Your argument is invalid. First you said that y was a necessary condition for x, and then you implied (by deducing from its presence) that it is a sufficient condition, or the reverse. The answers are in the back of the book. 1. You cannot have faith without hope. Only love can bring you hope. If you are a loving person then you are a kind person. So you see, if you are kind, ultimately you have faith. 2. If only love can bring you faith, then John must be a very religious individual. He certainly is a loving individual. 3. You cannot truly love unless you are kind. Joanne is mean to Eddie, so obviously she cant truly love him. 4. You can hope if you have genuine faith. There can be hope only in a world filled with love. This is a world of faithless people. So it will never be a world filled with love. 5. Kind people are loving people. However, only people with faith can really love. So one can hardly say that faith is dead in the modern world. After all, this world is filled with kind individuals. 6. You can only love if you have faith. On the other hand, you cannot love if you have no charity. There is very little charity in this world. So I fear that we must conclude that there is very little faith. 7. We have got to give the patient either drug A or drug B. We cannot give drug A unless the patient is at least ten pounds underweight. We can give drug B only if his blood pressure is high. This patient is fifteen pounds overweight, but his blood pressure is normal. So we should give him drug B. 8. You will have integrity if you are kind. Anyone with integrity will value honor. Since the world is filled with people who are kind, that shows that honor is not dead yet. 9. Hope will come to those who have faith. Unless you have faith, you cannot have peace. So it is important that we never lose hope. If we do, we will never achieve peace.

174 PART II PRACTICE


10. It says in the good book that to love others means that you must be a genuinely moral person. However, you can only love others if you take the time to seriously listen to them. Graham is always working. He has no time to socialize at all, let alone seriously listen to what others have to say. I guess it follows that Graham is not moral.

BAD DEDUCTIVE REASONING CAN MISPLACE VALUES (AND ASSOCIATED RESPONSES)

SECTIon 1 LEARnIng ThE InTRICACIES of PRACTICAL REASonIng

175

EXErCISE 14D
Argument forms
1. Diagram the following arguments: p q p _____ q 2. Name theme.g., affirming the necessary condition (or the consequent). 3. Indicate, in a well-phrased sentence, whether the argument is valid or invalid and why. For example: Your argument is certainly valid. If it is true that . . . , and if it is true that . . . , then it certainly follows that. . . . Or: Your argument is invalid. First you said that y was a necessary condition for x, and then you implied (by deducing from its presence) that it is a sufficient condition or the reverse. 1. Elephants have been known to bury their dead. However, elephants bury their dead only if they have a concept of their own species and understand what death means. If elephants understand what death means, they have a substantial capacity for abstraction. Therefore, elephants have a substantial capacity for abstraction. 2. The trainer said that in order to get picked for the national three-day event team, your horse has to be able to gallop at five hundred meters per minute. She also said that if you trained your horse according to the schedule set up in the Olympic Conditioning Manual, it should be able to do an easy gallop at five hundred meters per minute in about six months time. Natalie has been following that schedule for just about six months now. She is bound to get picked for the team. 3. Morality has a basis only if there is a God. There is a God only if the world makes sense and is ordered. The world is ordered, so there is a basis for morality. 4. Children will not be interested in reading unless schools supply them with interesting books. The books supplied in our schools are deplorably boring and simpleminded. They are not interesting. We can expect our children will not be interested in reading. 5. Please go and talk to Tony. He is devastated. The only thing he has ever wanted to be is a medical doctor, and now his chances are nil. The medical school told Tony that if he passed the biomedical ethics exam, he would be admitted. However, he found out this morning that he failed. Maybe you can convince him to apply to vet school. 6. You cannot claim to have a genuine sense of justice unless you have the emotional underpinning of love for others. All great religions preach that we should love one another. It is for that reason that we should have more confidence that our great religions can set us on the path to justice. 7. What is the matter with you? You told me the other day that you only go out with boys who are good-looking, so how could you turn down Paul? He is the best-looking guy in school! 8. I am not going to believe that you really care about learning anything unless you study hard. I will believe that you studied hard only if you devoted a significant amount of time to reviewing. You cannot possibly have devoted a significant amount of time to reviewing if you are coming to me for help the night before the exam. So, frankly, I have no sympathy for you. I really do not believe that you care about learning anything.

176 PART II PRACTICE


9. Our professor always said that if you are tough on yourself and on those with whom you communicate about using precise terms, you will be more successful in communicating. If you are successful in communicating, you will be able to get a more accurate picture about how others view any given situation. The more points of view that you are able to access, the better position you will be in to move toward truth on any issue. Being able to move toward truth on any issue, in turn, increases the possibility of attaining a good life. So I understand now why our professor was so tough on us about using precise terms. She loves us! She wants us to have a good life! 10. Either the murderer used a kitchen knife or the murderer carried an unusually large pocketknife. If the murderer carried an unusually large pocketknife, then he was wearing loose clothing or he would have been noticed by the neighbors. The neighbors did not notice the murderer. If he was wearing loose clothing, then he would have to have been very thin. We know the murderer was not thin. Therefore, we know that he used a kitchen knife.

EXErCISE 14E
Argument forms in multiple-choice questions
Answers are in the back of the book. 1. In a game of Monopoly, if a player owns a hotel on Boardwalk, he must own both Boardwalk and Park Place. If he owns a hotel in Marvin Gardens, he must own Marvin Gardens and either Boardwalk or Park Place. If he owns Park Place, he also own Marvin Gardens. If the player described previously does not own Park Place, which of the following conclusions may be drawn? a. The player owns a hotel on Boardwalk. b. The player owns a hotel on Marvin Gardens but does not own a hotel on Boardwalk. c. The player owns Marvin Gardens and Boardwalk but does not own a hotel on either property. d. The player does not own a hotel in Marvin Gardens. e. The player does not own a hotel in Boardwalk. 2. Every year, the members of the school board PTA select a new student representative. If the school board PTA selects a senior as the student representative, then the PTA will give the high school money for a spring musical. However, the school board PTA has already given the school money for a spring musical. If all the statements in the preceding argument are true, which of the following conclusions must also be true? a. The PTA should not select a senior as the student representative. b. The PTA already had given enough money to the high school. c. The current student representative is a junior. d. If the PTA does not give any additional money to the high school, then the PTA must not have selected a senior as its student representative. e. If the PTA gives more money to the high school this year, then its student representative must be a senior. 3. If Alice told Bob about the surprise party, then Bob told Carl about the surprise party. If Carl knew about the surprise party, then Danielle knew about it. However, Danielle did not know about it. Which of the following statements can be deduced from the information given?

SECTIon 1 LEARnIng ThE InTRICACIES of PRACTICAL REASonIng

177

4.

5.

6.

7.

I. Alice did not tell Carl about the party. II. Alice did not tell Bob about the party. III. Carl did not know about the party. a. I only. b. II only. c. I and II only. d. I and III only. e. I, II, and III. Some politicians are boating enthusiasts. All Republicrats favor industrial development along the shoreline. All boating enthusiasts oppose industrial development along the shoreline. If the preceding statements are true, which of the following must be true? a. All politicians are Republicrats. b. Some Republicrats are boating enthusiasts. c. Some politicians are not Republicrats. d. Not all Republicrats are politicians. e. Some boating enthusiasts are Republicrats. Four students are competing in the final round of a debating match. If Alan scores more points than Beth or if Cathy scores more points than David, then Cathy will be the winner. If David scores more points than Alan, then Beth will be the winner. If Cathy scores more points than Beth, then Alan will be the winner. If David is the winner, then which of the following statements must be true? I. David scored more points than Alan. II. Neither Cathy nor Alan scored more points that Beth. III. David did not score more points than Alan. a. III only. b. I and II only. c. I and III only. d. II and III only. e. I, II, and III. Jan really wants to go to the party, but she said wouldnt go to the party unless Sally went. However, Sally said she would only go to the party if she had a new dress. Sally cant buy any new clothing until she gets a raise. If Sally goes to the party, we can assume that I. Jan goes to the party. II. Sally has a new dress. III. Sally got a raise. a. I only. b. II only. c. II and III only. d. I and III only. e. I, II, and III. A philosopher makes the following statements: I think, therefore I am. If I am not, then I think not. If I think, then life means nothing. applying the preceding argument, if life does not mean nothing, then what more can the philosopher conclude? a. I am. b. I think. c. I do not think. d. I think and I am e. I think not and I am.

178 PART II PRACTICE


8. Price and wage controls are the only way to control inflation. However, wage controls limit worker spending, which, in turn, results in reduced corporate profits if price controls are in place. assume the preceding statements are true. Which of the following statements must also be true if corporate profits are not decreasing? a. If there is inflation, wage controls are not in place. b. If there is inflation, it is not being controlled. c. Workers have less money to spend. d. Price controls are in effect. e. Wage controls are in effect. 9. If Robert attends the party, then Sally, Tess, and Victor will also attend the party. If the preceding statement is true, which of the following statements must also be true? a. If Robert does not attend the part, then Sally does not attend the party. b. If Sally, Tess, and Victor attend the party, then Robert will also attend the party. c. If Robert and Sally attend the party, then Tess and Victor will not attend the party. d. If Victor does not attend the party, then Robert does not attend the party. e. If Tess does not attend the party, then Sally does not attend the party. 10. Eight authors are competing for a literary prize, to be awarded according to the votes of a panel of judges. The panel is divided into various voting blocs. If Anderson receives more votes than Benchley, and Carson receives more votes than Dinsmore, Ellman will win the prize. If Benchley receives more votes than Anderson, or Farley receives more votes than Gonzalez, Hawthorne will win the prize. If Dinsmore receives more votes than Carson, Farley will win the prize. If Hawthorne wins the prize, which of the following must be true? a. Anderson received more votes than Benchley. b. Benchley received more votes than Anderson. c. Farley received more votes than Gonzalez. d. Carson received more votes than Dinsmore. e. Dinsmore did not receive more votes than Carson.

EXErCISE 14F
Argument forms in multiple-choice questions
1. If the wind is strong, kites are flown. If the sky is not clear, kites are not flown. If the temperature is high, kites are flown. assuming the preceding statements are true, if kites are flown, which of the following statements must be true? (1) the wind is strong. (2) the sky is clear. (3) the temperature is high. a. b. c. d. e. 1 only. 2 only. 3 only. 1 and 3 only. 2 and 3 only.

SECTIon 1 LEARnIng ThE InTRICACIES of PRACTICAL REASonIng

179

2. When the International Restaurant serves bacalao and mondongo on the same day, it also serves empanadas. Empanadas are never served on Tuesday. Manuel will eat at the International only if bacalao is being served. assuming the preceding statements are true, which of the following must also be true? a. Manuel does not eat at the International on Tuesday. b. Manuel does not eat both bacalao and mondongo on the same day at the International. c. The International does not serve mondongo on Tuesday. d. Tuesday is the only day that the International does not serve bacalao. e. Bacalao and mondongo are not both served at the International on Tuesday. 3. All weavers are members of the union. Some carders are women. Some weavers are women. All union members are covered by health insurance. No carders are covered by health insurance. all of the following can be drawn from the preceding statements except a. All weavers are covered by health insurance b. Some women are covered by health insurance. c. Some women are not covered by health insurance. d. Some members of the union are not covered by health insurance. e. No carders are members of the union. 4. Which of the following workers would provide a counterexample to the preceding statements? a. A man who is a carder. b. A weaver who is not covered by health insurance. c. A carder who is not covered by health insurance. d. A worker covered by health insurance who is not a weaver. e. A worker covered by health insurance who is not a carder. 5. The workers will only get a holiday if the boss is not in town. When there are torrential rains, workers will get a holiday. The workers will get a holiday if the machinery breaks down. assuming the preceding statements are true, if the workers get a holiday, which of the following statements must be true? (1) the boss is not in town. (2) there are torrential rains. (3) the machinery has broken down. a. 1 only. b. 2 only. c. 1 and 2 only. d. 1 and 3 only. e. 1, 2, and 3. 6. A philosopher makes the following statements: I am; therefore, I think. If I am not, then I think not. I cannot think unless there is peace. applying the preceding argument, if there is no peace, then what more can the philosopher conclude? a. I am. b. I think. c. I am not, but I think. d. I think not, but I am. e. I think not, and I am not.

180 PART II PRACTICE


7. Either Alice loves Bill or David loves Carly. If Alice loves Bill or David loves Carly, then Elizabeth will be happy. If Peter is happy, then it is clear that David loves only Alice. If Peter is not happy, then Paul will be happy. Paul is not happy. assuming the preceding statements are true, which of the following are true? (1) Elizabeth is happy. (2) David does not love Carly. (3) Peter is happy. a. 1 only. b. 2 only c. 2 and 3 only. d. 1 and 3 only. e. 1, 2, and 3. 8. Picassos Guernica has merit only if it is appreciated, which it is. Hence, I think we can conclude that it does have artistic merit. a. Your argument is valid. b. Your argument is invalid. First you say that having merit is a sufficient condition, and then you imply it is a necessary condition. c. Your argument is invalid. First you say that being appreciated is a necessary condition, and then you imply it is a sufficient condition. 9. Kind people are loving people. However, only people with faith can really love. So one can hardly say that faith is dead in the modern world. After all, this world is filled with kind individuals. a. Wow, that is amazing. I guess religion is not dead after all. b. Your argument is invalid because you are affirming the necessary condition. c. Your argument is invalid because you are denying the sufficient condition. 10. Our teacher said that you can get into graduate school only if you get first-class grades. She also said that anyone who puts in two hours of homework for every hour in class is bound to get first-class grades. Ben is taking a full course load and he puts in forty hours of homework per week. He is sure to get into graduate school. a. You are absolutely right. I am sure that Ben is going to get into graduate school. b. Your argument is invalid because you affirmed a necessary condition. c. Your argument is invalid because you denied a sufficient condition.

15. Overview
To think impartially, one needs to master the way, which is the focus of Part II (as well as the will, which was, it is hoped, inspired by Part I). This section of Part II has explored some of the logical skills necessary to acquire the capacity to objectively judge the merits of any argument. To solidify those skills, it is suggested that you take some time to reflect on the degree to which those skills are actually beginning to infiltrate into your daily living. To that end, it is suggested that you answer the following questions, each of which is based on the subsections contained herein. Then put together a short personal synopsis of what you think will have the most powerful impact on your life. Include in your answer anything touched on in this textbook, including pre- and post-test scores.

SECTIon 1 LEARnIng ThE InTRICACIES of PRACTICAL REASonIng

181

EXErCISE 15a
Is the Knowledge Transferring?
1. reasons versus evidence Are you sufficiently comfortable with the distinction between empirical and nonempirical to know when it is appropriate to ask, What is your evidence? and when it is appropriate to ask instead, What are your reasons? Give a precise example if possible. 2. Precision To what degree do you believe that you are more diligent in requiring precision both from yourself and others? Give a precise example if possible. 3. What is a good question? Are you beginning to wake up to your own world? Are you beginning to ask questions and see potential dilemmas in your own life that heretofore you might have missed? Give precise examples where possible. Are you able to immediately recognize when a thesis statement is sufficiently contentious? 4. Fallacies Are you sufficiently familiar with common informal fallacies that you are able to hear them and to respond appropriately? Give precise examples if possible. 5. Deductive arguments Will you remember always that the definition of a valid deductive argument is that if the premises are true, the conclusion must be true? Are you convinced that knowing the difference between validity and soundness can serve as a powerful logical tool? 6. Finding the hidden premise Will you forever remember that a reason becomes a reason only in a whole argumenti.e., one that has at least two premises? Have you had sufficient practice in finding the hidden premise that you are able to hear it even when supplied with only one reason? 7. Evaluating reasons Are you completely convinced that the value of any reason is always a function of the degree to which the premises of the entire argument survive falsification by exposure to counterexample? 8. Establishing local sufficiency through parallels Are you confident that you can test the local sufficiency of your own reasoning as it applies to issues that are of personal importance? 9. Establishing global sufficiency through minis Have you had sufficient practice in objectively comparing the merits of your original position on a particular issue with the merits of your strongest opposition such that you are confident that you will now follow reasons wherever they lead, even if it requires that you back down from your original position? 10. Standardizing and hence counterexamples Have you sufficiently mastered standardizing ordinary-language claims so that you are confident that you can accurately pick out counterexamples? Are you confident that you can use the Rottweiler/dog tactic to confirm the order of a standardized claim?

182 PART II PRACTICE


11. Sufficient and necessary conditions Are you confident that you can translate any hidden all claim into one about sufficient and necessary conditions so as to ensure accurate meaning transmission? Are you convinced that Rottweiler flips are as dangerous as they are frequenti.e., that there is all the difference in the world between saying, for example, that an operation is sufficient (but not necessary) to save a life as opposed to saying an operation is necessary (but not sufficient) to save a life? 12. Seeing the hidden premise in forced-choice situations If you are ever required to take an exam that contains questions that require an understanding of assumptions that underlie argumentsi.e., hidden premises will you remember to review the logic contained herein? 13. responding to incorrect counterexamples Are you able to correct Rottweiler flips that result in people believing, incorrectly, that they have falsified a claim by posing an inaccurate counterexample? 14. Learning when to appropriately grant validity but not soundness and to show wherein invalidity resides (i.e., in rottweiler flips) Have you had sufficient practice so that you are confident you are able to accurately distinguish between valid and invalid arguments; that when confronted with a valid argument, you have the tools to grant validity but not soundness; and that when confronted with invalidity, you are able to show wherein the formal fallacy lies by exposing the Rottweiler flip? 15. reorganizing the whole package in your own mind Having now reviewed the road recently traveled, and after reviewing in chart form what both a good and a poor thinker look like (Appendix IV), are you confident that if, in the future, you lose your way, you will know where to look to find guidance for reinvigorating your capacity to think your way to freedom?

EXErCISE 15B
Travels with Diva
There is no doubt that different folks need different strokes, but we can all benefit by exploring seemingly tangential learning strategies that others may find central. The comics provided throughout this text may have seemed to many as simply a welcome distraction. To others, however, particularly those who are visual learners, they may have been the impetus to transfer many of the messages contained herein into longterm memory. The comics, in other words, can be another route to learning. Following is a list of the comic headings. Please review all the comics, rank-order the three that you personally found most inspiring or memorable, note the page, and state why you so ranked them. 1. Introduction Squandering Self-Consciousness (1) Part I: theory Section 1: the Possibility of Freedom How Is Freedom Possible? (9) Self-Control Is Not Autonomy (I) (12) Self-Control Is Not Autonomy (II) (13) Learning through Linguistically Enhanced Imagination Is the Human Trump Card (14) Language Tightens the Strings of Social Control (16)

2. 3. 4. 5. 6.

SECTIon 1 LEARnIng ThE InTRICACIES of PRACTICAL REASonIng

183

7. Materialism Threatens Freedom (17) 8. Freedom Requires Courage (19) 9. Self-Consciousness Is a Double-Edged Sword (21) Section 2: Impartial thinking 10. Social Determinism Can Be Invisible (23) 11. To Be a Chooser Requires Genuine Dialogue with Those Who Think Differently (26) 12. You Have to Engage Others in Order to Be Free of Them (27) 13. Truth through Falsification (30) 14. Seeking the Impossible Quest (32) 15. Truth Seeking Is Always a Two-Step Process (33) 16. Not Everyones Opinions Are Equally Good (34) 17. What Is Objective Is Faulty Reasoning, Not Truth (35) 18. Truth Seeking, Not Truth, Shall Set You Free (37) 19. The Stop and Go Signs toward Autonomy (38) Part II: Practice Section 1: Learning the Intricacies of Practical reasoning 20. Seeking a Good You Rather than Good Times (50) 21. Doing Nothing Is Not an Option: It Is Doing Something (51) 22. Wisdom versus Intelligence (53) 23. Freedom Is about the Future (55) 24. Imprecision Is Inefficient in Action Guiding (58) 25. Without Precision, Value Judgments Cannot Act as Magnets (61) 26. If You Do Not Care, You Are Not in Charge (63) 27. The Mind Is Opened by What Is Interesting; What Is Interesting Is Contentious (64) 28. Labeling Relevant Behavior Can Be Action Guiding (67) 29. Your Present Values Are Your Window into Your Future (68) 30. Are Your Emotions Those That You Choose? (69) 31. Values Become Yours Only When They Inform Your Actions (71) 32. You May Be Blind to the Person You Are Becoming (73) 33. A Reason That Is Not a Reason: Begging the Question (I) (77) 34. A Reason That Is Not a Reason: Begging the Question (II) (79) 35. Attacking the Arguer Rather than the Argument Misses the Point (81) 36. An Appeal to Tradition Can Sometimes Be a Reason (85) 37. Reasons Are Strong Only by Comparison: Bowling for Trutha Strawpin Argument (87) 38. Reasons Can Sometimes Connect A to Z: Slippery Slope Arguments (89) 39. Wisdom Varies with the Options Considered: False Dilemmas (90) 40. Own Up to an Accurate Description of Your Attitude: Distinction without a Difference (92) 41. Bad Reasoning Can Misplace Values (and Associated Responses) (94) 42. Analogizing Your Way toward Truth (96) 43. Truth Is a Function of Reason and/or Evidence, Not Mathematics: Fallacy of the Golden Mean (98) 44. Reasoning Changes Behavior by Moving Value (106) 45. Breaking the Major Premise (110) 46. The Wonders of Fault Finding (114) 47. I Am Talking about Rapists, Not Enron Executives (116) 48. I Am Talking about Homosexuals, Not Glasses (118) 49. What Would Count as Falsifying Your Belief? (120) 50. The Positivity of Negativity (123) 51. Me Only because of We (124) 52. Getting to Global Sufficiency by Testing Local Sufficiency through Counterexamples (128)

184 PART II PRACTICE


53. All Rottweilers Are Dogsi.e., Being a Rottweiler Is a Sufficient Condition for Being a Dog (146) 54. A Rottweiler Flip: Mistaking a Necessary for a Sufficient Condition (147) 55. A Yorkie Flip (159) 56. Bad Deductive Reasoning Can Lead to Bad Decisions (I) (166) 57. Bad Deductive Reasoning Can Lead to Bad Decisions (II) (167) 58. Bad Deductive Reasoning Can Lead to Bad Decisions (III) (170) 59. Bad Deductive Reasoning Can Lead to Bad Decisions (IV) (172) 60. Bad Deductive Reasoning Can Misplace Values (and Associated Responses) (174) Section 2: thinking and Writing Your Way to truth 61. It Is More Important to Be Right than to Appear Right (189) 62. Begging the Question: Boring Hypocritical Repetition (195) 63. Strawperson Argument: Bow-wow Chow (197) 64. Language Games (200) 65. Writing Can Be a Form of Interactive Listeningor Not (202) 66. When Self-Consciousness Takes Up the Challenge, It Is a Wonder to Behold (205)

Post-tests
After completing the following two post-tests, check your scores against the answers in Appendix V and against your scores on the pre-tests. You might want to discuss the difference in your scores when you share your responses to Exercises 15A and 15B with your classmates. It is not uncommon for a small subset of students to fail to really get the logic. If you are among those students, it is important to keep in mind that although understanding the logic of your language can certainly make your thinking more efficient, by far the most important lesson for you to glean from this journey is that your freedom depends on your ability to think impartially and that that in turn depends on articulately listening to those with whom you disagree. Thus, if you can remember to embrace your opposition (i.e., if you score well on the post-test that measures what kind of thinker you are), you will have come a long way in securing your autonomy and hence your identity.

Post-test 1: What Kind of Thinker Are You?


The following questionnaire was designed to help you understand yourself a little better and to help you estimate the extent to which you have changed the way you think since taking the pre-test. Please answer the questions as honestly as you can. This questionnaire is for your benefit only. For items 1 to 13, tick the column with the word that most appropriately fits in the round brackets provided. For items 14 to 17, tick the appropriate answer.
Often 1. I()askalotofquestions. 2. hendiscussingissues,I()getfrustratedwhenpeopledisagreewith W me. 3. tis()awasteoftimetotalktootherswhothinkcompletelydifferently I fromthewayIdo. 4. I()questionothersindetailinordertoclarifywhattheyhavejustsaid. 5. naheatedinterchange,whenothersaretalking,I()usethatopportunity I toconsiderhowIcanrestatemyownpositionmorepersuasively. 6. I()findithumiliatingtobackdown. 7. am()reluctanttochangemypositiononcertainissues,becausemy I positiononthoseissuesispartofwhoIam. 8. ()expressconfidenceinapositionevenif,atthetime,Iamunableto I backitupconvincingly. 9. have()saidthatsinceIdonothaveadequatedataorinformation,Iam I notinapositiontogiveanopiniononanissue. 10. I()findcomplexityannoying. 11. Iam()reluctanttodisagreewithpeoplewhoareimportanttome. 12. ()seriouslyquestionwhatagoodlifeisandwhatkindofpersonIwant I tobecome. 13. I()spendtimejustthinking. Yes 14. Ihaveapreciseideaofwhatagoodthinkerlookslike. 15. Icandefinegoodthinking. 16. hensomeoneelseusespoorthinking,Iunderstandandcouldexplain W whythethinkingisfaulty. 17. Havingalotofmoneyisnecessaryformywell-being. Dr. S. T. Gardner, Vancouver Institute of Philosophy for Children, 1998. Sort of No Sometimes Rarely Mark

185

Post-test 2: Logic
Circle the answer you think is the most appropriate. 1. A wise man once said, You can be truly successful only if you love what you are doing. If this is true, it follows that a. If you are not successful, that shows you do not love what you are doing. b. If you can find something you love to do, you know that you will be successful. c. You can know that there is no point in trying to do something you do not love to do, since you will never be successful at it. 2. If it is true that people are popular if they are beautiful, it follows that a. You can be sure your sister will not be popular, because she is not beautiful. b. You can be sure your cousin is going to be very popular, because she is beautiful. c. Both a and b. 3. If it is true that only citizens are allowed to serve on a jury, it follows that a. Since Manuel told me he is not allowed to serve on a jury, we can conclude that he has not become a citizen yet. b. All citizens are allowed to serve on a jury. c. Danielle must be a citizen of the United States, since she served on a jury when she was living in America. 4. If it is true that people cannot acquire diabetes in adult life unless they eat too many carbohydrates, it follows that a. All people who acquire diabetes later in life must eat too many carbohydrates. b. Your aunt is bound to get diabetes eventually, since she eats far too many carbohydrates. c. If your mother did not acquire diabetes in adult life, that shows she did not eat too many carbohydrates. 5. If it is true that only the lonely know how I feel tonight, it follows that a. All people who are lonely know how I feel tonight. b. You cannot possibly know how I feel tonight, since you said yourself you are not lonely. c. Since you do not understand how I feel tonight, obviously you are not lonely. 6. If it is true that without love, there can be no happiness, it follows that a. Since John is not happy, that shows he feels no love in his life. b. Jane is loved deeply by her family and friends; therefore, she is bound to be happy. c. Neither a nor b. 7. If it is true that you will never get elected to the student council if you tell people the truth about the issues, it follows that a. If you want to get elected, you should practice being deceitful, as being deceitful is necessary to get elected. b. Since John is a master at deceit, he is bound to get elected. c. Both a and b. 8. If it is true that intelligent people are trustworthy people, it follows that a. We can know that John is stupid, since he is completely untrustworthy. b. Since you can trust Harry with your life, he must be very intelligent. c. Neither a nor b.

186

9. If it is true that the person who ends up being the boss is the person who has been the least critical when working in the ranks, it follows that a. Since Joan is the first person to speak up when there is a problem, she will never be the boss. b. Since Mr. Jones is now the boss, he could not have complained much when he was working in the ranks. c. Both a and b. 10. There is a saying that strong men eat quiche. If this is true, it follows that a. If a man eats quiche, he must be strong. b. If a man refuses to eat quiche, he is not strong. c. If a man is weak, you can be sure he will not eat quiche. 11. Let us suppose that someone tells you that s/he has a magic ring and that if you put it on and it turns red, it means that you are angry. On the assumption that the ring is truly magical, it follows that a. Since the ring does not turn red, you are not angry, even if you think you are. b. Since you are not angry and you know it, it will not turn red. c. Since you are furious and you know it, you can be sure it will turn red. 12. The teacher said, This composition must have been written by a girl, because the handwriting is so neat. The teacher is assuming that a. All girls have neat handwriting. b. Some girls have neat handwriting. c. Only girls have neat handwriting. 13. John said, All cheerleaders are dumb. Jane said, That is not true. If Jane is right, it follows that a. Cheerleaders are not dumb. b. There are a lot of dumb people who are not cheerleaders. c. At least one cheerleader is not dumb. 14. The doctor told us that if children are vaccinated, they cannot get polio. Johnny got polio. If what the doctor said was true, it follows that a. Johnny was vaccinated. b. Johnny was not vaccinated. c. We cannot tell whether Johnny was vaccinated. 15. You heard someone at a party say, All women are emotional, by which I mean that women always cry when they get upset. If you wanted to demonstrate that the statement is false, the best way to do so would be to a. Show that many men are emotional (i.e., they cry when they get upset). b. Show that at least one woman is not emotional (i.e., she does not cry when she gets upset). c. Show that women in general are not emotional (i.e., they do not cry when they get upset).

187

188 PART II PRACTICE


A Personal Good-bye
NOTE: You should have already made good use of Part II, Section 2, in writing your mini-essays (see Exercises 9A and 9B). There can be no greater gift to yourself than genuinely becoming your own person. Knowing how to think for yourselfbecoming confident that you can impartially think through highly complex value issueswill give you an enviable sense of peace. It will supply you with the ballast to keep you steady against all the pressures to which you will be constantly subjected. Regardless of the energy and effort with which you have tackled the journey that has been laid out for you here, your journey is by no means over. Becoming your own person is a process that must be resumed, with vigor, every day of your life. However, now that you have glimpsed the means, the way is open to you. The way will never be easy, but it will always be worth the effort. I wish for you that you become, in the final analysis, the person you want to be.

section 2
Thinking and Writing Your Way to Truth
IT IS MORE IMPORTANT TO BE RIGHT THAN TO APPEAR RIGHT

190 ParT II PracTIce


Interactive Learning in Your Imagination
Local evaluation examines the strength of the premises that are offered in support of a conclusion. Global evaluation compares the strength of an argument to the strongest possible opposition.

Your biases are not your own. What you believe to be true may very well be a function of the massive persuasive forces to which you are constantly subjected. If you seek the dignity of becoming your own person, then you must strive to wash out bias. You can do this only by subjecting all opinions, claims, and judgments to the strongest possible opposition and then embracing the position that is least vulnerable to falsification by counterexample. In other words, you must seek to embrace that which ranks highest on a global sufficiency scalei.e., the position that is the least weak of all available options. You must strive, in still other words, to be right, not just to appear to be right. The following mini-essay format will set you on that path.

What a Good (Impartial) Argument Looks Like


One begins an argument by first postulating a claim that one believes sincerely, even if only intuitively, to be true. In language arts, this claim is referred to as your thesis statement (1). In posing your thesis statement, you should assume that the persons whom you are addressing do not believe that your thesis statement is true. This is an important assumption, because not only would there be little point in attempting to convince the already converted but also (and even worse) you might be seduced into offering a weak argument precisely because in the company of like-minded people, you are apt to get away with it. By contrast, to convince a disbeliever or skeptic, you will have to offer strong, convincing reasons why s/he should change his or her mind. This is referred to as support (2) for the thesis statement. Unhappily, too many people assume that this is the chief or, worse, the only important constituent of a good argument. This fallacious assumption is one of the leading causes of the intransigent, chronic disagreements that plague our society. For one thing, this one-sided approach is ineffective: Everybody is arguing and nobody is listening. For another, it can create a good deal of ill will. To be satisfied with merely presenting your own case demonstrates a profound lack of respect for those who disagree with you. That you do not have the patience or the time to seriously consider their concerns suggests that you view your opposition as badly misinformed, stupid, emotionally deranged, willfully argumentative, or any combination thereof. A more constructive attitudeone that might be appropriately labeled the golden rule of truth seekingis that you should view others as you view yourself,49 namely, as rational beings for whom truth is important. The difficulty that your opposition is experiencing, despite your persuasive efforts, is that s/he has highly convincing reasons (not addressed by your support) for believing that your thesis statement is false, misleading, perhaps even dangerous. If you do not address these concerns, they will inevitably subvert your attempts to convince your opposition of the truth of your claim. A crucial constituent in formulating an argument, therefore, is articulating the major concerns of your opposition (3). In doing so, you genuinely and respectfully invite your opposition into the playing field of judicious reasoning, and you thereby acknowledge that his or her beliefs and opinions are worthy of the same consideration as yours. This approach opens up the way for objective (i.e., unbiased) judgment, something most rational beings can tolerate even if the judgment goes against them. Now that you are both on the playing field, this is not the time for dirty tricks. It is imperative, once the concerns of your opposition surface, that you handle them with respect and sensitivity. Your job is to demonstrate (if it is legitimately demonstrable) that your oppositions concerns can be surmounted. Perhaps they are not as strong as they first appeared; or they are founded on misleading evidence; or they correctly capture an inevitable negative, but necessary, side effect of your position. Your responsibility, in other words, is to respond to your opposition (4) (specifically by showing

The golden rule of truth seeking: View others as you view yourselfi.e., presume that those whose positions differ from your own are rational beings for whom truth is important.

SecTIon 2 ThInkIng and WrITIng Your WaY To TruTh

191

that, relatively speaking, it is weaker than your support). If you cannot respond convincingly, then, in the name of truth, you should go back and change your thesis statement. The tracking of truth is serious business. None of us have the time or the spare energy to waste on empty rhetoric or sophisticated manipulation. Approximating truth is too important: Our combined welfare depends on it. And it is this seriousness that should be reflected in your conclusion (5). This is the finale. This is your last chance to tie it all togetherto demonstrate that the intricacies of your argument (which include the investigation of both local and global sufficiency) in fact establish the truth of your original claim. At its best, your conclusion should wow your audience with its elegance, its eloquence, its creativity, and its power to convince all in its wake.
conTenT oF a good (IMParTIaL) arguMenT
A good (impartial) argumenti.e., one that genuinely tries to estimate truth must contain the following: (1) A clear thesis statement in support of one side of a highly contentious issue (2) A convincing support for that statement (3) A strong opposition (4) A convincing response to that opposition (5) A convincing resolution or conclusion to the proposed problem

Detailed Analysis of the Five Essential Argument Constituents


1. A Clear Thesis Statement in Support of One Side of a Highly Contentious Issue
A. Your thesis should be contestable

If you pick a claim that has no believable opposition, that claim succeeds (if such a term is appropriate at all) by default, not on the strength of your reasoning. In principle, therefore, arguments in support of noncontentious claims cannot reflect the competence of the arguer and hence cannot be the conveyor of grades. Such claims are also uninteresting, a waste of time, and insulting to your reader. Your choice of topic suggests that those for whom you write are so dim-witted that they need convincing of the otherwise obvious.
B. Your thesis statement should be relevant and unique (avoid Internet topics)

You cannot show that you are strong unless you pick up something heavy. You cannot show that you are clever unless you tackle a topic that is contentious. The test of whether your thesis statement is sufficiently contentious: Can you think of a strong opposition?

As far as is humanly possible, pick a topic for which the truth genuinely matters to you. If the topic is personally important, if you can imagine how it will intersect with your life now or at some future time, it will be evident from the outset that the hard work necessary to track truth is worth the effort. Aside from being relevant, your topic should be uniqueavoid topics found on the Internet! This is important because the only way to get serious practice in creating an argument is to create an entire argument. Thus, although the topic of whether or not marijuana should be legal may be relevant, the arguments on both sides of this issue have been done to death; in choosing this for your topic, your only practice will be in reorganizing the bits. This is not to say, however, that you should avoid familiar topics altogether; rather, you should learn to put an unfamiliar twist on them. You

192 ParT II PracTIce


can do this by personalizing the issue. Thus, if you are a marijuana smoker, you could write about how much, if any, is too much, or whether or not you should smoke up with your children. You can also write about a difficult personal situation that you have encountered and extract a thesis statement by deciding what you should or should not have done.
C. Your thesis statement should be tentativei.e., you should be prepared from the outset to embrace its contrary if that is where reasoning ultimately points
You cannot make true what is not true.

You must always remember that the business here is to track truth, and therefore you must be prepared from the outset for the possibility that you may be required to embrace the position of your opposition. Another way of putting this important point is that when you first articulate your thesis statement, it must be genuinely tentative. If at the outset you pick a topic in the hope that you can utilize the strong argumentative tools herein provided to support a position that is dear to your heart, you must nonetheless be prepared to follow the reasoning where it leads, which may very well be into the arms of your opposition. If that is the case, you must be willing to flip your thesis statement. Keep in mind that you cannot make true what is not true. If there is little in terms of evidence or argument to back up your claim, ultimately your attempt will fail, and your grades will probably follow.
D. Your thesis statement should be clear and precise with respect to definitions, detail, example, and quantification

The cardinal rule for articulating a thesis statement is that it must be clear. Clarity is important for two reasons. First, the truth-potential of your claim cannot be evaluated unless you, and others, know precisely what it is you are claiming. Second, if what you believe to be true is not clear, it cannot guide your actions; it cannot act as a stimulus-pull on your behavior. If you make the claim, for instance, that women will be free only if patriarchy is dismantled, you will not be able to help bring about this apparently laudable goal unless you know precisely what would count as patriarchy being dismantled, to say nothing of what would count as women being free.50 Clarity thus may require a definition of terms. If you are arguing that pornography should be banned, for example, you will have to explain what you mean by pornography. Does it include anything and everything sexual, or are you referring only to that which is violent and degrading? Are you concerned only about visual images, or are you also concerned about the printed word? What about music? Just as you must define terms, you must also supply detail, where necessary, about what would count as fulfillment of the state of affairs advocated by your thesis statement. If you claim that immigration rates should be reduced, you must indicate what sort of numbers would count as being sufficiently reduced. If you argue that we should have zero tolerance for child abuse, you need to be more specific about what would count as zero tolerancee.g., no spanking, immediate jail time as punishment, or what? If you argue that parents should be held responsible for the illegal acts of their young offspring, you must supply detail about what would count as being held responsiblee.g., paying a fine, going to jail. Sometimes clarity can be achieved through the use of examples. Thus, if you make the claim that students who break school rules should be expelled, examples will help clarify your position. Are you referring to such major infractions as carrying weapons or selling drugs, or are you referring to all infractions, regardless of how smalle.g., chewing gum in the hallway? In Critical Thinking Textbook: Logical Self-Defense, R. H. Johnson and J. A. Blair51 offer three criteria (although not in the following order) by which to judge whether or not a claim that you make is clear. They suggest (1) that you should be prepared to restate

SecTIon 2 ThInkIng and WrITIng Your WaY To TruTh

193

the meaning of your claim in other words that express the same meaning (as reassurance, you might want to put an alternative formulation of your thesis statement in a footnote or in brackets after the original); (2) that others should be able to restate the meaning of your claim in other words; and (3) that you should be able to explain the conditions that would show whether or not your statement is true or false (e.g., if you claim that parenting without physical punishment is better than parenting that includes it, you should be prepared to suggest the conditions that would demonstrate the truth of this assertione.g., that such children would be more amenable to rational persuasion and less apt to resort to physical violence when attempting to solve problems).

Clarity is critical. If your beliefs, opinions, and admonitions are vague or unclear, they will be inefficient in guiding action.

In summary, then, your thesis statement should be 1. Contentious 2. Relevant and unique 3. Tentative (i.e., you should be prepared from the outset to embrace its contrary) 4. Clear and precise with respect to definition, detail, example, and quantification

2. A Convincing Support for the Thesis Statement


What you are attempting to do in your support is to establish local sufficiencyi.e., to provide your reader/listener with a sufficient reason for believing your claim. For that reason, you should keep in mind the following points.
A. Know whether your thesis statement can be supported by reference to reasons alone or whether it also requires reference to empirical evidence or data

If you are making a statement about what should be the case (e.g., people should not engage in casual sex), you are making a value claim, and hence your argument will rest primarily on the strength of your reasons. If you are making a claim about a fact (e.g., photo radar doesnt work), then your argument will rest primarily on the strength of the empirical data you offer in support. Sometimes there is a mix. Thus, if you are arguing that the forest industry should be allowed to continue clear-cutting because the termination of this practice will result in a devastating loss of jobs, you must be prepared to state, at least approximately, how many jobs will be lost, to say nothing of the challenge of defining what would count as devastating. How else can your reader estimate the truth of your claim?
B. Be aware of the hidden premise

A reason becomes a reason only within an argument. An argument always contains at least two reasons or premises. Often, when a reason is offered in support of a conclusion or thesis statement, its argumentative partner is invisible. Thus, if you argue that homosexuality is wrong because it is unnatural, the principle of charity requires that I attribute to you the necessary, validating major premise that anything that is unnatural (including chewing gum, for instance) is immoral. If you do not accept that principle, you cannot offer the former reasoning as support for your thesis. Or again, if you argue that clear-cutting should not be limited because to do so would have a negative impact on our economy, you must also accept the hidden premise that anything that negatively impacts our economy should be avoided.52

194 ParT II PracTIce


It is absolutely essential that you be aware of the hidden premise. This is so because a reason is only as strong as the weakest link in the entire argument. Thus, if your hidden premise is extremely weak (i.e., vulnerable to a strong counterexample) but you are unable to see it, you will remain unaware of how weak your support is.
C. Beware of Begging the Question!

Constructing a convincing argument can be described as a process of cognitive fishing; you are trying to hook onto a principle that your opposition holds dear and then, on the strength of that principle, reel in your claim. Thus, if you wanted to convince a skeptic that all drugs should be legal, you would not argue that the prohibition of drugs violates the principle of nonpaternalism if you already knew that your listener had no problem whatsoever in embracing paternalism. Likewise, if you wanted to convince a skeptic that God exists, there is no point in doing so by referring to the fact that it says so in the Bible and that the Bible is the word of God. Obviously if your opposition believed that the Bible was the word of God, your opposition would not have doubted the existence of God in the first place. In situations in which you do not hook into a principle or claim that your opposition believes to be true, all you are doing is fishing around in your own mind rather than attempting to reach across into the mind of your opposition. You have hooked one of your own beliefs, not one of your oppositions, so your reeling-in does nothing but create a circle; you have convinced nobody but yourself. This method of faulty, or fallacious, argumentation is referred to as circular reasoning, or begging the question. In such instances, since you have given no reason at all (i.e., no reason acceptable to this opposition), you are reduced to begging your opposition to believe your original claim. And since no reason at all is being offered, your opposition can give into your plea only at the price of jeopardizing his or her own passion for truth.
eXaMPLeS oF BeggIng The QueSTIon SuPPorT
Note: The support comes in the form of the reasoning after because. Gay and lesbian couples should receive the same benefits as heterosexual couples because all adults should be considered equal under the law. New immigrants should be required to speak a countrys official language before being allowed to immigrate because, in order for immigrants to make a positive contribution, they must be able to communicate in the official language. Casual sex should not be socially acceptable or encouraged because sex is a serious step and should be undertaken only by people who really love each other. There should be no federal cutbacks in education because quality education is a right. The criminally insane have the right to view any material that any other citizen has a right to view because restricting their viewing material would constitute discrimination.53

BEGGING THE QUESTION: BORING HYPOCRITICAL REPETITION

195

196 ParT II PracTIce


D. Treat your opposition with respect: Do not offer offensively weak support

If the support of your thesis statement is extremely weak, you are implicitly making the claim that you are prepared to believe any old thing on any old grounds. The moon is made of green cheese or chalk, or is the center of our solar system, for all you care. Such fanciful thinking may be productive for poetry, music, or art, but it has no place in the serious business of truth tracking. If you invite rational dialogue, as you do when you formulate an argument, you insult those who listen to your words, or read your work, if you offer them nothing but fluff. None of us has time to waste: Do not enter the business of truth tracking unless you are prepared to offer support that is sufficiently strong and believable that it justifies the evaluative participation of other rational agents.

In summary, then, when constructing support for your thesis statement, remember the following: 1. You need to know whether reference to reasons is sufficient or whether you will also need to refer to empirical evidence or data. 2. Be aware of the hidden premise. 3. Beware of begging the question. 4. Do not offer offensively weak support.

3. Articulation of a Strong Opposition


Since truthi.e., global sufficiencylies in siding with the least weak of available options, obviously one should try to articulate as many available options as possible. However, if one is pressed for time and space, the next best alternative is to articulate the strongest possible opposition. The strongest possible opposition is the strongest support for the thesis statement that contradicts your original, which usually means simply inserting a not into the original claim. Thus, if you are attempting to support the claim that one is justified in not giving spare change to beggars, then the opposing viewpoint would be that one is not justified in not giving spare change to beggars. Since your opposition is simply support for the negative of your thesis statement, all the aforementioned suggestions for articulating a strong support apply to articulating a strong opposition. Keep in mind that your opposition is not opposing the reasoning that you have articulated in your support. If that were the case, you would be stuck in the process of merely trying to establish local sufficiency. Your job here is to move on to global sufficiency and thus to investigate a different set of reasons that support the opposing view. Keep in mind, too, that if you create a strawperson argument (i.e., an argument with a dishonestly weak opposition), you rob yourself of the opportunity to test your thesis against a worthy opponent, and in so doing you forfeit the chance to evaluate the real strength of your position in terms of global sufficiency. This is a risky strategy if anything at all rides on establishing truth.

STRAWPERSON ARGUMENT: BOW-WOW CHOW

197

198 ParT II PracTIce


4. A Convincing Response to the Opposition
This is the critical point of the argument. By the time you are ready to respond to your opposition, what you have before you are two deductive arguments (both with hidden premises): one in favor of x and one opposed to x. Your task now is to show which of the two is stronger. If your opposition is stronger than your support, now is the time to switch positions! How do you decide which is stronger: your support or your opposition? You decide by flipping the question to which is the weakest?i.e., which argument is easiest to respond to in the sense of finding fault. Since the test is a negative one, the strength of the overall endeavor depends on starting off with the strongest possible options. Try always to keep in mind the point of the enterprise. Remember that a good philosopher, like a good scientist, does not attempt to prove his or her claim to be true. Rather, s/he examines as much evidence as possible, in a manner that is as precise and objective as possible, and only after s/he has failed to prove his or her theory false is s/he justified in proclaiming its truth. That is what you are trying to do here: You are trying to eliminate one of the two contending arguments by demonstrating its weakness;

Truth seekingi.e., global sufficiency requires that we try to estimate which of the alternatives is the least weak contender.

As an example, let us say that you have thus far written the following argument:
thesis

Violent and degrading pornography should be banned.


support

Since we can assume that, like advertising, violent and degrading pornography influences some people to behave in the manner portrayed, for the sake of the physical protection and the preservation of the dignity of women, violent and degrading pornography should be banned.
opposition

A democracy can flourish only if its citizens are willing to tolerate different beliefs and stated opinions. Banning pornography is a form of censorship, which interferes with that vital freedom of speech that is the major defense against the possibility of one groups tyrannizing another. Now divide them into their component deductive arguments.
supports argument

(Hidden premise) All [actions that are necessary to stop the positive powerful advertising of female brutalization and degradation and hence are essential for the physical safety and dignity of women] are actions that should be done. (Stated premise/reason) (Banning violent and degrading pornography) is [an action that is necessary to stop the positive powerful advertising of female brutalization and degradation and hence is essential for the physical safety and dignity of women]. (Conclusion/thesis) Therefore, (banning violent and degrading pornography) is an action that should be done. *

SecTIon 2 ThInkIng and WrITIng Your WaY To TruTh

199

oppositions argument

(Hidden premise) All [actions that are a form of censorship that threaten democracy and its essential ingredient, namely, an active commitment to freedom of speech that requires that we tolerate different beliefs and opinions and is thus a major defense against the possibility of one groups tyrannizing another,] are actions that should be done. (Stated premise/reason) (Banning violent and degrading pornography) is [an action that is a form of censorship that threatens democracy and its essential ingredient, namely, an active commitment to freedom of speech that requires that we tolerate different beliefs and opinions and is thus a major defense against the possibility of one groups tyrannizing another]. (Conclusion) Therefore, (banning violent and degrading pornography) is an action that should not be done.
response

Now decide which of the two arguments is easier to respond towhich one is the weakest. Which is the most vulnerable to a convincing counterexample? The weaker argument becomes your final opposition. Against the support, you might suggest that banning pornography will not work because of the inability to censor the Internet. Against the opposition, you might suggest that a vibrant democracy does not, in fact, require that we tolerate all stated opinions. We do not tolerate hate literature, for example, on the grounds that it may result in genuine harm. Libel is also against the law. Now that you have articulated potential flaws in the reasoning that underpins both the support and the opposition, you have to decide which of the positions is rendered the weakest as a result.

you are not trying to reemphasize the winning characteristics of the original thesis statement. The method of choice here is falsification, not verification. Like a good scientist, this is how a philosopher tracks truth. In this particular situation, you may decide that the support is the least weak because the fact that a law is relatively ineffective is not a huge liability. After all, rape and fraud are common occurrences; nonetheless, laws against them are presumed to be important in helping to decrease the rates of those crimes. If you decide that the support is the least weak, your opposition stays as your opposition, and you must now, in responding to your opposition, show how your opposition is weak, perhaps by saying something like the following: Although concerns over censorship are always important and relevant in a democracy, the fact is that there are some instances of censorship that promote rather than undermine democratic principles. Censoring untruths that could damage the reputations of individuals is one of them. Just as we have laws against libel, so we should have laws against violent and degrading pornography. In both instances, the victims of such slander have little hope of speaking truth to the power of the sensational negative slur to which they are subjected. Since the point of freedom of speech is to try to ensure that all sides have equal airing, and since in the case of libel and pornography, one side is completely outshouted by the other, a law protecting the voice of the shouting side actually diminishes the voice of the other and thus undermines the point of having a law to protect freedom of speech to begin with.

The response to your opposition is a response only to your opposition. The response to your opposition is not an opportunity to reiterate support for the original claim.

LANGUAGE GAMES

200

SecTIon 2 ThInkIng and WrITIng Your WaY To TruTh

201

5. A Convincing Resolution or Conclusion to the Posed Problem


By the time you get to your conclusion, you will have engaged in an arduous journey that has resultedif you have done your jobin the gift of truth. Your conclusion therefore should reflect your exhilaration for having stabilized a vision that you believe should guide action. You are, quite literally wiser. Thus, it is appropriate that you punctuate the end of this journey with a pearl of wisdoma succinct creative synopsis of both the learned truth and the journey that led to it; it is the accumulation of such pearls that will ultimately light your way to autonomy. Unhappily, if you have not done a good job so farfor example, if your thesis statement is too general, if your support is not adequate, if your opposition is weak, or if the response to your opposition is unconvincingthere is very little that can be done at this point. The acid test of any argument is to ask whether, in the final analysis, it is convincing. If it is not, it is hard to see how the conclusion could be judged as adequate, no matter how efficiently the details of the argument are summarized.
eXaMPLeS oF good concLuSIonS
Good conclusions mirror the argument and contain a pithy pearl of wisdom (an aphorism). A conclusion to the thesis statement Little white lies (i.e., lies that are used without the intention to hurt) are necessary for coexistence in our society. In an uncomplicated world, lies would not be necessary, because the truth would be self-evident. However, in this world, if lies were not part of regular interpersonal communication, social interchange would be cold, mean, and harsh. There are other values in life besides the value of truthfulnesssuch as caring about the welfare of othersthat sometimes, perhaps even often, take precedence. Values are valuable only within a context. If nothing else is at stake, you should never lie. However, if the preservation of life, liberty, and the pursuit of happiness requires the grease of a few little white lies, so be it. A conclusion to the thesis statement Countries should set aside much larger areas of protected wilderness. Civilizations and even species have historically peaked and then collapsed. We are faced with the possible extinction of our own civilization. We must not follow the patterns and paths of our forefathers, which have led toward disaster. We have the unique knowledge and ability to change our direction in a gradual and progressive manner instead of exhausting our potential. If we allow our generation to destroy our biodiversity, we will be guilty of the indefensible selfishness of stabilizing our own economic wealth at the expense of limiting the future possibilities and choices of our grandchildren and their grandchildrento say nothing of the animal life that we destroy in the process. Although responsible environmental stewardship will undoubtedly bring economic hard times in the short term, in the longer term all will be much richer. A conclusion to the thesis statement Individuals should not use words, such as f*ck, that connote aggression and force; rather, they should use more neutral terminology such had intercourse with. Every individual should use neutral sexual language in speech and writing and should instruct others to do the same. Few of us will have the opportunity to stop a rape in progress; however, all of us have the opportunity to insist that the words used to describe sexual interaction be imbued with the values of equality and care. If we are ever going to create a society in which violence and sex are disassociated, we must all do our part.

WRITING CAN BE A FORM OF INTERACTIVE LISTENINGOR NOT

202

203

204 ParT II PracTIce


Summary for Evaluating the Five Essential Argument Constituents
CHECKLIST FOR MINI-ESSAYS 1. Remember that your parallels contain only the bare bones of your reasoning. In your mini-essays, you need to add flesh. Try using analogies. 2. Be precise. Use examples. Keep it real. 3. Make sure you are not presenting a strawperson argument. 4. If your opposition is stronger than your support, flip your thesis. 5. Make sure your response is only and entirely a response that shows what is weak about the opposition and that it is not more support. 6. You may not change your mind in the conclusion. If, in the first draft, you change your mind, go back and rewrite the essay.

When evaluating an argument, ask yourself the following questions: With regard to your thesis statement Is it an essentially contestable problem? Is it relevant and unique? Could you easily find this argument on the Internet? Is it genuinely tentative? Are you prepared to embrace the opposite position if that is where the reasons point? Is it clear? Are terms adequately defined? Is sufficient detail supplied so that it is clear what would count as fulfillment of the suggested claim? Are examples needed? In articulating support for the thesis statement Is reference to reasons sufficient, or will you also need to refer to empirical evidence or data? Do you accept the hidden premise? Is there evidence of begging the question? Is the support offensively weak? Overall, is your support genuinely convincing? Have you articulated the strongest imaginable opposition, and is it presented with the same degree of care as your support? Or have you indulged in a strawperson tactic by portraying the opposition either as having a silly or irrelevant or unconvincing objection or as begging the question and so forth? Is there a convincing response to the opposition? If you were the opposition, would your concerns have been allayed? Or is this merely an illegitimate repeat or, worse, even another support of the original thesis statement? After having read the conclusion, are you convinced? Is your conclusion more than merely a restatement of your thesis statement? Does your conclusion honor both your support and your opposition? Would somebody reading your conclusion be able to accurately estimate the preceding argument? Does the conclusion go further than effectively summarizing the argument components by suggesting novel ways to deal with, or avoid, such problematic issues or situations? Does your conclusion contain a pithy pearl of wisdom that appears to summarize a principle that can be extracted from the preceding argumentative process?

WHEN SELF-CONSCIOUSNESS TAkES UP THE CHALLENGE, IT IS A WONdER TO BEHOLd

205

206 ParT II PracTIce


WHEN SELF-CONSCIOUSNESS TAkES UP THE CHALLENGE, IT IS A WONdER TO BEHOLd (continued)

Interactive Reasoning
We live in a world in which the points of contact between desperate perspectives seem to be dwindling to nonexistence.54 We live in a world in which too many adults role model intolerance of differing viewpoints. We live in a world in which the terror of the fragility of ones own ideology is so profound that scrutiny through a critical eye from an outsider is considered insulting and scrutiny through a critical eye from an insider is considered blasphemy. Yet we live in a world in which the confidence in the infallibility of those same ideologies is so strong that many are willing to die in their names and to take others with them. We live in a world, in other words, in which the very possibility of striving toward autonomythat disposition that is uniquely possible for self-conscious language usersseems ever more seriously threatened. It is no mundane clich to say that wars are won only by winning the minds of the people. However, winning minds does not mean convincing others that we are right and they are wrong. That which divides people can be transcended only if all concerned seriously attempt to tackle the issues that divide them in an impartial way. Peace may well be the payoff for such interactive thinkingalthough perhaps not the most important payoff. The prime value of interactive thinking is that through it autonomy is possible. It is how individuals are, as it were, reasonably born. There is thus no sacrifice in genuinely communicating with your enemy. There is no risk in articulately listening to those with whom you radically disagree. In fact, quite the opposite is true. There is a huge risk if you do not genuinely communicate, if you do not articulately listen to those with opposing viewpoints, and that risk is the loss of yourself.

appendix I
Answers to Exercises
Exercise 1A: Answers
Empirical: What Is Your Evidence? 1. Sexwithoutcommitmentiswrong. 2. Photoradardoesntwork. 3. fyourfriendhasaseriousdrugaddiction,youshouldtell I hisorherparents. 4. Smokingcausescancer. This claim is so embedded in common knowledge that it would be odd to ask for evidence, even though it is empirical. 5. oddlerswhospendmostoftheirtimeindaycareend T upbeingmoreindependentthantheirpeers. One could set up an experiment to test out this hypothesis. It is somewhat of a mixture in that one could try to reason its conclusion if there was no evidence available. However, one must always be prepared to admit that, ultimately, empirical evidence is always the deciding factor for empirical claims. X X X Mixture Nonempirical: What Are Your Reasons? X

207

208 AppEndix i
Exercise 2A: Answers
Vague 1.Childrenshouldtakemoreresponsibilitydoingchoresathome. R. So do you think that primary schoolchildren should do all the cooking and cleaning? 2.Allprospectiveparentsshouldberequiredtotakeparentingcourses. R. What kind of courses? The kind that tell you when it is OK to feed baby solid food or the kind that will help parents ensure that their kids have a healthy measure of self-esteem? If the latter, what would those courses look like? This is also vague as it is not obvious how this would be enforced, although one might suggest a tax credit for every parent who took the course. 3.Ourcityshouldhavemoreparks. R. Oh, really? Does anyone know any square inch of any city that is not already owned and used? Should we start tearing down skyscrapers in order to make way for parks? 4.Thegovernmentshouldensurethatallpetsaretreatedhumanely. R. How are we going to do thathave cameras in homes and gardens? Note: This person may very well have a precise problem in minde.g., not securing dogs that ride in the back of trucks. If so, then this response may prompt the individual to be more precise. 5.Ourcityshouldbuildacomprehensivemasstransitsystem. R. And how is the government going to pay for it? Shall we double the tax on gas? X X X X X Precise Enough

Exercise 4A: Answers


1. Youhavejustcommittedthefallacyofanillegitimate ad hominemattack.Johnpresentedanargument.Theassumptioninrationaldiscourseisthatallargumentsshouldbejudgedontheirownmerit.ThefactthatJohnisa memberoftheKuKluxKlanhasnobearingonthemeritoftheargument.Sowhatdoyouthinkofhisargument thatimmigrationlevelsshouldbecurtaileduntilunemploymentlevelsaredown? 2. Itseemstomethatthatisafaulty analogy. Thepointofopeningshootinggalleriesistosavethelivesofaddicts bydecreasingoverdoses,theuseofcontaminateddrugs,andthedangerouspracticeofsharingneedles.Openingshootinggalleries,inotherwords,hasanumberofhighlypositivebenefits;lettingkidslooseinacandystore hasfew.Ithinkyouranalogydecreasesourunderstandingoftheintentofopeningshootinggalleriesratherthan enhancingourunderstanding. 3. Yourreasoningbegs the question.Whatisatissueiswhetherornotcriminaloffendersundertheageofeighteen shouldreceivespecialprotectionunderthelaw.Simplysayingtheyshouldnotisnotthesameasprovidinga reasonexplainingwhytheyshouldnot.Thereasonthatyouoffer(i.e.,theyshouldnotreceivespecialtreatment) isjustarewordingoftheconclusion.Theargument,therefore,iscircular. 4. Youarebeingparalyzedbyafalse dilemma.Goingtothepoliceordoingnothingarenottheonlyoptionsthat areopentoyou.Youmight,forexample,decidethatthebestcourseofactionistotryandpersuadeyourfriend toturnhimselfin.Ifherefuses,youmightdecidethathewasnotthefriendyouthoughthewasinthefirstplace, andhencemaynotdeserveyourloyalty. 5. Sofiasreasoningisanexampleofapost hoc fallacy.Onthebasisofaten-daytrial,Sofiahasjumpedtothe conclusionthatthemixtureoflemonjuiceandRindgoldsRosewatercausedherblemishestodisappear.How-

AnswErs to ExErcisEs

209

ever,thisassumptionmaybefaulty.Herblemishesmighthaveimprovedasafunctionofherhormonecycle,or becauseshewaswashingherfacemoreoften,orbecauseshehappenedtospendmoretimeoutinthesunduring thosetendays,orsolelybecauseofthelemonjuice.Tolegitimatelymakeacausalclaim,onefirstmusteliminate allotherpossibilitiesthatmightexplainthecorrelation.Makingacausalclaimrequiresenergyandhardwork. 6. Someofyoumighthavedescribedthisasanabusive ad hominem attack.Ifyoudid,youarewrong.Anabusive adhominemattackoccursonlywhensomeoneattacksthearguerratherthantheargument.Inthiscase,noargumentisbeingpresentedandnotruthclaimisbeingmade.Thisisjustanexampleofsomeonebeingabusive.It maynotbenice,buttheindividualcannotbeaccusedofcommittingafallacyi.e.,ofpresentingafaultyargument.Soyourresponseshouldfocusondeflectingtheinjuryofsuchanunkindremark.Youareentirelywelcome.Itwasmypleasure. 7. Yourreasoningexemplifiesthefallacyofadistinction without a difference.Yousaythatyouarenotasexist,but believingthereisnoplaceinthearmyforwomenispreciselythesortofreasoningthattypifiesasexist.Thatis whatyouare,asexist. 8. Yourreasoningbegs the question.Theissueunderscrutinyiswhetherornotgayandlesbiancouplesshould receivethesamebenefitsunderthelaw(i.e.,whetherornotalladultsshouldbetreatedequallyunderthelaw). Merelysayingtheyshouldbetreatedequallyisnosubstituteforgivingareasonwhytheyshouldbetreated equally.Canyougivemeanactualreason? 9. Youhavejustpresentedthefallacyofanillegitimate ad feminam attack.Jenniferpresentedanargument,and herargumentdeservestobejudgedonitsownmerits.Whetherornotshehassomethingtogain(i.e.,nomore bedpans)ifherargumentisacceptedisirrelevant.Iamwonderingifyouwouldliketocommentonherargument thatbecausenurseshavesuchahighlevelofeducation,theyshouldnotbewastingtheirtimechangingbedpans. 10. Youhavejustcommittedanillegitimate appeal to authority(specifically,totradition).Thefactthatthereisnow evidencetosupporttheclaimthatmarinemammalsareself-conscioussuggeststhatkillingwhalesmayverywell beonaparwithkillinghumans.Thus,inthefaceofthepotentialofinflictinggreatharm,onewouldneedextremelystrongreasonsforresumingthepracticeofwhaling.Simplyappealingtothefactthatitwasalwaysdone orwasoncedonethiswayisvacuous,asisevidentfromthefactthatmerelyappealingtothefactthatslaverywas oncepartofwhitepeoplestraditionhardlyjustifiesitsresumption.

Exercise 4B: Answers


1. Theargumentthatyouhavejustgivenisanillegitimate slippery slopeargumentbecauseyouhavenotgivenme anyreasonwhyonesituationofbreakingarulewillinevitablyleadtoanother.Afterall,exceptionsaremadeto rulesallthetime.Thismightbeacaseinwhichanexceptionisjustified,inwhichcasethereisnoreasontopresumethatchaoswillfollow.Inorderforaslipperyslopeargumenttobejustified,onemustshowwhytheslopeis slippery. 2. Althoughthisissimilartoitem1,thefallacyisnotthesame.Herewehaveanexampleofafalse dilemma.Like thepreceding,thealternativemaybethat,insomeinstances,exceptionsarejustified,soyourresponseshouldbe Tosaythateithereveryonefollowsalltherulesorweshouldnothaveanyrulesatallisafalsedilemma.There aremanyinstancesinwhichbreakingarulewithoutbreakingthemallisjustified,andIbelievethisisoneof them,because.... 3. Youhavejustcommittedanillegitimate abusive ad hominem attack.Ipresentedanargument,andeveryargumentdeservestobejudgedonitsownmerits.Iamwonderingifyouwouldliketocommentonmyargument, namely,thatwomensstayinghomewiththeirchildrenmaybeanecessaryconditionofbonding. 4. Yourreasoningbegs the question.Whatisatissuehereiswhetherornotwomenshouldhavearighttodecide whathappenstotheirownbodiesevenincasesinwhichitinvolvesthetakingofalife.Anyonecansaythat anyonehasaright.Icouldjustaseasilysaythatthefetushasarighttolife.Simplysayingthatabeinghasaright isnotasubstituteforgivingareasonwhythereshouldbearight.Wouldyouliketotrytotellmewhyyouthink womenshouldhavearighttoanabortion? 5. Well,Joan,eventhoughyoumightnotlikebeingcalleddumb,nonethelessyouleftyourselfopentothislegitimate ad feminam attack.Thetruthofanyclaimcanbejudgedonlyonthestrengthofthereasonsthatbackit.If therearenoreasonstoattackandyoudidnotgiveanyreasonsthenitislegitimatetoattackthesource.You areindeedlookingprettydumbwhenyouasksomeonetobelieveapositionwithoutanyreasonssupportingit. Andifyounowsayyouarenotaskinganyoneelsetobelievethatposition,thenwhysayanythingatall?

210 AppEndix i
6. RaymondshouldsaytoMarco,Thatisanillegitimate abusive ad hominem attack.Ipresentedanargument, andtheargumentdeservestobejudgedonitsownmerit.Wouldyoucaretocommentonmyargumentthat condommachinesmaypromotepromiscuitybysuggestingthattheschooladministrationthinksthatsexisOK orcool? 7. Itseemstomethatthatisafaulty analogy.Certainlythecustomeristheexpertwithregardtomostconsumer goods.Ifthecustomerwantspinkcarswithpurplespots,sobeit.Withregardtoknowledge,however,itisthe teacherwhoistheexpert.Theteacher,notthestudent,knowswhatknowledgeisandisnotimportant.Ithink youranalogydecreasesratherthenenhancesourunderstandingofeducation,andhenceIthinkitisfaulty. 8. Well,mysweetchild,thatisanillegitimate ad hominem attack,specificallyatu quoque.EventhoughIindulge inadailymartini,itdoesnotmakemyargumentagainstsmokingpotanylesslegitimate.Argumentsmustbe judgedontheirownmerit.Thus,evenifIsmokedpotmyself,Icouldstilllegitimatelyadviseyouagainstsmokingit.Infact,somemightarguethatbeingapotsmokermyselfputsmeinaparticularlyprivilegedpositionto knowitsharms.Inanycase,Ihaveachievedmostofmygoals,whereasyouarejustbeginningthecompetitive race.Youneedtoensurethatallyourfacultiesareattheirverybest.Ithinkgettinghookedbypotmayseriously decreaseyourcompetitiveedgebothbymuddlingyourmindandbyseducingyouintotime-wastingwaysto socializeandrelax.Wouldyoucaretocommentonthatargument? 9. Yourargumentbegs the question.Simplysayingthatallstudentsshouldbetreatedthesameinsupportofthe conclusionthatforeignstudentsshouldnothavetopayhigherfeesisjustrepeatingtheconclusioninadifferent form.Itiscircular.Perhapsyoucouldgivemeareasonwhyyouthinkforeignstudentsshouldnothavetopay higherfees,particularlyinlightofthefactthatonlytheparentsoflocalstudentssupportacademicinstitutions withtheirtaxdollars. 10. Appealingtotraditionwhenthetraditionresultsinharmisanillegitimate appeal to tradition.Thepracticeof killingelephantsmayactuallybeintrinsicallyunethicalifitturnsout(astheevidencesuggests)thatelephants areself-conscious.However,evenifelephantsarenotself-conscious,thepresentcontextmakeskillingelephants wrong.Elephantsarenolongerplentiful.Indeed,therearesofewelephantsleftthattheyareonthevergeof extinction,andthatiswhatmakeskillingelephantswrongnowinawaythatitmaynothavebeenwrongwhen yourtraditionwasfounded.

Exercise 4C: Answers


1. Youareusingafalse dilemmaasanexcuse.Youareassumingthatthereareonlytwochoices,eitheryoudrivefast oryouwillnevergetanywhereontime.However,therearemanyotheroptions,themostobviousbeingthatif youleaveforyourdestinationinplentyoftime,youwillbeabletodriveatareasonablespeed. 2. Well,myfriend,thatisanillegitimate appeal to authority,specificallytheauthority of the crowd.Justbecause everyonethinksitisOKtosleeparounddoesnotmakeitright.Onceuponatimemostpeoplethoughtitwas OKtoownslaves;thatdidnotmakeitright.Theonlygroundsonwhichonecanlegitimatelyjudgeaclaimright orwrongisbyjudgingthereasonsthatbackit.Wouldyoucaretogivemereasonsexplainingwhyyouthinkitis OKtosleeparound? 3. Thatisanillegitimate appeal to authority.Einsteinwasanexpertinmakingtheatomicbomb;hewasnotan experteitherinethicsorinpoliticalscience.SoifyoubelievethatitiswrongfortheUnitedStatestostockpile nuclearweapons,youneedtogivemereasonsratherthanappealingtowhatsomeoneelsethought.Wouldyou caretodothat? 4. Thatisastrawpersonargument.Itmisrepresentsthegovernmentsposition.Giventheamountofmoneythat thegovernmentlosesinhealthcarecostsandlostworkhours,allowingsmokingisfarmorecostlythanthetax collected.Amoreappropriaterepresentationofthegovernmentspositionisthatitwouldnotdaretrysucha paternalisticmove.Citizensinademocracyareextremelyjealousabouttheirfreedom,includingtheirfreedomto smokethemselvestodeathiftheysochose. 5. Thatisadistinction without a difference.Notwantingyourdaughtertomarryablackmanmerelybecauseheis blackispreciselythesortofattitudethatcharacterizesaracist. 6. Someofyoumayhavedescribedthisasaposthocfallacybecausethereisahintthatthemothermaybeassumingthatthereasonhersonisactingpoorlyisbecausehestartedtohangaroundthisgang.However,themother isnotmakinganycasualclaims.Inaddition,someofyoumighthavedescribedthisasanillegitimateadhominem

AnswErs to ExErcisEs

211

attack:Sincethesonishangingaroundwithcreepycharacters,hisargumentmustbefaulty.However,ifyou rereadthisexample,youwillrealizethatthesonmademerelyaclaim,notanargument.Intheabsenceofreasons orevidence,theonlywaytojudgethetruthvalueofthatclaimistojudgeitssource.This,then,isalegitimate ad hominem attack.Soifthesonobjectstothefactthatheisnotbeingbelieved,youmightrespond,Youwere subjectedtoalegitimateadhominemattackbyyourmother.Youmadetheclaimthatyouwerenottherewhen yourfriendsbrokeintothestorelastnight.However,theonlywaytojudgethetruthofaclaimisbythereasons orevidencethatbackit.Iftherearenoreasonsorevidencepresented,thentheonlyotherwayisbyjudgingthe source,andinthisinstancethesourcesuggeststhereismorelikelihoodthatyourclaimisfalsethantrue. 7. Thatisapost hoc argument.Acorrelationnomatterhowstrongisnotsufficienttomakeacausalclaim.You havetoruleoutallotherexplanationsfirst,suchasthatthecorrelationwasamatterofchance,oryoumaybesick becauseofsomethingthatyouateordrankattheparty,orperhapsyouhavetheflu.Inordertohavelegitimate groundsformakingacausalclaim,allotherexplanationsneedtoberuledout. 8. Youarebegging the question.Anyonecansayanybodyhasarighttoanything;thatdoesnotmeantheright exists.Youcouldjustaswellsaythatstudentshavearighttobepaidafullsalarywhenattendingcollege.Ifyou aregoingtousethewordrightandyouarenotreferringtoalegalright,youneedtoexplainwhysucharight shouldexist.Sowhydoyouthinkstudentsshouldhavearighttohigh-quality,no-costeducation? 9. Thatisafallacy of the golden mean.Justbecausebothblamedtheother,itdoesnotfollowthatthetruthlies somewhereinthemiddle.Youcannotestimatethetruthofthesituationuntilyouheartheevidence.Inthis instance,JanemaybenaggingJohnonlybecauseheisabusive;i.e.,inthisinstance,thefaultmayliesolely withJohn. 10. Thatisastrawperson argument.Thereisnoreasontobelievethattheschoolsystemwantsanythingotherthan whatisbestforthestudents.Becausetheschoolsystemissuchanenormousbureaucracy,thereisrealdifficulty inimplementingnewapproaches.Inaddition,thereisrealdifficultyintryingtoestimatetheworthofnewapproaches.Oftenthesafestrouteforaschoolsystemisjusttotryanddobetteratwhatithasbeendoing.

Exercise 4D: Answers


1. Youaremisrepresentingyouropposition.Incriticalthinkingcircles,thatisreferredtoasastrawperson fallacy. Thereasonthatenvironmentalistsareconcernedaboutglobalwarmingisthatrapidtemperaturechangewillhave devastatingconsequencesforallbiospheresthatcannotkeepupwiththechange,tosaynothingofchanging weatherpatterns,whichmayincludemanymorekillerstorms,andsoforth. 2. IamdelightedthatyoubelievethatSt.Lauraiswatchingoveryou,buttosaythatthefactthatyousustainedno injuriesinacarcrashbecauseyouwerewearinghermedalisapost hoc fallacy.Inordertomakeacausalclaim, youhavetoeliminateallotherexplanations.Youmayhavebeensavedfrominjurybythefactthatyouhappened tobesittinginaparticularseat,forinstance. 3. Thatisanillegitimate slippery slope argument.IfyoumaketheclaimthatlisteningtoMarilynMansonwill progressthroughallkindsofstepsuntilkidsendupactinglikefreaks,thenyouhavetoexplainhowonestep leadstoanother. 4. Thatisclearlybegging the question.Yourreasonisalmostidenticaltotheconclusion,soyouarearguingina circle.Youneedtogiveareasonexplainingwhyitisimportantthatonlypeopleinacommittedrelationshipor onlypeoplewhogenuinelycareforoneanothershouldengageinsex. 5. Withallduerespect,sir,thatisafalse dilemma.Thereweremanyotheroptionsavailabletoyou,suchasallowingtheUNweaponsinspectorstocontinuetheirwork,orcontinuingnegotiationsthroughtheUN,orincreasing theeffectivenessofyourownintelligenceuntiltherewasproofpositivethattherewereweaponsofmassdestruction.Eventhen,apreemptivestrikemaynothavebeenthebestrecourse. 6. Tothemomyoushouldsay,Youhavejustcommittedafallacy of the golden mean.Truthisafunctionofreason orevidence.Thefactthattwopeoplehappentobefightingisnoevidencewhatsoeverthattheyarebothatfault. Itmaybeentirelythefaultofonlyone.Ifyoureallywanttofindoutwhoisatfault,youneedtolistentothe reasonsandevidencethateachcanproduce. 7. Well,myfriend,youaretryingtomakeadistinction that has no difference.Yousaythatyouarenotlazy,but tryingeverythinginyourpowertogetoutofhelpingthegroupisexactlythekindofbehaviorthatshowsyou arelazy.

212 AppEndix i
8. Well,Mr.Bertuzzi,youhavejustcommittedanillegitimate appeal to authority.Youarenotanexpertinthe fieldofautoqualityandcomparisons,soyoucannotaskustotakeyouradvicewithoutreasonsorevidence. 9. ToJohnyoushouldsay,Thatisanillegitimate ad feminam attack.Marypresentedyouwithanargumentthat shouldbejudgedonitsownmerits.Wouldyoucaretocommentonwhetherornotyoufeeljustifiedinsupportingapracticethatinflictsunnecessarypainonanimals? 10. Thatisafaulty analogy.Keepingwhalesincaptivityisentirelydifferentfromkeepingdogsincaptivity.Whales haveanaturalhabitatotherthanincaptivity,butadogsnaturalhabitatiscaptivity.Onecanimaginethatwhales flourishandaremuchhappierifleftintheocean.OnecannotimagineaYorkshireTerrierlikeBennie,forinstance,flourishingorbeinghappierifleftinthewoods.

Exercise 5A: Answers


1.Valid,butunsound. 2.Sound.(Technically,youdonotneedtosayvalid,asthedefinitionofasoundargumentisavalidargument whosepremisesaretrue.) 3.Valid,butunsound. 4.Technically,thisisavaliddeductiveargument,althoughitisprobablymorecorrectlylabeledasinductive.What thisargumentshowsisthatadeductiveargumentformsthebasisofallinductivearguments.Soifyouanswered deductive,thereisasenseinwhichyouareright.Ifyouansweredinductive,youarealsoright.Thedegreeto whichthiscouldbelabeledastronginductiveargumentdependsontheefficacyofgatheringthecorrectinformation(e.g.,shouldweassumethatallstudentsinthesampletoldthetruth?)andthenonthesizeandrepresentative natureofthesample. 5.Valid.Itwouldrequirealotofresearchtoinvestigateitssoundness.

Exercise 5B: Answers


1. Inavaliddeductiveargument,ifthepremisesaretrue,whatdoyouknowabouttheconclusion?It must be true. 2. Inaninductiveargument,ifthepremisesaretrue,whatdoyouknowabouttheconclusion?It is always only more or less probable. 3. Inadeductiveargument,iftheargumentisvalid,whatdoyouknowaboutthepremises?Nothing. See 1, 3, and 6 in Exercise 5A. 4. Inadeductiveargument,iftheargumentissound,whatdoyouknowaboutthepremises?They must be true. The definition of a sound argument is a deductive argument whose premises are true. 5. Inadeductiveargument,iftheargumentisvalid,whatdoyouknowabouttheconclusion?Nothing. Note that item 1 in Exercise 5A is valid but unsound and has a false conclusion, whereas items 3 and 6 are valid but unsound but have true conclusions. In the latter two instances, the person making the argument is right for the wrong reason. Remember, ones opinion or judgment is only ever as good as the reasons that back it.

Exercise 6A: Answers


1. (P1)All[actsthattakealife]areactsthatarewrong. (P2)(Abortion)is[anactthattakesalife]. (C)(Abortion)isanactthatiswrong. 2. (P1)All[actsthatdenywomentherighttochooseanything]areactsthatarewrong. (P2)(Outlawingabortion)is[anactthatdenieswomentherighttochoose]. (C)(Outlawingabortion)isanactthatiswrong. 3. (P1)All[thingsthatareoffensivetomany]arethingsthatshouldbeoutlawed. (P2)Nudity)is[somethingthatisoffensivetomany]. (C)(Nudity)issomethingthatshouldbeoutlawed.

AnswErs to ExErcisEs

213

4. (P1)All[actsthatmightresultinthedeathofaninnocentman]areactsthatshouldbeoutlawed. (P2)(Capitalpunishment)is[anactthatmightresultinthedeathofaninnocentman]. (C)(Capitalpunishment)isanactthatshouldbeoutlawed. 5.(P1)All[peoplewhoperpetrateaharmthatlastsforever]arepeoplewhoshouldbecastrated. (P2)(Rapists)are[peoplewhoperpetrateaharmthatlastsforever]. (C)(Rapists)arepeoplewhoshouldbecastrated.

Exercise 7A: Answers


1. (P1)All[thingsthatwillincreasethesafetyofcyclistsandmakedrivingeasierformotorists]arethingsthat shouldbeprovidedbythecity. (P2)(Pavedbicyclelanes)are[thingsthatwillincreasethesafetyofcyclistsandmakedrivingeasierfor motorists]. (C)Therefore,(pavedbicyclelanes)arethingsthatthecityshouldprovide. premise 1 counterexample:Takingdownallthemajorbuildings Rating: premise 2 counterexample Rating:++ Overallrating: question:Areyoutellingmethatthecityshouldbepreparedtodoanythingthatwillincreasethesafetyof cyclistse.g.,takedownallthemajorbuildings? NOTE:Thisisaparticularlyinterestingexample.Thestatedreasongetsaveryhighmarkinitsrating;however, thehiddenpremisegetsaverylowmark.Thus,ifonewerenotabletoseethehiddenpremise,onewouldmistakethis verypoorreasoningforverygoodreasoning. 2. (P1)All[proceduresthatincreasethechancesoffindingcuresformanyofthedeadlydiseasesfromwhichmany humanssuffer]areproceduresthatareethicallyacceptable. (P2)(Usinganimalsinmedicalresearch)is[aprocedurethatwillincreasethechancesoffindingcuresformany ofthedeadlydiseasesfromwhichmanyhumanssuffer]. (C)Therefore,(usinganimalsinmedicalresearch)isaprocedurethatisethicallyacceptable. premise 1 counterexample:Usinghealthychildrenasguineapigsinmedicalresearch Rating: premise 2 Rating:++ Overallrating: question:Areyoutellingmethatanyprocedurethatincreasedthechancesoffindingacureforadeadlydisease fromwhichhumanssufferisacceptablee.g.,experimentingonhumanorphans? 3. (P1)All[individualswholackself-control]areindividualswhoshouldbeashamed. (P2)(Individualswhoaresignificantlyoverweightandwhohavenounderlyingendocrineproblem)are[individualswholackself-control]. (C)Therefore,(individualswhoaresignificantlyoverweightandwhohavenounderlyingendocrineproblem)are individualswhoshouldbeashamed. premise 1 counterexample:Crying Rating: premise 2 counterexample:Manyoverweightindividualsmaynotcareaboutthesocialidealofbeingslimand maybewillingtorisktheassociatedhealthproblemssotheycanfullyenjoytheirfood.Premise2mayrequirea secondqualifier,andwhoarenotgourmands. Rating:+/ Overallreasoning:

214 AppEndix i
e.g.,crying?Note:Inthisinstance,thepersonmayverywellbelievethatallsuchbehaviorisbehavioraboutwhich oneshouldbeashamed.Ortheindividualmaywanttotryaqualificationthateliminatesemotionalresponses. 4. (P1)All[behaviorsthatmightthreatenafriendship]arebehaviorsthatshouldbeavoided. (P2)(Tellingtheteacherthatyourfriendcheatedonanexam)is[abehaviorthatmightthreatenafriendship]. (C)Therefore,(tellingtheteacherthatyourfriendcheatedonanexam)isabehaviorthatshouldbeavoided. premise 1 counterexample:Yourfriendisachildmolester. Rating: premise 2 counterexample Rating:++ Overallrating: question:Areyoutellingmethatyoushouldavoidanybehaviorthatmightthreatenafriendship?Whatifyour friendwasmolestinghischild? 5. (P1)All[peoplewhoarejustfriends]arepeoplewhoshouldalwaysinsistonpayinghalf. (P2)(Peoplewhogooutonafirstdate)are[peoplewhoarejustfriends]. (C)Therefore,(peoplewhogooutonafirstdate)arepeoplewhoshouldalwaysinsistonpayinghalf. premise 1 counterexample:Whatifyourfriendhassubstantiallymoremoney?Orwhatifafriendasks anotherto,say,thetheater?Shouldnttheaskerpay? Rating:+/ premise 2 counterexample:Theycouldbeclassedasmorestrangersthanfriends. Rating:+ Overallrating:+/ question:Areyoutellingmethatfriendsshouldalwaysinsistonpayinghalfe.g.,evenifonehassubstantially moremoneythantheother?Evenifoneaskedtheotherto,say,thetheater? 6. (P1)All[actionsthatareeffectiveinchangingachildsbehavior]areactionsthatareethicallyacceptable. (P2)(Physicalpunishment)is[anactionthatiseffectiveinchangingachildsbehavior]. (C)Therefore,(physicalpunishment)isanactionthatisethicallyacceptable. premise 1 counterexample:Boilingachildinoilwould,nodoubt,changehisorherbehavior. Rating: premise 2 counterexample:Insomeinstancesitmaycreaterebellionandhencebecounterproductive. Rating:+ Overallrating: question:Areyoutellingmethatanyactionthatiseffectiveinchangingachildsbehaviorisacceptablee.g., boilingachildinhotoil? 7. (P1)All[actionsthatsendamessagetochildrenthattheuseofviolenceisanacceptablewaytosolveproblems] areactionsthatshouldbeavoided. (P2)(Usingphysicalpunishmentonachild)is[anactionthatsendsamessagetoachildthattheuseofviolence isanacceptablewaytosolveproblems]. (C)Therefore,(usingphysicalpunishmentonachild)isanactionthatshouldbeavoided. premise 1 counterexample:Howaboutcartoons?Althoughthepersoncouldjustaddliteraltoactiontoget aroundthisproblem. Rating:+ premise 2 counterexample Rating:++ Overallratings:+ question:Areyoutellingmethateverythingthatsendsamessagetochildrenthatviolenceisanacceptableway tosolveproblemsshouldbeavoided?Whataboutcartoons?
question:Areyoutellingmethatpeopleshouldbeashamedofallbehaviorsthatindicatealackofself-control

AnswErs to ExErcisEs

215

8. (P1)All[relationshipsinwhichtrustisbroken]arerelationshipsthatshouldbeterminated. (P2)(Amonogamousrelationshipinwhichonepartnercheatsonanother)is[arelationshipinwhichtrustis broken]. (C)Therefore,(amonogamousrelationshipinwhichonepartnercheatsonanother)isarelationshipthatshould beterminated. premise 1 counterexample:Trustisoftenbrokeninparent-childrelationships,yetsuchrelationshipsoften survive.Friendsalsobreakoneanotherstrust,andoftenlearntoforgiveeachother.Thispremiserequiresmore thought.Istheresomethingspecialaboutbreakingatrustinanintimaterelationship?Istheresomethingspecial aboutthefactthatthetrustwasbrokenbyasexualact? Rating:+/ premise 2 counterexample Rating:++ Overallrating:/+ question:Areyoutellingmethatallrelationshipsinwhichtrustisbrokenshouldbeterminated?Whatabout whenteenagersmisusethetrustoftheirparents? 9. (P1)All[practicesinwhichyouuseyourownmoneytobuysomethingthatyoureallylike]arepracticesthatare perfectlyjustified. (P2)(Buyingreallyextravagantlyexpensiveclothing)is[apracticeinwhichyouuseyourownmoneytobuy somethingthatyoureallylike]. (C)Therefore,(buyingreallyextravagantlyexpensiveclothing)isapracticethatisperfectlyjustified. premise 1 counterexample:Buyinghighlydestructiveweaponsordrugs. Rating: premise 2 counterexample:Theargumentbegsthequestion.Sinceonecanlegitimatelyassumethatifyouspend alotofmoneyonsomething,youmustreallylikeit,thefirstpartofthisclaimisidenticaltothesecond.Itthereforelacksfalsifiability. Rating: Overallrating: question:Yourargumentbegsthequestion.Thequestionatissueiswhetherornotyouarejustifiedinspending anextravagantamountofmoneyonsomethingyoureallylike.Wouldyoucaretoofferareasonexplainingwhy thatisjustified? 10. (P1)Asfaraspossible,all[behaviorsthatcanbepersonallyinsulting]arebehaviorsthatshouldbeavoided. (P2)(Usingpoliticallyincorrectlanguagewhenreferringtoothers)is[abehaviorthatcanbepersonally insulting]. (C)Therefore,(usingpoliticallyincorrectlanguagewhenreferringtoothers)isabehaviorthatshouldbe avoided. premise 1 counterexample:Somemightarguethatusingpoliticallyincorrectlanguage,suchasreferringto someoneasfat,mightbeagoodwake-upcall. Rating:+ premise 2 counterexample Rating:++ Overallrating:+ questions:Dontyouthinkthereareinstancesinwhichitmightprovebeneficialtocallitthewayitis,evenif doingsoisnotpoliticallycorrecte.g.,callingafatpersonfatratherthanhorizontallychallenged?

216 AppEndix i
Exercise 7C: Answers
1. (P1)All[thingsthatcanhelpdevelopeye-handcoordination,nurturememoryskills,andensurethatchildrenwill becomecomfortablewiththetechnologyofthefuture]arethingsthatshouldbeencouragedbyparents. (P2)(Childrenplayingcomputergames)is[somethingthatcanhelpdevelopeye-handcoordination,nurture memoryskills,andensurethatchildrenbecomecomfortablewiththetechnologyofthefuture]. (C)(Childrenplayingcomputergames)issomethingthatshouldbeencouragedbyparents. premise 1 counterexample:Theskillsthatchildrenacquirethroughplayingcomputergamesmaybeoffsetby thefactthattoomuchscreentimemaydestroytheircapacitytointeractwithreal-lifepeople Rating:+ premise 2 counterexample:Itisnotclearthatsuchgamesnurturememoryskills.Inaddition,giventheadvances intechnology,itisnotclearhowgamespersecanensurethatchildrenbecomecomfortablewiththetechnology ofthefuture.ResearchontheInternetmaybemoreproductive,forinstance. Rating:+/ Overallrating:+/ question:Doyoureallythinkthattryingtocapturethebadguysinacomputergameisgoingtohelpprepare youngsterstogetcomfortablewithfuturetechnology?DontyouthinkthatdoingseriousresearchontheInternet mightbemorehelpful? 2. (P1)All[practicesthatcausepaininanimals,thatresultinknowledgethatmaynotbesafelyappliedtohumans, andthatareofquestionablevalue,asweseemtobeapproachingthelimitofhumanlife]arepracticesthatshould beavoided. (P2)(Theuseofanimalsinscientificresearch)is[apracticethatcausespaininanimals,thatresultsinknowledge thatmaynotbesafelyappliedtohumans,andthatisofquestionablevalue,asweseemtobeapproachingthelimit ofhumanlife]. (C)(Theuseofanimalsinscientificresearch)isapracticethatshouldbeavoided. premise 1 counterexample Rating:++ premise 2 counterexample:Someresearchdoesnotcausepaine.g.,operationsdoneonananesthetizedanimal thatistheneuthanized.Someresearchisclearlysafelytransferred.Ifsomeofthetransferredknowledgeisrisky,it maybethelasthope. Rating:+ Overallrating:+ question:Dontyouthinkthatsomeresearch,suchaslearningnewsurgicaltechniques,canbedonesothatthere isnopainfortheanimalandthattheacquiredknowledgecanbetransferredtothefieldofhumanhealth?

Exercise 10A: Answers


NOTE:Theplaceholdersinanyofthefollowinganswerscanbeswitchedaslongasthesignsarechangedi.e.,(all Rottweilersaredogs)=(allnon-dogsarenon-Rottweilers). 1. All(fools)are(peoplewhoknowthat). counterexample:Afoolwhodoesnotknowthat. 2. All(whales)are(mammals). counterexample:Awhalethatisnotamammal. 3. All(papersthatareacceptable)are(papersthataretyped). counterexample:Apaperthatisacceptedthatisnottyped. 4. All(peoplewhoareallowedinthisclub)are(girls). counterexample:Someonewhoisallowedintheclubwhoisnotagirl. 5. All(peoplewhopayadmission)are(peoplewhogetin). counterexample:Someonewhopaysadmissionwhodoesnotgetin.

AnswErs to ExErcisEs

217

6. All(peoplewhoserveonjuries)are(peoplewhoareeligibletovote). counterexample:Someonewhoservesonajurywhoisnoteligibletovote. 7. All(peoplewhovotefortheabortionact)are(Democrats). counterexample:SomeonewhovotedfortheabortionactwhoisnotaDemocrat. 8. Some(conservatives)are(non-Republicans).ThisisthecontradictoryofallconservativesareRepublicans. counterexample:AllconservativesareRepublicans. 9. All(peoplewholive)are(peoplewhohavethisoperation). counterexample:Someonewholiveswhodoesnothavethisoperation. 10. All(peoplewhohavethisoperation)are(peoplewhowilllive). counterexample:Someonewhohasthisoperationbutwhodoesnotlive. 11. All(peoplewhogetgoodjobs)are(peoplewhohaveexperience). counterexample:Someonewhogetsagoodjobbutwhodoesnothaveexperience. 12. Some(membersofthisclass)are(peoplewhowerehomewiththeflu)andsome(membersofthisclass)arenot (peoplewhowerehomewiththeflu). counterexample:Technicallyboththenoandallclaimswouldbecounterexamples,althoughinreallife,the noclaimwouldprobablybethemostusualcounterexample. 13. All(peoplewhoremainemployedhere)are(peoplewhoattendweeklymeetings). counterexample:Someonewhoremainsemployedherebutwhodoesnotattendweeklymeetings. 14. All(peoplewhohavegoodjobs)are(peoplewhoknowsomeoneofinfluence). counterexample:Someonewithagoodjobbutwhodoesnotknowanyoneofinfluence. 15. All(peoplewhoworkhard)are(peoplewhoaresuccessful). counterexample:Apersonwhoworkshardandwhoisnotsuccessful.

Exercise 10B: Answers


NOTE:Theplaceholdersinanyofthefollowinganswerscanbeswitchedaslongasthesignsarechangedi.e.,(all Rottweilersaredogs)=(allnon-dogsarenon-Rottweilers). 1. All(suicides)are(actsthatarewrong). counterexample:Asuicidethatisnotawrongact. 2. Some(suicides)are(actsthatarenotwrong)andsome(suicides)arenot(actsthatarenotwrong). counterexample:All,no,orfew. 3. All(peoplewhocommittedsuicidelastyear)are(peopleovertheageoften). counterexample:Onepersonwhocommittedsuicidelastyearwhowasundertheageoften. 4. All(timesthatyouhurtthepeoplewholoveyou)are(timesthatyouhavedonesomethingwrong). counterexample:Atimethatyouhurtsomeonewholovedyoubutyoudidnotdosomethingwrong. 5. All(peoplewhocommitsuicide)are(peoplewhoarenotsaneatthetime). counterexample:Someonewhocommittedsuicidewhowassaneatthetime. 6. All(peoplewhocommitsuicideeasily)are(peoplewhohavehelpfromothers). counterexample:Someonewhocommitssuicideeasilybutwhodoesnothavehelpfromothers. 7. All(timesthatcropsgrow)are(timeswhenitrains.). counterexample:Atimewhenthecropsgrowbutitdoesnotrain. 8. All(peoplewholikeexistentialisttheory)are(peoplewholikeCamussThe Plague). counterexample:SomeonewholikesexistentialisttheorybutwhodoesnotlikeCamussThe Plague. 9. All(peoplewholikeexistentialisttheory)are(pessimists). counterexample:Someonewholikesexistentialisttheorybutwhoisnotapessimist. 10. All(existentialistworks)are(nonphilosophicalworks). counterexample:Anexistentialistworkthatisrealphilosophy.

218 AppEndix i
Exercise 10C: Answers
NOTE:Theplaceholdersinanyofthefollowinganswerscanbeswitchedaslongasthesignsarechangedi.e.,(all Rottweilersaredogs)=(allnon-dogsarenon-Rottweilers). 1. All(peoplewhoareautonomous)are(peoplewholearntothinkimpartially). counterexample:Someonewhoisautonomousbutwhodoesnotlearntothinkimpartially. 2. All(entitiesthatareself-conscious)are(entitiesthatcanlearnasymboliclanguage)and(allentitiesthatlearna symboliclanguage)are(entitiesthatareself-conscious). counterexample:Anentitythatisnotself-consciousbutthatcanlearnasymboliclanguageoranentitythatis self-consciousbutthatcannotlearnasymboliclanguage. 3. All(peoplewhomastertheartofthinkingimpartially)are(peoplewholearntoarticulatelylistentoothers). counterexample:Someonewhomasterstheartofthinkingimpartiallybutwhodoesnotlearntoarticulatelylisten toothers. 4. All(peoplewhoaskalotofquestions)are(peoplewhoaregoodthinkers). counterexample:Someonewhoasksalotofquestionsbutwhoisnotagoodthinker. 5. All(timeswhensomeonegivesintogroup-think)are(timeswhensomeonesautonomyisjeopardized). counterexample:Someonewhogivesintogroup-thinkbutwhoseautonomyisnotjeopardized.

Exercise 10E: Answers


NOTE:Theplaceholdersinanyofthefollowinganswerscanbeswitchedaslongasthesignsarechangedi.e.,(all Rottweilersaredogs)=(allnon-dogsarenon-Rottweilers). 1. All(horses)are(things/animalsthatarenotself-conscious). counterexample:Ahorsethatisself-conscious. 2.All(horsesthatcanrunlikethewind)are(Arabianhorses). counterexample:AhorsethatcanrunlikethewindthatisnotArabian. 3.All(healthyhorses)are(horsesthataregivenhigh-qualityfood). counterexample:Ahealthyhorsethatisnotgivenhigh-qualityfood. 4. All(goodhorsepeople)are(peoplewhoputshoesonahorsewhenitwalksonpavement). NOTE:Theclaimisanimpliedvaluestandard.Thetranslationmustshowthatavaluestandardisbeingclaimed. counterexample:Agoodhorsepersonwhodoesnotputshoesonahorsewhenitwalksonpavement. 5. All(riders)are(peoplewhoshouldpicktheirhorseshoovesbeforeriding). counterexample:Ariderwhoshouldnotpickhisorherhorseshoovesbeforeriding. NOTE:Theclaimisanimpliedvaluestandard.Thetranslationmustshowthatavaluestandardisbeingclaimed.

Exercise 11A: Answers


NOTE:Theplaceholdersinanyofthefollowinganswerscanbeswitchedaslongasthesignsarechangedi.e.,(all Rottweilersaredogs)=(allnon-dogsarenon-Rottweilers). 1. All(peoplewhomakealeftturnthreekilometersback)are(peoplewhodonotgetstuckintheswamp). Areyoutellingmethatmakingaleftturnthreekilometersbackwouldhavebeenasufficientconditionforensuringthatwewouldnothavegottenstuckintheswamp? Areyoutellingmethatifanyonemakesaturnthreekilometersback,notgettingstuckintheswampwillnecessarilyfollow? 2. All(peoplewhovote)are(peoplewhoregister). Areyoutellingmethatifsomeonevotes,thatisasufficientconditiontoknowthatthatpersonhasregistered? Areyoutellingmethatregistrationisanecessaryconditionforvoting?

AnswErs to ExErcisEs

219

3. All(peoplewhogetpastthedoormanandintotheclub)are(peoplewhoarerichandfamous). Areyoutellingmethatifsomeonegetspastthedoormanandintotheclub,thatisasufficientconditiontoknow thatthatpersonmustberichandfamous? Areyoutellingmethatbeingrichandfamousisanecessaryconditionforgettingintotheclub? 4. All(peoplewhohavejusteaten)are(peoplewhoshouldstayofftheride,theHowlinScreamer). Areyoutellingmethatjusthavingeatenisasufficientconditionforstayingofftheride,theHowlinScreamer? Areyoutellingmethatifsomeonehasjusteaten,stayingofftheHowlinScreamershouldnecessarilyfollow? 5.All(peoplewhoweartiestoschool)are(geeks). Areyoutellingmethatwearingatietoschoolissufficienttobelabeledageek? Areyoutellingmethatbeingageekisanecessaryconditionforwearingatietoschool? 6.All(peoplelikeyouwhocouldseemenow)are(peoplewhowouldbeveryproudofmenow). Areyoutellingmethatseeingyounowwouldbeasufficientconditionforbeingproudofyou? AreyoutellingmethatifIcouldseeyounow,beingproudofyouwouldnecessarilyfollow? 7. All(peoplelikemewhogoonthehikeonSaturday)are(peopletowhomyouhavelentraingear). AreyoutellingmethatifsomeonelikeyouisonthehikeonSaturday,thatisasufficientconditiontoknow thatthatpersonhasbeenlentraingear? AreyoutellingmethatlendingyouraingearisanecessaryconditionforyourcomingonthehikeonSaturday? 8. All(peoplewhoshouldnotfeelcompelledtocommitthecheatsheettomemory)are(peoplewhoarecompletely comfortablewithusingtheRottweiler/dogtactic). Areyoutellingmethatknowingthatsomeonedoesnotfeelcompelledtocommitthecheatsheettomemory shouldbeasufficientconditionforknowingthatthatpersoniscompletelycomfortablewithusingtheRottweiler/ dogtactic? AreyoutellingmethatbeingcompletelycomfortablewithusingtheRottweiler/dogtacticshouldbeanecessary conditionfornotfeelingcompelledtocommitthecheatsheettomemory? 9. All(peoplewhoareallowedtousetheelevator)are(bankemployees). Areyoutellingmethatifsomeoneisallowedontheelevator,thatisasufficientconditiontoknowthatthatpersonisabankemployee? Areyoutellingmethatbeingabankemployeeisanecessaryconditionforusingtheelevator? 10. All(peoplewhocleantheirrooms)are(peoplewhocangoout)andall(peoplewhocangoout)are(peoplewho cleantheirrooms). Areyoutellingmethatcleaningmyroomisbothanecessaryandsufficientconditionforgoingout?

Exercise 11B: Answers


NOTE:Theplaceholdersinanyofthefollowinganswerscanbeswitchedaslongasthesignsarechangedi.e.,(all Rottweilersaredogs)=(allnon-dogsarenon-Rottweilers). 1.All(peoplelikeyouwholoveme)are(peoplewhomIwillfollowanywhere). Areyoutellingmethatmylovingyouisasufficientconditionforensuringthatyouwillfollowmeanywhere? AreyoutellingmethatifIloveyou,yourfollowingmeanywherewillnecessarilyfollow? 2.All(peoplewhodonotfireretro-rocketsnow)are(peoplewhomissthereentrywindow). Areyoutellingmethatnotfiringourretro-rocketsnowissufficienttoensurethatwewillmissourreentry window? Areyoutellingmethatfiringtheretro-rocketsnowisanecessaryconditionforreentry? Or Areyoutellingmethatifwedonotfiretheretro-rockets,missingthereentrywindowwillnecessarilyfollow? 3.All(peoplewhofireretro-rocketsinthreeminutes)are(peoplewhowillbeabletoreenterEarthsatmosphere). Areyoutellingmethatfiringtheretro-rocketsinthreeminuteswillbesufficienttoensurethatwereenterEarths atmosphere?

220 AppEndix i
Areyoutellingmethatifwefiretheretro-rocketsinthreeminutes,beingabletoreenterEarthsatmospherewill necessarilyfollow? 4.All(peoplewhoshouldnotstrivetocreateacareerthattheycouldcommittofortherestoftheirlives)are (peoplewhoareindependentlywealthy). Areyoutellingmethatifyouknowthatsomeoneshouldnotstrivetocreateacareerthatheorshecancommit tofortherestofhisorherlife,thatisasufficientconditionforknowingthatthatpersonisindependentlywealthy? Areyoutellingmethatbeingindependentlywealthyshouldbeanecessaryconditionforbelievingthatyou shouldnotstrivetocreateacareerthatyoucancommittofortherestofyourlife? 5.All(timeswhenrosesareredandvioletsareblue)are(timeswhenwritingshortpoemsiseasytodo). Areyoutellingmethatthefactthatrosesareredandvioletsareblueissufficienttoensurethatwritingshort poemsiseasytodo? Areyoutellingmethatifrosesareredandvioletsareblue,shortpoemsbeingeasytowritewillnecessarily follow? 6.All(professorslikemyselfwhocomeupwithmoresentences)are(professorswhoperceivearealneedforthem). Areyoutellingmethatifaprofessorlikeyoucomesupwithmoresentences,thatisasufficientconditionto knowthatthatprofessormustperceivearealneedforthem? Areyoutellingmethatourreallyneedingthemisanecessaryconditionforyoutocomeupwithmoresentences? 7.All(manageablerefugeesituations)are(situationsinwhichimmigrationstandardsarenottoostrict)andall(situationsinwhichimmigrationstandardsarenottoostrict)are(situationswithmanageablerefugeeproblems). Areyoutellingmethatliberalimmigrationstandardsarebothanecessaryandsufficientconditionformanageable refugeesituations? 8.All(thingsthatleadtooccasionalbizarreaccidents)are(thingsthatweknowhaveunpredictableeffects). Areyoutellingmethatknowingthatsomethingleadstotheoccasionalbizarreaccidentissufficienttoknowthat theentityinquestionhasunpredictableeffects? Areyoutellingmethatifsomethingleadstooccasionalbizarreaccidents,thenknowingthatthatthinghasunpredictableeffectsnecessarilyfollows? 9.All(peoplewhodowellinacademia)are(peoplewhohaveacomputer). Areyoutellingmethatifsomeonedoeswellinacademia,thatisasufficientconditiontoknowthatthatperson musthaveacomputer? Areyoutellingmethathavingacomputerisanecessaryconditionfordoingwellinschool? 10.All(peoplewhothinkwell)are(peoplewhoarticulatelylistentothepointofviewofothers). Areyoutellingmethatifsomeonethinkswell,thatisasufficientconditiontoknowthatthatpersonarticulately listenstothepointofviewofothers? Areyoutellingmethatbeingabletoarticulatelylistentothepointofviewofothersisanecessaryconditionfor thinkingwell?

Exercise 11C: Answers


1. Sufficient 2. Sufficient 3. Necessary 4. Necessary 5. Sufficient 6. Necessary 7. Necessary 8. Necessary 9. Sufficientandnecessary

AnswErs to ExErcisEs

221

10. Necessary 11. Sufficient;sufficient 12. Sufficient 13. Neithersufficientnornecessary 14. Sufficient 15. Necessary

Exercise 11D: Answers


NOTE:Theplaceholdersinanyofthefollowinganswerscanbeswitchedaslongasthesignsarechangedi.e.,(all Rottweilersaredogs)=(allnon-dogsarenon-Rottweilers). 1. All(peoplewithfaith)are(peoplewhohavehope). Areyoutellingmethatifsomeonehasfaith,thatisasufficientconditionforknowingthatthatpersonhashope? Areyoutellingmethathopeisanecessaryconditionoffaith? counterexample:Apersonwithfaithwhodoesnothavehope. 2.All(peoplewithfaith)are(peoplewholove). Areyoutellingmethatifsomeonehasfaith,thatisasufficientconditionforknowingthatthatpersonisaperson wholoves? Areyoutellingmethatloveisanecessaryconditionoffaith? counterexample:Someonewhohasfaithbutwhodoesnotlove. 3.All(peoplewhotrulylove)are(peoplewhoarekind). Areyoutellingmethatifsomebodyistrulyloving,thatisasufficientconditiontoknowthatthatpersoniskind? Areyoutellingmethatbeingkindisanecessaryconditionofbeingabletolove? counterexample:Apersonwhoistrulylovingbutwhoisnotkind. 4.All(peoplewhohavefaith)are(peoplewhohavehope). Areyoutellingmethatifsomeonehasfaith,thatisasufficientconditiontoknowthispersonhashope? Areyoutellingmethatifsomeonehasfaith,havinghopewillnecessarilyfollow? counterexample:Someonewhohasfaithbutwhodoesnothavehope. 5.All(peoplewhoarekind)are(peoplewhoareloving). Areyoutellingmethatifoneiskind,thatisasufficientconditiontoknowthatoneisloving? Areyoutellingmethatbeinglovingisanecessaryconditionforbeingkind? counterexample:Someonewhoiskindwhoisnotloving. 6.All(peoplewhohavefaith)are(peoplewhoarecharitable). Areyoutellingmethatifsomeonehasfaith,thatisasufficientconditiontoknowthatthatpersonischaritable? Areyoutellingmethatbeingcharitableisanecessaryconditionforhavingfaith? counterexample:Someonewhohasfaithbutwhoisnotcharitable. 7.All(peoplewhohavehope)are(peoplewhohavefaith),andall(peoplewhohavefaith)are(peoplewhohave hope). Areyoutellingmethatfaithisanecessaryandsufficientconditionforhope? counterexample:Eithersomeonewhohasfaithbutnohopeorsomeonewhohashopebutnofaith. 8. All(peoplewithintegrity)are(peoplewhovaluehonor). Areyoutellingmethatifsomeonehasintegrity,thatisasufficientconditiontoshowthatthatpersonvalues honor? Areyoutellingmethatvaluinghonorisanecessaryconditionofhavingintegrity? counterexample:Someonewhohasintegritybutwhodoesnotvaluehonor.

222 AppEndix i
Exercise 11E: Answers
NOTE:Theplaceholdersinanyofthefollowinganswerscanbeswitchedaslongasthesignsarechangedi.e.,(all Rottweilersaredogs)=(allnon-dogsarenon-Rottweilers). 1. All(peoplewhohaveautonomy)are(peoplewhoareimpartial). Areyoutellingmethatifyouknowsomeonewhoisautonomous,thatisasufficientconditionforknowingthat thatpersonisimpartial? Areyoutellingmethatimpartialityisanecessaryconditionofautonomy? counterexample:Anautonomouspersonwhoisnotimpartial. 2.All(peoplewhospendalotoftimewithnonautonomouspersons)are(peoplewhoseautonomyisthreatened). Areyoutellingmethatknowingthatsomeonespendsalotoftimewithnonautonomouspeopleisasufficientconditionforknowingthatthatpersonsautonomyisthreatened? Areyoutellingmethatonesautonomybeingthreatenednecessarilyfollowsspendingalotoftimewithnonautonomouspeople? counterexample:Apersonwhospendsalotoftimewithnonautonomouspeoplebutwhoseautonomyisnot threatened. 3. All(peoplewhohate)are(peoplewhoareunabletobeautonomous). Areyoutellingmethatbeingfilledwithhateisasufficientconditionforensuringthatonecannotbeautonomous? Areyoutellingmethatrefusingtohateisanecessaryconditionofautonomy?(NotethatthatflipisOKbecause thesignschanged.) counterexample:Someonewhohatesbutwhoisautonomous. 4. All(peoplewhoareautonomous)are(peoplewhoarepreparedtostandupandbecounted). Areyoutellingmethatifyouknowthatsomeoneisautonomous,thatisasufficientconditionforknowingthat thatpersonwillstandupandbecounted? Areyoutellingmethatifsomeoneisautonomous,beingpreparedtostandupandbecountedwillnecessarily follow? counterexample:Someonewhoisautonomousbutwhoisnotpreparedtostandupandbecounted. 5.All(peoplewhoareautonomous)are(peoplewhogenuinelycareaboutatleastsomeothers). Areyoutellingmethatifyouknowthatsomeoneisautonomous,thatisasufficientconditionforknowingthat thatpersongenuinelycaresaboutatleastsomeothers? Areyoutellingmethatbeingabletogenuinelycareaboutatleastsomeothersisanecessaryconditionofbeing autonomous? counterexample:Someonewhoisautonomousbutwhodoesnotgenuinelycareaboutanybody.

Exercise 11F: Answers


1.Ifsomeonesays,Youwontgetintomedicalschoolbecauseyoudidntgetfirst-classgradesincollege,thatperson isassumingthatgettingfirst-classgradesisanecessaryconditionforgettingintomedicalschool. 2.Ifsomeonesays,Wegavethepatienttengrainsofaspirinsothathisfeverwouldcomedown,thatpersonisassumingthattengrainsofaspirinisasufficientconditionforbringingdownafever. 3.Ifsomeonesays,Sheisgoingtobeasdrunkasaskunk:Shepolishedoffthreemartinisinthelasthalf-hour,that personisassumingthatdrinkingthreemartinisisasufficientconditionforgettingdrunkasaskunk. 4. Ifsomeonesays,Wehaverunoutofflour,sowecantmakebread,thatpersonisassumingthatflourisanecessaryconditionformakingbread.

AnswErs to ExErcisEs

223

Exercise 11G: Answers


1. Ifrosesarered,thenvioletsareblue. a. Thefactthatvioletsgrowblueinthespringbeforetherosesbloomshowsthatthatclaimisfalse. b. Thefactthatrosesgrowtoalovelyredrichcolorlongafterthevioletsaredeadshowsthatthatclaimisfalse. c. Bothaandb. item: (red roses) (blue violets) a. (Blue violets) but (~ red roses) is the counter to a Rottie flip. *b. (Red roses) but (~ blue violets). 2. Itisnottruethatonlyhappypeoplearegenerous. a. Allhappypeoplearegenerous. b.Allgenerouspeoplearehappy. c.Neitheranorb. item: opposition to (generous) are (happy) = some (generous) but (~ happy). F a. rom the fact that it is false that all generous people are happy, you cannot deduce its flip that all happy people are generous. b. This is precisely what is being denied. *c. 3.Therecanbenopeacebecausethereisnojoy. a.Therecanbenojoyunlessthereispeace. b.Thefactthatmanypeoplearemiserableinsituationsofrealpeaceshowsthatthatclaimisfalse. c.Alljoyfultimesarepeacefultimes. item: (no joy) (no peace). a. ( Joy) (peace), which is a Rottie flip. *b. he counterexample to the initial claim is some (no joy) but (~ no peace)the last instance is a double T negative, so it is (miserable) but (peace). c. A Rottie flip. 4.Youcannotbefairifyouarenotintelligent. a.Well,Hitlerwascertainlyintelligent,buthewashardlyfair.Thatshowsthatthatstatementisfalse. b.Areyoureallytellingmethatonlyintelligentpeoplearefair? c.Neitheranorb. item: (not intelligent) (not fair). a. The counterexample to this would be (not intelligent) but (fair). *b. (Fair) (intelligent), which is the same as the initial claim flipped with changed signs. 5.Dullwomenhaveimmaculatehouses. a.Thatisterrific.Myhouseisalwaysamess.IguessthatmeansthatIamaveryinterestingperson. b.Ohno.Myhouseisalwaysclean.IguessthatmeansthatIamdull. c.Bothaandb. item: (dull) (immaculate). *a. (Mess) (interesting), which is the flip of the aforementioned statement with changed signs. b. A Rottie flip. 6.Youshouldhugyourcoworkersinordertoelevateofficespirit. a.Huggingwillraiseofficespirits. b.Everybodyintheofficeismiserable.Thatshowstherewasnotmuchhugginggoingon. c.Neitheranorb. item: (office spirit) (hugs). a. A Rottie flip. b. If no joy, then no hugging. A Rottie flip. *c.

224 AppEndix i
7.Itisnottruethatonlyempatheticpeoplearekind. a.Thefactthatsomeempatheticpeoplearekindshowsthatthatstatementisfalse. b.Thefactthatyoucanonlybeempatheticifyouarekindshowsthatthatstatementisfalse. c. Thefactthatallkindpeopleareempatheticshowsthatthatstatementisfalse. item: this is the opposition to (kind) (empathetic), which would be some (kind) but (~ empathetic). a. The opposition to the Rottie flip. b. A Rottie flip. *c. This is what the original claim is saying is false. 8. Onlycitizenscanserveonajury. a.Youcanserveonajuryifyouareacitizen. b.Allcitizenshavetherighttoserveonajury. c.Neitheranorb. item: ( jury) (citizens). a. A Rottie flip. b. A Rottie flip. *c. 9.Hopefollowslove. a.ThefactthatJanehasbeenhopelesslyinloveforyearsshowsthatthatclaimisfalse. b.Thefactthattherearemanyinstancesinwhichpeoplebravelyhope,eventhoughtheymayfeelloveless, showsthatthatclaimisfalse. c.Bothaandb. item: (love) (hope). *a. This says (love) but ( ~ hope), which is the counterexample to the initial claim. b. This says (hope) but (~ love), which is a counterexample to the Rottie flip. 10.Savetheplanet.Itistheonlyonewherechocolategrows. a.Areyoutellingmethattheonlyreasonthatweshouldtrytosavethisplanetisbecauseyourfatgutcannotgo adaywithoutchocolate? b.Areyoutellingmethateveryonewholoveschocolateshouldbeanenvironmentalist? c.Neitheranorb. item: (chocolate) (planet). a. A Rottie flip. *b. The is a rewording of the original claim.

Exercise 11H: Answers


1.Peoplearegreedybecausetheyareignorant. a.Allgreedypeopleareignorant. b.ThefactthatScroogewasgreedybutnotignorantshowsthatthatclaimisfalse. c.Neitheranorb. item: (ignorant) (greedy). a. Rottie flip. b. The counterexample to the initial claim is (ignorant) but (~ greedy). *c. 2.Courageisessentialforhonesty. a.ThefactthatJohnisdeadhonestbutnonethelessacowardshowsthatthatclaimisfalse. b.Youcannotbecourageousunlessyouarehonest. c.AlexandertheGreatmusthavebeenanhonestman.Weknowforsurethathewascourageous. item: (honest) (courage). *a. The counterexample to the initial claim is indeed (honest) but (~ courageous). b. A Rottie flip. c. A Rottie flip.

AnswErs to ExErcisEs

225

3.Lifeisnotworthlivingwithoutchocolate. a.Jeanisboundtofeelbettersoon.Iboughtherthebiggestboxofchocolatesinthestore. b.Theysaythatthecacaobeanisbecomingendangered.Thatisbadnewsforhumankind.Nochocolatewill meanmasshumanmisery. c.Neitheranorb. item: (life worth living) (chocolate). a. A Rottie flip. *b. Flip of the initial claim, with changed sign. 4.Youshouldeatredmeatifyouwanttohaveagoodbloodcount. a.Noveganshavegoodbloodcounts. b.Everyonewhoeatsmeathasagoodbloodcount. c.Neitheranorb. item: (blood) (meat). *a. The flip of the initial claim, with changed signs. b. A Rottie flip. 5.Youcannotfailunlessyouarelazy. a.Janedidnotfail.Thatprovesthatsheisnotlazy. b.Areyoutellingmethateverysinglepersonwhohaseverfailedisapersonwhoislazy? c.JohnisthelaziestpersonIknow.Heisboundtofail. item: (fail) (lazy). a. A Rottie flip. *b. Yes. c. A Rottie flip. 6. Youcanonlyconsideryourselfbeautifulifyouhavelotsofdates. a.IguessthatmeansthatJaneisanuglyduckling.Shenevergoesout. b.Beautifulpeoplealwayshavelotsofdates. c.Bothaandb. item: (beautiful) (dates). a. This is the flip of the initial claim, but it makes the same statement because the signs are changed. b. This is the initial claim. *c. 7.Lonelypeoplemakegoodpoets. a.Youcanonlybecomeagoodpoetifyouexperienceloneliness. b.Johnhasnofriends.Heisboundtowinthepoetryprize. c.Bothaandb. item: (lonely) (poets). a. A Rottie flip *b. This is a rewording of the initial claim. 8.Religionisthebreedinggroundforarrogance. a.ThefactthatBuddhawasnotarrogantshowsthatthatclaimisfalse. b.Thefactthattherearelotsofarrogantpeopleinthisworldwhoarenotreligiousshowsthatthatclaimisfalse. c.Thefactthattheworldisfullofpeoplewhoareneitherreligiousnorarrogantshowsthatthatclaimisfalse. item: (religion) (arrogance). *a. (Religion) but (~ arrogance) is the counterexample to the initial claim. b. (Arrogance) but (~ religion) is the counterexample to a Rottie flip. c. ~ Religion) but (~ arrogance) is a counterexample to the claim that all nonreligious people are arrogant ( or all nonarrogant people are religious. 9.Unlessyouarereallycareful,youwillnotbeabletocope. a.Danishopelesslycareless.Thatmeanshewillnotbeabletocope. b.Ifyouarecoping,thatshowsthatyouareacarefulperson. c.Bothaandb.

226 AppEndix i
item: (cope) (careful). a. This is the flip of the initial claim, but it makes the same statement because the signs are changed. b. This is a rewording of the initial claim. *c. 10.Youshouldbekindtoherbecausesheissad. a.AreyoutellingmethatIshouldbekindtoeverysinglesadpersonImeet? b.ThefactthatIshouldnotbekindtothelocalserialkillerwhoissadshowsthatthatisastupidthingto sayi.e.,thatitisfalse. c.Bothaandb. item: (sad) (kind). a. This is exactly what the person is saying. b. (Sad) but (~ kind) is the counterexample to the initial claim. *c.

Exercise 12A: Answers


1. b 2. c 3. e 4. b 5. b

Exercise 13B: Answers


1. e 2. c 3. c 4. c 5. a 6. b

Exercise 14A: Answers


1.PTS ~PT __________ ~S Denyingthesufficient. Yourargumentisnotvalid.Firstyousaythatthinkingpositivelyissufficienttoensuresuccess,andthenyou implythatitisanecessarycondition. 2. SM M __________ S Affirmingthenecessary. Yourargumentisinvalid.Firstyousaythatagoodmarketisanecessaryconditionforsellingyourhouse,and thenyouimplythatitisasufficientcondition.

AnswErs to ExErcisEs

227

3.R~S ~(~S) __________ ~R Denyingthenecessary. Yourargumentiscertainlyvalid.Ifitistruethatnotstrikingisanecessaryconditionforrecoveryoftheplant, andifitistruethatastrikehasindeedbeencalled,thenitcertainlyfollowsthattheplantwillnotrecover. (But....) 4.CSO ~C __________ ~SO Denyingthesufficient. Yourargumentisinvalid.Firstyousaythathavingcourageisasufficientconditiontoensurethatsomeonewill speakout,andthenyouimplythatitisanecessarycondition. 5. ELL ~LL __________ ~E Denyingthenecessary. Yourargumentiscertainlyvalid.Ifitistruethatlovingtolearnisanecessaryconsequenceofagoodeducation, andifitistrueStevehatestolearn,thenitcertainlyfollowsthathemusthavehadapooreducation. (But....) 6.DYG ~DY __________ ~G Denyingthesufficient. Yourargumentisinvalid.Yourteachersaidthatdyingyoungisasufficientconditiontoshowthatsomeoneis good,notthatitisanecessaryconditionforsomeonetobeconsideredgood.Or,yourargumentisinvalid. Yourteachersaidthatbeinggoodisanecessaryconditionofdyingyoung,notthatitisasufficientcondition. 7. JM ~J __________ ~M Denyingthesufficient. Yourargumentisinvalid.Firstyousaythathavingthejewelswouldbeasufficientconditionofknowingthat someoneisthemurderer,andthenyouimplythatitisanecessaryconditionofsomeonebeingamurderer. 8.OS S __________ O Affirmingthenecessary. Babysittertoteenager:Yourargumentisinvalid.Yourparentssaidthatasafeenvironmentwasanecessaryconditionofyourgoingout,notthatitwasasufficientcondition.

228 AppEndix i
9.MB M __________ B Affirmingthesufficient. Yourargumentiscertainlyvalid.Ifitistruethatbeingamodelisasufficientconditionforknowingthatsomeone isbeautiful,andgiventhatmycousinisamodel,itcertainlyfollowsthatshemustbebeautiful.(But....) 10.CH ~C __________ ~H Denyingthesufficient. Yourargumentisinvalid.Isaidthatcyclingisasufficientconditionforsomeonebeingrequiredtowearahelmet, notthatitwasanecessarycondition.Infact,Iagreewithyou.Ithinkthatskiingisalsoaconditionthatshould requirehelmets.

Exercise 14B: Answers


1. LD DEM EMOT ~OT __________ ~L Denyingthenecessary. Yourargumentiscertainlyvalid.Ifitistruethatgettingthedrugsisanecessaryconditionforyoutolive,andifit istruethatmakingextramoneyisanecessaryconditionforyourgettingthedrugs,andifitistruethatworkingovertimeisanecessaryconditionforyourmakingextramoney,andifitistruethatyouwillbetoosickto workovertime,thenitfollowsthatyouwillnotlive.(But....) 2.GSM HM H __________ GS Affirmingthesufficient;affirmingthenecessary. Yourargumentisinvalid.Althoughitmayverywellbetruethatdoingallthathomeworkisasufficientcondition forgettingfirst-classgradesanditmaybetruethatifBenisdoingallthathomework,thatinturnisasufficientconditiontoensurethathewillgetfirst-classmarks,nonetheless,yourteachersaidthatgettingfirst-class gradesisanecessaryconditionforbeingadmittedtograduateschool,notasufficientcondition. 3.SDIM MnopartS Mnopart __________ DIM Affirmingthesufficient. Yourargumentiscertainlyvalid.Ifitistruethatstrikingisasufficientconditionforensuringthatprospectsfor recoveryaredim,andifthefailureofmanagementtodoitspartisasufficientconditionforensuringthatthere willbeastrike,andifitistruethatmanagementinfactwillnotdoitspart,thenitcertainlyfollowsthatprospectsforrecoveryaredim.(But....)

AnswErs to ExErcisEs

229

4.BL ~B __________ ~L Denyingthesufficient. Yourargumentisinvalid.Firstyouclaimthatbeingbeautifulisasufficientconditionforbeingloved,andthen youimplyitisanecessarycondition. 5.MA A __________ M Affirmingthenecessary. Yourargumentisinvalid.Firstyousaythatappreciationisanecessaryconditionforthepaintingtohavemerit, andthenyouimplyitisasufficientcondition. 6.LOH ~LO __________ ~H Denyingthesufficient. Yourargumentisinvalid.FirstyousaythatthelightsbeingonisasufficientconditionforknowingthatDeniseis home,andthenyouimplythatitisanecessarycondition. 7.FQ Q __________ F Affirmingthenecessary. Yourargumentisinvalid.FirstyousaythatCynthiasquittingwillnecessarilyfollowifshefails,andthenyou implythatifshehasquit,thatisasufficientconditionforknowingthatshefailed. 8.DJ ~D __________ ~J Denyingthesufficient. Yourargumentisinvalid.Firstyousaythatdeterrenceisasufficientconditionforcapitalpunishmenttobejustified,andthenyouimplythatitisanecessarycondition. 9.WUvAU ~TD DOI OIEW EW~WU ~T __________ AU Affirmingthesufficientconditionandvaliddisjunction. Yourargumentiscertainlyvalid.IfitistruethatnotsendingtroopstoIraqwilldestabilizethecountry,andifitistrue thatdestabilizationwillinevitablyleadtoaninterruptioninIraqsoilproduction,andifitistruethataninterruptioninIraqsoilproductionwillwreckoureconomy,andifitistruethatnotcaringthatoureconomymaybe wreckedshowsthatwearenotwithyou,andifitistruethatweareeitherwithyouoragainstyou,thenitcertainly followsfromthefactthatwearetryingtopreventtroopsfrombeingsenttoIraqthatweareagainstyou.(But....)

230 AppEndix i
10. PvW FPP WPer FP~R R ~Per _________ P Denyingthenecessaryconditionandavaliddisjunction. Youargumentiscertainlyvalid.ThereisnothingtoshowthatitwasnotMr.Plum.However,thefactthatthe smellofperfumewasanecessaryconditiontoshowthatMrs.Whitediditandthefactthattherewasnosmell ofperfume(whoseabsencewecouldhavediscountedhadtheairconditioningbeenon)showthatitwasnot Mrs.White.Thus,sinceitwaseitherMr.PlumorMrs.White,thefactthatitwasnotMrs.Whiteshowsthat itmusthavebeenMr.Plum.

Exercise 14C: Answers


1.FH HL LK K __________ F Affirmingthenecessarycondition. Yourargumentisinvalid.Firstyousaythatkindnessisanecessaryconditionforlove,whichinturnisanecessary conditionforhope,whichinturnisanecessaryconditionforfaith.However,thenyouassumethatkindnessis asufficientconditionforlove,hope,andfaith. 2.FL L __________ F Affirmingthenecessarycondition. Yourargumentisinvalid.Firstyousaythatloveisanecessaryconditionoffaith,andthenyouimplyitisasufficientcondition. 3.LK ~K __________ ~L Denyingthenecessarycondition. Yourargumentiscertainlyvalid.Ifitistruethatkindnessisanecessaryconditionofloving,andifitistruethat JoanneisinfactunkindtoEddie,itcertainlyfollowsthatshedoesnotlovehim.(But....) 4.FH HL ~F __________ ~L Denyingthesufficientcondition. Yourargumentisinvalid.Firstyousaythatfaithisasufficientconditionforhope,whichinturnisasufficient conditionforlove.Thenyouimplythatfaithisanecessaryconditionforhopeandthereforelove.

AnswErs to ExErcisEs

231

5.KL LF K __________ F Affirmingthesufficientcondition. Yourargumentiscertainlyvalid.Ifitistruethatallkindpeoplearelovingpeople,andifitistruethatbeinglovingisasufficientconditiontoshowthatonehasfaith,andifitistruethattheworldisfilledwithkindindividuals,thenitcertainlyfollowsthatfaithisanythingbutdead.(But....) 6.LF LC ~C __________ ~F Denyingthenecessaryconditionanddenyingthesufficientcondition. Yourargumentisinvalid.Althoughitmaybetruethatthereisverylittlecharityinthisworld,andalthoughit maybetruethatcharityisanecessaryconditionforlove,yousaidthatloveisasufficientconditionforfaith, notthatitisanecessarycondition. 7.AvB Aunder10 BHBP ~under10 ~HBP __________ B Twicedenyingthenecessarycondition,andaninvaliddisjunctivemove. Yourargumentisinvalid.Becausethepatientisnottenpoundsunderweight,youcannotgivehimdrugA,but becausehedoesnothavehighbloodpressure,youcannotgivehimdrugB.Soitisfalsethatyoushould givehimAorB. 8.KI IH K __________ H Affirmingthesufficientcondition. Yourargumentiscertainlyvalid.Ifitistruethatkindnessisasufficientconditionforintegrity,andifitistruethat integrityisasufficientconditionforvaluinghonor,andifitistruethattheworldisfilledwithpeoplewhoare kind,thenitcertainlyfollowsthattherearelotsofpeopleintheworldwhovaluehonor.(But....) 9.FH PF ~H __________ ~P Denyingthenecessarycondition. Yourargumentiscertainlyvalid.Ifitistruethathopeisanecessaryconditionoffaithandifitistruethatfaithis anecessaryconditionofpeace,thenitcertainlyfollowsthatifwelosehope,wewillneverachievepeace. (But....)

232 AppEndix i
10.LM LLis ~Lis __________ ~M Denyingthenecessaryconditionanddenyingthesufficientcondition. Yourargumentisinvalid.Althoughitmaybetruethatseriouslylisteningtoothersisanecessaryconditionof lovingthem,andsinceGrahamhasnotimetolistentoothers,itcertainlyfollowsthathehasnotimeforlove. However,thegoodbooksaysthatlovingothersisasufficientconditiontoensurethatyouaremoral,nota necessarycondition.SothefactthatGrahammaynotlovesaysnothingwhatsoeveraboutwhetherornothe ismoral.

Exercise 14E: Answers


1. e 2. d 3. e 4. c. 5. d 6. c 7. c 8. b 9. d 10.e

appendix II
Analyzing Arguments
The following are examples of good and not so good arguments that have been written according to the instructions of this textbook. The arguments are on the right-hand side. The comments are on the left-hand side. Cover the comments, and try to judge whether each separate component is adequate. Assume that each component can be given a full mark of 1, a partial mark of 0.5, or (if it is completely inadequate) a 0. NoTe: Students should note that the following arguments are in a 10.5-point font. Students who normally use a 12-point font will find that good arguments are rarely shorter than one full page.

233

234 Appendix ii
Argument 1: Evaluation
THESIS This is an excellent thesis statement. For one thing, it is the flip of the students original thesis statement, which indicates that this student was prepared to follow reasons wherever they might lead, even if into the arms of the opposition. For another, it is clearly relevant. Who has not had these feelings? It is evident, therefore, that working through this argument is going to have an impact on this students future attitude and perhaps behavior. Mark: 1 SUPPORT The argument here is as follows: All [feelings that prevent one from absorbing the bigger educational picture and from benefiting from the giveand-take of communicating with fellow learners] are feelings that are not justified. (Feelings of gladness about doing better than others) are [feelings that prevent one from absorbing the bigger educational picture and from benefiting from the give-and-take of communicating with fellow learners]. (Feelings of gladness about doing better than others) are feelings that are not justified. This is a fairly strong support, although one can imagine a counterexample right away, namely, competition for a scholarship. So we will look to see if that is tackled in the opposition. Mark: 1 OPPOSITION The argument here is as follows: All [feelings that motivate me to continue my hard work and that increase the chance of getting a scholarship] are feelings that are justified. (Feelings of gladness about doing better than others) are [feelings that motivate me to continue my hard work and increase the chances of getting a scholarship.] (Feelings of gladness about doing better than others) are feelings that are justified. This is a fairly strong opposition, particularly because it accurately paints what actually happens in reality. A lot of students, and indeed a lot of people in all kinds of competitions, care more about doing better than others than about how well they do relative to their own potential. (The weakness in this argument is pointed out in the response.) Mark: 1 RESPONSE The weakness in the oppositions position is that it fails to note that there is so much more to life than grades. If you become a mean, selfish, spiteful individual in the pursuit of bettering others, you may find that a scholarshipwhich may well be the result of a highly competitive attitudeis poor compensation. Mark: 1 CONCLUSION This lovely conclusion is filled with words of wisdome.g., It is important to learn to appreciate the benefit of being a member of a competent, prosperous team, group, or community, as well as to appreciate being a competent prosperous individual. Mark: 1 Final mark: 5/5

AnAlyzing Arguments

235

Argument 1: Feeling Good When You Beat Others


THESIS When I ask others how well they did on an exam, I feel glad when I did better, and really annoyed when they did better than I did. I used to believe that these feelings were justified. Now I do not. SUPPORT Feeling glad for doing better than another is viewing school as a competition. Competition can be highly stressful and can lead to an extremely self-centered attitude. By putting competition aside, students will become more relaxed and better able to communicate with their peers. As a result, they will be better able to absorb the bigger educational picture, and they will be in a much better position to benefit from the give-and-take of communicating with fellow learners. OPPOSITION When I work hard studying for a test and I do better than an equal peer, I feel glad because my hard work paid off. I use these feelings of gladness to further motivate me to keep working hard to achieve those same high grades. Conversely, when I get a worse grade than someone else, I use the feelings of frustration and annoyance to push myself toward working harder to get a higher grade next time. Also, with the limited amount of money available to students for scholarships, if I do better on a test than someone who is potential competition for this money, I feel glad that I did better because it may mean the difference between my receiving a scholarship or not. RESPONSE The grade you receive on a test should be reward for the hard work that is put into the test and should be the motivator for future work. Feeling glad that you beat another peer on a test may be satisfying, but you may not be achieving your full potential by pegging your grade to that of another person, who may have different goals than you. The school system does base many of its scholarships on grade point average alone. However, there are countless other achievements that focus instead on community participation and extracurricular activities. The lack of hard work preparing for a quiz as a result of time spent helping out at a homeless shelter may lose you the difference of a grade point for one scholarship, but it may put you in the running for countless other achievements that have different requirements. Besides, no amount of scholarship money can compensate for the personal price of becoming a mean, selfish, spiteful person. CONCLUSION Pursuing ones goals with passion is a good thing. However, these goals should not come at the expense of the growth of classmates, nor should they be perceived as more glorious because they are out of reach of others. Who you are as a person is more important than any scholastic recognition. Besides, if a class of students prospers and grows as a community, everyone in that community will benefit from that generalized growth. It is important to learn to appreciate the benefit of being a member of a competent, prosperous team, group, or community, as well as to appreciate being a competent prosperous individual. Being happy about ones own success but sad about the failure of others shows that one has learned that important lesson.

236 Appendix ii
Argument 2: Evaluation
THESIS This is the same thesis statement as Argument 1, and therefore also receives a mark of 1 for the same reasons. Mark: 1 SUPPORT The argument here is as follows: All [feelings that tend to prevent one from contributing to the growth of ones classmates] are feelings that are not justified. (Feelings of gladness when classmates do less well than you do) are [feelings that prevent you from contributing to the growth of those classmates.] (Feelings of gladness when classmates do less well than you do) are feelings that are not justified. This argument tends to beg the question. The thesis is investigating the issue of whether one is justified in being concerned only with oneself. This support simply states that one should not be solely concerned with oneself. However, it does not say why. Still it hints that you will pay a personal price, so we will give it a 0.5 mark. OPPOSITION The argument here is as follows: All [feelings that correctly mirror the situation, namely, that only a very limited number can win, for example, the olympics] are feelings that are justified. (Feelings of gladness when others do less well than you do) are [feelings that correctly mirror the situation, namely, that only a very limited number can win, for example, the olympics]. (Feelings of gladness when others do less well than you do) are feelings that are justified. Although this opposition makes an interesting point about the olympics, it weakens its own position by making the claim that this competition (i.e., school) is like any other, which clearly it is not. So it is leaning toward being a strawperson. Mark: 0.5 RESPONSE This response correctly points out the obvious weakness in the opposition, namely, that school is not like every other competition. However, it weakens its position by begging the question once more by stating that it is your responsibility to contribute to the learning experience of your classmates without stating why. It also contains a faulty analogy. Refusing to help an elderly person who has fallen on the street is hardly of the same magnitude of harm as private feelings of joy because one has done better than others, to say nothing of the fact that whether or not one helps others is a different topic than whether or not one should be pleased when one does better than others. Mark: 0.5 CONCLUSION The conclusion has gone completely off topic. The topic is about justifying feelings. The conclusion focuses almost entirely on the degree to which one should help others. Mark: 0 Final mark: 2.5/5 NoTe: This is the same topic as the previous one, but the argument here is substantially weaker. This is yet more evidence that shows that not everyones opinion is as good as everyone elses. An opinion is only as good as the argument that backs it.

AnAlyzing Arguments

237

Argument 2: Feeling Good When You Beat Others


THESIS When I ask others how well they did on an exam, I feel glad when I did better than they did and really annoyed when they did better than I did. I used to believe that these feelings were justified. Now I do not. SUPPORT It is wrong to feel glad when someone else does poorly. obviously, one should be proud of doing well oneself, but that should have nothing to do with how anyone else performs. If someone feels happy because of anothers misfortune, then that person will be the sort who does not cooperate in a class, who does not help others if they need it, and who is less likely to share his or her knowledge with classmates to give them a better understanding of how to answer a question or do an assignment. This one individual is keeping information to him- or herself, which may lead to the class missing out on a peers much needed perspective, such as is provided in the community of inquiry. OPPOSITION of course you have the right to feel happy about doing better than others. Being in school is just like taking part in any other competition. If you are in the olympics, are you supposed to be glad when someone else does better than you? There are only three people who are going to stand on that podium. The only way you are going to get on that podium is by ensuring that you are better than everyone else in the race. It is not good enough to just do well yourself. You must do better than others. So feeling glad that others do less well than you do is justified. RESPONSE Being in school is not like being in the olympics. Being in school is a preparation for life. You have to learn to care for others. What kind of person would you be if you did not stop to help an elderly person who had fallen on the street just because you were in a race to get to school on time? Winning the race is not everything. It is your responsibility to contribute to the learning experience of your classmates, just as it is their responsibility to contribute to yours. CONCLUSION It is unethical to refuse to come to the assistance of others. You never know when one day you might be in need. Should that day come, you would want others to help you. You should always do unto others as you would have them do unto you. If you are lucky enough to be able to pick up concepts quicker than others, if you are able to master more material more quickly, this is a gift that should be shared. Your concern for others will not go unnoticed.

238 Appendix ii
Argument 3: Evaluation
THESIS This is an excellent thesis statement. It is clearly relevant, as the answer to the question embedded in this thesis statement will have an impact on this individuals behavior. Mark: 1 SUPPORT The argument here is as follows: All [acts that show respect for a particular woman and for women in general] are acts that should not elicit anger. (opening the door for women) is [an act that shows respect for the individual woman and for women in general]. (opening the door for women) is an act that should not elicit anger. This is a strong support, particularly as it is hard to think of a counterexamplei.e., something that shows respect but that nonetheless should elicit anger. Mark: 1 OPPOSITION The argument here is as follows: All [acts that retain the negative connotations that they had in the past (i.e., that women were the weaker sex)] are acts that should elicit anger. (opening the door for women) is [an act that retains the negative connotations that it had in the past (i.e., that women were the weaker sex)]. (opening the door for women) is an act that should elicit anger. This is an excellent opposition. At this juncture, you should feel the tension between the support and the opposition and be genuinely unsure which is stronger. However, if you pause for a moment and think of a possible counterexample to the opposition, you may be able to think of some gesture that has been held over from the past but that now has a completely different meaning. This would be the counterexample to the aforementioned (see response). Mark: 1 RESPONSE This is a fairly interesting counterexample to the oppositions claim that traditions always maintain the meaning of the past. It is particularly elegant in the mirroring of the three-part advantage of mounting ones horse on the left and men opening the door for women. Mark: 1 CONCLUSION This is a first-class conclusion. By citing a tradition that continues to promote discrimination (e.g., the use of non-genderneutral terms) that should be eliminated, the arguer shows that he empathizes with the concerns of his opposition. The last point reinforces the argument made in the support, that rules of etiquette are not valueless, meaningless rules, but ways of demonstrating an attitude that otherwise may be difficult to get across. This is an excellent argument. Mark: 1 Final mark: 5/5

AnAlyzing Arguments

239

Argument 3: Opening Doors for Women


THESIS My girlfriend feels offended (angry) when a guy walks faster ahead of her to open the door for her. Her anger is not justified. SUPPORT In todays world, there are only so many ways left that one can be polite and hence show respect for other individuals. opening the door for my girlfriend is simply a gesture of politeness and respect. It also shows respect for our elders and the traditions in which we were brought up. When I was growing up, my mother insisted that I open the door for women, pull out their chairs, and so on. In continuing those gestures, I am showing respect for my mother and her beliefs. Such gestures thus show respect not only for a particular woman but also for women in general. OPPOSITION As a feminist, my girlfriend believes that all gestures that make women look weaker than men are offensive. When a guy walks faster than a girl to get the door, although he might not realize it, he is portraying her as somebody who cannot easily open the door herself. The reason that opening the door became a tradition in the first place was because women could not easily reach the door handle because of their wide skirts. Today, women wear more comfortable clothing, and hence there is no reason for men to continue the habit of opening the door. It simply serves as a symbol for all the discrimination that women have endured for centuries. Such symbolic gestures are understandably offensive. RESPONSE In horseback riding, it became a tradition to mount ones horse on the left because one wore ones sword on the right. Today, good riders maintain that tradition largely because it shows respect for the sport of horseback riding in general, it promotes uniformity of behavior and expectations, and it infuses style into the whole action pattern, without in the least maintaining the original meaning. It seems to me that the meaning of the tradition of men opening the door for women has also evolved. Although the tradition emerged as a result of the assistance women needed because of the wide skirts they wore, maintaining the tradition now only shows respect for intergender interaction in general, while promoting uniformity of expectations and infusing style into the whole action pattern. CONCLUSION There are all sorts of apparently innocent acts that not only symbolize past discrimination against women but also perpetuate that discrimination. These rightly elicit anger. The use of non-gender-neutral terms, for example, sends the message that women are somehow invisible, or at least second-class, and we will be none the poorer for their elimination. opening doors for women is a different matter. In todays rough-and-tumble world, there seem to be very few acts that we know send a message of respect. Men opening doors for women is one of them. We will all be much the poorer with the elimination of this tradition.

240 Appendix ii
Argument 4: Evaluation
THESIS An Internet topic! Boring! This topic has been done to death. No one is interested in reading about this topic unless the writer has something new to add. Nonetheless, since the thesis is crystal clear, it gets a 1. However, this writer is treading on dangerous ground. If you are going to tackle a common topic, you should be familiar with the common pros and cons. Mark: 1 SUPPORT The argument here is as follows: All [acts that enhance the lives of prostitutes, help stem widespread health problems, and increase revenue for the government] are acts that should be done. (Legalizing prostitution) is [an act that will enhance the lives of prostitutes, help stem widespread health problems, and increase revenue for the government]. (Legalizing prostitution) is an act that should be done. This is very comprehensive support. Mark: 1 OPPOSITION The argument here is as follows: All [acts that encourage more young people to become prostitutes and that encourage more people to engage in loveless sex] are acts that should not be done. (Legalizing prostitution) is [an act that will encourage more young people to become prostitutes and will encourage more people to engage in loveless sex]. (Legalizing prostitution) is an act that should not be done. This student has just fallen into the trap s/he set for her- or himself. The well-known and strongest opposition, one the arguer could have read in any standard text or on the Internet, is that legalizing prostitution would be tantamount to legalizing the exploitation of a group of women who are already unfairly disadvantaged. To suggest that legalizing prostitution will encourage a lot of young people to become prostitutes is just silly, and bringing up the problem of loveless sex is a reference to a Victorian value that has little relevance in modern liberal society, which makes this a strawperson argument. Mark: 0 RESPONSE A response to nothing is nothing. Since the opposition is 0, the response must be 0. Showing the weakness of an obviously weak argument is of no merit. Besides, this response has its own faults by using the word right without explaining why the purported right exists. So it begs the question. Mark: 0 CONCLUSION This conclusion has some merit by bringing up the analogy of prohibition. However, the failure to understand that the main opposition to the thesis is that this may just institutionalize, and hence perpetrate, the exploitation of women also renders this student unable to evaluate whether this analogy is insightful or faulty. (Note: Try to avoid questions when you are making an argument.) Mark: 0.5 Final mark: 2.5/5

AnAlyzing Arguments

241

Argument 4: Legalizing Prostitution


THESIS Prostitution should not be illegal. SUPPORT Making prostitution legal will allow the business to be handled more maturely. Instead of prostitutes hanging out on corners, discrete parlors in a red-light district could be set up. Instead of pimps controlling the lives of their prostitutes with violence and drugs, a madam or caretaker could provide a safe, even friendly place to provide the service. Another issue pertains to health. With the sexual diseases of old and new being transmitted, it makes practical sense to monitor this behavior. Legalization could require prostitutes to be medically examined on a regular basis by the managers of the house where they worked. Right now prostitutes can easily obtain and spread diseases, causing serious, widespread health problems. Also, legalizing it may even benefit the government through taxation of the profession like any other and also by the amount of money it would save, as cash would not be wasted trying to prevent it. OPPOSITION If we legalize prostitution, we are sending the message that prostitution is oK. If it is oK, then many more young people will engage in the profession than do now. This will cause a lot of disruption in those families if other members of the family do not think that prostitution is oK. Also, making prostitution legal and providing a hotel of sorts for those who are willing to pay makes prostitutes more accessible. This in turn will encourage more people to partake in these services. Consider how such legalization would upset family values. Such legalization sends the message that sexual activities without the intimacy of a relationship are oK. Is this what we want to teach our children? RESPONSE every individual has the right to decide his or her line of work. Perhaps family members would be upset if you decided to become a garbage collector. However, if it was good money, why not? And any business or service has the right to promote its product, be it a massage parlor or a sex parlor. In the end, it is up to the consumer whether or not s/he wants to buy. As for family values, it depends entirely on ones point of view. Sexual intercourse alone is not immoral. Putting a price on it should not be any different. CONCLUSION The legalization of prostitution would bring benefits to any city that would greatly outweigh the problems, if any. A waste of effort, time, and money could be avoided if we allowed this practice to continue lawfully. Should we continue to ban prostitution just as the American government banned alcohol during the 1920s? Is this logical?

242 Appendix ii
Argument 5: Evaluation
THESIS Clearly this thesis is not sufficiently precise. What does properly mean? This could refer to someone who walks his or her dog infrequently or to someone who beats his or her dog. However, even if it were precise, it borders on the obviousi.e., it is not very contentious. Mark: 0.5 SUPPORT The argument here is as follows: All [behavior that causes dogs to suffer] is behavior that is unethical. (Not looking after dogs properly) is [behavior that causes dogs to suffer]. (Not looking after dogs properly) is behavior that is unethical. This is a problem support. For one thing, it continues to suffer from the imprecise thesis statement. We still do not know precisely what improper care refers to. In addition, the support uses two faulty analogies. Clearly, leaving a dog alone all day does not result in the same harm as leaving a child alone all day. Similarly, the claim that dogs feel the same emotions as humans is questionable. The last sentence is just question-begging fluff. However, the support does make a valid point in saying that suffering is bad regardless of what animal is experiencing it. Mark: 0.5 OPPOSITION The argument here is as follows: All [behavior that is not illegal] is behavior that is not unethical. (Not looking after dogs properly) is [behavior that is not illegal]. (Not looking after dogs properly) is behavior that is not unethical. This opposition is totally vacuous. It is simply not true that all things that are not illegal are not unethical. Breaking a promise is unethical, not illegal, as is not showing up for a date or cheating on an exam. The point about not being able to create a police force to prevent dogs from being harmed is irrelevant. Ultimately, the problem with this opposition is embedded in the thesis statement. It is hard to imagine an opposition to such an imprecise althoughmore importantnoncontentious thesis statement. Mark: 0 RESPONSE A response to nothing is nothing. Since the opposition is 0, the response must be 0. Showing the weakness of an obviously weak argument is of no merit. Besides, most of this response is just a repeat of the thesis statementi.e., begging the question. Mark: 0 CONCLUSION It is evident from the conclusion that this writer loves dogs, so the topic is certainly relevant. However, the entire enterprise is faulty because the thesis statement is not sufficiently precise, but more important it is not sufficiently contentious. This was more a voyage into emotion than into logic. Ultimately, a conclusion can be only as good as its argument. So although this makes some pretty points, it still gets only 0.5. Mark: 0.5 Final mark: 1.5

AnAlyzing Arguments

243

Argument 5: Treating Dogs Poorly


THESIS People who do not look after their dogs properly are unethical. SUPPORT People who do not look after their dogs properly are no different from people who abuse their children. What would you think of someone who left his or her kid alone all day or drove the kid around in the back of a truck? There is no difference. Animals can feel pain and emotions just as humans do. Suffering, whether it exists in an animal or a human is something that we should prevent if at all possible. Anyone who is not prepared to put in the time and effort to care for an animal properly is acting in a highly immoral manner. OPPOSITION We have laws against both child abuse and animal abuse. As long as a person is not breaking the law, that person cannot be seen as acting unethically. Besides, we have a hard enough time protecting people. We certainly do not have the money to create a police force whose job it would be to protect animals. RESPONSE Whether or not we are able to stop animal abuse is beside the point. If you do not treat your animal with the love and respect that it deserves, you are a very unethical person. Animals have feelings just as humans do. You should not have an animal unless you are prepared to treat it properly. CONCLUSION Dogs have been known to rescue their owners from burning buildings. Dogs provide companionship for those who are lonely. Dogs are used to guide the blind. Dogs have even been trained to sense the beginning of a seizure in epileptics so that these people can get to a safe place. Dogs have been trained to sense the onset of diabetic comas and to warn their owners. Dogs have been used to find illegal drugs. Dogs help rescue avalanche victims. Dogs really are mans best friends. They are loyal, playful, and unconditionally loving. They deserve to be treated with love and respect. Anyone who does not treat a dog with the love and respect that it deserves is acting unethically.

244 Appendix ii
Argument 6: Evaluation
THESIS This is a precise clear thesis statement, which is all the more powerful because it uses an example. Although it tends toward the obvious, the fact that so many still resist using gender-neutral terms keeps this topic relevant. Mark: 1 SUPPORT The argument here is as follows: All [practices that show that one is refusing to put thoughts of equality into action and that make women feel excluded] are practices that are unethical. (Refusing to use gender-neutral terms) is [a practice that shows that one is refusing to put thoughts of equality into action and that makes women feel excluded]. (Refusing to use gender-neutral terms) is a practice that is unethical. This is a very strong support. It is difficult to imagine a counterexample of either of the premises. Mark: 1 OPPOSITION The argument here is as follows: All [practices that help ensure the flow of language and keep communication simple by avoiding the silliness of being politically correct] are practices that are not unethical. (Refusing to use gender-neutral terms) is [a practice that helps ensure the flow of language and keeps communication simple by avoiding the silliness of being politically correct]. (Refusing to use gender-neutral terms) is a practice that is not unethical. This is a surprisingly strong response to a thesis statement that leans toward the obvious. Many resist using gender-neutral terms precisely because it results in awkward wording or because it seems like senseless pressure to be politically correct. Mark: 1 RESPONSE This is a strong response to both the points made by the opposition. A few bumps in the flow of communication do seem like a small price to pay for showing respect. And the suggestion that it is more important to be empathetically respectful than politically correct is persuasive. Mark: 1 CONCLUSION What a lovely conclusion. There is much wisdom in the claim that language creates an atmosphere and that, for too long, women have lived in a linguistically foreign world. There is also wisdom in the claim that both genders should feel equal in both word and deed. Mark: 1 Final mark: 5/5

AnAlyzing Arguments

245

Argument 6: Gender-Neutral Terms


THESIS People who refuse to try to consistently use gender-neutral terms, such as humankind for mankind, are unethical. SUPPORT Changing language by using gender-neutral terms is about putting new philosophies into action. As is demonstrated in legislation prohibiting discrimination based on gender, society has undergone a change in how it views women. By using gender-neutral terms, we are putting thought into action. It is easy to say that you believe in gender equality, but making it a reality is what is really important. Changing language is a way of demonstrating to yourself and others your belief in gender equality, and it helps to make it a reality. Failing to use gender-neutral terms leaves half the population feeling excluded and, as a result, unequal. If we are going to say that we believe in the equality of women, ethics demands that our actions and behaviors demonstrate that. OPPOSITION Demanding the constant use of gender-neutral terms is just an example of feminism gone too far. The constant use of gender-neutral terms is awkward and interrupts the flow of conversatione.g. There was a student who went to college. S/he paid his or her tuition on the first day. Getting upset because someone uses the term mankind instead of humankind is just a result of hypersensitivity. The term mankind has been used for centuries to refer to both men and women. The person who uses the term is in fact referring to both men and women, not just to men. Life is complex enough without changing ones language in order to be in sync with what is politically correct. Are we now going to insist on using horizontally challenged instead of fat? It all seems a bit silly. RESPONSE Thoughts and intentions alone cannot define people as ethical. What people do is the ultimate test. Thinking that equality is important is one thing; doing your little bit to contribute to ensuring that that equality comes about is another. A few bumps in the flow of communication seem like a small price to pay to demonstrate ones respect for women. This is not merely a matter of political correctness. Many women are insulted by the use of non-genderneutral terms. Many women believe that it defines them as invisible or insignificant. By definition, to treat someone with respect is to take account of his or her feelings. Thus, by definition, failing to use gender-neutral terms fails to give women respect. And if fat people are insulted by that term and prefer to be referred to as horizontally challenged, so be it. This is not about being correct; it is about being empathetically respectful. CONCLUSION Language creates an atmosphere that can be inviting or dismissive, comforting or insulting. For too long, women have lived in a linguistic world that is virtually foreign to them. The use of gender-neutral terms is the only way in which both genders will feel equal both in word and in deed. If one says that one believes in gender equality, then one should say it by using humankind, not mankind. one word stratifies; the other promotes equality.

246 Appendix ii
Argument 7: Evaluation
THESIS This is an excellent thesis statement. For one, it is crystal clear as a result of using an example. For another, it shows that this student changed his or her mind as a result of going through this processi.e., this student was prepared to follow reasons wherever they lead. Mark: 1 SUPPORT The argument here is as follows: All [acts that allow women to simply see what is in fashion and that do not pressure women into trying to have model bodies any more than watching violent TV induces them to be violent] are acts that are not unethical. (Buying fashion magazines) is [an act that allows women to simply see what is in fashion and does not pressure women into trying to have model bodies any more than watching violent TV induces them to be violent]. (Buying fashion magazines) is an act that is not unethical. This is a good support that is made particularly strong by the analogy. Mark: 1 OPPOSITION The argument here is as follows: All [acts that support an industry that contributes to womens dissatisfaction with their bodies and sidetracks them from so much else that is important in life] are acts that are unethical. (Buying fashion magazines) is [an act that supports an industry that contributes to womens dissatisfaction with their bodies and sidetracks them from so much else that is important in life]. (Buying fashion magazines) is an act that is unethical. This is a very strong opposition. Too often we forget the power and the responsibility of the consumer in a market economy. At this juncture, you may be unsure which is stronger, the support or the opposition. This is a sign that this is a good, contentious thesis statement. Mark: 1 RESPONSE This is a superb response in that it recognizes a distinction between industries that do direct harm and industries that do harm only indirectly. The analogy with Bugs Bunny is extremely persuasive. Mark: 1 CONCLUSION It is hard to imagine a more powerful conclusion. This whole issue has been taken to a much deeper level. The sarcasm in We havent come a long way, baby! really hits it home. Mark: 1 Final mark: 5/5

AnAlyzing Arguments

247

Argument 7: Buying Fashion Magazines


THESIS I used to believe that women who bought magazines that portrayed women as waifs (i.e., virtually every fashion magazine) were acting unethically. I no longer believe this. SUPPORT Women who buy fashion magazines do not buy them for the purpose of comparing themselves to the models but rather to see what is in fashion and to read the various articles. Women are not stupid; most are not influenced by the ads they see, as they realize that the majority of women cannot have a model body. Buying fashion magazines is no more unethical than watching violence on TV. Just as one can watch violence on TV and know it is not real, one can look at a fashion magazine without thinking one must have the same body. OPPOSITION When women buy fashion magazines, they are supporting the fashion magazine industry. As long as women buy these magazines, the industry will continue to use thin and unrealistic models. Many women who look at these models begin to feel stress and anxiety about their own bodies, because it appears as though they do not measure up. Supporting an industry that contributes to womens dissatisfaction with their own bodies and sidetracks them from so much else that is important in life is unethical. RESPONSE Some industries destroy our forests and pollute our rivers. These industries should be boycotted by the consumer. However, the fashion industry is not directly harmful in this way. Rather, it is the response to the product of this industry that is harmful. We should work on the inappropriate response, not the stimulus. We cannot wrap our world in cotton batting. We all need to learn to create our own priorities regardless of outside pressures. We must teach our children, for example, that enjoying Bugs Bunny does not mean that it is oK to bop a friend on the head. And we must teach our young women to enjoy fashion without becoming anorexic in the name of fashion. CONCLUSION Unfortunately, in todays society there are pressures all around us to be unrealistically thin. However, no real changes will be realized by boycotting the fashion magazine industry. The root of the problem lies with the fact that both women and men still judge women in terms of their potential to be beautiful objects. We havent come a long way, baby! Real change will be realized only when women strive to have the perfect mind or personality rather than the perfect body and men reinforce that goal by seeking out smart and funny women rather than dumb blondes with big boobs.

248 Appendix ii
Argument 8: Evaluation
THESIS This student is to be commended for picking a topic that speaks to his or her own life. This topic clearly is relevant. It has a fatal flaw, however: It is imprecise. What does getting on my case mean? This could be anything from mild complaining because s/he does not help around the house to autocratic intrusion, including insisting on knowing everything that this student does. Mark: 0.5 SUPPORT The argument here is as follows: All [things that my parents have no right to do at my age] are things that I am justified in being furious about. (My parents being constantly on my case) is [something they have no right to do at my age]. (My parents being constantly on my case) is something I am justified in being furious about. This support is riddled with bias. This person clearly sees the situation only from his or her own point of view. S/he wants to live at home in order to maintain his or her standard of living but is annoyed with the frustration and responsibilities that come with living with others. It also clearly begs the question by stating that parents have no right to comment. Why dont they have a right? This claim needs a reason to back it. Mark: 0 OPPOSITION This argument sort of looks like this: All [things that are usually just a matter of a difference of opinion] are things that I am not justified in being furious about. (My parents being constantly on my case) is [something that is usually just a matter of a difference of opinion]. (My parents being constantly on my case) is something I am not justified in being furious about. There is really no opposition at all. one gets the feeling that this student doth protest too much, that in fact the troubles that arise at home are because his or her parents do not believe that s/he is carrying his or her weight. If the problem is about opinions, then specifics are needed. It is impossible to know what this issue is all about at this juncture. There also seems to be a change of subject here, namely, whether or not s/he should move out. Mark: 0 RESPONSE even if there had been anything in the opposition, it is evident that this student sees the response as simply an opportunity to restate his or her own position. This student has not understood that the response is supposed to be an answer to the concerns of the opposition. Mark: 0 CONCLUSION If all parents want what is best for their children, then why is this student complaining about his or her parents being on his or her case? And it is not true that all children want to make their parents proudthat is why this issue has come up in the first place. Nor is it true that young people turn out well just by giving them space. This is obviously an utterly biased exercise that is impossibly vague and fueled by emotion rather than logic. Mark: 0 Final mark: 0.5/5

AnAlyzing Arguments

249

Argument 8: Getting Annoyed with Parents


THESIS I am a twenty-year-old student living at home. My parents are constantly on my case about not spending time at home and helping out. I am justified in being furious with them. SUPPORT This issue has been going on since I got out of high school. No matter what mood or state I am in, they do not seem to care, and they constantly give me a hard time. I think that my age allows me to make decisions for myself. I am no longer a kid, so they have no right to tell me that I do not understand what I am doing or whether what I am doing is right or wrong. It is true that I am living under their roof, but the reality is that I could easily be living on my own without worrying about what someone else might think or say. However, living on my own would not allow me to live life the way I should, because I would be working to support myself, and that would make me lose out. OPPOSITION I come from a very traditional family in which it is not usually the case that children leave home at a young age, so I cannot just move out. My parents are also older now and need companionship. They also feel that since I am living under their roof, I have to take responsibilities, which I try to do. Usually our arguments are about differences of opinions. RESPONSE As elders, my parents should understand that I am going through a phase of life in which everything is meant to be experienced. This requires time outside of the house. I cannot always be there when they want me. I try to carry out the stuff they ask me to do. Adding more only makes me want to do less. They are being unreasonable. CONCLUSION I think if my parents gave me some space without being on my case every second, I would turn out to be what they want. After all, children want to make their parents proud, and all parents want what is best for their children.

250 Appendix ii
Argument 9: Evaluation
THESIS This is a highly relevant thesis statement. Who has not been annoyed by someone talking on a cell phone in a supermarket or at a park? It is also to be admired for avoiding trying to make a legal case of the matter, instead going the very powerful route of tagging a label to the behavior. Mark: 1 SUPPORT The argument here is as follows: All [people who use a status symbol in order to make themselves look important in the eyes of others] are people who are not autonomous. (People who are at the constant beck and call of their cell phone) are [people who are using a status symbol in order to make themselves look important in the eyes of others]. (People who are at the constant beck and call of their cell phone) are people who are not autonomous. Again, it is evident that this topic is relevant to this writer. S/he is trying to figure out whether or not his or her annoyance with his or her brothers use of the cell phone is justified. It is difficult to think of a counterexample to this argument. Mark: 1 OPPOSITION The argument here is as follows: All [people who use a piece of technology to ensure that they have maximum access to multiple points of view, which increases their capacity to make wise decisions] are people who should not automatically be perceived as not autonomous. (People who are at the constant beck and call of their cell phones) are [people who are using a piece of technology that ensures that they have maximum access to multiple points of view, which increases their capacity to make wise decisions]. (People who are at the constant beck and call of their cell phones) are people who should not automatically be perceived as not autonomous. This is a surprisingly strong and innovative response in that it uses one of the necessary conditions of being able to achieve autonomynamely, the capacity to access multiple points of viewas a counter to the claim that people who use cell phones are not autonomous. It is also a wise opposition because it is not making the claim that those who use cell phones are autonomous, but rather that those who use cell phones are not necessarily not autonomous. That is an important difference. Mark: 1 RESPONSE This is a continuation of the support. It makes an interesting comparison between an old and new-age symbol of adulthood. However, it is not a response to the oppositioni.e., it does not show how the opposition is faulty. That is the job of the response, because only in so doing does it show why the oppositions case is weaker than the supports case. Unhappily, therefore, this response gets 0. Mark: 0 CONCLUSION This conclusion makes some interesting points (e.g., that insecure people are owned by what they own), and it hints at what might have been the response to the oppositionnamely, that it is worthwhile communication, rather than communication per se, that feeds autonomy. However, it continues to just ignore the point made by the opposition, so it gets 0.5. Mark: 0.5 Final mark: 3.5/5

AnAlyzing Arguments

251

Argument 9: The Overuse of Cell Phones


THESIS People who are constantly at the beck and call of their cell phonesexcluding business people, professionals, or those who otherwise require a cell phone for career or safety purposescould be accurately described as people who are not autonomous. SUPPORT My brother is a good example of someone at the beck and call of his cell phone. He carries his cell phone almost everywhere and answers it almost everywhere. Calls are answered in the shower, on the toilet, at breakfast, or in the car. Many high school teenagers act similarly, talking to friends on their cells on the way to school, in the hallways, and in the school cafeterias. Why cant they wait until they get home? Probably because they think that using a cell phone will make them appear important or popular. To many, the cell phone has become a status symbol. When people have to use an external object as a means to define themselves in the eyes of others, that is a sign of personal insecurity. Dependency and personal insecurity are incompatible with autonomy. OPPOSITION People who use cell phones want and indeed have the power to constantly be available and be able to communicate with others. Having a cell phone and being at its constant beck and call is a decision to take advantage of an option that will increase communicative freedom, which in turn will allow them to access multiple points of view about various issues. This will increase their capacity to make wise decisions. Having maximum communicative freedom and accessing as many points of view as possible are necessary conditions of autonomy. Using a piece of technology that allows a person access to multiple points of view, therefore, should not be perceived as an act that precludes autonomy. RESPONSE Being constantly at the beck and call of your cell phone is not a way to obtain freedom and control over your life. It is, rather, giving other people freedom and control to instantly and constantly interrupt your life, at every instant of your life. It is a false status symbol. In the past, kids used to smoke in order to feel grown-up. Now they use cell phones. However, a grown-up use of a cell phone would be one that is important, for example, a doctor on call. Kids who use their cell phones just to gossip are just trying to make themselves feel important. CONCLUSION People who are at the constant beck and call of their cell phones are overly concerned about being connected with their friends. These are insecure individuals. They are deceived by a false sense of freedom and control. They are very much owned by the things they own. If autonomy is what these people seek, then they should learn to reserve the use of their cell phones for cases of emergency.

252 Appendix ii
Argument 10: Evaluation
THESIS This is obviously a highly relevant thesis. This student is trying to decide what to do in the situation in which she finds herself. This thesis tackles a topic that is much more profound than the usual Internet issue of whether or not animals should be used in testing cosmetic products. Mark: 1 SUPPORT The argument here is as follows: All [acts that support an industry that takes advantage of subjecting animals to cruel and unnecessary testing] are acts that should not be done. (Wearing cologne that has been tested on animals, even if given as a gift) is [an act that supports an industry that takes advantage of subjecting animals to cruel and unnecessary testing]. (Wearing cologne that has been tested on animals, even if given as a gift) is an act that should not be done. This is a strong argument. It is hard to think of a good counterexample. Mark: 1 OPPOSITION The argument here is as follows: All [acts that will do no more harm than has already been done through the buying of the cologne and will prevent harm from coming to both my friend and me] are acts that are permissible. (Wearing cologne that has been tested on animals, particularly if it is given as a gift,) is [an act that will do no more harm than has already been done through the buying of the cologne and will prevent harm from coming to both my friend and me]. (Wearing cologne that has been tested on animals, particularly if it is given as a gift,) is an act that is permissible. This is a surprisingly strong opposition. It suggests that in reality, in this situation, this abstract principle will do more harm than good. It is hard to think of a counterexample to any of the preceding premises except that this persons principles will be violated. Mark: 1 RESPONSE What a lovely response. It picks up a counterexample to the oppositions claim that violating ones principles will result in no harm by claiming that harm is in fact done not just because a principle has been violated but because of the message that it sends. Mark: 1 CONCLUSION This is a well-balanced conclusion. It recognizes that in reality we are all forced to break some principles but that, nonetheless, the reality of this situation does not justify breaking a principle in this instance. The last sentence captures the lesson learned through thinking through this process in a particularly poignant way. Mark: 1 Final mark: 5/5

AnAlyzing Arguments

253

Argument 10: Using Gifts That Have Been Tested on Animals


THESIS For my birthday last week, I received from a good friend a bottle of cologne that had been tested on animals. I am not justified in using it even though I got it as a gift. SUPPORT I oppose animal testing, particularly for cosmetics, because I oppose unnecessary cruelty to animals. Manufacturers who take advantage of animal testing have survived in the market for such long periods of time because of the consumer tendency to buy their products. If I wore their products, I would be supporting their industry as much as every other consumer. I would be acting as a free endorsement agent by advertising their product when I used it. As a vegetarian, I would never eat a burger just because someone bought it for me. This situation is no different. Whether or not it is a gift, this product is an insult to my principles. As someone who values her principles, it would be hypocritical to use such a product and claim to hold such principles. OPPOSITION My friend bought me this cologne while traveling in europe. He has obviously gone to a great deal of trouble and paid a substantial amount of money to buy me this gift. I should be prepared to use it as a symbol of our friendship. What good am I doing if I throw it down the drain? The product has already been bought. The industry has already been supported. Throwing it away will not do anyone any good and will hurt my friends feelings. Besides, I will rarely have a chance to use such expensive cologne, so I might as well enjoy it. RESPONSE Wearing cologne is not a necessity of life. I can live without it, if it is against my beliefs. everybody has to pay a price for being true to worthy principles. This just happens to be the price that I have to pay. even if the damage has already been done, profiting from the damage makes one part of the perpetration. It would be hypocritical and inexcusable for a person to claim to be concerned about the welfare of elephants but have her house decked out in ivory that had been given to her by her grandmother. What is important here is the message that one sends by the actions that one undertakes. I am sorry that my friend did not know me better. However, I can use this as an opportunity to explain my position. He may even come to hold similar beliefs. CONCLUSION People have to make choices in different conditions that they are not always happy about. Sometimes I have to make choices that do not sit well with my principles. I drive to school because of serious time constraints, even though I hate to contribute to environmental pollution. This choice, however, is different. Wearing cologne is not a necessity; it is not even important. Refusing to wear this cologne, even if it is wonderful cologne, seems like a small price to pay in exchange for breathing real life into what I believe. By making the public decision of refusing to wear this cologne, I can be an advertisement for the principle of respect of animals, instead of being an advertisement for an industry that shames humans.

appendix III
Examples of Good Arguments
The following are arguments constructed by students in either their first or their second year of university. Some are from critical thinking courses and some are from other knowledge-based courses that were founded on critical thinking principles. Use the following to help you evaluate the quality of the arguments you construct.

Critical Thinking Course


THESIS The proposed AIDS memorial monument should not be built in Stanley Park. SUPPORT To begin with, there is no reason to assume that such a monument would do anything to help prevent the spread of the disease. Certainly, war memorials have done little in that regard. Second, putting up this wall would ruin the purpose of the park, which is to provide people with an escape from the realities of the world. Finally, agreeing to put up an AIDS wall would open the opportunity for other medical casualties to have monuments put up as well. Eventually the park could resemble a cemetery. OPPOSITION AIDS cases continue to increase, and raising awareness is an important step in eliminating ignorance on this subject. Putting up this wall would remind the public of the realities of this disease and the outcome. It would also demonstrate that we are sympathetic and we care about those who have passed away. In addition, the artist in charge stated that the area chosen is not used for anything important anyway. RESPONSE Certainly awareness is important, but the cost of this monument is estimated at $150,000, and it would take approximately eighteen months to raise this money. Instead of investing this money and time in a wall that is placed in an unimportant area of the park, we could use it to fund educational programs and research in preventing and curing this disease. Plus, there are other diseases, such as cancer and leukemia, that are also increasing in our population. It would be unfair to exclude them or to indirectly state that those who died of other diseases are not as important as those who died of AIDS. CONCLUSION It would be a very thoughtful and caring gesture to put up an AIDS memorial wall in Stanley Park, but it would not promote its purpose effectively. Rather, it would destroy the parks definition as a place of tranquility and retreat. It would also cause problems of discrimination against those who die of other diseases. We must go forward in our battle against AIDS, but let us go forward with the power of direct education rather than with a memorial to the dead in a place that celebrates life.

255

256 AppEndix iii


Critical Thinking Course
THESIS The proposed steel AIDS memorial monument should be erected in Stanley Park. SUPPORT Stanley Park is a natural environment, and nature represents life and death. The AIDS memorial, too, is about life and death: the lives and very tragic deaths of those individuals who have succumbed to this disease. Thus, this setting seems conducive to this monuments message. Also, AIDS is a worldwide problem, and Stanley Park hosts people from all over the world. Thus, the message of this monument would not be limited to the people of our city. As part of humanity, we are all affected, and we should all be united in the fight against this disease. OPPOSITION The proposed AIDS memorial is to be a large steel monument, and this will contrast to and be an eyesore in the natural setting of Stanley Park. Many people use the parks natural and peaceful setting to escape from their personal and the worlds problems, and such an in-your-face monument may inhibit their ability to do so. This monument should be erected in another high-volume and visible location, such as the city center, or perhaps the Burrard Street Bridge. Also, there are other diseases, such as cancer, that do not have any monuments, let alone monuments in Stanley Park. Why should we single out this one disease? RESPONSE The purpose of this monument is to celebrate the lives of the victims of AIDS and to bring awareness about the increasing numbers of people afflicted with this disease. Unlike other diseases, such as cancer, without awareness leading to preventive measures and a cure, the AIDS epidemic will continue to grow and has the potential of killing enormous numbers of people. Not only will the contrast of steel in nature draw peoples attention and bring some awareness but the harshness and boldness of the steel will also illustrate the harsh reality that AIDS is an epidemic killing millions of our fellow human beings. Whether or not people want to accept it, this is a disease that affects everybody in some form or another. The beautiful and natural setting of Stanley Park helps people unwind, slow down, and relax, leaving the commotion of the rat race behind, even for a short while. Because of this mental state, people may be more likely to take notice of this monument and perhaps feel more compassionate and receptive to its message. If this monument were to be erected in the city center, where people are busy going from one place to another with their minds focused on their daily problems and where there is vehicle and pedestrian congestion, they may not notice it or be receptive to its message. CONCLUSION This proposed AIDS memorial monument in Stanley Park may not blend into the natural setting. However, it is this very contrast, along with the relaxed atmosphere, that will likely increase the effectiveness of its message. This disease is entirely preventable. A slight aberration in the general tranquility of the park seems a small price to pay for doing our bit to stamp out this plague. In addition, the very beauty of the park is surely a perfect setting for celebrating the lives of those whom we were unable to save. Let us at least give them, and those who loved them, this remembrance.

Critical Thinking Course


THESIS Little white lies, lies that are used without the intention to hurt, are necessary for coexistence in our society. SUPPORT People cannot live together without lies because they have a truth threshold. The truth, more often than not, hurts, and constantly having to hear and deal with the truththe cold, hard truthespecially from people who are near and dear to us, such as family, lovers, and friends, is certainly not a pleasant situation. Humans cannot bear very much reality (T. S. Eliot). People often lie to protect others, for example, saying, It is a lovely dress, even though they do not really believe it. These lies are designed to keep the wheels of social interaction going. How many friends would you have if you were to express your true feelings all the time? You know, John, you are a fun guy to hang out with, but I think you are a moron. Actually, Cheryl, I do think that you have a fat ass. In these cases, people want to be

ExAmplEs of Good ArGumEnts

257

fooled. You also lie to protect yourself from feeling guilty; out of fear of punishment, failure, or disgrace; or to seem more impressive to others. If a lie saves you from a difficult situation, wins you a job that you really need, or gets you a date for Saturday night, then why not tell it? OPPOSITION Lies hinder the pursuit of truth. If people constantly lie in order to conveniently resolve a situation, then the situation will never be resolved. Honesty is fundamental to society. People cannot live together well if no one is able to believe what anyone else is saying. Lies are meant to mislead others deliberately, either by concealing or by falsifying the truth. Lying to get a job or a date or to get out of a sticky situation is simply immoral and wrong. Lies are never harmless; someone is always eventually hurt or deceived. Small lies lead to bigger lies to cover up the first lies, and so on, until it is out of control. If you do not tell your friend that the dress she is wearing looks silly and someone on the street laughs at her, she will be hurt then. Lies create uncertainty, and uncertainty creates skepticism. People will think others are lying even when they are not. So, when you ask that girl out, despite the fact that you may be completely truthful, she will not trust you. RESPONSE Lies are often less hurtful than the truth. It would hurt someone more to tell them, Look, I was drunk in the bar when I met you and, quite frankly, I find you physically unappealing, and have already thrown away your telephone number, rather than say, No, I cant go out with you tonight because Im busy a couple of times until the other person clues in. Besides, people seem to prefer a pleasant lie to the harsh truth. George Bush was elected with the infamous read my lips, no more taxes, whereas Walter Mondale lost by a landslide after he told voters he would raise taxes. And what if you do not know the absolute truth because of a lack of information, lack of knowledge, or inability to see into the future? Is telling a country that the economy will get better lying? If it is, then the phrase I love you is probably the most common lie around. CONCLUSION In an uncomplicated world, lies would not be necessary because the truth would be self-evident. However, in this world, if lies were not part of regular interpersonal communication, social interchange would be cold, mean, and harsh. There are other values in life besides the value of truthfulnesssuch as caring about the welfare of othersthat sometimes, perhaps even often, take precedence. Values are valuable only within a context. If nothing else is at stake, you should never lie. However, if the preservation of life, liberty, and pursuit of happiness requires the grease of a few little white lies, so be it.

Critical Thinking Course


THESIS Parents should support a childs interest in computer games as long as it is in moderation. SUPPORT There are a large variety of computer games that have an array of different beneficial characteristics. Eye-hand coordination can be developed through shoot-em-up games (games that typically involve an aircraft of some sort that navigates through enemy fire) and sports games. Memory is constantly tested when you are forced to recall the sequence you need in order to make a certain move in fight games. Strategic games make a person think laterally. Strategy and memory are also tested with run through games such as Bandicoot or Sonic. Games are also a good way to entice children to use computers. Technology is moving ahead rapidly, and computer skills will be necessary in the future. Games make computers friendlier to children. OPPOSITION Computer games are a waste of time. A child who sits in front of a computer for hours on end playing a game may actually be harmed. Schoolwork may be neglected or not completed as well as it could have been had more time been devoted to it. In addition, a child benefits from a lot of social interaction with other children. Eye-hand coordination and strategy can be exercised through sports, and memory can be challenged with the help of a book. If a child uses the computer for games, then the true benefits of a computer will be undiscovered. Instead of being a powerful educational tool, the computer will be reduced to a toy.

258 AppEndix iii


RESPONSE A child is easily bored with academics unless it is fun. Trying to persuade a child to use a computer for research and educational purposes alone would be a difficult battle. Since school homework requires research, the child will refer to the computer for educational purposes in any case. Games relieve stress, and although getting past level four does not mean a lot to an adult, it gives the child a small sense of accomplishment. It may also be helpful socially if a child can take his knowledge of a game to his or her friends. Some people are just not built for sports; computer games enable them to experience competition. CONCLUSION Computer games, as with most things in life, should be utilized in moderation. They allow children to escape the daily routine and exercise some basic functions in life and are basically challenging and fun. Of course, excessive use could have negative results, such as the development of antisocial behavior and the neglect of responsibilities. However, computers are going to be a big part of everyday life for the foreseeable future. Parents may as well help their children learn to play with them appropriately, just as they help them handle other potentially helpful or dangerous practices, such as the use of alcohol.

Biomedical Ethics
THESIS If an individual attempts to commit suicide and is taken to the hospital, no heroic measures should be taken to save that persons life. SUPPORT According to Immanuel Kant, respect for the autonomy of others requires that we allow them to make their own choices. In a suicide attempt, this choice is death. Also, utilitarians, who argue that we should always attempt to produce the greatest good for the greatest number, would no doubt suggest that a better use of scarce medical resources would be to reserve them for those who want to live rather than to use them on those who want to die. OPPOSITION People who try to end their lives are often very confused and depressed. They see suicide as their only option and lack other choices in their lives. This would make them incompetent and irrational, and we are not morally obligated to stand by these types of decisions. These people should be revived and then rehabilitated. RESPONSE Suicide is often a planned event, with lots of reasoning and rationale put into the decision. In order to override the survival instinct, this decision must have been thoroughly thought through, and the reasons behind the decision must be extremely powerful. The notion of respect for persons would be completely vacuous if all it meant is that we respect the decision of others when, and only when, we agree with them. Autonomy demands that we respect others choices with regard to the time and manner of their deaths, even if it is not a path that we ourselves would choose. CONCLUSION It is hard for those of us who cling so desperately to life to believe that some people truly wish to die. However, respect for others demands that we respect their choices. We do not have to agree with them or help them, but we must honor them in their final choice. If we honored all suicides, the resources in the hospitals could be dedicated to saving those who truly want to live, thus honoring those choices as well.

Biomedical/Environmental Ethics
THESIS Scientific research and medical procedures should not be tested on animals in order to benefit human beings from the results of these experiments. SUPPORT Most experiments are conducted on sentient animals (those that feel pain, fear, anxiety, loneliness, boredom, and pleasure). We confine them to isolated cages where they are continuously subjected to excruciating experiments, where they are traumatized and violated. As Tom Regan says, The pulsing pain of the chimp with electrodes planted deep

ExAmplEs of Good ArGumEnts

259

in her brain is repulsive (43). We inhumanely treat these innocent animals like resources, as if their sole value is reducible to their usefulness to humans. The fact that these beings have the capacity for suffering mandates that we discontinue sacrificing them for our gain. OPPOSITION The moral rights of humans and nonhuman animals are not identical in strength because humans are more intelligent than animals and have the ability to listen to reason (Warren 46). A utilitarian view seeks to maximize the satisfaction of interests, which sometimes requires the killing of animals in order to produce the greatest good for the greatest number. It should certainly be justifiable to use a few animals in cancer research if it would result in finding a cure that could save thousands of lives. RESPONSE This view is a form of speciesism (Singer 34), as it has an unjustified bias of promoting the best interests of humans alone (as these procedures are performed in the attempt to find medical breakthroughs for human beings and not other animals). There are some humans who are clearly below the level of awareness, self-consciousness, and intelligence of many animals, such as humans with brain damage or even infants (Singer 35). If we can justify exploiting animals in our scientific studies, then we should be prepared to include certain individuals from the human population. If we are not, then we should stop using animals. CONCLUSION The undeniable fact that the animals we cruelly use in scientific research have the ability to suffer calls for the elimination of the scientific research to which they have been subjected. The psychological and physical trauma that we inflict on these innocent creatures cannot be justified, as is evident from the fact that we would be outraged if humans of a similar or lesser level of sentience were treated in a similar fashion. If we consider these experiments too vile even for humans who are unable to experience pain, it is certainly immoral to perform them on the animals that can!
Regan, Tom. The Radical Egalitarian Case for Animal Rights. Environmental Ethics: Reading in Theory and Application. 3rd ed. Ed. Louis P. Pojman. Belmont, Calif.: Wadsworth/Thompson Learning, 2001. 4045. Singer, Peter. A Utilitarian Defense of Animal Liberation. Environmental Ethics: Reading in Theory and Application. 3rd ed. Ed. Louis P. Pojman. Belmont, Calif.: Wadsworth/Thompson Learning, 2001. 3339. Warren, Mary Anne. A Critique of Regans Animal Rights Theory. Environmental Ethics: Reading in Theory and Application. 3rd ed. Ed. Louis P. Pojman. Belmont, Calif.: Wadsworth/Thompson Learning, 2001. 4650.

Environmental Ethics
THESIS Animals used in farming should be given a set of basic rights that would guarantee humane treatmenti.e., decent food, water, and a set amount of open space. This law would outlaw the present method of production of veal, chicken, and some dairy operations. Raising calves in boxes to improve quality would be illegal. SUPPORT The capacity for sentience in animals makes it immoral to treat them in a cruel way, regarding them as a means only. Our actions toward factory farm animals disregard the pain and suffering they experience. We extend moral rights to all people because our inherent inequalities are not relevant criteria for granting moral rights. So, for example, we extend moral rights to mentally disabled humans and infants regardless of their lack of reason. Since we treat mentally disabled humans in a moral way because we understand they can suffer, sentience should be the criterion for animals as well. Peter Singer (35) asks, If possessing a higher degree of intelligence does not entitle one human to use another for his own ends, how can it entitle humans to exploit nonhumans? OPPOSITION Instead of just banning inhumane treatment of animals, we should ban meat eating altogether. If we grant nonreasoning creatures the same rights as mentally disabled humans, it follows that we should stop eating them, or else start eating infants and the mentally infirm. Also, the recognition of rights for animals shows that we should not own them. After all, we do not own our children or our mentally disabled citizens. In addition, although the real wrong is the imprisonment and torture of animals, imprisonment without torture is still imprisonment. Laws to soften the system are not needed; consumer awareness is. Peter Singer (36) cites an unnamed source: Cruelty is acknowledged only when profitability increases.

260 AppEndix iii


RESPONSE Mentally disabled humans and infants have moral rights, but not the same moral rights as autonomous moral agentsi.e., healthy human adults. We do not allow infants to drive. We withhold rights to maintain the subjects best interest. Giving moral rights to animals does not mean equal rights. Millions of freed cattle, chickens, and pigs would be an environmental disaster. Ending the use of farm animals altogether would be saving them by ensuring their extinction. CONCLUSION Not everyone will stop eating meat, and not everyone will care where their meat comes from. That is why we need laws to protect animals. Consumer awareness, one hopes, would help encourage the acceptance of animal rights and could diminish the profitability of factory farming, encouraging the many producers to use humane methods. Once that happens, it will seem no more extraordinary to enact laws that attempt to prevent cruelty to farm animals than it does to enact laws that attempt to prevent cruelty to pets.
Singer, Peter. A Utilitarian Defense of Animal Liberation. Environmental Ethics: Reading in Theory and Application. 3rd ed. Ed. Louis P. Pojman. Belmont, Calif.: Wadsworth/Thompson Learning, 2001. 3339.

Environmental Ethics
THESIS Parts of our country should be set aside as sacred spaces with the value and untouchability of the trees in the sacred spaces as in China, of which Garrett Hardin speaks (220). That is to say, under no circumstances, even economic collapse or widespread starvation or the building of roads, should these areas be subject to human use and abuse. These wilderness areas must be large enough to sustain all the life presently inhabiting them and should be interconnected by safe corridors to give a greater guarantee of accomplishing a varied and sustainable biodiversity. They should cover all the regions and ecosystems within Canada (north coastal, south coastal, mountainous, interior, and subarctic). SUPPORT Donella Meadows (157) says, Biodiversity contains the accumulated wisdom of nature, and the key to its future and explains how preserving biodiversity is crucial economically, environmentally, and informationally. Biodiversity is of critical importance to our healthy existence and to the existence of future generations of both humans and animals. OPPOSITION In limiting business access to our plentiful resource base, we would be crippling our ability to take part in the worlds economic market. We would be sending ourselves and our children and future generations into poverty. Who are we to decide that the next generation should have a standard of living far below what we have hitherto experienced? RESPONSE Although we would not be able to participate as easily in the global economy as we have in the past, we would not be impoverished in the absolute sense. It is true that we would have to fundamentally change almost every aspect of our lives, as Lester Brown, Christopher Flavin, and Sandra Postal have outlined in Vision of a Sustainable World (497). However, since by definition unsustainable economic activity must come to a halt sooner or later, surely it makes more sense to bring it to a halt sooner, when there are still thriving ecosystems left that vitally contribute to the welfare of all living entities. CONCLUSION Civilizations and even species have historically peaked and then collapsed. We are faced with the possible extinction of our own civilization. We must not follow the patterns and paths of our forefathers, which have led toward disaster. We have the unique knowledge and the ability to change our direction in a gradual and progressive manner instead of exhausting our potential. If we allow our generation to destroy biodiversity, we will be limiting future possibilities and the choices of our grandchildren and their grandchildren. Although such wisdom will undoubtedly bring economic hard times in the short term, in the longer term we will all be much richer.
Brown, Lester, Christopher Flavin, and Sandra Postal. Vision of a Sustainable World. Environmental Ethics: Reading in Theory and Application. 1st ed. Ed. Louis P. Pojman. Boston: Jones and Bartlett, 1994. 493500.

ExAmplEs of Good ArGumEnts

261

Hardin, Garrett. Who Cares for Posterity? Environmental Ethics: Reading in Theory and Application. 1st ed. Ed. Louis P. Pojman. Boston: Jones and Bartlett, 1994. 21924. Meadows, Donella H. Biodiversity: The Key to Saving Life on Earth. Environmental Ethics: Reading in Theory and Application. 1st ed. Ed. Louis P. Pojman. Boston: Jones and Bartlett, 1994. 15657.

Philosophy and Gender


THESIS We define pornography as the graphic, sexually explicit subordination of women, through pictures or words, that also includes women dehumanized as sexual objects, things, or commodities; enjoying pain or humiliation or rape; being tied up, cut up, mutilated, bruised or physically hurt; in postures of sexual submission of servility or display; reduced to body parts, penetrated by objects or animals, or presented in scenarios of degradation, injury, torture; shown as filthy or inferior; bleeding, bruised or hurt in a context that makes these conditions sexual. Erotica, defined by distinction as not this, might be sexually explicit materials premised on equality (MacKinnon 394). Understanding this, all forms of pornography should be illegal (i.e., require time served in prison). Erotica should replace all pornographic material to promote tenderness and discussion of lovemaking preferences. SUPPORT Pornography teaches like advertising teaches, and what it teaches is that women should be subservient to mens sexual demands, however physically and emotionally painful, and therefore, it teaches that women are to be at the mercy of men. This normalizes the idea of men harming women. Since rape, battery, torture, and murder are punishable by law, the production and possession of materials that advocate this behavior as natural and acceptable should be made punishable by law (as is the case with child pornography). Since there is such a demand for sexually explicit material, erotica should replace pornography. This would promote sexual equality. OPPOSITION Since men desire sex more than women, pornography serves as a masturbation tool that helps to vent their sexual urges. David Steinberg claims that porn films are also educational in that they offer real learn-by-watching information (the information we should but do not all receive as emerging adults) on all kinds of sexual practicesand that there is no problem in this as long as we bring a critical eye to tell the fake from the real (there is plenty of both), and the friendly from the nasty (also both well-represented) (398). He also points out that if pornography becomes illegal, it, like prostitution, will only come to represent the notion that sex is dirty, even more strongly than it does today (398). RESPONSE Men who masturbate to pornography (defined in the thesis) are conditioning themselves (if not conditioned already) to be sexually aroused by the idea of women submitting themselves to every sexual demand, no matter how brutal. This is harmful to women as a social group. Men such as these should masturbate to erotica, since it is as sexually explicit as pornography but it is founded on sexual equality. For emerging adults, porn does not serve as healthy education, as it teaches both men and women that the sexual difference between them is that men should dominate and women should submit to any and every sexual demand in order to be sexually desirable. Pornography should be illegal because it perpetrates brutal treatment of women. It is pornography, not its lack, that promotes the idea that sex is dirty. CONCLUSION A necessary (but not sufficient) condition for achieving equality between men and women, and decreasing the utterly unacceptable rate of violence against women, is to ensure that all forms of pornography are illegal and their production punishable by law. On the other hand, in order to avoid reinforcing the idea that sex is dirty, and as a form of healthy sex education and communication, we should welcome, indeed encourage, erotica.
MacKinnon, Catherine. Francis Biddles Sister: Pornography, Civil Rights, and Speech. Gender Basics. Ed. Anne Minas. Belmont, Calif.: Wadsworth Publishing, 1993. 39195. Steinberg, David. The Roots of Pornography. Gender Basics. Ed. Anne Minas. Belmont, Calif.: Wadsworth Publishing, 1993. 39699.

262 AppEndix iii


Philosophy and Gender
THESIS Sex education classes should teach both sexes never to participate in sexual activities (i.e., kissing, fondling, engaging in intercourse) with new partners without first receiving verbal consent from the partner. (Both partners literally have to say yes.) SUPPORT Boys learn that they must initiate sexual activity, that they may meet with reluctance from girls, but if they just persist, cajole, and refuse to let up, that ultimately they will get what they want (Warshaw 362). This leads to womens wishes being ignored, and in some cases it leads to rape. Both partners should be encouraged to seek verbal consent, as it will provide clarification of limits for one another. OPPOSITION We do not need to train women and men to ask for consent because if one does not wish to participate in the activity, then all she or he need do is say no. Body language is what indicates consent. Asking for verbal consent will take the romance out of the moment. RESPONSE Warshaw states that womens socialization has most likely taught her that she must not express her own wishes forcefully, that she should not hurt other peoples feelings or reject them, that she should be quiet, polite, and never make a scene (359). Since women are socialized in this way, it is difficult (in some cases impossible) for a woman to verbally reject a sexual situation. Instead, she may physically hint the rejection, which could be confused as putting up resistance to maintain a good reputation. Body language is ambiguous. This situation then becomes a game, which each side silently struggles to win. Rape may be the end result. Explicit verbal consent need not take away from the heat of the moment. In a healthy sexual situation, two willing people talking about the adventure on which they are about to embark should be a turn-on. Verbal consent can be requested in a very sexual waye.g., I would love to have sex with you, and I am wondering if you are feeling the same way? CONCLUSION Sex education classes should teach both sexes never to participate in sexual activities without verbal consent from their partner. This will not only help prevent unwanted sexual advances but also help both men and women become more comfortable talking about sex, which in the long run will enhance, not detract from, the sexual experience.
Warshaw, Robin. I Never Called It Rape. Gender Basics. Ed. Anne Minas. Belmont, Calif.: Wadsworth Publishing, 1993. 35864.

Philosophy and Gender


THESIS Under no circumstances should women fake an orgasm. (In the interest of convenience and simplicity, this essay pertains to heterosexual sex.) SUPPORT Faking orgasm is unethical because it is intentionally deceptive. Within a relationship, deceit of any kind should be vigilantly avoided because it is harmful: inducing secrets, betrayal, and distance. Individually, the woman gives away her opportunity for sexual climax and in doing so gives away the mans opportunity to learn how to satisfy her. Indeed, she reinforces unsatisfying behavior. OPPOSITION Women are sexually different from men; they do not always experience orgasm. By faking orgasm, a woman will not have to endure a drawn-out sexual act intended to produce an orgasm she knows she is not going to experience at that particular time. It also helps to avoid the long discussion about why she did not reach orgasm. Therefore, it is beneficial to a woman to occasionally fake orgasm. The partner does not feel any less a man, and the woman does not feel responsible for putting a damper on the sexual act.

ExAmplEs of Good ArGumEnts

263

RESPONSE There is plenty of harm done by faking orgasm: The woman and man each suffer separately, as do the relationship and the state of women and men at large. The woman will hardly feel sexy if she has to pretend her lusty climax; instead, she will harbor the guilt or discomfort of creating deception. If she is looking for an exit from sex, she should communicate that exact message (verbally or physically). Also, she forfeits her opportunity for sexual climax because he assumes he has done his job and can quit. She should not take responsibility for padding the ego of her lover, as it puts undo co-dependent pressure on her, and he is robbed of the opportunity to learn more effective sexual methods of encouraging her satisfaction. Faking an orgasm also encourages the idea that sexual climax is the embodiment of sexual purpose, propagating the idea that sex is about orgasm, not love. Women as a group suffer harm because faking orgasm reinforces the notion that a womans role is to serve her ma(ster)n. Men as a group suffer because it reinforces the idea that a mans self-worth rests in getting the penis to do what it ought (Teifer 317) and that his ego is wrapped up in his libido. The woman thus reinforces a false sense of masculinity at the expense of her sense of integrity and physical pleasure. If she cares so much about her partners manhood, she should take the time to teach and show him how to make her have an orgasm. If he defines his manhood through his partners orgasm, then now he will truly be a man. CONCLUSION Faking an orgasm is detrimental to a woman as it virtually ensures that failure to achieve orgasm will become a common occurrence. Faking orgasms is detrimental to a man as it ensures that he cannot learn to become a good lover. Faking orgasms is detrimental to a relationship, as this mismatch will inevitably create frustration and differing sexual appetites. Faking orgasms is detrimental to our whole notion of the role that sex can play in bringing two people together by overfocusing on the Big O. Be honest, ladies! Your man will love you for it, that is, if he is a man worth loving.
Teifer, Leonore. In Pursuit of the Perfect Penis: The Medicalization of Male Sexuality. Gender Basics. Ed. Anne Minas. Belmont, Calif.: Wadsworth Publishing, 1993. 31519.

Philosophy and Gender


THESIS Every individual should promote the use of sexual language that does not connote aggression, force, or objectification by using neutral diction in speech and writing and by instructing others to do the same. Specifically, words like f*ck, bang, nail, screw, and so forth should be replaced by have sex with, have intercourse with, make love with, and so forth. SUPPORT Language leads men not only to objectify women but also to objectifyand so dissociate fromtheir own sexual organs (Warshaw 362). This objectification and dissociation enables the man to regard his penis as something out of his control, thereby justifying his not taking responsibility for his actions. Hypermasculine behavior is encouraged by messages sent through phrases that describe sex through aggressive means, in which the woman is the passive, hurt element. We internalize the correlation between violence and intimacy, as our thoughts and consciousness are built out of the exact meaning carried by words. OPPOSITION It is a hassle and an inconvenience to pour conversations through a sieve of political correctness. Words are not that important, as they do not change the rising rape incidence or erase other sexually violent crimes. F*cking is not generally seen as a derogatory term when referring to sex; it is simply slang. This diction is so widespread it would be difficult, if not impossible, to change. Having intercourse or having sex is too clinical, while making love is too mushy. RESPONSE It may be inconvenient or awkward to consciously choose neutral language at first, but with practice it will become second nature. Words are important, as the Sapir-Whorf hypothesis claims, because they are vehicles of our consciousness, our thoughts, our beliefs, and our worldview. Since you f*cking . . . or f*ck you carry negative connotations, you

264 AppEndix iii


f*cked her must, at least subliminally, carry some of that same negative charge. To dissociate the relation of sex and violence in our actions, it is necessary to dissociate them in our minds. The reason f*cking is not seen as a derogatory term is that violence and sex are so tightly ingrained in our way of thinking and speaking, and therefore of acting, that we are blind to this derogatory association. CONCLUSION Every individual should use neutral sexual diction in speaking and writing and instruct others to do the same. Few of us will have the opportunity to stop a rape in progress; however, all of us have the opportunity to insist that the words used to describe sexual interaction are imbued with the values of equality and care. If we are ever going to create a society in which violence and sex are disassociated, we must all do our part.
Warshaw, Robin. I Never Called It Rape. Gender Basics. Ed. Anne Minas. Belmont, Calif.: Wadsworth Publishing, 1993. 35864.

appendix IV
What Good and Poor Thinkers Look Like
Given the theoretical framework thus far presented, a picture of what a good and a bad thinker look like should be emerging. Precise images of what both a good thinker and a poor thinker look like are important, as they can serve as appetitive and aversive stimuli, respectively. These stimuli, in turn, will compete successfully, one hopes, with all the other influences that pressure us into thinking poorly.
A GOOD THINKER

Figuresoutwhichquestionsareimportanttoaski.e.,isawake. ccesses asmuchrelevant data/reasons aspossiblefromas A manyanglesaspossible. Assesseshowthedata/reasonsbearonthequestionathand. udgeswhichofcompetinganswersareleast plausible(are J underpinnedwithfaultyreasoning)orgeneratesnovelsolutionsin thefaceofuntenableconflict. Ispreparedtoabandonanswersthatareinadequate. odulates confidenceinproportiontothestrengthofthedata/ M reasonstowhichs/he hasaccess. nderstandsthathisorherfallibilityrequiresthats/healways U remainopentonewdata. Putsgood thinkeratornearthetopofidentitycharacteristics. Ismoved by reasons. Hasanimageofwhatgood thinkinglooks like.

265

266 APPENDIx IV
A POOR THINKER

Asksveryfewquestions. Findsithard to keep quietwhenothersarespeaking. Doesnot really listentothepointofviewofothers. arely tries to clarifywhatanotherpersonistryingto R communicate. Listenstoothersmainlytorefute. Becomesannoyedinadiscussionwhenothersdisagree. Findscomplexity frustrating. elievesthateveryonesopinionsareas good as everyone elses. B Tendstoacceptwhatimportant others believe. Isincapableofdifferentiating good from poorreasons. Infusesopinionswithan unwarranted amount of confidence. Rarely backs downfromhisorheroriginalposition. Sticks to positionsthroughthickandthin. Isnot courageousinstandingupforwhats/hebelievesisright. Isnot courageousinfollowingreasonswheretheylead. oes not think that agonizing over questions of value is D important. Does not think that living ones values is important.

appendix V
Answers to Pre-tests and Post-tests
Pre-test 1: What Kind of Thinker Are You? Scoring
Often 1. I()askalotofquestions. 2. hendiscussingissues,I()getfrustratedwhenpeopledisagreewithme. W 3. tis()awasteoftimetotalkwithotherswhothinkcompletelydifferently I fromthewayIdo. 4. I()questionothersindetailinordertoclarifywhattheyhavejustsaid. 5. naheatedinterchange,whenothersaretalking,I()usethatopportunityto I considerhowIcanrestatemyownpositionmorepersuasively. 6. I()findithumiliatingtobackdown. 7. am()reluctanttochangemypositiononcertainissues,becausemy I positiononthoseissuesispartofwhoIam. 8. ()expressconfidenceinapositionevenif,atthetime,Iamunabletoback I itupconvincingly. 9. have()saidthatsinceIdonothaveadequatedataorinformation,Iamnot I inapositiontogiveanopiniononanissue. 10. I()findcomplexityannoying. 11. am()reluctanttodisagreewithpeoplewhoareimportanttome. I 12. ()seriouslyquestionwhatagoodlifeisandwhatkindofpersonIwantto I become. 13. I()spendtimejustthinking. 3 0 0 4 0 0 0 0 3 0 0 4 2 Yes 14. Ihaveapreciseideaofwhatagoodthinkerlookslike. 15. Icandefinegoodthinking. 16. hensomeoneelseusespoorthinking,Iunderstandandcanexplainwhy W thethinkingisfaulty. 17. Havingalotofmoneyisnecessaryformywell-being. Total 3 3 3 0 Sometimes 1 1 1 1 2 1 1 1 1 1 1 1 1 Sort of 1 1 1 1 Rarely 0 3 3 0 4 3 3 3 0 4 3 0 0 No 0 0 0 3 Mark

267

268 APPendix V
What Kind of Thinker Are You? Ifyouscoredbetween46and54,youareagoodthinker,andyouhavelearned toappreciatetheimportanceofrigorous,complex,impartialthinking. Ifyouscoredbetween36and45,youareaprettygoodthinker.Ifyouarediligentinforcingyourselftoreflectonthemeritsofopposingviewpointsimpartially,youwillcontinuetoimprove. Ifyouscoredbetween26and35,youmaybeontheroadtobecomingagood thinker,butyouareinseriousjeopardyofbackslidingintothequagmireofyour ownprejudices.Youneedtokeepremindingyourselfthatyourautonomyyour veryexistenceasauniqueself-legislatordependsonyourcapacitytotestthe meritsofyourownviewsagainstthoseofcompetingcontendersseriously. Ascorebelow26suggeststhatyouareonthehookofsomeseriouslypowerful puppetstrings.Italsosuggeststhatyouarehostiletotheprocessthathasbeen laidoutforyouhere.Ifso,thenrememberthatthechallengeremainsopento youtoshowwhereinitisfaulty(seetheinvitationattheendofPartI).Remember,too,thatthechoicetochooseyourselfisalwaysopen,anditisalways rewarding,eveniftheroadtofindingyourselfseemslikeatreacherousone. Note on Comparison Thepre-testmayhavebeentransparenttosomei.e.,someofyouwillhave guessedwhatthepoliticallycorrectanswerswouldbe.Ifthisisthecase, therewillbelittledifferenceinthetwofairlyhighscores.Nonetheless,manyof youmaybelievethatyouhaveamuchmoreprofoundinsightintopreciselywhat thesetestsaregettingat,andyoumaybelieve,therefore,thatalthoughthereis littledifferenceinthescores,thereisaprofounddifferenceinthewayyounow think.Ifso,goodforyou! Forsomeofyou,therewillbeasignificantdifferenceforthebetter.Ifso,congratulations!Youhavecomealongwayinashortperiodoftime. Forsomeofyou,bothscoresmaybelow.Ifso,seethepreviouscomments.

Post-test 2: Logic Answers


1. c 2. b 3. c 4. a 5. b 6. c 7. a 8. a 9. c 10. b 11. b 12. c 13. c 14. b 15. b

notes
1. NeilPostman,The End of Education(NewYork:AlfredA.Knopf,1995),x. 2. ManyofthethoughtsfoundunderthisheadingarepublishedinSusanT.Gardner,TeachingFreedom,Analytic Teaching21,no.1(November2000):2128. 3. Clearly,manyanimals,perhapsmost,areconsciousofthemselvesinrelationshiptootherentities.Thecapacity tobeawareofoneself,however,isamatterofdegree.Languageisacatalystthatgreatlynurturestheexpansion ofself-consciousness(inparticular,thecapacitytoimagineoneselffarintothefuture)hence,theimportanceof thephraselinguisticallyenhancedself-consciousness. 4. GabrielMarcel,Mortality,Hope,andFreedom,inExistentialism and the Philosophical Tradition,ed.D.Barsoum Raymond(UpperSaddleRiver,N.J.:PrenticeHall,1991),341. 5. Externalhasbeenplacedinquotationmarkstosignalthattheforceofanyexternalstimulusisnotstrictlyexternal.Itisalsoafunctionoftheinternalstateoftheconsciousbeing.Forexample,theappetitivepulloffoodisat leastpartiallyafunctionofthedegreetowhichtheconsciousbeinginquestionishungry. 6. ManyoftheideasincorporatedunderthisheadingcanbefoundinSusanT.Gardner,Autonomy:APhilosophicalCapture,Practical Philosophy3,no.3(April2001):1922. 7. Thisaccountofself-consciousnessfindsitsrootsintheworksofGeorgeHerbertMead,On Social Psychology, ed.A.Strauss(Chicago:UniversityofChicagoPress,1934).Meadsworkgaverisetoamovementreferredto assymbolicinteractionism.Thereaderwillnotethatthetermsymbolicinteractionisusedfrequentlyinthe presentwork.Note:Unlikesignlanguage,symboliclanguagecanbeusedtorefertoeventsoutsidethehere andnow. 8. Deductiveargumentationisthekindofargumentthatmovesvalue;itisstudiedindetailinPartII. 9. Asexplainedinnote5,externalhasbeenplacedinquotationmarkstoindicatethattheforceofanyexternal stimulusisnotstrictlyexternal.Itisalsoafunctionoftheinternalstateoftheconsciousbeing. 10. Andhasbeenitalicizedtoemphasizetheimportanceofsymboliclanguage.Whereastheemergenceofself- consciousnessappearstobeanecessaryconditionforthedevelopmentofsymboliclanguage,symboliclanguage, inturn,isnecessaryforthedevelopmentofthekindofself-consciousnessthatallowsustoimagineourselvesinto varyingdepthsofourfuture. 11. Animaginedactionandanimaginedreinforcementbecomeassociatedthroughlinguisticdeductiveargumentation(e.g.,youshouldsharetoysbecauseitwillshowthatyouareakindandcaringperson). 12. Recognizinghiddenpremisesisnecessaryifoneistobecomecompetentinseeingthevaluesthatanchorpractical reasoning.PracticeinfindinghiddenpremisesisincorporatedintoanumberofexercisesinPartII. 13. KantianscholarswillnotethesimilaritytoImmanuelKantsKingdomofEndsbutthereisadifference.Seethe nextsection. 14. Specifically,Kantarguedfortheimportanceofutilizingauniversalizationprocess.Itisarguedherethatalthoughthisprocessmaybehelpfulineliminatingbias,amorepowerfulprocessissubjectingourjudgmenttothe actualcritiqueofourstrongestopposition. 15. RichardHare,Freedom and Reason(Oxford,England:OxfordUniversityPress,1967),15985. 16. Sentimentsintheirdevelopmentwillbeverygreatlydeterminedbyaccidentalcauses....Tosatisfyourdoubts, therefore,itisnecessarythatamethodshouldbefoundbywhichourbeliefsmaybedeterminedbynothing human,butbysomeexternalpermanencybysomethinguponwhichourthinkinghasnoeffect....Suchisthe methodofscience.CharlesSandersPeirce,The Fixation of Belief, inPhilosophical Writings of Peirce,ed.JustusBuchler (NewYork:DoverPublications,1955),18. 17. Theattainmentofaunifiedmethodmeansthatthefundamentalunityofthestructureofinquiryincommon senseandscienceberecognized,theirdifferencebeingoneintheproblemswithwhichtheyaredirectlyconcerned,notintheirrespectivelogics.JohnDewey,Logic: The Theory of Inquiry(NewYork:HenryHolt,1938),81.

269

270

notes

18. JrgenHabermas,The Theory of Communicative Action,vol.1,Reason and the Rationalization of Society,trans.Thomas McCarthy(Boston:BeaconPress,1992[Germantext:1981]).Specifically,Habermasarguesthatyoucannot judgetheadequacyofanagentsreasonedsupportofherassertionsand/oractionsanadequacythatcannonethelessbeobjectivelymeasured(22,25)accordingtohowitstandsuptosuchcriticalevaluationascoherence, prediction,etc.unlessyoufirstunderstandtheinvisiblepersonallargercontext(i.e.,apersonslife-worldor lebenswelt)(112). 19. InhistreatiseOn LibertyinUtilitarianism and Other Writings (NewYork:NewAmericanLibrary,1962),JohnStuart Milllendsstrongsupporttothisintersubjectiveviewofimpartialitywhenhearguesthattheonlywaywehumans cangainabroadunderstandingofanyissueisbyseeingitfromasmanyperspectivesaspossible:Theonlywayin whichahumanbeingcanmakesomeapproachtoknowingthewholeofasubject,isbyhearingwhatcanbesaid aboutitbypersonsofevery variety of opinion,andstudyingall modeswhichitcanbelookedatbyeverycharacterof mind.No wise man ever acquired his wisdom in any mode but this; nor is it in the nature of human intellect to become wise in any other manner(146,italicsadded). Millgoesontoarguethatitisimportantthatthisexposureberealratherthanimaginaryi.e.,thatin ordertobenefitfromthemultipleviewpointsofothers,thoseviewpointscannotbeartificiallycontrivedbythe teacherorthroughtheexaminationoftextbooks.Individualsmusthavetheopportunitytolistentothosewith whomtheydisagree,toempathizewiththeirperspectives,andtogenuinelyseetheissueasothersseeit:Noris itenoughthatheshouldheartheargumentsofadversariesfromhisownteachers,presentedastheystatethem, andaccompaniedbywhattheyofferasrefutations.Thatisnotthewaytodojusticetothearguments,orbring themintorealcontactwithhisownmind.Hemustbeabletohearthemfrompersonswhoactuallybelievethem; whodefendtheminearnest,anddotheirveryutmostforthem.Hemustknowthemintheirmostplausibleand persuasiveform;hemustfeelthewholeforceofthedifficultywhichthetrueviewofthesubjecthastoencounteranddisposeof;elsehewillneverreallypossesshimselfoftheportionoftruthwhichmeetsandremovesthat difficulty.Ninety-nineinahundredofwhatarecallededucatedmenareinthiscondition;evenofthosewhocan arguefluentlyfortheiropinions.Theirconclusionmaybetrue,butitmightbefalseforanythingtheyknow:they have never thrown themselves into the mental position of those who think differently from them, and considered what such persons have to say;andconsequentlytheydonot,inanypropersenseoftheword,knowthedoctrinewhichtheythemselves profess(163164,italicsadded). 20. SusanT.Gardner,InteractiveReasoning:TheRoadtoFreedom,Critical and Creative Thinking: The Australian Journal of Philosophy of Education 12,no.2(November2004):112. 21. Foramoredetailedcommentontheimportanceofarticulatelistening,seethesubsectionentitledInteractive Reasoningonpage206. 22. ManyofthethoughtsfoundinthissectionarepublishedinSusanT.Gardner,TruthinEthics(andElsewhere), Analytic Teaching19,no.2(April1999):5562. 23. KarlPopper,Conjectures and Refutations: The Growth of Scientific Knowledge(London:Routledge,1963). 24. DetailsofvalidandinvalidformsofreasoningarediscussedinPartII. 25. Relatedtothispoint,itisofinteresttonotethatthefactthatwemortalscanonlyfalsifyandnotverifyrenders thetruth-seekingprocessaninherentlycreativeone;itlogicallydemandsofusthatourconfidencebeproportionaltothenumberofimaginedalternativesgenuinelytakenintoaccount. 26. Propositionsaboutperceptionsmaybeanexceptionhere. 27. Itmaybethatalotofconfusionwithregardtotruthmighthavebeenavoidedifphilosophyhadcalledsuch theoriesthecorrespondencetheoryoffalsehood,thepragmatictheoryoffalsehood,orthecoherencetheory offalsehoodinsteadoftheoriesoftruth. 28. Inphilosophicalcircles,knowledgeistypicallydefinedasjustifiedtruebelief.Accordingtothistheoretical framework,amoreaptdefinitionofknowledgewouldbeajustifiedbeliefi.e.,onebackedbyreasonsand/or evidencethathassurvivedarigorouspublicfalsificationprocess. 29. ImmanuelKant,The Groundwork of the Metaphysics of Morals,trans.H.J.Paton(NewYork:HarperandRow,1967). 30. JohnStuartMill,Utilitarianism and Other Writings(London,1863). 31. JohnRawls,A Theory of Justice(Oxford,England:ClarendonPress,1971). 32. RichardHare,The Logic of the Moral Words(London:OxfordUniversityPress,1967);idem,Freedom and Reason. 33. InthethirdeditionofR.H.JohnsonandJ.A.Blair, Critical Thinking Textbook: Logical Self-Defense(Toronto:McGrawHill,1993),theauthorscontendthatanargumentthatdoesnotinsomesenseaddressthesecompetingpointsof

notes

271

viewfailstosatisfytherequirementofglobalsufficiency(61)asopposedtolocalsufficiency,whichfocuseson thedegreetowhichapremisesupportsaconclusion. 34. Thoughonedoesnotjudgeallclaimsasequal,oneshouldjudgeallpeopleasequalintermsoftheirability tomakeclaims.MarkKingwell,The World We Want: Virtue, Vice, and the Good Citizen (Toronto:PenguinBooks, 2000),125. 35. Thisanalogyisparticularlyappropriate,asitremindsusthatweneedtotakeinfuelinordertoprocessitintoforwardmotion.Intermsofmakingprogresstowardtruthonanygivenissue,weneedtotakeinthefuelofasmany pointsofviewaspossible. 36. InT.EdwardDamer,Attacking Faulty Reasoning: A Practical Guide to Fallacy-Free Reasoning (Belmont,Calif.:Wadsworth,1995),thedescriptionofpoisoningthewelldiffersfromthedescriptioninS.MorrisEngel,With Good Reason: An Introduction to Informal Fallacies (NewYork:St.MartinsPress,1994). 37. TheSupremeCourtofCanadausedaslipperyslopeargument(similartotheonepresentedhere)todenyfortytwo-year-oldSueRodriguez,whowassufferingfromamyotrophiclateralsclerosis(LouGehrigsdisease),the righttodie.Rodriguezv.BritishColumbia(AttorneyGeneral),[1993]3S.C.R.519. 38. ThedistinctionbetweenavalidandasoundargumentisexploredindetailinSubsection5,WhatKindofArgumentIsIt?onpage102. 39. ThesetermsareborrowedfromTrudyGovier,A Practical Study of Argument,5thed.(Belmont,Calif.:Wadsworth/ ThompsonLearning,2001),350.Seeibid.foramorethoroughanalysisofanalogies. 40. Ibid. 41. RussellVannoy,SexwithLovevs.SexwithoutLove,inPhilosophy of Sex and Love,ed.R.Trevas(UpperSaddle River,N.J.:Prentice-Hall,1997). 42. C.S.Lewistohisgodson,LaurenceHarwood,letter,inOxford Today11,no.1(1998):55. 43. Itshouldbenotedthatsomemightconsiderthefollowinganargument:Benisinthegarden;therefore,there isonehumaninthegarden.Thisappearstobeanargumentwithonlyonepremise.However,inorderforthis reasoningtobeprecise,itmustindicatewhetherBenisahumanbeingoradog(orwhatever).Onceitdoesso,it simplybegsthequestion:Benisahumanbeinginthegarden;therefore,thereisonehumanbeinginthegarden. Aroseisaroseisaroseisnottypicallyconsideredanargument.Inanycase,forallpracticalpurposes,itissafe toassumethatallargumentsconsistofatleasttwopremisesandaconclusion. 44. Alsoincludedininductivereasoningarecausalreasoningandinductiveanalogicalreasoning. 45. Thereasonthatthedefinitionofdeductionneedsthisawkwardqualifieristhatvalidityisanallornothingcharacteristic:Eitheranargumentisvalidoritisnot.Thus,theonlywaytopickoutaninvaliddeductivemoveisto saythatthearguerapparentlyintendeditasvalidi.e.,intendedthattheconclusionfollownecessarilyfromthe premisesbutwasunsuccessful.Thiscontrastswithinductivearguments,whichcanbemoreorlessstrong. 46. Visualpicturesorwhatevercouldalsobesubstitutedforthings. 47. Themiddleterminapracticalsyllogismisthefactwithwhichtheminortermisassociated,inthiscasefood. Themajortermisthepredicateoftheconclusion(orthevalue),inthiscasesalivation.Inapracticalsyllogism, movingfromthetopdown,itisofnotethatthemiddletermdropsouti.e.,thefactthatwastheoriginalanchor launchesitsvaluetoanewfactandthendisappears.Movingfromthebottomup,theminortermdropsout,thus demonstratingthatnewfact-valueconnectionscanbeestablishedonlybyborrowingfromanoldfact-valueconnection.Inotherwords,newvaluecannotbecreated.Changecomeswithacontinuousmovingandrefiningof valueswithwhichentitiesareborn. 48. Manyofthequestionsthatfollowaretakenfrom ThomasH.Martinson,Super Course for the LSAT,2nded.(New York:SimonandSchuster,1989),andPetersons Logic and Reading Review for the GRE, GMAT, LSAT, MCAT(Lawrenceville,N.J.:ThompsonPeters,2002). 49. Thecorrelatewouldbethetraditionalgoldenrule,whichadmonishesthatyoutotreatothersasyouwouldhave themtreatyou. 50. InCritical Thinking Textbook:Logical Self-Defense,JohnsonandBlaircommentthatthereasonthatvaguenesscreates aproblemforusasreasonersissimilartothereasonthatablurryimagepresentsaproblemtousasperceiv- ers(150). 51. Ibid,145. 52. SeeSubsections6and7inPartII,SectionI,forexercisesonfindingthehiddenpremise.

272

notes

53. Thisexampleoccurredinanactualcase.ThepersoninquestionwasallowedtoviewthefilmSilence of the Lambs on thebasisofthefactthatdenyinghimviewingaccesswouldconstitutediscrimination.Hewentontobecomea serialmurderer. 54. Theremarksinthissection,alongwithsomeofthetheoreticalargumentspresentedearlier,arefromGardner, InteractiveReasoning.

glossary
Ad hominem/ad feminam attack: An illegitimate attack on the arguer rather than the argument. This is an informal fallacy, because all arguments must be judged on their own merits. It sidetracks the discussion, moving it away from rather than toward truth. (See the comic on page 81.) Analogy (empirical, a priori): An attempt to explain something (the primary subject) by likening it to something with which the listener is familiar (the analogue). An analogy is empirical if the suggested similarity is literal (e.g., invading this country will result in a situation just like the one in Vietnam). The analogy is a priori (nonempirical) when the similarity resides in some underlying principle or structure. (See the comic on pages 9697.) Analogy (faulty): An analogy that leads away from rather than toward truth. An empirical analogy is faulty when the primary subject is literally dissimilar to the analogue in some relevant way. An a priori analogy is faulty when the suggested similarity in an underlying principle or structure confuses rather than clarifies our understanding of the primary subject. Antecedent: The first clause of a conditional claim (e.g., if its a Rottweiler, then its a dog). The antecedent of a conditional claim always makes a claim about a sufficient condition. In this case, if you know someone has a Rottweiler, that is a sufficient condition for knowing that that person has a dog. Note: Conditional claims are exactly the same as all claims. Saying that if its a Rottweiler, then its a dog is exactly the same as saying that all Rottweilers are dogs. (See the comic on page 146.) Appeal to authority: An appeal to what an authority has said or written instead of the presentation of reasons or evidence. Since it is impossible for any of us to be an expert in all fields, this is legitimate except (1) when the authority is not an authority in the field, (2) when there is disagreement between authorities, and (3) when there is no way of accessing what an authority actually said (e.g., improper referencing). Two subcategories of appeal to authority are appeal to the crowd, which is always illegitimate, and appeal to tradition, which is sometimes illegitimate (see Appeal to the crowd; Appeal to tradition). Appeal to the crowd: A reference to what many others believe (or are doing) instead of the presentation of reasons or evidence. This is always illegitimate. The fact that everyone once believed the world was flat did not make it so. (See the comic on page 94.) Appeal to tradition: A reference to tradition instead of the presentation of reasons or evidence. Since traditions help bind people together, this type of appeal serves as a legitimatealthough not necessarily an overridingreason, except in a situation in which the tradition results in harm, in which case it is illegitimate. (See the comic on page 85.) Appetitive and aversive stimuli: Aspects of the environment that elicit approach and avoidance responses from conscious entities. (See the comic on pages 3839.) Argument: A pattern formed by a number of claims, or propositions, such that the listener is invited to believe the truth of one claim (the conclusion) on the basis of the truth of the others (premises). Premises are also referred to as reasons. Argument, deductive: An argument that carries the assumption that if the premises are true, the conclusion must be true. Deductive reasoning is the sort of reasoning used primarily to move values. Argument, inductive: An argument in which the truth of the premises only increases the probability that the conclusion will be true. Inductive reasoning is the sort of reasoning used primarily to move facts. Figuring out ways to increase the probability that a claim is true is the core art of scientific reasoning. Autonomy: A state of being to which one can reasonably lay claim on the basis of the degree to which one has freed ones judgments (not just ones actions) from the determining power of outside influence. (See the comics on pages 12, 13, 17, 26, 27, 3839, and 205.) Begging the question: A tactic in which one assumes the truth of what one is trying to prove. This tactic is obviously illegitimate, since the point of argument is to offer support for the conclusion in terms of reasons and evidence. If one simply assumes what one is trying to prove, one is reduced to begging ones audience to believe the conclusion without offering any grounds. Begging the question occurs in one of two ways: (1) when the conclusion is inserted into the major premises but in other words or (2) when the minor premise (which is a statement of fact) is assumed to be true instead of being left open to genuine falsification (e.g., all my enemies are unworthy of respect). (See the comics on pages 77, 79, 11617, and 195.)

273

274

glossary

Coherence theory of truth (falsehood): A view that truth is a function of the coherence of beliefs, assumptions, or positions. If ones beliefs contradict another belief, one of those beliefs must be faulty in some way. This is why falsification, as a method of evaluating truth with regard to evaluative claims, works. One cannot believe without contradiction that all things that are unnatural are immoral and that it is perfectly all right to wear glasses. (See the comics on pages 11617 and 118.) Referring to this viewpoint as the incoherence theory of falsehood might be more helpful than the more common notion of the coherence theory of truth, as incoherence indicates that something must be faulty; coherence by itself does not necessarily indicate truth. Complex question: A question that traps a respondent into affirming an underlying assumption, regardless of whether s/he says yes or no (e.g., have you stopped cheating on your taxes yet?). This is usually classed as a form of begging the question. Conclusion: A claim that is said to be true on the basis of the truth of other claims or premises. Consequent: The second clause of a conditional claim (e.g., if it is a Rottweiler, then it is a dog). The consequent of a conditional claim always makes a claim about a necessary condition; in this case, having a dog is a necessary condition of having a Rottweiler. Note: Conditional claims are exactly the same as all claims. Saying that if its a Rottweiler, then its a dog is exactly the same as saying that all Rottweilers are dogs. Contentious: Relating to a claim in which one might reasonably predict that out of a large number of people, many would agree and many would disagree. Topics that are contentious, or controversial, are inherently interesting because they allow virtually everyone to explore unfamiliar territory. In academia, only contentious claims offer the writer the opportunity to demonstrate his or her reasoning capacity, as only contentious claims require that one deal with a strong opposition. (See the comic on page 64.) Correspondence theory of truth: A view that the truth of a proposition is a function of its relationship to reality. If what one believes is inconsistent with what is in fact the case, that is evidence to suggest that there is something faulty about what one believes (e.g., believing that the moon is made of green cheese). Counterexample: An instance or situation that is incompatible with a more general claim. (See Coherence theory of truth (falsehood); Correspondence theory of truth. See also the comic on page 128.) Deductive argument: See Argument, deductive. Determinism: A view that every event is necessitated by antecedent events and conditions, together with the laws of nature (i.e., that every event is causally determined by an unbroken chain of prior occurrences). Distinction without a difference: A claim that ones action, although typical of a certain characterization, is exempt from that label (e.g., Im not racist or anything; I just dont think we should allow blacks in our club). (See the comic on page 92.) Empirical claim: A claim whose truth can be estimated through experiential evidence (e.g., smoking causes cancer). Epistemology: The study of how we know things. With regard to truth, it can be argued that the product is not epistemologically independent of the process. This means that our confidence in truth claims should be entirely a function of our confidence in the process used to test the truth. Fallacy, formal and informal: A fault in reasoning. A formal fallacy is fault in logic (i.e., the conclusion does not follow from the premises). An informal fallacy is a strategy in interpersonal exchange (verbal or written) in which the discussion is moved away from rather than toward truth. Fallacy of the golden mean: Assumption that the truth lies somewhere between diametrically opposed views (e.g., in a divorce, both parties must be at fault). This is a fallacy, because truth can be approached only through reason or evidence, and it may very well lie at one of the extreme poles. (See the comic on page 98.) False dilemma (trilemma, quadrilemma): False assumption from the outset that there are only a limited number of options from which to choose. (See the comic on page 90.) Falsification: A method (used in science but equally effective in the field of ethics) by which we can move toward truth by throwing out what is false. (See the comics on pages 30, 35, 11415, and 120.) Freedom: A term that may refer to (1) lack of confinement, or the capacity to do whatever one wants to do (physical freedom); (2) the capacity to free ones behavior from the determining influence of the immediate external environment by imaginatively adhering to the wishes or demands of others (negative freedom, or self-control; see the comics on pages 12, 13, 16, 17, 19, and 23); or (3) the capacity to free ones judgment from the determining influence of others by washing out bias (positive freedom, or autonomy). When one is autonomous, one has the capacity for self-legislation (i.e., one is free to create oneself). (See the comic on pages 2056.) Inductive argument: See Argument, inductive.

glossary

275

Introject: To incorporate into ones personality subconsciously. Language, sign: Communication that can transmit information only about the here and now. This is the sort of language that animals use (e.g., a growl means I am unhappy right now). (Note: Although the method of communication used by deaf individuals is often referred to as sign language, it is, in fact, symbolic language. See Language, symbolic.) Language, symbolic: A symbolic method of communication that differs from sign communication in that it can transmit information about events and situations outside the here and now (e.g., I can tell you in words, I will be very unhappy if tomorrow you dont do what you say you are going to do). Linguistically enhanced imagination: Imagination that is transformed by language. All animals can imagine the future by remembering the past, as is evidenced by the fact that animals learn by anticipating reward. However, because they possess symbolic language, humans can also imagine themselves far into the future and can think and talk about what might happen. Because of this capacity to imaginatively experience what might happen (not just what actually happens or happened), they can learn in their imaginations. (See the comic on page 14.) This capacity is a double-edged sword, because such imaginative learning is often invisible. (See the comics on pages 16, 17, and 19.) Metaphysical: Relating to a reality beyond what is perceptible to the senses. Nonempirical claim: A claim whose truth value can be estimated only through reasoning alone (e.g., casual sex is wrong). Value claims are nonempirical. Paradigm: A theoretical framework that constitutes a way of viewing reality. Poisoning the well: A subclass of an ad hominem/ad feminam attack in which one illegitimately attacks the arguer (rather than the argument) on the specific grounds that the arguer is associated with a group that is not to be trusted (e.g., shes a radical feminist; no wonder she thinks women should have equal pay for equal work). Post hoc fallacy: An assumption of causality based on correlation alone. A legitimate causal claim requires that one rule out all other possible explanations (e.g., that the correlation may have been by chance). A post hoc fallacy, in other words, lacks global sufficiency. (See the comic on page 94.) Practical reasoning: Reasoning that ultimately focuses on value and, in so doing, fuels action. (See the comic on page 106.) Practical reasoning contrasts with theoretical reasoning, which focuses on matters of fact. In addition, unlike theoretical reasoning, practical reasoning can never be put on hold, since doing nothing is just as much doing something as actually doing something. (See the comic on page 51.) Pragmatic theory of truth: A view that estimates the truth of a proposition on the basis of whether or not believing that it is true works in practice. Precision: Exactness. Precision is vitally important in practical reasoning because vagueness may fuel the wrong action or may indeed paralyze action altogether. (See the comic on pages 6162.) Premise: A claim or position used in an argument to prove the truth of a conclusion. Premises are also referred to as reasons. Note that a reason becomes a reason only in a valid deductive argument. Premise, hidden: An implicit but unstated premise. Since a reason becomes a reason only in a valid deductive argument, and since an argument requires at least two premises, anytime a reason is offered, it is inevitably accompanied by a hidden (invisible) premise. However, the strength of any reason is always a function of the strength of the entire argument. Thus, one cannot judge the strength of any reason without first determining the strength of the hidden premise. (See the comics on pages 110, 114, 11617, and 118.) Note: If one offers two reasons, it simply means that one now has two hidden premises to determine. Project (verb): To generate value-laden images of various potential actions, attitudes, or situations. These imagined stimuli, in turn, have the potential to affect ones own behavior. Note: Although this term is borrowed from psychoanalytic theory, the definition as worded is idiosyncratic to the paradigm presented here. Reason: See Premise; Premise, hidden. Relativism: The view that truth is always relative to some frame of reference. This view, which leads many to believe that everyones opinion is as good as everyone elses, is false. As reasoning beings, we have the capacity to estimate the worth of any opinion on the basis of the strength of the reasons that back it. The strength of any reason is a function of the degree to which it is invulnerable to falsification by counterexample. (See the comics on pages 30, 34, and 128.) Rottweiler flip: The switching of necessary and sufficient conditions. Note: This term is idiosyncratic to this text. In argument forms, this appears as denying the antecedent or sufficient condition and affirming the consequent or necessary condition. (See the comics on pages 146, 147, 159, 166, 167, 170, 172, and 174.) Self-consciousness: The capacity to imagine oneself from the point of view of others. Self-consciousness is not an all-or-nothing phenomenon; it is a matter of degree. Symbolic language exponentially increases a beings capacity for

276

glossary

self-consciousness. However, even among language users, self-consciousness can be presumed to vary as a function of the number of alternative views entertained. (See the comics on pages 1, 21, 27, and 205.) Self-control: The capacity to free ones behavior (although not necessarily ones judgments) from the determining influence of the immediate environment by imaginatively adhering to the wishes or demands of others. This is also referred to as negative freedom. (See the comics on pages 12, 13, 16, 17, 19, and 23.) Self-creation: The ability to choose which goals or ends to pursue, and in so doing, map out the kind of person one wants to become. Self-creation is open only to language-using self-conscious beings who can imagine themselves into the far future and who, because of practical reasoning, can move value around in their heads. (See the comics on pages 50, 55, 68, 73, and 2056.) Self-legislation: The capacity to free ones judgments from the determining power of outside influence by subjecting ones judgments to the test of both local and global sufficiency. This is also referred to as positive freedom or autonomy. (See the comics on pages 26, 63, and 67.) Shotgun argument: An argument in which a conclusion is said to be true on the basis of several reasons tied together (e.g., he died because he was a hemophiliac, he cut himself severely, and no medical help was available). Sign language: See Language, sign. Slippery slope argument: An argument that takes the following form: We should not do a, not because there is anything intrinsically wrong with a but because a leads to b, which leads to c, which will eventually lead to z, and it is z that it unacceptable. Therefore we cannot do a. A slippery slope argument is legitimatei.e., it has local sufficiency (although not necessarily global sufficiency)when the argument demonstrates why the slope is slippery. It is illegitimate when it does not. (See the comics on pages 89 and 94.) Sound argument: A valid deductive argument whose premises are true. Strawperson argument: An argument in which one woefully underestimates the strength of ones opposition. This is a particularly self-defeating fallacy, as an estimate of the truth of a proposition can be made only on the basis of its strength relative to the strength of opposing contenders. (See the comic on page 87 [note that strawpin refers to the metaphor contained in the comic]; see also the comics on pages 189 and 197.) Sufficiency, local and global: Adequacy of an argument. The local sufficiency of an argument is estimated by determining the degree to which each of its premises is vulnerable to counterexample. Thereafter, its global sufficiency is estimated according to its strength relative to that of competing contenders, with relative strength estimated as a function of the relative degree of vulnerability to counterexample. (See the comics on pages 33, 34, 3839, 128, and 200.) Symbolic language: See Language, symbolic. Theoretical reasoning: Reasoning that focuses on matters of fact, as opposed to practical reasoning, which focuses on matters of value. Theoretical reasoning alone cannot fuel action. Tu quoque: Subclass of an ad hominem/ad feminam attack in which one illegitimately attacks the arguer (rather than the argument) on the grounds that since the arguer him- or herself is guilty of ignoring the advice s/he is offering, his or her argument can be discounted. This attack is illegitimate because, no matter how hypocritical the arguer might appear, all arguments must be judged on their own merits. Tu quoque is Latin for you also. Valid: Relating specifically to a correctly formed deductive argument such that if the premises are true, then the conclusion must be true. An invalid argument thus refers to a failed attempt to construct a valid argument. Note: Validity refers only to the form, not to the content. The following is a valid deductive argument: All dogs have wings. Diva is a dog. Therefore, Diva has wings. Values: That which we hold as desirable. Values are on a par with stimuli that elicit animal responses but differ in that they have been netted by symbols and thus can be manipulated in a language users mind (which may reflect the efficacy of talk therapyi.e., that linguistically captured remembered stimuli can be linguistically manipulated and hence muted). Since values are what controls behavior, if you want to control your own actions, you need to get control of your own valuesi.e., you need to get control of your own practical reasoning. (See the comics on pages 106 and 2056.) Wisdom (versus intelligence): Thoughtful organization of ones pursuit of goals in a way that is defensible with regard to the kind of person one hopes to become. Since only language-using self-conscious beings can imagine themselves into the far future and, through practical reasoning, move value around in their heads, only language-using selfconscious agents have the capacity for wisdom. Wisdom contrasts with mere intelligence. Intelligence can be defined as the capacity to choose ever more efficient means to ends already given. (See the comics on pages 50, 5354, 55, 68, and 73.)

index
abusive ad hominem/ad feminam attacks, 80, 209 addiction, 74 ad hominem/ad feminam attacks, 208, 273; abusive, 80, 209; circumstantial, 80, 81; examples of, 8082; fallacies, 8082; illegitimate, 8082, 209, 210, 212; legitimate, 82, 209, 211; poisoning the well, 82, 271n36, 275; tu quoque, 80, 276 analogies, 273; Analogizing Your Way toward Truth and, 9697; a priori, 95; Bad Reasoning Can Misplace Values (and Associated Responses) and, 94; empirical 95; examples of, 95; fallacies and, 9397; faults and, 9397, 208, 210, 212; response to illegitimate, 94, 97; truths and, 9697 analytic truths, 29 antecedent clauses, 273; denying, 165; if . . . , then sentences and, 14243, 163 appeal to authority, 273; accessing source and, 83; appeal to the crowd and, 8384, 273; An Appeal to Tradition Can Sometimes Be a Reason and, 85; fallacies and, 8385; illegitimate, 8384, 209, 212; legitimate, 84; response to illegitimate, 8384 appeal to the crowd, 8384, 273 appeal to tradition, 84; An Appeal to Tradition Can Sometimes Be a Reason and, 85, 210, 273 appetitive stimuli, 11, 18, 273 a priori analogies, 95 argument constituents: articulation of strong opposition and, 19698; convincing response to opposition and, 198200; interactive reasoning and, 206; resolution or conclusion to posed problems and, 2013; summary of essential, 2046; support for thesis statements and, 19396; thesis statements and, 19193; truth and analysis of essential constituents of, 191206; When Self-Consciousness Takes Up the Challenge, It Is a Wonder to Behold and, 2056 arguments, 273; analyzing, 23353; avoiding Rottweiler flips and getting counterexamples straight in, 13642; back to seeing whole, 15658; conclusions and, 102, 2013; conditional claims and, 165, 16980; constituents of, 191200, 2013, 2046; content of good, 191; deducing from conditional or all claims with, 16380; deductive, 51, 10212, 16570, 172, 174, 181, 271n45, 273; evaluating global sufficiency of ones own, 13136; evaluating local sufficiency in both sides of, 12730; examples of good, 25564; getting comfortable with deductive, 103 5; inductive, 1024, 273; invalid, 1023, 16365, 16869, 276; local sufficiency and, 11327, 12930; making claims about sufficient or necessary conditions with, 14256; oppositions and, 196200; overview of, 18088; parallel, 127, 129 30; post hoc, 9193, 2089, 211, 275; practical reasoning and types of, 1025; practical reasoning, evaluating reasons, and deductive, 10512; premises and, 102; reasons and, 102, 10512, 12627, 276; responding to incorrect counterexamples in, 15963; responses and, 88, 89, 93, 99, 198200; shotgun, 12627, 276; slippery slope, 8788, 89, 209, 211, 276; sound, 88, 103, 105, 11227, 168, 276; strawperson, 8687, 130, 197, 204, 211, 276; symbolic, 15; testing for local sufficiency through counterexamples with, 11327; thesis statements and, 12930, 19196; truths and, 87, 98, 190, 191206; types of, 1023; valid, 88, 103, 105, 112, 115, 163, 168, 276; wisdom and, 90. See also practical reasoning Aristotles square of opposition, 136 articulate listening, 27, 28, 42, 43, 184, 206 associations, 13; physical, 15; process, 15 autonomy, 2, 10, 1112, 13, 17, 18, 2528, 35, 3840, 43, 48, 50, 201, 206, 273 aversive stimuli, 11, 18, 273 begging the question, 208, 209, 210, 211, 273; cognitive fishing and, 77; complex questions and, 79, 274; examples of, 78; fallacies and, 7680; A Reason That Is Not a Reason: Begging the Question (I), 77; A Reason That Is Not a Reason: Begging the Question (II), 79; response to, 80 behavior, 12, 70; evolution of, 20, 42; freedom and evolution of, 20; modifications of, 14 bias neutralization, 41, 43; impartial thinking and, 2528; processes, 24 Blair, J. A., 192, 271n50 bodies, 66 cheat sheets, 143, 164 checklist for mini-essays, 204 checklist for parallel arguments, 130 children: parents relationships with, 7475; self-control acquired by, 41 circumstantial ad hominem/ad feminam attacks, 80, 81 claims: all, 16380; arguments and deducing from conditional or all, 16380; empirical, 52, 54, 56, 207, 274; nonempirical, 52, 54, 56, 275; reasons and, 105; sufficient or necessary conditions and making, 14256; value, 54. See also conditional claims cognitive fishing, 77, 194 coherence theory of truth (falsehood), 31, 33, 119, 274 common informal faults. See fallacies communication, 28 communities of inquiry, 2526, 28 complex questions, 79, 274 conclusions, 274; arguments and, 102; examples of good, 201; indicator words for, 111; necessary conditions and, 144; posed problems and, 2013. See also arguments conditional claims: argument forms and, 165, 16980; Bad Deductive Reasoning Can Lead to Bad Decisions (I) and, 166; Bad Deductive Reasoning Can Lead to Bad Decisions (II) and, 167; Bad Deductive Reasoning Can Lead to Bad Decisions (III) and, 170; Bad Deductive Reasoning Can Lead to Bad Decisions (IV) and, 172; Bad Deductive Reasoning Can Misplace Values (and Associated Responses) and, 174; cheat sheet for valid/invalid deductions from all claims or, 164; deductive arguments and, 165, 166, 167, 16869, 170, 172, 174; logic symbols and, 164; practical reasoning and deducing from all claims or, 16380; responding to deductive arguments and, 165; responding to invalid deductive moves, 16869; responding to valid deductive moves, 168; Rottweiler flip and, 16869; triggers and, 165 conditions. See necessary conditions; sufficient conditions consequent clauses, 28, 14243, 163, 165, 274, 276 contaminated judgments, 24 content, 24, 191 contentious criteria, 64, 191, 193, 274 cooperative capacities, 15 correspondence theory of truth, 31, 33, 119, 274 counterexamples, 274; practical reasoning and responding to incorrect, 15963; responding to misunderstandings and, 16063; Rottweiler flips and, 15960; standardizing and, 181; A Yorkie Flip and, 159 Critical Thinking Textbook: Logical Self-Defense (Johnson and Blair), 192, 271n50 death, 75 decisions, 12 deductive arguments, 2, 18, 52, 102, 17580, 181, 271n45, 273; conditional claims and responding to, 165, 166, 167, 16869, 170, 172, 174; definition of, 103; features

277

278 index
deductive arguments (cont.) of, 1045; getting comfortable with, 103; practical reasoning and, 51; practical reasoning, evaluating reasons, and, 10512 determinism, 2, 40, 274 Dewey, John, 25, 28, 40, 269n17 dilemmas, 89, 274. See also false dilemma arguments disjunctive syllogisms, 88, 165 distinction without a difference, 209, 211, 274; fallacies, 9091; Own Up to an Accurate Description of Your Attitude: Distinction without a Difference and, 92; response to, 91 drugs, 74 emotions, 6870 empirical claims, 52, 54, 56, 95, 193, 204, 207, 274 empirical analogies, 95 epistemology, 31, 274 ethics, 31 evidence versus reasons, 5257 exercises, answers to, 20732 Existentialists, 36 external stimuli, 12, 15, 269n5 fallacies, 181; ad hominem/ad feminam attacks, 8082; good and faulty analogies and, 9397; appeal to authority, 8385; arguing and, 76; begging the question, 7680; distinction without a difference, 9091; false dilemma, 8890; formal, 76, 274; of the golden mean, 9899, 274; informal, 52, 76, 99102, 274; logic and, 76; post hoc, 9193, 275; practical reasoning and informal faults and common, 76102; recognizing informal, 99102; slippery slope, 8788, 89, 209, 211, 276; strawperson, 8687, 197, 211, 276. See also arguments false alternatives. See false dilemma arguments false dilemma arguments, 208, 210, 211, 274; disjunctive syllogisms and, 88; fallacies, 8890; responses to, 89; Wisdom Varies with the Options Considered: False Dilemmas and, 90 falsification, 30, 31, 33, 35, 36, 112, 119, 274; impartial thinking and estimating truths through, 2831; objectivity, verification, and, 199 faults, 35, 11415; analogies with, 9397, 208, 210, 212; good analogies and, 9397; practical reasoning and common fallacies and informal, 76102. See also fallacies faulty analogy, 9397, 208, 210, 212 freedom, 274; autonomy, or positive, 1112; controlling personal values and, 18; dynamics of value and, 1318; evolution of behavior and, 20; Freedom Requires Courage and, 19; goal of, 10; How Is Freedom Possible? and, 9; through impartial examination of values, 1820; impartial thinking, determinism, and, 40; Language Tightens the Strings of Social Control and, 16; Learning through Linguistically Enhanced Imagination Is the Human Trump Card and, 14; Materialism Threatens Freedom and, 17; negative, 11, 12, 41, 274; positive, 12, 41, 274; possibilities of, 1011; principle of, 19; Self-Consciousness Is a Double-Edged Sword and, 21; Self-Control Is Not Autonomy (I) and, 12; Self-Control Is Not Autonomy (II) and,13; self-direction, or negative, 11; theory of, 721 global sufficiency, 2, 27, 31, 35, 36, 42, 48, 86, 91, 127, 128, 181, 196, 276; arguments and evaluating ones own, 13136; impartiality with local and, 33; opposition and, 197; practical reasoning and evaluating, 13136; reasons and local sufficiency are no guarantee of, 119; writing mini-essays and, 13136 Gods-eye view, 25, 29, 30 golden mean arguments, 211, 274; examples of, 99; fallacies and, 9899; response to, 99; Truth Is a Function of Reason and/or Evidence, Not Mathematics: Fallacy of the Golden Mean and, 98 Govier, Trudy, 95 Habermas, Jrgen, 25, 270n18 Hare, Richard, 25, 31 hidden all, 137 hidden premises, 107, 11112, 15658, 181, 19394, 196, 204, 275 How Is Freedom Possible? 9 if . . . , then sentences, 14243, 163 illegitimate ad hominem/ad feminam attacks, 8082, 209, 210, 212 illegitimate analogies, 94, 97 illegitimate appeal to authority, 8384, 209, 212 imagination, 15, 18, 275 impartiality, 24, 25, 28, 33, 51 impartial reasoning, 2, 1820. See also impartial thinking impartial thinking: To Be a Chooser Requires Genuine Dialogue with Those Who Think Differently and, 26; bias neutralization as intersubjective process and, 2528; freedom needs determinism and, 40; Gods-eye view and, 29; invitation and, 40; judging quality: estimating truth through falsification and, 2931; message of, 3639; Not Everyones Opinions Are Equally Good and, 34; practical reasoning as two-step falsification process and, 31 33; Seeking the Impossible Quest and, 32; social determinism and, 2325; Social Determinism Can Be Invisible and, 23; The Stop and Go Signs toward Autonomy and, 3839; talking to relativists and, 3436; theory, 2345; thinking through values and, 19; truth seeking in ethics and, 31; Truth Seeking Is Always a Two-Step Process and, 33; Truth Seeking, Not Truth, Shall Set You Free and, 37; Truth through Falsification and, 30; What Is Objective Is Faulty Reasoning, Not Truth and, 35; You Have to Engage Others in Order to Be Free of Them and, 27 indicator words, 111 inductive arguments, 95, 102, 103, 104, 273 informal fallacies, 76, 99102, 274 intelligence, 50, 5354, 276 interactive reasoning, 26, 190, 206 intersubjective processes, 2528, 34 intimate relationships, 7274 intrasubjective processes, 26 introjection, 16, 275 invalid arguments, 1023, 16365, 16869, 276 invitation, 40 Johnson, R. H., 192, 271n50 judgments, 12; contaminated, 24; impartial thinking, estimating truth through falsification, and, 2931; precision and, 6162 Kant, Immanuel, 12, 24, 2527, 28, 31, 269n14 language, 15, 16; games, 200; Learning through Linguistically Enhanced Imagination Is the Human Trump Card and, 14; self-consciousness and users of, 10, 42; sign/symbolic, 2, 11, 1316, 18, 275, 276 least weak options, 33 legitimate ad hominem/ad feminam attacks, 82, 209, 211 legitimate appeal to authority, 84 linguistically enhanced self-consciousness (imagination), 1011, 16, 275; Learning through Linguistically Enhanced Imagination Is the Human Trump Card and, 14 linguistic interactions, 15; To Be a Chooser Requires Genuine Dialogue with Those Who Think Differently and, 26 local sufficiency, 2, 31, 33, 35, 36, 42, 48, 181, 193, 276; arguments, counterexamples, and testing for, 11327; Getting to Global Sufficiency by Testing Local Sufficiency through Counterexamples and, 128; parallel arguments and, 127; parallel arguments using thesis statements and, 12930; practical reasoning and evaluating, 12730; reasons and no guarantee of, 119; reasons, counterexamples, and testing for, 11315; truth, practical reasoning, and, 119 logic: fallacies and, 76; symbols, 146, 164; tests and demonstrating the need for, 56 Marcel, Gabriel, 10 Metaphysical reality, 13, 275 Mill, John Stuart, 31, 270n19

index

279

mini-essays: checklist for, 204; writing, 13136, 190206 modus ponens, 164, 165 modus tollens, 28, 164, 165 morality, 27 necessary conditions, 10, 163, 168, 274, 276; All Rottweilers are Dogsi.e., Being a Rottweiler Is a Sufficient Condition for Being a Dog and, 146; cheat sheet for sufficient and, 143; counterexamples and sufficient and, 15256; double negatives and, 144; if . . . , then and, 14243; logic symbols and, 146; only and, 14344; practical reasoning and, 14256; A Rottweiler Flip: Mistaking a Necessary for a Sufficient Condition and, 147; standardization, counterexamples, and, 15051; sufficient and, 14849; translations and, 14445; working quickly with reasons and, 151 negative freedom, 11, 12, 41, 274 negative tests, of rationality, 25 nonempirical claims, 52, 54, 56, 275 objectivity, 24, 199 On Liberty in Utilitarianism and Other Writings (Mill), 270n19 opposition: argument constituents and articulation of strong, 19698; argument constituents and convincing response to, 198200; To Be a Chooser Requires Genuine Dialogue with Those Who Think Differently and 26; conclusion or thesis and, 198; falsification, verification, and, 199; global sufficiency and, 197; hidden premise and, 198; language games and, 200; Reasons Are Strong Only by Comparison: Bowling for Trutha Strawpin Argument and, 87; response and,199; stated premise or reason and, 198; Strawperson Argument: Bow-wow Chow and, 197; truth and, 191, 197; You Have to Engage Others in Order to Be Free of Them and, 27. See also arguments paradigms, 19, 40, 41, 45, 275 parallel arguments: checklist for, 130; local sufficiency and, 127; thesis statements and, 12930 parent-child relationships, 7475 particular reasoning process, 24 Pavlovian conditioning, 1089 Peirce, Charles, 25, 25n16, 28, 40, 269n16 physical associations, 15 poisoning the well, 82, 271n36, 275 Popper, Karl, 28 positive freedom, 12, 41, 274 post hoc arguments, 2089, 211; Bad Reasoning Can Misplace Values (and Associated Responses) and, 94; example of, 93; fallacies, 9193, 275; response to, 93 Postman, Neil, 1 potentiality, 10, 15

practical reasoning, 2, 18, 275; argument types and, 1025; common informal faults or fallacies and, 76102; counterexamples and avoiding Rottweiler flips with, 13642; deducing from conditional or all claims and, 16380; deductive arguments and, 51; Doing Nothing Is Not an Option: It Is Doing Something and, 51; evaluating global sufficiency of, 13136; evaluating local sufficiency of ones own and ones oppositions, 12730; evaluating reasons, soundness, and, 11227; hidden premises in forced-choice situations and, 15658; impartiality and, 51; impartial thinking, falsification process, and, 3133; intelligence and, 50; interactive, 206; local sufficiency and theories of truth in, 119; looking at values and, 6276; overview of, 18088; Pavlovian conditioning and, 1089; practice and learning intricacies of, 47188; pushing toward precision and, 5862; reasons versus evidence and, 5257; responding to incorrect counterexamples and, 15963; Seeking a Good You Rather than Good Times and, 50; sufficient or necessary conditions with, 14256; valid deductive arguments, evaluating reasons, and, 10512; value reasoning and, 51; wisdom and, 50; Wisdom versus Intelligence and, 5354. See also reasons practice: learning intricacies of practical reasoning and, 47188; thinking and writing toward truth in, 189206 pragmatic theory of truth, 31, 33, 275 precision, 181, 275; Imprecision Is Inefficient in Action Guiding, 58; practical reasoning and pushing toward, 5862; Without Precision, Value Judgments Cannot Act as Magnets, 6162; responding to ensure, 59 premises, 275; arguments and, 102; counterexamples, reasons, and hidden, 12122, 12526; finding hidden, 107, 11112, 15658, 181, 19394, 196, 204, 275; forced-choice situations and hidden, 15658; opposition and, 198; practical reasoning, forced-choice situations, and hidden, 15658; reasons and breaking major, 110; thesis statements and hidden, 19394; tricks for finding hidden, 108. See also reasons probabilities, 31, 103 projection, 11, 1518, 4142, 275 projected values, 1516, 18, 41 quadrilemmas, 89, 274 quantifier, 136, 138 questions: addiction and drugs, 74; begging, 76, 77, 78, 79, 80, 19495, 196, 208, 209, 210, 211, 273; behavior, 70; bodies and, 66; about child-parent relationships, 7475; clear and precise, 63; complex, 79, 274; contentious, 64; emotions and, 6870; God, death, and religion, 75; good, 63, 181; about intimate relationships and

sex, 7274; about lack of action, 7072; about owing others, 66, 68; practical, 63; relevant, 64; tentative, 64; values and criteria for good, 6364; values, practical issues, and self-directed, 65 Rawls, John, 31 reasons, 2; arguing about how classes of things relate and, 1079; arguments and, 102; Breaking the Major Premise and, 110; claims and, 105; coherence and, 119; conclusions and, 119; correspondence and, 119; evaluating, 181; falsification and, 112; finding hidden, 11112; hidden premises, counterexamples, and evaluating, 12122, 12526; I Am Talking about Homosexuals, Not Glasses and, 118; I Am Talking about Rapists, Not Enron Executives and, 11617; imaginatively mediated practical, 109; indicator words for, 111; linguistically mediated practical, 109; local sufficiency and theories of truth with practical, 119; local sufficiency does not guarantee global sufficiency with, 119; Me Only because of We and, 124; practical reasoning and evidence versus, 5257; practical reasoning, deductive arguments, and evaluating, 10512; The Positivity of Negativity and, 123; practical reasoning, soundness, and evaluating, 11227; Reasoning Changes Behavior by Moving Value and, 1067; shotgun arguments and, 12627, 276; sufficient conditions and, 144; testing for local sufficiency through counterexamples and, 11315; tricks for finding hidden premises or, 108; What Would Count as Falsifying Your Belief and, 120; The Wonders of Fault Finding and, 11415. See also hidden premises; practical reasoning; premises reasons versus evidence, 181; empirical claims and, 52, 54, 56; Freedom Is about the Future and, 55; nonempirical claims and, 52, 54, 56; practical reasoning and, 5257; Wisdom versus Intelligence and, 5354 relationships: children-parent, 7475; sex and intimate, 7274 relative adequacy, 27 relativism, 3436, 44, 275 relevant criteria, 64, 191, 193 religion, 75 responses: analogies and illegitimate, 97; analogies, bad reasoning, and, 94; appeal to authority and illegitimate, 8384; arguments and, 88, 89, 93, 99, 198200; begging the question and, 80; conditional claims and, 174; distinction without a difference and, 91; false dilemma arguments and, 89; golden mean arguments and, 99; opposition, argument constituents, and, 198200; post hoc arguments and, 93; slippery slope arguments and illegitimate/ legitimate, 88; truths and, 191. See also arguments Rodriguez, Sue, 271n37

280 index
Rottweiler/dog tactic, 139 Rottweiler flips, 275; Aristotles square of opposition and, 136; Bad Deductive Reasoning Can Lead to Bad Decisions (I) and, 166; Bad Deductive Reasoning Can Lead to Bad Decisions (II) and, 167; Bad Deductive Reasoning Can Lead to Bad Decisions (III) and, 170; Bad Deductive Reasoning Can Lead to Bad Decisions (IV) and, 172; Bad Deductive Reasoning Can Misplace Values (and Associated Responses) and, 174; conditional claims and, 16869; contradictory patterns and, 136; counterexamples and, 15960; determining order and, 139; hidden alls and, 137; necessary conditions and, 146, 147; practical reasoning, counterexamples, and avoiding, 13642; standard forms and, 136; standardization, counterexamples, and, 14042; standardization formula and, 138 self-consciousness, 10, 15, 42, 2056, 27576 self-control, 11, 12, 15, 16, 276; children and, 41; social control and, 17 self-creation, 2, 1011, 36, 50, 276 self-direction, 10, 11, 17 self-generated stimuli, 12 self-legislation, 2, 10, 12, 63, 276 sex, 7274 shotgun arguments, 12627, 276 sign/symbolic language, 2, 11, 1316, 18, 275, 276 Silence of the Lambs, 272n53 slippery slope arguments, 89, 276; examples of legitimate and illegitimate, 88; fallacies, 8788; illegitimate, 8788, 209, 211; legitimate, 8788, 209; Reasons Can Sometimes Connect A to Z: Slippery Slope Arguments and, 89; response to legitimate and illegitimate, 88 social control, 15, 16, 17 social determinism, 2325 sound arguments, 88, 103, 105, 11227, 168, 276 soundness. See reasons Squandering Self-Consciousness, 1 standardization, 13642 stimuli: appetitive, 11, 273; aversive, 11, 18, 273; external, 12, 15; self-generated, 12 strawperson arguments, 28, 130, 197, 204, 211, 276; examples of, 86; fallacies, 8687; misrepresenting opposing positions and, 86; Reasons Are Strong Only by Comparison: Bowling for Trutha Strawpin Argument, 87; Strawperson Argument: Bow-wow Chow and, 197 sufficiency: global, 2, 27, 33, 36, 48, 119, 128, 13136, 181, 197, 276; local, 2, 33, 36, 48, 11326, 12730, 181, 276 sufficient conditions, 163, 276; practical reasoning and, 14256; reasons as, 144. See also necessary conditions support: argument constituents and, 19396; truths and convincing, 191. See also arguments syllogisms: conditional claims and disjunctive, 165; disjunctive, 88, 165; middle and major terms with, 271n47 symbolic arguments, 15 symbolic interaction, 14, 15, 18 symbolic/sign language, 2, 11, 1316, 18, 275, 276 synthetic truths, 29 tentative criteria, 64, 192, 193 tests: arguments, local sufficiency through counterexamples, and, 11327; demonstrating the need for logic, 56; reasons, counterexamples, and, 11315; types of thinkers and, 4 theoretical reasoning, 2, 18, 31, 50, 276 theory: impartial thinking, 2345; possibility of freedom, 721 thesis statements, 6265, 76, 204; arguments and, 19193; arguments and support for, 19396; articulation of strong opposition and, 196; begging the question and, 19495, 196; Begging the Question: Boring Hypocritical Repetition and, 195; clear and precise, 19293; contestable, 191; hidden premises and, 19394; parallel arguments and, 12930; reasons, empirical evidence, and, 193; relevant and unique, 19192; tentative, 192; treating opposition with respect, 196. See also arguments thinkers: good and bad, 26566; types of, 4, 26768. See also thinking thinking: bias neutralization and impartial, 2528; falsification, estimating truths, and impartial, 2931; freedom, determinism, and impartial, 40; Gods-eye views and impartial, 29; impartial, 2345; practice, writing toward truth, and, 189206; values and impartial, 19 translation rules, 145 translations, 14445 triggers, 165 trilemmas, 89, 274 truths: analogies and, 9697; analytic, 29; arguments and, 87, 98, 190, 191206; coherence theory of, 274; conclusions and, 191; content of good arguments and, 191; convincing support and, 191; correspondence theory of, 274; detailed analysis of five essential argument constituents and, 191206; golden mean arguments and, 98; identifying good arguments and, 19091; impartial thinking and, 2931, 33, 35, 37; impartial thinking, ethics, and seeking, 31; impartial thinking, judging quality, and, 2931; interactive learning with imagination and, 190; It Is More Important to Be Right than to Appear Right and, 189; opposition and, 191, 197; practice, thinking, and writing toward, 189206; pragmatic theory of, 275; reasons, local sufficiency, and theories of, 119; response and, 191; strawperson arguments and, 87; synthetic, 29; thesis statements and, 191 tu quoque ad hominem/ad feminam attacks, 80, 276 valid arguments, 88, 103, 112, 115, 168, 275, 276 validity, 88, 1023, 105, 112, 275, 276 value claims, 54. See also nonempirical claims value reasoning, 51 values, 276; Are Your Emotions Those That You Choose? and, 69; attempting to prove what is obvious and, 65; categories for thesis statements and, 6675; claims, 54; criteria for good questions with, 6364; freedom and controlling personal, 18; freedom and dynamics of, 1318; freedom and impartial examination of, 1820; If You Do Not Care, You Are Not in Charge and, 63; impartially thinking through, 19; Labeling Relevant Behavior Can Be Action Guiding and, 67; The Mind Is Opened by What Is Interesting: What Is Interesting Is Contentious and, 64; practical reasoning and looking at, 6276; self-directed questions, practical issues, and, 65; symbolic interaction and, 18; Values Become Yours Only When They Inform Your Actions and, 71; You May Be Blind to the Person You Are Becoming and, 73; Your Present Values Are Your Window into Your Future and, 68 Vannoy, Russell, 95 verifications, 30 wisdom, 90, 276; practical reasoning and, 50; Wisdom versus Intelligence and, 5354 Wittgenstein, Ludwig, 25 writings, 2023; mini-essays and global sufficiency, 13136; practice, thinking, and truth with, 189206 A Yorkie Flip, 159

Susan T. Gardner is Professor of Philosophy at Capilano University in North Vancouver, Canada.

Вам также может понравиться